You are on page 1of 222

Basic-Sciences Review for Medical Licensure

Developed at
The University of Oklahoma, College of Medicine

Suitable Reviews for:


United States Medical Licensing Examination
(USMLE), Step 1
Federation Licensing Examination (FLEX)
Preface to the
()~'1ttJte4,

In 1973, the University of Oklahoma College of Medicine instituted a require-


ment for passage of the Part 1 National Boards for promotion to the third year.
To assist students in preparation for this examination, a two-week review of the
basic sciences was added to the curriculum in 1975. Ten review texts were writ-
ten by the faculty: four in anatomical sciences and one each in the other six
basic sciences. Self-instructional quizzes were also developed by each discipline
and administered during the review period.
The first year the course was instituted the Total Score performance on Na-
tional Boards Part I increased 60 points, with the relative standing of the school
changing from 56th to 9th in the nation. The performance of the class since then
has remained near the national candidate mean (500) with a range of 467 to 537.
This improvement in our own students' performance has been documented (Hyde
et al: Performance on NBME Part I examination in relation to policies regarding
use of test. J. Med. Educ. 60:439-443, 1985).
A questionnaire was administered to one of the classes after they had com-
pleted the Boards; 82% rated the review books as the most beneficial part of the
course. These texts were subsequently rewritten and made available for use by
all students of medicine who were preparing for comprehensive examinations
in the Basic Medical Sciences. Since their introduction in 1987, over a quarter
of a million copies have been sold. Assuming that 60,000 students have been
first-time takers in the intervening five years, this equates to an average of four
books per examinee.
Obviously these texts have proven to be of value. The main reason is that they
present a concise overview of each discipline, emphasizing the content and con-
cepts most appropriate to the task at hand, i.e., passage of a comprehensive ex-
amination over the Basic Medical Sciences.
The recent changes in the licensure examination structure that have been made
to create a Step 1/Step 2 process have necessitated a complete revision of the
Oklahoma Notes. This task was begun in the summer of 1991; the book you are
now holding is a product of that revision. Besides bringing each book up to date,
the authors have made every effort to make the texts and review questions con-
form to the new format of the National Board of Medical Examiners tests.
I hope you will find these review books valuable in your preparation for the
licensure exams. Good Luck!

Richard M. Hyde, Ph.D.


Executive Editor
"The time has come," the Walrus said,
"To talk of many things:
Of shoes-and ships-and sealing wax-
Of cabbages-and kings-
And why the sea is boiling hot-
And whether pigs have wings."

Lewis Carroll
Pathology
Third Edition

John H. Holliman

Springer-Verlag
New York Berlin Heidelberg London Paris
Tokyo Hong Kong Barcelona Budapest
John H. Holliman, M.D.
Department of Pathology
College of Medicine
Health Sciences Center
The University of Oklahoma
Oklahoma City, OK 73190
USA

Library of Congress Cataloging-in-Publication Data


Holliman, John H.
Pathology / John H. Holliman.-3rd ed.
p. cm.-(Oklahoma notes)
ISBN-13: 978-0-387-97780-5
1. Pathology-Outlines, syllabi, etc. I. Title. II. Series.
[DNLM: 1. Pathology-examination questions.
2. Pathology-outlines. QZ 18 H739p]
RB32.H65 1992
616.07'076-dc20
DNLM/DLC
for Library of Congress 91-5246

Printed on acid-free paper.

© 1992, 1988, and 1987 Springer-Verlag New York, Inc.


All rights reserved. This work may not be translated or copied in whole or in part without
the written permission of the publisher (Springer-Verlag New York, Inc., 175 Fifth Ave-
nue, New York, NY 10010, USA), except for brief excerpts in connection with reviews or
scholarly analysis. Use in connection with any form of information storage and retrieval,
electronic adaptation, computer software, or by similar or dissimilar methodology now
known or hereafter developed is forbidden.
The use of general descriptive names, trade names, trademarks, etc., in this publication,
even if the former are not especially identified, is not to be taken as a sign that such
names, as understood by the Trade Marks and Merchandise Marks Act, may accordingly
be used freely by anyone.
While the advice and information in this book is believed to be true and accurate at the
date of going to press, neither the authors nor the editors nor the publisher can accept
any legal responsibility for any errors or omissions that may be made. The publisher makes
no warranty, express or implied, with respect to the material contained herein.

Production managed by Henry Krell; manufacturing supervised by Jacqui Ashri.


Camera-ready copy prepared by the author.

9 8 7 6 5 4 3 2 1

ISBN-13: 978-0-387-97780-5 e-ISBN-13: 978-1-4684-0435-7


DOl: 10.1007/978-1-4684-0435-7
Preface

These notes were developed as a synopsis of general and systemic pathology in


order to present to those students who are preparing for national examinations
a relatively concise review of the discipline. These notes are not intended to
replace the many fine textbooks of pathology whose scope, detail, and mission
are more suited to the in-depth study of pathology. Rather, these notes are in-
tended to serve as "memory joggers" to rekindle and refresh the recall of salient
features of various disease processes and to point out to the student possible
areas of weakness which they may need to address in more detail by referring
to one of the standard textbooks of pathology.
Hopefully, the notes will allow a systematic review of the material in a rea-
sonable amount of time. Some major topics in Pathology (genetic diseases, im-
munologic disorders, and infectious diseases) are covered in other areas of our
curriculum and are therefore not included in an organized fashion in these notes.
You may wish to refer to other titles in the Oklahoma Notes series for more
detailed discussions of these areas. -
The first portion of the notes reviews some of the basic principles and patterns
of disease. The remairider of the notes reviews disorders of individual organ sys-
tems. There are two self assessment exams at the end of the notes containing
questions that have previously been used on local exams and, although I have
tried to filter it out, they may contain wording that is not presented in the notes.
H so, please refer to your standard pathology textbooks.
Most "disease" results from deviation of normal cell/tissue structure and func-
tion. A thorough understanding of what can induce these changes, how these
changes evolve to produce signs and symptoms, and how you may be able in-
tervene in this process will enable you to take better care of your patients. And
that's why we're here, isn't it?

John H. Holliman
Contents
Preface to the Oklahoma Notes ..................................... v
Preface ........................................................... vii

General Pathology

Principles of Cell/Tissue Injury .................................... 1


Principles of Fluid Balance and Hemodynamics ..................... 8
Principles of Inflammation ......................................... 13
Principles of Wound Healing and Tissue Repair ..................... 18
Principles of Neoplasia ............................................ 21

Systemic Pathology

Cardiovascular System ............................................. 29


Congestive Heart Failure ......................................... 29
Congenital Heart Disorders ....................................... 30
Atherosclerosis .... " ....................................... " . . . 32
Ischemic Heart Disease/Coronary Heart Disease .................... 35
Rheumatic Fever/Rheumatic Heart Disease ........................ 38
Endocardial/Valvular Heart Disease ............................... 39
Myocardial Heart Disease ........................................ 41
Pericardial Heart Disease ......................................... 42
Cardiac Neoplasms .............................................. 43
Vascular Disease ................................................ 44

Respiratory Tract .................................................. 48


Congenital Disorders . . . . . • . . . . . . . . . . . . . . . . . . . . . . . . . . . . . . . . . . . . . . . 48
Respiratory Distress Syndromes .................................. 49
Inflammatory Disorders .......................................... 50
Chronic Obstructive Pulmonary Disease ........................... 54
Restrictive Pulmonary Diseases ................................... 56
Respiratory Tract Neoplasms ..................................... 57

Hematopoietic/Lymphoreticular Systems ............................ 60


Basic Laboratory Hematology ..................................... 60
Coagulation Disorders ........................................... 62
Anemia......................................................... 66
Acute Leukemia ........... . . . . . . . . . . . . . . . . . . . . . . . . . . . . . . . . . . . . . . 68
Myelodysplastic Syndromes ...................................... 70
Myeloproliferative Syndromes. ... . . . . .. . .. .. . .. . . . . . . . . . . . . . .. . . . 72
Lymphoproliferative Disorders ................................... 73
x Contents

Immunosecretory Disorders ...................................... 73


Malignant Lymphomas........................................... 75
Histiocytoses . . . . . . . . . . . . . . . . . . . . . . . . . . . . . . . . . . . . . . . . . . . . . . . . . . . . 79
Thymic Disorders ............................................... 79

Endocrine System ................................................. 81


General Principles... . ... . . .. .. . .. . . . ... . . .. . . . ... . . . . . .. .. . . . . .. 81
Pituitary ........................................................ 81
Thyroid......................................................... 84
Parathyroid ..................................................... 89
Adrenal Cortex .................................................. 90
Endocrine Pancreas .............................................. 93
Diffuse Endocrine System ........................................ 96
Multiple Endocrine Neoplasia Syndromes ......................... 97

Breast......................................... ................... 98
Fibrocystic Change .............................................. 99
Neoplasms......................................... ........ ..... 100

Female Genital Tract .............................................. 104


Sexually Transmitted Diseases ................................... 104
Vulva/Vagina.................. ................................. 105
Cervix.......................................................... 107
Uterus.......................................................... 109
Fallopian Tubes ................................................. 111
Ovary ...................................... _.................... 112
Gestational Trophoblastic Neoplasms ............................. 117

Male Genital Tract ................................................ 118


Penis. . . .. . . . .. . .. .. . .. .. . .. .. . .. .. . .. . . . . . . . . .. .... . . .. .. . . . .. . 118
Testis... ............................................. ........ ... 118
Prostate. . . . . . . . . . . . . . . . . . . . . . . . . . . . . . . . . . . . . . . . . . . . . . . . . . . . . . . . . 120

Urinary Tract ..................................................... 122


Congenital/Developmental Disqrders .............................. 122
Glomerular Disease .............................................. 124
Pyelonephritis ........................................... " . ... .. 127
Hypertension. . . . . . . . . . . . . . . . . . . . . . . . . . . . . . . . . . . . . . . . . . . . . . . . . . . . 127
Acute Renal Failure ............................................. 128
Chronic Renal Failure ........................................... 129
Renal Neoplasms ................................................ 129
Renal Manifestations of Systemic Disease ......................... 130
Urinary Bladder. .. . .. .... ... .. . .. . . ... . .. .. .. .... ......... . .. . . . 131

Gastrointestinal Tract .............................................. 132


Esophagus ...................................................... 132
Stomach ........................................................ 134
Small Bowel/Colon/Rectum ..................................... 137

Hepatobiliary System and Exocrine Pancreas ........................ 147


Qilirubin Metabolism/Jaundice ................................... 147
Inflammatory Disease ............................................ 151
Cirrhosis . . . . . . . . . . . . . . . . . . . . . . . . . . . . . . . . . . . . . . . . . . . . . . . . . . . . . . . . 154
Contents xi

Hepatic Neoplasia ............................................... 156


Hepatic Failure . . . . . . . . . . . . . . . . . . . . . . . . . . . . . . . . . . . . . . . . . . . . . . . . . . 158
Gallbladder and Biliary Tract . . . . . . . . . . . . . . . . . . . . . . . . . . . . . . . . . . . . . 158
Exocrine Pancreas............................................... 161

Skin.............................................................. 163
Acute Inflammatory Disease...................................... 163
Infectious Disease ............................................... 164
PapulosquaEnousDisease ......................................... 165
Vesiculobullous Disease ......................................... 166
Connective Tissue (Autoimmune) Disorders ....................... 167
Epidermal Tumors and Tumor-like Conditions .................... 168
Dermal Tumors and Infiltrates .................................... 170
Melanocytic Tumors............................................. 171

Skeletal System ...................................... .-............ 173


Congenital Disorders................ . ........ .... . .... ... ..... .. . 173
Metabolic Bone Disorders ........................................ 173
Bone Neoplasia ................................................. 174
Joint Disorders .................................................. 175

Neuromuscular System ............................................. 177


Reaction to Injury ............................................... 178
Traumatic Injury ................................................ 179
Vascular Disease .. ~..... ...... ........... . . ... ... ......... . .... . 180'
Infectious Disease .............................. ~................ 183
Dementia and Degenerative Disease ............................... 186
Demyelinating Disease........................................... 187
Metabolic Disease ............................................... 187
CNS Neoplasia .................................................. 188
Peripheral Nervous System ..... . . . . . . . . . . . . . . . . . . . . . . . . . . . . . . . . . . 191
Skeletal Muscle ................................................. 192
PRINCIPLES OF CELL/TISSUE INJURY

I. THE STEADY STATE - In a dramatic break with the thinking of the time,
Rudolf Virchow showed that cellular alterations correlate with clinical
aspects of disease. In general, the viability of a cell is dependent on
its ability to produce energy, to manufacture vital proteins, to maintain
structural integrity and, in most cases, to replicate. "Health" therefore
depends on the ability of the cells to maintain stability (a steady state)
in the face of a constantly changing microenvironment (a process termed
homeostasis) while "disease" results from impairment of any of these
functions and may be reflected by structural, physiologic, biochemical, or
molecular changes in the cell.
II. CELLUIJAR RFBPONSETOAN ALTERED STEADY STATE - Change in the cellular
environment whether by substantially altered physiologic stimuli or patho-
logic injury induces an " altered" steady state. The degree of " alteration"
is a product of the type of change, its intensity, its duration, and the
innate capacity of that particular cell to withstand change. An acute
injury or change in the microenvironment, if mild, may only transiently
alter cellular function. Persistent injury, excessive physiologic demands,
or substantial alterations in the cellular microenvironment, however, may
result in cellular adaptation. This adaptation to an altered steady state
is potentially reversible when the original cellular microenvironment is
restored. At a biochemical and cellular level, there are numerous adaptive
strategies (use of alternate biochemical pathways, induction of new
metabolic products, alteration of cellular receptors, etc.) to cope with
the change in environment.
A. ATROPHY - is an adaptive decrease in the size or number of individual
cells that had previously been of normal size and number (thus a
decrease in the size of a tissue or organ). Like many adaptive re-
sponses, this can either be due to normal physiologic or abnormal
pathologic changes in the environment. When pathologic, atrophy re-
sults from a number of influences that restrict oxygenation, nutri-
tion, or stimulation of a cell. Cellular organelles (mitochondria,
endoplasmic reticulum, etc.) are decreased in number resulting from
either increased catabolism (as illustrated by increased numbers of
autophagic vacuoles and residual bodies) or decreased synthesis of
cell constituents.
B. HYPERTROPHY - is an adaptive increase in the size of individual cells
(thus an increase in the size and weight of a tissue or organ) in
response to increased stimulation or functional demand. Generally,
this involves an increase in structural components of those cells
that are not generally capable of mitotic division. The cell only
has a limited capacity to enlarge, however, and persistent excessive
stimulation or demand will ultimately result in cell exhaustion and
failure.
C. HYPERPLASIA - is an adaptive increase in the number of cells (thus
leading to an increase in volume, and perhaps size, of a tissue or
organ) usually in response to increased hormonal or growth factor
stimulation. Generally, this involves an increased stimulation of
cells that are capable of mitotic division and, since hyperplasia
involves increased mitotic division, it is not surprising that
hyperplasia and neoplasia are often closely associated.
D. ~APLASIA - is an adaptive substitution of one differentiated cell
type for another differentiated cell type which is better suited to
the new microenvironment. Metaplasia is generally regarded not as a
physiologic but rather as a pathologic process in which a harsher
environment induces a change to a more protective tissue type. In
some instances, persistence of the adverse environment may ulti-
mately induce neoplastic transformation.
E. DYSPLASIA - is an atypical, generally non-adaptive but reversible
growth of cells that is usually induced by chronic irritation or
stimulation. Varying degrees of cytologic and nuclear atypia are
present including alteration in the shape and size of the cell
and/or nucleus, loss of cell orientation, and increased mitoses.

1
Similar changes are observed in neoplastic cells and, indeed,
dysplasia is generally regarded as a potential precursor to cancer.
F. NEOPUSIA - is an abnormal, non-adaptive and irreversible growth of
cells which exceeds and is uncoordinated with the growth of normal
cells.
G. DEGENERATIVE AND/OR DEATH - if a cell is unable to adapt to a new
environment or if there are overwhelming or unusual injuries, cells
may undergo degeneration and ultimately death. Degenerative changes
are potentially reversible if the inciting cause is eliminated;
however, the line between reversibly and irreversibly injured cells
may at times be quite indistinct.
III. MECHANISMS OF CELL/TISSUE INJURY
A. O~EN - can injure cells through deficiency or absence (hypoxia/-
anoxia) and through formation of byproducts (free radicals). Hypoxic
injury can serve as a prototype of the processes involved in both
reversibly and irreversibly injured cells.
1. (In general, those cells that have high metabolic
HYPOXIA/ANOXIA
activity are most sensitive to the effects of hypoxia/anoxia.)
a. Reversible injury - Whether due to poor 02 saturation of
the blood, decreased hemoglobin, or insufficient vascu-
lar supply (ischemia), oxygen deficiency interferes with
mitochondrial oxidative phosphorylation and the produc-
tion of ATP. To maintain energy production, the anaero-
bic glycolytic pathway is activated which reduces cellu-
lar glycogen and increases cellular lactic acid. The
resulting drop in pH causes clumping of the nuclear
chromatin. The loss of ATP interferes with the energy-
dependent sodium pump in the plasma membrane, and the
resulting influx of sodium, calcium, and water causes
cellular swelling. Ultrastructurally, cytoskeletal
damage may be apparent. "Blebs" may develop indicating
damage to the plasma membrane. The endoplasmic reticulum
dilates, ribosomes become detached from the rough endo-
plasmic reticulum, and polysomes dis aggregate thereby
interfering with protein production. Initially, mito-
chondria condense but subsequently swell and develop
amorphous densities consisting of lipids, proteins and
calcium.
b. Irreversible injury - At this point, although the exact
biochemical mechanisms are unknown, if oxygen depriva-
tion continues, the cell becomes irreversibly injured.
In addition to the increase of cytoplasmic calcium
resulting from failure of the membrane ionic pump, there
is added release of calcium that is normally sequestered
in the mitochondria and endoplasmic reticulum. Excess
calcium may activate the endogenous calcium-dependent
phospholipases which then be~in to break down the cell
membranes. Ultrastructural myelin figures" indicate
cellular membrane damage and this leads to further per-
meability changes that perpetuate the damage. Synthesis
of replacement phospholipids is hampered by the lack of
ATP production in the injured mitochondria which now
show internal vacuolization. Other mechanisms (breakdown
of the cytoskeleton, free radical formation, accumula-
tion of catabolic products, etc) may also contribute to
additional plasma and/or organelle membrane damage.
Ultimately, the lysosomal membranes become porous enough
to release degradative enzymes into the cytoplasm, and
cellular digestion (autolysis) occurs.

2
2. In the cell, partial reduction of oxygen
FREE RADICAL FORtlATION -
is an important source of free radicals (superoxides, hydroxyl
ions and hydrogen peroxide) - atoms that contain a single
electron in an outer orbital and are therefore extremely
unstable and reactive. They are able to provoke inappropriate
disulfide bonding of proteins, peroxidation of lipids, and
damage to DNA. They can be created by ionizing radiation,
metabolism of drugs, or as byproducts of normal metabolism and
are inactivated by spontaneous decay, by naturally occurring
antioxidants (superoxide dismutase, etc), or by interaction
with specific enzymes. In reacting with other substances
however (especially in peroxidation of lipids), additional
free radicals may be formed which can initiate a chain of
autocatalytic events.
B. CHEHICALS - are an important cause of cell injury and cell death and
can cause tissue damage by a wide variety of pathways. Chemicals can
be inhaled, ingested, or absorbed through the skin. They can
stimulate, suppress, or disrupt normal cellular function and alter
membrane permeability either directly or through the action of
metabolic byproducts. The toxicity of many chemicals may be related
to the induction of intracellular free radicals or to triggering
immunologic reactions.
C. RADIATION - the type of radiation, dosage, mode of delivery, and
oxygen supply to the tissues all influence the effect of radiation.
1. IONIZING RADIATION (alpha and beta particles, gamma rays, X-rays)
causes cellular injury by the transfer of radiant energy which
may, through radiolysis of intracellular water, produce free
radicals. Ionizing radiation disrupts molecular bonding in DNA
resulting in single or double stranded breaks that lead to
mutation, altered function, or even cell death. It is not
surprising, therefore, that morphologic alterations of the
nucleus (giant cells, bizarre pleomorphism, etc) may become
apparent by light microscopy and mimic changes seen in malig-
nant neoplastic cells. Indeed, radiation may actually induce
certain neoplasms. Striking changes may also be observed in
the vasculature of irradiated tissue. Acutely, the vessels may
dilate, thrombose, or rupture. Over time, however, reactive
endothelial cell proliferation and mural scarring may lead to
narrowing or even obliteration of the vessel lumens causing
hypoperfusion of tissue. Cells and tissues vary in their
sensitivity to ionizing radiation. In general, cells with a
high degree of specialization and low turnover rate are most
radioresistant. .
2. ULTRAVIOLET RAYS - penetrate the skin only superficially and
induce injury after a latent period of several hours (sun-
burn). Systemic symptoms include fever, headache, nausea,
vomiting. Repeated exposure causes progressive pigmentation
and skin thickening and increases the risk of developing
various cutaneous malignancies.
3. INFRARED RAYS AND MICROWAVES - can produce heat and, in overdos es ,
may cause thermal burns, edema, and vesiculation of the skin.
D. cell injury and/or cell death result if tissue
TEMPERATURE EXTREMES -
is maintained at a temperature greater than 1S0C below or more than
SOC above normal body temperature. The severity of injury is related
to the duration of the exposure.
1. LOCALIZED HYPOTHERMIA - cooling or freezing of tissue damages
vascular endothelium and increases vascular permeability
leading to edema and blister formation. Crystallization of
intracellular water interferes with ionic concentrations,
denatures proteins, and can physically disrupt cell membranes
leading to cell death (frostbite). The circulatory changes
(including vasoconstriction, vasodilation, and increased
permeability) may also cause extensive injury due to poor
blood perfusion of surrounding tissue.

3
2. SYS'fEHIC HYPOTHERtiIA -if compensatory mechanisms (increased BMR,
shivering, etc) are overwhelmed, continued dissipation of the
internal core heat leads to metabolic depression, mental
confusion, lethargy, and coma. Vascular collapse and cardiac
arrhythmias are major causes of death.
3. LOCALIZED HYPERTHERtiIA (BURNS) can cause coagulation of
intracellular protein, membrane damage, and interruption of
metabolic pathways. Circulatory changes include vasodilatation
and increased vascular permeability. The clinical significance
of cutaneous burns depends on the depth and the surface area
covered. Third degree (full thickness) burns are the most
severe and destroy not only the epidermis but the underlying
dermis and dermal appendages as well. First and second degree
(partial thickness) burns leave basal epithelium or at least
some of the dermal appendages intact (from which epithelial
cells can be regenerated). Inhalation of superheated air can
cause similar injury throughout the respiratory tract.
Complications of burns include shock (loss of circulating
blood volume), hemoconcentration, electrolyte imbalance, and
infection (responsible for over 50% of deaths).
4. SYS'fEHIC HYPERTHERtiIA results from either increased heat
production (increased BMR, increased muscular activity, etc),
decreased heat loss (peripheral vasoconstriction, interference
with sweating mechanisms, etc), or alteration of the "set
point" of the hypothalamic regulatory centers (macrophage
release of interleukin-l in infections, etc). Compensatory
peripheral vasodilation leads to pooling of blood and hypo-
tension. Excessive sweating may lead to severe fluid and
electrolyte imbalances which could induce shock.
E. BIO~IC AGEm5 - include a wide spectrum of microbial organisms from
viruses to higher forms of parasites.
F. DtMUNO~IC REACfIONS - Although immunologic competency is necessary for
normal life function, hypersensitivity or immunodeficiency reactions
may result in cell injury and cell death.
G. GENETIC MUTATIONS - may alter a cell's ability to maintain normal
homeostasis by interfering with the regulation and/or structure of
the protein products of the genes.
H. NUTRITIONAL DlBALANCES - may interfere with a cell's ability to maintain
structure and function and includes disorders such as starvation,
avitaminosis, hypervitaminosis, and obesity.
I. MECHANICAL TRAUMA - produces wounds such as abrasions (loss of super-
ficial cells as the result of friction), contusions (disruption of
blood vessels produced by blunt force), lacerations (the tearing of
tissue resulting from excessive stretching), and incisions (cuts
produced by a sharp instrument).
J. ELECfRICITY - electrothermal injury occurs when some part of the body
completes a circuit between two conductors. It can cause cell injury
or death either due to interruption of neural transmissions in the
cardiac conduction system, or by the generation of heat. The outcome
of electrothermal injury depends on the voltage, amperage, conduc-
tion path through the body, and the electrical resistance of the
tissue (a function of the water content) in that conduction path.
K. ATMOSPHERIC PRESSURE - injury depends on the rate of change of pressure.
Increased pressure is tolerated better than decreased pressure.
L. SOUND - injury to various cells may result from repeated exposure to
loud volume (e.g. > 100 decibel).
IV. MORPHOl,OGIC P ATIERNS OF CEU, IN.JURY (DEGENERATION vs NECROSIS)

A. REVERSIBLEINJURIES - may be reflected by cellular alterations that


sometimes are referred to as degenerations. Whether by overproduc-
tion, interruption of metabolic pathways or interference with normal
control mechanisms, degenerations are often manifested by the exces-
sive accumulation, either in the cytoplasm or nucleus, of cellular
products. Although intracellular accumulations do not always indi-
cate cell injury, they may nevertheless interfere with the normal
functioning of the cell.
4
1. HATER

a. Cellular swelling - is the first morphologic change to


occur after disturbances of membrane permeability alter
the regulation of fluid volume and ionic concentrations.
Injured cells appear enlarged with a pale cytoplasm but
normally positioned nucleus. A concomitant compression
of the microvasculature can occur which may contribute
to further cell injury. Cellular swelling can be grossly
appreciated by increased weight of an organ or tissue.
b. Hydropic change (vacuolar degeneration) - is an accen-
tuation of cellular swelling. Segments of distended
endoplasmic reticulum appear in the cytoplasm as
vacuoles, and may displace the nucleus to the periphery
of the cell.
2. LIPm

a. Fatty degeneration - refers to an absolute increase in


neutral fats within parenchymal cells (most often liver,
heart, muscle, and kidney). With any interruption of
normal cellular lipid uptake, production, or metabolism,
clear lipid vacuoles appear in the cytoplasm. As they
coalesce or enlarge, they displace the nucleus to the
periphery of the cell.
b. Fatty infiltration - is a separate process in which
lipids accumulate within stromal connective tissue cells
(most frequently seen in the heart and pancreas).
3. GLYCOGEN (glycogen infiltration) - involves an increase in
intracellular glycogen due to abnormal glucose or glycogen
metabolism. It is generally stored as clear vacuoles in the
cytoplasm (hepatocytes, however, tend to accumulate glycogen
in the nucleus).
4. PROTEIN (hyaline degeneration) -- accumulations appear as

:iEc:e
hom~,eneous, glassy, pink-staining hyaline. ; !i!rm

5.
&:3a:!fe:c:mayt\ew::~lf~~l~r~::t!:
PIGItEN'f -
t:Jh:O:::l:Jf:\\e
be of either exogenous (carbon, iron,silica,
ink, etc.) or endogenous origin. Its presence may not in
itself be injurious, but may indicate an underlying disorder.
Common endogenous pigments include:
a. Lipofusfiin (lipochrome) - is a finely granular yellow-
brown, wear and tear" pigment that signifies previous
free radical injury and lipid peroxidation of cell mem-
branes. It frequently appears with aging of the cell
(brOTm atrophy) and is deposited in the cytoplasm within
perinuclear lysosomes. It does not interfere with cell
function.
b. Kelanin - is a brown-black pigment that normally is
produced by melanocytes and, in a somewhat different
form (neuromelanin) in specific areas of the brain.
Cells containing large amounts of melanin are usually an
indication of some disorder of melanocytes.
c. Hemosiderin - is a granular gold-brown pigment derived
from the breakdown of hemoglobin and represents aggre-
,ates of ferritin micelles. Hemosiderin bearing cells
(usually macrophages) are frequently found around areas
of hemorrhage but may also be found in a variety of
parenchymal cells in systemic disorders such as
hemosiderosis and the more severe hemochromatosis.
d. Bilirubin - is a green-brown pigment that is also a
breakdown product of hemoglobin but unlike hemosiderin,
does not contain iron. It is the major pigment of bile,
and can accumulate in fluid and tissue whenever there is
a disturbance in bile metabolism or excretion.

5
B. IRREVERSmLE INJURIES - occur when there are sufficient biochemical
disturbances to induce cell death. The morphologic changes caused by
the deterioration of an irreversibly injured cell in living tissue
is termed necrosis. Necrosis results from cellular degradation by
either endogenous digestive enzymes liberated from injured cellular
lysosomes (autolysis) or digestive enzymes released from invading
leukocytes (heterolysis). There is a multi-hour time lag between the
time a cell dies biochemically and the time the morphologic changes
of necrosis appear. Morphologic changes in necrosis include:
1. this is, in part,
HCX1OGENIZATION AND EOSINOPHILIA OF THE CYTOPLAStI -
due to loss of the normal cytoplasmic basophilia and is the
result of the disaggregation of polysomes and denaturation of
cytoplasmic proteins.
2. DENSE CONDENSATION OF THE NUCLEAR CHRmATIN (pyknosis) - which then
either fragments (karyorrhexis), dissolves (karyolysis), or is
extruded from the cell. Although irreparable damage to the
cell membrane that alters its ability to control the cytoplas-
mic environment actually spells doom for the cell, these
nuclear changes are definitive morphologic evidence of
irreversible injury and cell death.
V. MORPHOLOGIC PATIERNS OF NECROSIS - the manner in which necrotic
changes progress within a tissue depends in part on the type of injury,
the tissue involved, and the environment surrounding the dying cell(s).
Identification of these different morphologic forms of necrosis can give
some clue as to the etiology of the insult.
A. COAGULATION NECROSIS - is the most common pattern of necrosis and is
generally the result of sudden cessation of oxygen supply. Although
nuclear material is lost, denaturation of the endogenous catalytic
enzymes prevents cellular digestion. This allows preservation of
"ghost-like" cellular outlines and underlying tissue architecture.
Eventually, the cells are removed by the action of exogenous
proteolytic enzymes and phagocytic "scavengers" and the tissue is
replaced by scar tissue.
B. LIQUEFACfION NECROSIS - results when proteolytic digestion of dead cells
(either by endogenous or exogenous catalytic enzymes) is not delayed
by denaturation. This is characteristic of cells injured by those
bacterial infections which attract large numbers of leukocytes
(creating an abscess) and ischemic destruction of brain tissue.
C. GANGRENOUS NECROSIS - generally refers to ischemic coagulation necrosis
complicated by secondary bacterial infection and liquefaction of the
necrotic tissue by neutrophilic enzymes. This occurs most frequently
in the lower extremities and is a common complication of
uncontrolled diabetes mellitus.
D. CASEOUS NECROSIS - is a form of necrosis in which the preservation of
the underlying tissue outlines is lost and replaced b~ a granular,
amorphous, acellular substance which has a "cheese-like r consistency
on gross examination. It is encountered principally in infectious
diseases involving mycobacteria and fungi and generally is seen in
association with a specialized form of chronic inflammation known as
granulomatous inflammation.
E• FAT NECROSIS

1. ENZYHATIC FAT NECROSIS - is seen when pancreatic lipases are


released into abdominal fatty tissues. The lipases convert
triglycerides to free fatty acids which complex with calcium
to form calcium soaps. Grossly this produces white chalky
deposits in fatty tissue.
2. TRAUMATIC FAT NECROSIS - is produced by traumatic rupture of fat
cells with subsequent phagocytosis of the lipid material by
macrophages. Seen primarily in female breast tissue, traumatic
fat necrosis histologically does not have the enzymatically
"digested" appearance seen with enzymatic fat necrosis.

6
F. FIBRINOm NECROSIS - refers to the smudgy, amorphous, eosinophilic
deposits characteristically associated with immune complex deposits
(usually in the walls of small blood vessels). The deposits consist
of immunoglobulins, fibrinogen, and complement and resemble fibrin
deposits histologically - hence the term fibrinoid (fibrin-like).
VI. CALCIFICATION

A. DYSTROPHIC CALCIFICATION - refers to the deposition of calcium salts in


necrotic tissue. Histologically, calcium may appear as small or
large amorphous basophilic deposits or concentrically laminated
spheres (psammoma bodies). Within dying cells, calcium accumulates
in the irreparably damaged mitochondria but extracellular calcium
deposits also develop utilizing membrane bound vesicles as a nidus
for propagation.
B. tlETASIATIC CALCIFICATION - refers to the deposition of calcium in normal
tissues of patients with high serum calcium levels. The histologic
appearance is similar to dystrophic calcification but the
distribution is generally more widespread.

7
PRINCIPLES OF FLUID BALANCE
a:n.d.
HEMODYNAMICS
I. EDE~A - refers to the accumulation of excess fluid in the interstitial
(extracellular and extravascular) spaces or natural body cavities. It may
be a regional process involving a localized area (ascites, hydrothorax,
effusions, etc.), or it may be a diffuse process involving all tissues of
the body (anasarca).
A. PATHOGBNESIS - The normal exchange of fluid between the plasma and
interstitial tissues is dependent on opposing forces which, under
normal circumstances, are fairly well balanced. On the arteriolar
side of the capillary bed, the hydrostatic pressure of the intra-
vascular fluid and the osmotic pressure of the interstitial fluid
(primarily reflected by its sodium content) drives intravascular
fluid into the interst:i.tial tissues. Conversely, on the venous side
of the capillary bed, the hydrostatic pressure of the interstitial
fluid in conjunct:i.on with the oncotic pressure of the plasma
proteins (primarily reflected by albumin content) draws interstitial
fluid back into the vasculature.

II
35 mmHg 15 mmHg
Hydrostatic
Pressure t t

Arteriole
II t Capillary
Bed
I t Venule

Oncotic
Pressure
!I
20 mmHg
H
25 mmHg

Fluid which is not returned to the venule is removed by lymphatics


and ultimately returned to the blood via the thoracic duct. Edema,
therefore, can result from 1) increased hydrostatic pressure of the
intravascular fluid (generally involves increased pressure on the
venous side due to right heart failure, constrictive pericarditis,
venous obstruction, etc); 2) increased osmotic pressure of inter-
stitial fluid (generally due to increased sodium retention); 3)
decreased oncotic pressure of plasma protein (generally due to a
decrease in albumin concentration); 4) increased endothelial perm-
eability (generally the result of inflammation, immunologic reac-
tion, or other tissue injury); or 5) lymphatic obstruction (lymph-
edema) .
Protein-poor transudates (specific gravity < 1.012) develop from
imbalances in the normal mechanisms of hemodynamics while protein-
rich exudates (specific gravity> 1.020) develop from endothelial
damage and alteration of vascular permeability.
B• I'IAJOR CAUSES OF EDEMA

1. LOCALIZEDEDEMA generally results from impaired venous


drainage, increased vascular permeability, or lymphatic
obstruction.
2. SYSTEMIC EDEMA

a. Heart failure - results in increased hydrostatic pres-


sures in the veins returning to the heart (both pulmon-
ary and systemic) as well as a decreased cardiac output.

8
Transudative fluid escapes from capillaries into the
interstitium and exceeds the capacity of the lymphatics
to drain the excess fluid. The resulting edema is com-
pounded by retention of sodium and water (any mechanism
which results in a decreased cardiac output or hypovol-
emia will decrease renal blood flow, activate the renin-
angiotensin-aldosterone system, and cause retention of
sodium and water).
b. Renal disease - may result in loss of plasma protein or
increased sodium retention.
c. GI disease (starvation, malabsorption, enteropathy, etc)
- may result in plasma protein deficiencies.
d. Liver disease - may result in decreased synthesis of
plasma protein, increased hydrostatic pressure and
pooling of blood in portal venous circulation, or
hepatic lymphatic obstruction.
C. depends on the severity, location, rapidity of
CLINICAL SIGNIFICANCE -
development, and underlying cause.
II. HYPEREMIA (active) - refers to increased blood flow through dilated
arteries, arterioles, and capillary beds. Clinically, this results in
increased warmth and redness in affected tissue. Basically, hyperemia is
a reflexive mechanism (neurally and/or chemically mediated) designed to
supply more blood to areas of inflammation, to tissues needing more
oxygen, or as a mechanism of heat dissipation.
III. CONGESTION (passive hyperemia) - refers to pooling of blood in veins,
venules, and capiilaries usually due to impaired venous drainage. Clini-
cally, it results in a bluish discoloration of tissue (cyanosis) due to
accumulation of reduced hemoglobin. Since impaired venous drainage also
leads to increased hydrostatic pressure, edema is a common accompaniment
of congestion.
A. ACUTE CONGESTION leads simply
to an excessively bloody organ or tissue.
B. CHRONIC CONGESTION, however,leads to impaired tissue oxygenation,
accumulation of acids, and may result in degeneration or necrosis of
the affected tissue. The effects of chronic passive congestion are
most often seen in:
1. LIDmS (due to left heart failure) - Chronically distended sep-
tal capillaries may rupture or leak blood into the alveolar
spaces. The red blood cells are phagocytized by alveolar
macrophages, the hemoglobin is broken down to hemosiderin (a
brown pigment), and these hemosiderin-laden macrophages
("heart failure' cells) in sufficient numbers will impart a
brown discoloration to the lungs. Over time, there is also
irrevers:f.ble fibrous thickening of the alveolar walls contri-
buting to pulmonary hypertension and the grossly appreciated
"brown induration".
2. LIVER (due to right heart failure, inferior vena cava or
hepatic vein obstruction) - centrilobular sinusoidal conges-
tion results in hypoxia and atrophy of centrilobular hepato-
cytes and fatty change of peripheral periportal hepatocytes.
This imparts a mottled red-brown and yellow-tan appearance
known as "nutmeg liver". Long standing chronic congestion may
induce centrilobular fibrosis known as cardiac sclerosis.
3. SPLEEN (due to portal hypertension, hepatic cirrhosis)
sinusoidal congestion and fibrosis may lead to congestive
splenomegaly.
IV. HEMORRHAGE - active bleeding into extravascular tissues or space
resulting from disruption of the integrity of vascular walls. Hemorrhage
into skin, mucous membranes, or serosal surfaces are usually referred to
as petechiae (pin-point), purpura « 1.0 cm), or ecchymoses ( > than
1.0 cm); extravascular blood clots are hematomas; blood in body cavities
are referenced to the location (hemothorax, hemopericardium, hemoperi-
toneum, hemarthrosis, etc); blood from the nose is epistaxis; coughing of

9
blood from lungs is hemoptysis; vomiting of blood is hematemesis; dark
"tarry" blood in the stool is melena; bright red blood in stool is
hematochezia. The clinical significance of hemorrhage depends on the
amount of hemorrhage, the location, and the rate at which blood is lost.
V. THROMBOSIS - Slowing and cessation of hemorrhage is accomplished by
forming an intravascular blood coagulum (thrombus) as the result of a com-
plex interaction between the vascular wall, blood cells, platelets, and
the plasma coagulation and anticoagulation factors. A blood clot, on the
other hand, refers to the formation of an extravascular blood coagulum or
a postmortem intravascular coagulum formed only from the plasma coagula-
tion factors. Although the formation of thrombi may be appropriate and
life-saving, it may also be inappropriate and life threatening.
A. PREDISPOSING FACTORS TO TIlRtl'tBUS FORtlATION - VIRCHOH r S TRIAD

1. ALTERATION OF VASCULAR ENDOTHELIUM (atheroscleros is, diabetes,


hypertension, immunologic reactions, chemical agents, bacter-
ial toxins, etc.)
2. ALTERATION IN BLOOD FLOH (stasis, turbulence)
3. ALTERATIONS IN BLOOD cmPONENTS (hypercoagulability)

B. CHARACTERISTICS

1. ARTERIAL TIlRtl'tBI - most often form in areas of atherosclerotic


damage to the endothelium or, in the heart over areas of
previous myocardial infarction, and are attached to the
underlying vessel wall (mural thrombi). Cardiac and aortic
thrombi are generally nonocclusive while thrombi in smaller
arteries may be occlusive and are most frequently seen in the
coronary, cerebral, iliac, and femoral arteries. As arterial
thrombi develop (particularly those in the heart and large
arteries), they tend to develop alternating layers of fibrin
and aggregated platelets (lines of Zahn) which grossly gives
the thrombus a grey laminated appearance (white thrombus).
2. VENOUS TIlRtl'tBI - usually form in areas of blood stasis or venous
inflammation (thrombophlebitis) or trauma. They are most
frequent in the deep leg veins and are less commonly found in
periprostatic, periovarian, and periuterine pelvic venous
plexi. Venous thrombi generally are occlusive. Since they
develop in areas of stasis, they have a greater concentration
of red blood cells (red thrombus) and are less likely to have
Lines of Zahn.
3. CAPILLARY TIlRtl'tBI - are usually due to local endothelial damage
and frequently consist of platelets and fibrin.
4. POSTMORTEM CLOTS - may occasionally be difficult to differentiate
from antemortem thrombi (especially venous thrombi) but in
general, post-mortem clots form a perfect cast of the vessel
in which they develop, do not contain lines of Zahn, are not
attached to the vessel wall, are not friable, and have a
"currant-jelly" and/or "chicken fat" appearance.
C. SEQUELAE

1. Lysosomal enzymes from polys and


TIlRa1BOLYSIS AIm/OR ORGANIZATION -
platelets will digest the coagulum and lead to softening.
Fibroblasts and capillary endothelial cells infiltrate the
thrombus from the underlying vessel wall. A combination of
fibrinolytic activity and capillary growth may reestablish
vascular flow (recanalization) through a thrombus. Eventually
fibroblastic contraction will shrink the thrombus and it may
become incorporated into the wall of the vessel.
2. CONTINUED PROPAGATION Atm GROHTII.
3. DETACHMENT AIm EMBOLIZATION.

D. depends on size, number, location, rapidity of


CLINICAL SIGNIFICANCE -
development, and availability of collateral circulation.

10
VI. INFARCTION - refers to the occurrence of localized ischemic necrosis
(generally coagulation necrosis) of an organ or tissue secondary to an
abrupt reduction in tissue oxygenation. This is usually the result of
interference of the arterial supply but in some instances may be due to
obstruction of venous drainage.
A. KmIFYING FAcroRS

1. CARDIOVASCULAR Sl'ATUS - any condition that decreases the oxygen


carrying capacity of blood or that decreases blood flow
through a tissue predisposes to infarction.
2. TISSUE SENSITIVITY TO ISCHEMIA - in general, those tissues that are
more highly specialized and/or are more metabolically active,
are most sensitive to hypoxia/anoxia.
3. RAPmITY OF OCCLUSION - slower developing occlusions are less
prone to cause infarction since collateral circulation around
the obstruction may develop.
4. ANATOMY OF THE TISSUE ARTERIAL SUPPLY

a. Single blood supply without significant anastomoses


(kidney, heart, etc) - occlusion of these vessels will
result in coagulation necrosis of the tissues supplied
by that artery. Since no blood can enter, the tissue
becomes pale (pale infarct).
b. Single blood supply with rich anastomoses (small bowel,
etc) - depending on the location of occlusion, infarc-
tion mayor may not occur. If it does, bleeding into the
necrotic tissue from neighboring anastomotic channels
may produce hemorrhage (red or hemorrhagic infarct).
c. Dual blood supply (lungs, liver) - occlusion of one
blood source mayor may not produce infarct depending on
the patency of the alternate blood supply. If infarct
does occur, it will tend to be hemorrhagic.
d. Parallel blood supply (brain) - occlusion of a central
vessel generally will not produce an infarct.
B. tIORPHOLOGY - although the color may vary according to the tissue
involved, grossly most arterial infarcts tend to be wedge-shaped
with the apex located close to the point of obstruction. Initially,
they are somewhat ill-defined but become progressively demarcated
with time.
e. CLINICAL SIGNIFICANCE - depends on location and size of infarct. A small
infarct of the myocardium may be clinically insignificant while a
small infarct of the brainstem may be fatal. On the other hand, a
large infarct of the cerebral cortex may result only in neurologiC
deficits while a large infarct of the myocardium may cause death.
VII. EMBOLISM - refers to a detached mass (solid, liquid, or gaseous) that is
carried in the vascular system to a point distant to its site of origin or
entry. The vast majority of emboli are derived from thrombi (thrombo-
emboli) but other material such as air, fat, atherosclerotic plaque, amni-
otic fluid, tumor cells, bone marrow, bacteria, foreign objects, etc. can
act as emboli if they gain access to the circulation. If the impaction of
an embolus occludes a vessel, infarction may occur. Thromboemboli, like
thrombi, may also undergo lysis or organization.
A. SYSTEHIC (ARTERIAL) EMBOLI -80-85% arise from mural thrombi in the left
ventricle or left atrial appendage of the heart, but valve vegeta-
tions, atherosclerotic plaque, etc. may also embolize. Depending on
the size of embolus and site of lodgement, it mayor may not cause
infarction. Major sites of impaction include lower extremities,
brain, kidney, spleen.
B. PUUlONARY (VENOUS) EMBOLI - are the third most common cause of sudden
death (after MI and eVA). More than 95% arise from thrombi in the
deep leg veins (popliteal, femoral, iliac), travel through the ven-
ous system, through the right heart, and into the pulmonary arter-
ies. Although rare, if a right-to-Ieft shunt is present in the heart

11
(patent foramen ovale, atrial or ventricular septal defect), a ven-
ous embolus can gain access into the systemic circulation (paradox-
ical embolus). Clinical significance depends on the size and number
of emboli as well as the general cardiovascular status of the
patient. Small emboli may be asymptomatic or, if numerous may cause
pulmonary hypertension (5%); medium size emboli may cause pulmonary
infarction or hemorrhage (10-15%); and large emboli may cause acute
right heart failure and sudden death (- 10%) if more than 50% of
the arterial flow is obstructed.
C. FAT BmOLI - may produce a clinical syndrome of progressive respir-
atory distress, mental deterioration and possible renal impairment
that generally develops 1-3 days after bone trauma. It is related to
the mechanical and chemical effects of fat in the circulation. Fat
emboli larger than 20~ are filtered in the lung while smaller aggre-
gates may pass through the lung and lodge in brain and/or kidneys.
D. AIR EtIBOLI (abortion, traumatic pneumothorax, Caisson's disease, etc.)
- small air bubbles may block microvasculature while larger amounts
(100 cc) may cause "air lock" in right heart.
VIII. SHOCK - can simply be defined as inadequate blood perfusion and the
resultant hypoxia of body tissues.
A. ETIOLOGY

1. HYPOVOLEMIC - acute loss of blood or fluid from the circuiation.


2. CARDIOGENIC - inability of heart to maintain adequate output.
3. VASCULAR

a. Neurogenic - inability to maintain peripheral vascular


muscle tone and subsequent pooling of blood (CNS trauma)
b. Septic (endotoxic) leads to peripheral vascular
pooling and is usually due to severe gram negative
bacterial infections.
c. AnQphylactic hypersensitivity reaction leads to
widespread vasodilatation and increased capillary
permeability resulting-in peripheral vascular pooling.
B. PATHOGENESIS

Reduced Blood Volume ---------+. Decreased Blood Return to Heart


+ I
I +
Pooling of blood and Decreased Cardiac Output
loss of fluid into
tissue
t
I
Arteriolar dilatation with
+
Decreased Blood Flow
increased capillary and
venule permeability
+
I +
Lactic acidosis +------------------ Decreased Oxygen to Tissues
C• CLINICAL SIGNS AND SYHP1'QI1S

1. REDUCED BLOOO VOLUME - hypotension, weak thready pulse.


2. DECREASED CARDIAC OUTPUT - compensatory tachycardia.
3. LACTIC AcmOSIS - hyperventilation, obtundation, restlessness.
4. TISSUE HYPOXIA - cyanos is.
5. RENAL RETENTION OF SALT AND WATER (compensatory attempt to increase
blood volume) - oliguria.
6. PERIPHERAL VASOCONSTRICTION (initial reflex to divert blood to
vital organs) - cool skin (hypovolemic, cardiogenic shock).
7. ARTERIOLAR DILATATION AND INCREASED VASCULAR PERHEABILITY - warm moist
skin (vascular shock).

12
PRINCIPLES OF INFLAMMATION

The inflammatory host defense reaction may be evoked by any insult or injury to
the body tissues and involves vascular, neurologic, and cellular r~sponses to a
variety of chemical mediators. Only occasionally however does the inflammation
become severe enough to be clinically evident. The inflammatory process is
designed to destroy, dilute, or contain the injurious agent and prepare the
tissue for repair, but like other self-protective mechanisms, exuberance of the
inflammatory response can itself result in significant tissue damage.
I. MAJOR CELLULAR PARTICIPANTS
A. POLYMORPHONUCLEAR LEUKOCYTES (GRANULOCYTES)

1. NEUTROPHILS or "POLYS" (50-65% of circulating white blood cells)


- can phagocytize and destroy bacteria and other minute par-
ticulate matter, elaborate chemotactic factors, and produce
digestive enzymes to degrade necrotic cellular debris. Small
cytoplasmic granules contain lysozyme, collagenase, alkaline
phosphatase, etc. while larger granules contain acid hydro-
lases, myeloperoxidase, and neutral proteases (elastase, etc).
2. EOSlNOPHILS (1-5% of circulating white blood cells) - although
their function is not completely clear, eosinophils produce
major basic protein (MBP) which is toxic to both parasites and
epithelial cells. They can phagocytize antigen-antibody com-
plexes, and they also contain histaminases which can dampen
the effects of the allergic hypersensitivity reaction.
3. BASOPHILS (1% of circulating white blood cells) - contain gran-
ules which release heparin and histamine. Basophils (as well
as neutrophils and monocytes), with the proper stimulus, can
produce and secrete platelet activating factor (PAF) which can
cause vasodilation, increased permeability of venules, and
synthesis of arachidonic acid metabolites.
B• HONONYCLEAR LEUKOCYTES

1. HONOCYTES(4-8% of circulating white blood cells) - are the


major source of wandering tissue macrophages (histiocytes).
These cells act to phagocytize large particulate matter and
are also capable of pinocytosis of soluble material. They have
surface receptors for C3b and the Fc fragment of immunoglob-
ulins and can be "activated" and "deactivated" according to
their microenvironment. They "process" antigens for the
immunologic host response and secrete numerous chemical pro-
ducts (cytokines, acid hydrolases, neutral proteases, free
radicals, growth promoting factors, etc.) that are integral to
the inflammatory and immune host defenses.
2. LYMPH~ (30-40% of circulating white blood cells) - are key
mediators in the humoral and cell-mediated immune responses
but are not phagocytic cells. They can synthesize lymphokines
that are involved in lymphocyte recruitment and proliferation.
C. PLATELETS- derived from bone marrow megakaryocytes, these enclose
electron dense granules that contain vasoactive amines, Ca++, etc;
alpha granules that contain platelet derived growth factor, coagula-
tion proteins, etc; and lysosomes that contain acid hydrolases.
D. EXIhvASCYLAR CELLS
1. PLASMA CELLS - are derived from B-Iymphocytes and do not
circulate through the blood but rather through the tissue and
lymphatic system. They produce the antibodies involved in the
humoral immune response.
2. HAST CELLS - are tissue-bound cells that secrete mediators
important in the early inflammatory response and are located
primarily around small blood vessels and serous membranes.

13
II. ACUTE INFLAMMATION - is a defensive action by the host that affords an
immediate response to tissue injury. Regardless of the injurious agent,
the acute inflammatory response consists of a defined series of events
which rapidly mobilizes host defenses to mitigate the severity of the
injury. Local manifestations of acute inflammation include calor (heat),
rubor (redness), tumor (swelling), dolor (pain), and functio laesa (loss
of function). A variety of diseases characterized by recurrent infection
are the result of defects at various points in this complex process.
A. VASCULAR RESPONSE TO IKJURY

1. VASOCONSTRICfION - often there is an immediate, but transient,


constriction of arterioles due perhaps to an adrenergic neuro-
genic response.
2. V~ILATION - in the area of injury, arterioles then rapidly
dilate and the precapillary sphincters of nonfunctioning cap-
illary beds are relaxed allowing an increased volume and rate
of blood flow (hyperemia) through the tissue. Mediated by
histamine, this results in the clinically appreciated heat and
redness of inflammation.
3. PERHEABILITY CHANGES - as hyperemia develops, the vessels become
abnormally permeable allowing the escape of plasma fluid into
the interstitium of the tissues resulting in the clinically
appreciated swelling. The fluid may also act as a dilutional
agent against toxins or antigens. Patterns of altered permea-
bility include:
a. Immediate-transient response (occurs in response to mild
injury) - An increase in permeability of yenules begins
1-10 minutes after onset of injury and lasts 15-30 min-
utes. This increase in permeability appears to be
mediated primarily by serotonin (released from plate-
lets) and histamine (released from mast cells). Venular
endothelial cells which normally are continuous and held
together by tight cellular junctions have a high concen-
tration of surface receptors for histamine. When stimu-
lated, they contract allowing fluid leakage into inter-
stitial tissue.
b. Delayed-prolonged response (occurs in response to moder-
ate injury) - The immediate-transient response is fol-
lowed by a period of low permeability but then, anywhere
from 2-10 hours later, a second increase in permeability
occurs. This second increase in permeability involves
both yenules and capillaries and is felt to be due to
direct injury to the endothelial cells.
c. Immediate-sustained response (occurs in response to
severe injury) - This involves an immediate increase in
the permeability of yenules. capillaries. and/or arteri-
01es, apparently mediated by direct physical damage to
the vessel walls. Most clinically significant injuries
result in this immediate sustained response.
B. HEtlOCONCEHIRATION
All) STASIS - as fluid escapes into the surrounding
tissue, the concentration of cellular elements remaining inside the
vessels increases. The normal laminar flow of blood is disrupted and
the red blood cells tend to clump together in the center of the
vessel while the white blood cells fall to the outer margins and
begin to line the endothelial surface, a process called margination.
C. ADHESION - as the blood flow slows down because of the loss of fluid
from the vessels, the marginating white blood cells begin to "stick"
to the endothelial lining and to each other. Activation, expression,
or induction of adhesion molecules between the endothelial cells and
leukocytes is likely the result of cytokines (interleukin-1, tissue
necrosis factor) that are released by macrophages. This results in
adherence of leukocytes to the endothelium of the vessel wall which
eventually becomes lined with leukocytes, a process called pavement-
ing.

14
D. EMIGRATION - is the process by which motile leukocytes escape from
blood vessels by squeezing through widened endothelial cell junc-
tions into the perivascular interstitial tissue. Soon after injury,
large numbers of neutrophils and monocytes escape from the yenules.
Later, another wave of white blood cells, this time principally
monocytes, escape from the venules and capillaries.
E. CHEMarAXIS - is the unidirectional migration of cells toward an
attractant, usually a chemical substance, along a concentration
gradient. Once released from the vasculature, leukocytes migrate
toward injury sites. Neutrophils have surface receptors for various
chemotactic agents such as bacterial products, CSa, arachidonic acid
metabolites, kallikrein, etc. Chemotactic products that attract
monocyte/macrophages include lymphokines, platelet-derived growth
factor, CSa, etc. Binding of these receptors ultimately leads to an
increase in intracellular Ca++ which activates contractile elements
of the cytoskeleton. As the chemotactic gradient increases, Ca++
influx persists and membrane phospholipids are converted to arachi-
donic acid metabolites. There is also degranulation of storage
vesicles and formation of free radicals within the leukocyte.
F. AGGREGATION - the types of cells that aggregate at a site of injury
depends somewhat on the causative agent of that injury. This may
reflect a sequential release of cell-specific chemotactic factors.
In general, with acute inflammation, neutrophils arrive at the site
of injury first. Later, the slower moving macrophages and lympho-
cytes arrive. In viral infections, however, lymphocytes are the
predominant cell, and in allergic hypersensitivity reactions and
parasitic infections, eosinophils may predominate.
G. PHAOOCYnSIS - is a clearing mechanism characteristic of neutrophils
and macrophages and involves three stages.
1. RECOGNITION AND ATrACHMENT - may require serum-derived opsonins
~C3b, IgG subclasses, etc) to coat the surface of the
, inductee". These opsonins are recognized and bound by
receptors on the leukocyte membrane.
2. ENGULFMENT - occurs by pseudopodial extensions of the cell cyto-
plasm which completely enclose the "inductee". Fusion of this
phagosome with one or more cytoplasmic lysosomes forms a
phagolysosome. Attachment of lysosomes to an incompletely
engulfed phagosome, however, may lead to discharge of degrada-
tive enzymes into the extracellular tissue and inadvertently
causing further tissue damage.
3. KILLING AND/OR DEGRADATION - strong antimicrobial activity is
provided by the production of free radicals and the H O~­
myeloperoxidase-halide system of neutrophils. MyeloperoxiJase
deficient phagocytes (such as macrophages) can also destroy
organisms, albeit at a slower rate, by the production of free
radicals, high concentrations of ~02' etc. Other intracellular
granules contain products which can also serve to destroy
susceptible organisms. Acid hydrolases contribute to the
ultimate degradation of bacteria and other particles within
the phagolysosome.
III. CHEMICAL MEDIATORS OF ACUTE INFLAMMATION - constitute the bridge
between injury and host inflammatory responses. There are a multitude of
mediators that act independently or by interaction with others. Mediators
can be preformed cellular products (histamine, serotonin, lysosomal
enzymes, etc), synthesized cellular products (PAF, arachidonic acid
metabolites, cytokines, etc), constituents of plasma (products of the
coagulation, complement, and kinin systems), or products of tissue injury.
Some of the more important mediators are:
A. VASOACTIVE AMINES (action is short-lived)
1. HISfAMlNE- is important in the immediate-transient increased
permeability response and in IgE-mediated hypersensitivity. It
is. present within granules of mast cells, basophils, and
platelets, and its release is triggered by numerous factors.

lS
2. SEmrrONlH(5-hydroxytryptamine) - is found primarily in plate-
lets and is released during platelet aggregation. Platelet
activating factor (PM), produced by endothelial cells and a
variety of inflammatory cells, also causes release during IgE-
mediated reactions.
B. PLASKA PRQTEASES

1. tDlPONENTS OF THE KININ SYSTEM - are produced by activation of


clotting factor XII (Hageman factor). Activated factor XII
(prekallikrein activator) converts inactive plasma prekalli-
krein into the active proteolytic enzyme, kallikrein. Kalli-
krein (which can activate additional Hageman factor) cleaves
high molecular-weight kininogen (HMWK) to produce bradykinin.
Bradykinin is a potent mediator of increased vascular permea-
bility. It also induces vascular dilation and pain.
2. tDlPONENTS OF THE tDlPLEHENT SYSTEM - C3a and C5a (anaphylatoxins)
are the primary permeability-increasing components of comple-
ment and act by releasing histamine from mast cells and plate-
lets. C5a also increases surface expression of leukocyte adhe-
sionmolecules, is chemotactic to neutrophils and monocytes,
and initiates arachidonic acid metabolism. C3b is an opsonin
and recognizes neutrophil, macrophage and eosinophil recep-
tors. C5b6789, the membrane attack complex (MAC), causes mem-
brane lysis of the cell to which it is attached.
3. tDlPONENTS OF THE COAGULATION AND FmRINOLYTIC SYSTEMS - sustain and
amplify the inflammatory response by interacting with both the
complement and kinin systems.
C. (prostaglandins and leukotrienes) - once
ARACHIDONIC ACID METABOLITES
unbound from its esterified state in the membrane phospholipids,
arachidonic acid is metabolized by either the cyclooxygenase pathway
(producing prostaglandins) or the lipoxygenase pathway (producing
leukotrienes). Important products of the cyclooxygenase pathway
include prostacyclin (PGI z ), thromboxane (~), and PGEl • Prosta-
glandins 12 and Ez act as vasodilators. In addition, prostaglandin
Ez may be responsible at least in part for some of the pain and
fever seen in inflammation. Of the lipoxygenase pathway products,
leukotrienes C, D, and Eft cause vasoconstriction and increase
vascular permeJbilfty. Leukotriene Bft is a potent chemotactic agent
for leukocytes.
IV. CLINICAL PATTERNS OF ACUTE INFLAMMAUON

A. CATARRHAL INFLAI1I'IATION -is associated with a profuse secretion of fluid


from a mucous membrane.
B. SEROUS INFLAI1I'IATION - is often the result of mild injury and consists
of the extravasation of an exudate derived from serum or mesothelial
linings.
C. FmRINOUS INFLAMtlATION- is often the result of moderate injury and
produced a fibrin-rich exudate which forms shaggy fibrin strands
that may ultimately produce adhesions.
D. SEROSANGUINOUS OR HEI'IORRHAGIC INFLAI1I'IATION - is often the result of severe
injury with vascular damage that results in the extravasation of red
blood cells.
E. SUPPURATIVE (PURULENT) INFLAMtlATION - indicates the presence of pus which
consists of tissue breakdown products, neutrophils, and in most
cases microorganisms. An abscess is a localized collection of pus
associated with liquefaction necrosis of tissue. An empyema is a
localized collection of pus in a natural anatomic cavity (usually
pleural cavity).
F. ULCERATIVE INFLAtltlATIOH - refers to a localized sloughing of the surface
of a tissue due to inflammation and necrosis of the tissue.
G. GANGRENOUS INFLAI1IIATION - implies enzymatic and bacterial decomposition
(putrefaction) of necrotic (usually ischemic coagulation necrosis)
tissue.

16
H. PRANOYS OR PSEUIKI'IEI1BRANOYS INFLAMHATION - refers to the formation of
membranes" composed of matted fibrin, mucus, and inflammatory cells
on focally necrotic epithelial surfaces.
V. CLINICAL MANIFESTATIONS OF ACUTE INFIJAMMATION

A. may include fever (effects of prostaglandin E2


SYSIB'lIC I'IANIFESTATIONS -
and endogenous pyrogens such as interleukin-1 and tissue necrosis
factor produced and released by activated macrophages), shaking
chills, weakness, muscle aching, etc.
B. LABORATORY FINDINGS - Any inflammation of clinical significance is
usually reflected by an increase in the total number of leukocytes
circulating in the peripheral blood (leukocytosis). Leukocytosis may
be accompanied by increased percentage of immature neutrophils in
the peripheral blood ("left shift"). Extreme benign elevations of
white blood cell count are referred to as leukemoid reactions. There
is also increased hepatic synthesis of "acute phase proteins" (C-
reactive protein, complement, fibrinogen, etc.) induced by the
action of interleukin-l and tissue necrosis factor. The erythrocyte
sedimentation rate increases and blood becomes hypercoagulable.
These findings, however, only indicate the probable existence of
inflammation. They do not reflect where the inflammation is
occurring or what the etiological agent might be.
VI. CHRONIC INFLAMMATION - may arise following an acute inflammatory
reaction in which the inciting agent is not destroyed or with repetitive
bouts of acute inflammation. Alternatively, chronic inflammation may begin
as a low grade, smoldering response to persistent infection by organisms
with low virulence, prolonged exposure to nondegradable but toxic sub-
stances (silica in the lung), or autoimmune reactions (rheumatoid arth-
ritis) without ever showing the classical signs of acute inflammation. In
any event, the host response is more of an immunologic response than an
acute inflammatory response. It should be noted, however, that in some
instances, acute and chronic inflammation may coexist for long periods of
time (i.e. chronic osteomyelitis).
A. - is characterized by a proliferative
NON-SPECIFIC CHRONIC INFLAI1I1ATION
(fibroblastic) response rather than the exudative response seen in
acute inflammation. Histologically, the inflammatory cells are pri-
marily mononuclear (macrophages, lymphocytes, and plasma cells)
rather than the polymorphonuclear cells that are seen in acute
inflammation. Lymphokines and other chemotactic mediators attract
mononuclear cells to the site of injury and they are interspersed
among proliferating fibroblasts and capillaries. As a result,
chronic inflammation often results in scarring with subsequent
deformity and damage to the involved tissues.
B. GRA.NUUI1ATOUS INFLAMHATION - refers to a specific type of chronic
inflammation which may occur in response to a variety of agents
(esp. mycobacteria, foreign bodies, fungi) which are indigestible or
have low antigenicity. Granulomas are the pathologic hallmark and
consist of small aggregates of modified macrophages known as
epithelioid cells. These are almost invariably surrounded by a rim
of fibroblasts and mononuclear cells, principally lymphocytes. There
mayor may not be central caseous necrosis. Giant cells (formed by
a syncytium or fusion of epithelioid cells) are often, but not
always, found within the granuloma.

17
PRINCIPLES OF WOUND HEALING AND
TISSUE REPAIR

As a continuum of the inflammatory process, wound healing is a phenomenon con-


sisting of sequential controlled steps which result in the replacement of dead
tissue with viable tissue and/or scar. In most instances there is some degree of
functional impairment of the involved tissue.
I. REPAIR BY REGENERATION

A. REGENERATIVE CAPACITY OF CELLS

1. LABILE CELLS - are cells which multiply continuously to replace


other cells that are lost through normal physiological proces-
ses (epithelial surfaces, hematopoietic cells, etc). These
cells are continuously in the cell cycle and replacement of
damaged cells is relatively rapid.
2. PERtIANEHf CELLS - are cells that do not have the ability to
regenerate and consist of striated and cardiac muscle cells
and eNS neurons.
3. STABLE CELLS - are cells which retain the ability to regenerate
but do not do so under normal circumstances (parenchymal cells
of liver, glandular organs, mesenchymal cells, endothelial
cells, etc). These cells are in the Go phase of the cell cycle
and the mechanism(s) by which these cells are stimulated to
re-enter the cell cycle and proliferate is not entirely known.
Current hypotheses include:
a. Stimulation by growth factors - growth factors are poly-
peptide proteins which attach to cell membrane receptor
sites on various types of cells and either promote DNA
replication (progression factors) or prepare the cell
for DNA replication (competence factors). Epidermal
growth factor (EGF) and transforming growth factor a
(TGFa) are progression factors which enhance epidermal
proliferation and stimulate, at least in culture, divi-
sion of fibroblasts. Platelet-derived growth factor
(PDGF) is a competence factor found in the alpha gran-
ules of platelets that stimulates migration and prolif-
eration of fibroblasts, smooth muscle cells, and mono-
cytes. Fibroblast growth factor (FGF), in addition to
the same stimulatory activity as platelet-derived growth
factor, will stimulate proliferation of endothelial
cells and promote angiogenesis.
b. Loss of contact inhibition
c. Decrease in cellular density
B. REQUIREI1ENTS FOR REGENERATION OF NORtiAL TISSUE are that there be
preservation of some original parenchymal tissue and that there be
preservation of the stromal architecture. If the stromal architec-
ture of an organ is damaged by the injury, complete restoration of
normal architecture and function is not possible.
II. REPAIR BY SCAR FORMATION

A. the repair of tissue injury generally involves, to


COLLAGEN SYNTHESIS -
some extent, the production of collagen and a scar. Approximately
eleven different forms of collagen have been identified and each has
a distinctive distribution in the various tissues of the body. Types
I, II, and III are fibrillar in structure and found in interstitial
spaces with Type I being the predominant collagen in skin, bone, and
most organs. Types IV - XI are non-fibrillar (amorphous) and in
addition to an interstitial presence are also found as a constituent
of basement membranes. The basic collagen molecule (tropocollagen)
is a coiled left-handed helix composed of three polypeptide alpha
chains which may be biochemically identical or biochemically (and

18
genetically) distinct. It is the biochemical character of the three
alpha chains which determines the collagen type.
B• TYPES OF CONNECfIVE TISSUE REPAIR

1. PRIMARYUNION (healing by first intention) - refers to the


repair process involved with surgical incisions where the
margins of the wound are closely coapted by sutures or other
methods. There is very little loss of tissue substance and a
minimal amount of inflammatory exudate and necrotic debris.
At first, the increased vascular permeability resulting from
the inflammatory response initiated by the tissue injury pro-
duces a fibrin-rich exudate (or blood clot) which fills the
space between wound margins. If on the skin, as this dehydrat-
es it forms a surface scab which, among other things, seals
the wound against invasion by microorganisms. Beneath the
surface crust, surface epithelial cells from the wound margins
begin to migrate toward the midline (also along fibrin
strands) within 24 hours of the injury, and by 48 hours a
tenuous single layered epithelium covers the surface of the
wound. Further proliferation and differentiation of the epi-
thelial cells subsequently occurs but once re-epithelializa-
tion is complete, the cells cease their proliferation possibly
due to a variety of growth inhibitors that are also elaborated
during the inflammatory and repair process.
Within the first twenty four hours, polymorphonuclear leuko-
cytes accumulate at the margins of the wound as part of the
acute inflammatory response to the tissue injury. By the
second day, fibroblasts from the healthy wound margins are
"activated' - perhaps by growth factors derived from plate-
lets, monocytes, or damaged tissue. The fibrin strands, coated
by plasma fibronectins, are chemotactic for macrophages and
fibroblasts and also act as "scaffolding" to facilitate the
influx of these cells to the area of injury. In addition to
collagen and elastic fibers, the fibroblasts secrete ground
substance (glycosaminoglycans, proteoglycans, glycoproteins)
and additional fibronectins.
Macrophages begin to replace the neutrophils around the third
day and, in addition to cleaning up cellular debris along the
wound margins, secrete factors which act in concert with the
fibronectins secreted by the fibroblasts to promote angiogen-
esis and neovascularization of the wound. Endothelial cells
proliferate and develop a lumen, but are IIleaky" and allow
protein and fluid to escape into the surrounding tissue creat-
ing edema but also supplying nutrients to the metabolically
active macrophages and fibroblasts. This combination of neo-
vascularity, II activated ll fibroblasts and mixed inflammatory
infiltrate (mostly macrophages) embedded in an edematous
ground substance is termed granulation tissue and is generally
present by the fifth day. From this point, the fibroblasts
continue to secrete collagen and other ground substances. As
the inflammatory infiltrate subsides and the collagen becomes
more abundant, the vessels are slowly crowded out to produce
a relatively acellular, avascular scar.
2. (healing by second intention) - refers to the
SECONDARY UNION
repair process involved when there is sufficient loss of
tissue to prevent coaptation of the wound margins. Generally,
there is extensive inflammatory exudate and necrotic debris
that must be removed before healing can occur.
A slow, gradual buildup of granulation tissue begins at the
margins and base of the wound and grows inward at a rate of
approximately 0.1 - 0.2 mm/-day. The exposed granulation
tissue is subject to trauma, and due to the delicate nature of
the newly formed capillaries, is prone to bleed. Migration of

19
surface epithelium can progress only so far as the underlying
granulation tissue and therefore it takes longer for the wound
to be isolated from the surrounding environment thereby
increasing the likelihood of infection.
Wound contraction is a phenomenon that occurs in secondary
union. Fibroblasts at the wound margins contain myofi1aments
(myofibrob1asts) which can contract and significantly reduce
the area of a defect that must be filled with granulation
tissue and subsequent scar. Although usually advantageous,
wound contraction can occasionally be deleterious in that it
may lead to disfiguring scars, "frozen" joints, etc.
c. ABERRATIONS OF CONHECfIVE TISSUE PROLIFERATION

1. EXUBERAHf GRANULATION (proud flesh) - refers to the excessive


buildup of granulation tissue which protrudes above the sur-
face of the wound and prevents re-epithe1ization.
2. KELOID FORHATION - refers to a buildup of excessive amounts of
collagen leading to disfiguring scars. Although the cause is
unknown, there does seem to be a genetic predisposition.
D. DEVELOPMENTOF HOUND STRENGTH - as soon as granulation tissue is
established and fibroblasts begin to secrete new collagen, the ten-
sile strength of a wound begins to increase but not in a linear
fashion. Much like the repair of bone fractures, there is resorption
and remodeling of collagen as it is deposited but the total collagen
content of a wound increases faster than the tensile strength of a
wound. Tensile strength depends as much on the type of collagen
deposited as on the amount. Early in wound healing, type III col-
lagen (which has a relatively low tensile strength) is secreted but
as the scar matures and the collagen is remodeled, Type I collagen
(which has a greater tensile strength) becomes predominant. Although
opinions vary, after approximately 100 days, seventy to ninety
percent of the original tissue strength is restored, but a scar is
never as strong as the original -tissue and, of course, is non-
functional in terms of parenchymal function.
III. FACTORS CAUSING A DELAY IN WOUND HEAIJING

A. include foreign material rema1n1ng in the wound,


EXOGENOUS FAcroRS
trauma, infection, certain drugs, movement, and radiation.
B. ENDOGENOUS FACfORS

1. INADEQUATE BLOOD SUPPLY loss of granulocytes facilitates


infection and inhibits lysosomal degradation of dead tissue.
2. INADEQUATE NUTRITION - protein deficiency interferes with proper
formation of ground substance; vitamin C deficiency interferes
with production and tensile strength of collagen; and zinc
deficiency impairs DNA and RNA synthesis.
3. EXCESS GLUCOCORTICOSTEROIDS

20
PRINCIPLES OF NEOPLASIA

Neoplasia refers to a persistent, abnormal growth of tissue which exceeds and is


uncoordinated with the growth of surrounding normal tissues. Normally, cells do
not replicate unless they are stimulated by exogenous growth factors released in
response to physiologic, pathologic, or reparative demands. Neoplastic cells,
however, are characterized by the ability to replicate in the absence of these
exogenous growth factors. Although inherited genetic influences unquestionably
play a direct role in the development of certain neoplasms, epidemiologic
evidence suggests that 75-90% of human neoplasia is due to environmental factors.
Indeed, tissues most directly exposed to the environment (skin, respiratory
tract, gastrointestinal tract, etc) tend to show higher rates of neoplastic
transformation. Evidence also suggests that neoplastic transformation is a
progressive process involving multiple steps and that in the majority of cases,
neoplastic transformation involves somatic mutations of the cellular DNA. It
appears, therefore, that neoplasia is, in essence, a "genetic" disease in that
the fundamental cellular changes occur at the level of DNA but that these changes
are induced by environmental factors.
1. NOMENCLATURE - "tumors" are denoted by the suffix "-oma". This usually
indicates a neoplastic process but occasionally may be applied to a non-
neoplastic mass (hematoma, granuloma, etc). An unmodified suffix generally
denotes a benign neoplasm while for malignant neoplasms, the suffix is
modified to either carcinoma (referring to epithelial malignancies) or
sarcoma (referring to mesenchymal/connective tissue malignancies). Notable
exceptions include, but are not limited to, the terms melanoma (malignant
neoplasm of melanocytes), lymphoma (malignant neoplasm of lymphoid
tissue), and glioma (malignant neoplasms of parenchymal cells of the CNS).
Teratoma is a neoplasm which contains cells from more than one germ cell
layer and may be benign or malignant. Hamartoma is a non-neoplastic
"tumor" that represents abnormal overgrowth or differentiation of cells
native to the tissue of origin. Prefixes generally denote the cell of
origin, microscopic architectural growth patterns, and/or gross features.
A. PREFIXES INDICATING CELL OR TISSUE OF ORIGIN - include squamous (squamous
epithelium), fibro- (fibrous connective tissue), leiomyo- (smooth
muscle), rbabdomyo- (skeletal muscle), lipo- (adipose tissue),
chondro- (cartilage) , osteo- (bone) , hemangio- (blood vessel),
lymphangio- (lymphatic vessel), etc.
B. PREFIXES INDICATING ARCHITEcruRAL GROWTH PATTERN - include adeno- (forming
glands), follicular (forming follicles), cyst (forming cystic
spaces), papillary' (forming "nipple-like I projections), villous
(forming shaggy, I finger-like" projections), etc.
C. PREFIXES INDICATING GROSS FEATURES - include scirrbous (hard), medullary
(soft, resembling marrow), colloid (gelatinous, mucinous), etc.
II. .'EATIJRES OF BENIGN ANDMAUGNANT NEOPLASMS - the distinction between
benign and malignant tumors is based on their appearance and clinical
behavior. Although there may be similarities between benign and malignant
tumors, differences generally relate to:
A. refers to the extent to which neoplastic cells
CELLULAR DIFFERENTIATION -
resemble their cell of origin. In general, benign neoplasias are
well differentiated (1. e. closely resemble their cell of origin) and
retain functional capabilities. Malignant cells, however, may range
from complete lack of differentiation (anaplasia) to well differen-
tiated. Anaplasia is characterized by cellular and nuclear
pleomorphism; increased nuclear/cytoplasmic ratio with increased"
nuclear chromatin which is frequently "clumped" along an irregular
nuclear membrane; large nucleoli; bizarre mitoses; loss of cellular
orientation; and, in general, loss of normal functional capacity.
B. RATE OF G80mB - tends to parallel the degree of differentiation of
the neoplastic cells. Benign neoplasias generally show slow growth
or' may , on occasion, spontaneously regress. Malignant neoplasms,
however, tend to grow more rapidly and rarely cease growth or
regress.

21
C. MODE OF GROHTH - benign neoplasms grow by expansion and tend to
compress the surrounding tissue into a "capsule" that separates the
tumor from normal tissue. Malignant tumors, however, grow by
infiltration and invasion of the surrounding tissue and are not
confined by a capsule.
D. METASTASIS - refers to spread of a neoplasm to points that are not
contiguous with the primary lesion. Benign tumors do not metasta-
size, but all malignant neoplasms have metastatic potential
(although they do not all do so). In general, metastases occur via:
1. LYMPHATIC DISSEMINATION - is the most common route of metastasis,
especially of epithelial neoplasms (carcinomas) and follows
the natural lymphatic drainage of the site of malignancy.
Regional lymph nodes may be enlarged due to metastatic tumor
or to immune reaction to the presence of tumor products.
2. HEMATOGENOUS DISSEMINATION - is characteristic of connective tissue
neoplasms (sarcoma). Carcinomas, however, are also spread by
a hematogenous route since there are vascular-lymphatic
anastomoses. Invasion and metastases are more likely to occur
via the venous system (as opposed to the arterial system) due
to their thin walled structure.
3. TRANSCOELOtIIC SEEDING - may occur with malignancies that involve
coelomic (peritoneal, pleural) surfaces.
4. TRAUMATIC SEEDING - excessive manipulation of a malignant tumor
may detach and carry small portions of the tumor to other
sites.
III. CLINICAL SIGNIFICANCE OF NEOPLASIA - may be related to:

A. ANATOMIC LOCATION -both benign and malignant neoplasias may be fatal


by virtue of their anatomic position. Small tumors (either benign or
malignant) may cause sudden death by interfering with vital
functions (brain stem, conduction system of heart, etc).
B. LOCAL EFFECTS include desmoplasia (excessive production of
connective tissue), compression, necrosis, hemorrhage, infarction,
ulceration, infection, etc. of surrounding tissues.
C. SYSTEMIC EFFECTS - many tumors may produce hormones or hormone -1 ike
substances that can have systemic effects (hypercalcemia, Cushing's
syndrome, SIADH secretion, etc). Other effects may relate to
migratory thrombophlebitis, systemic thromboses, embolization,
cachexia, etc.
IV. CARCINOGENIC AGENTS
A. CHEMICALS -from studies with known chemical carcinogens, it appears
that transformation of a normal cell to a neoplastic cell occurs in
two stages: initiation and promotion. Contact with either an initia-
tor or promotor alone is insufficient to produce transformation.
Additionally, the carcinogenicity of chemical agents appears to be
dose dependent so that multiple fractional doses over time have the
same transforming potential as a comparable one-time dose and dif-
ferent chemicals may act synergistically to induce cell transforma-
tion (cocarcinogenesis). Most chemical carcinogens require metabolic
activation for conversion into ultimate carcinogens (there are a few
direct acting alkylating and acylating agents), and a variety of
factors (age, sex, nutritional status, etc) may modify the carcino-
genic effect of a chemical by affecting its metabolism. Most of the
known carcinogens are metabolized by cytochrome P-4S0-dependent
mono-oxygenases.
1. - are chemical substances which can produce rapid,
INITIATORS
irreversible changes to a cell but which, by themselves, are
unable to induce neoplastic transformation. Since the change
is irreversible, it is thought that there is permanent
alteration of the DNA. Initiators tend to get converted either
directly or through a metabolic descendant into substances
that react with DNA to cause strand breaks, to alter

22
methylation, or to hinder DNA repair. For DNA changes to
become permanent and initiation to occur, however, the
carcinogen-altered cells must be able to undergo at least one
replicative cell cycle. Even so, initiated cells do not have
growth autonomy nor do they have readily identifiable unique
genotypic or phenotypic markers.
2. PROMarERS - are chemical substances that can induce neoplastic
transformation in a previously initiated cell, but cannot
cause neoplastic transformation in and of themselves. This
indicates that the cellular changes induced by promoters are
reversible and do not damage the DNA. Their action seems to
involve altered expression of genetic information in the cell.
Promoters seem to induce clonal proliferation of initiated
cells and alter their differentiation and maturation pathways.
B. ~IATION - ionizing radiation may directly ionize critical cellular
macromolecules or interact with cellular water to produce free
radicals that mediate cellular damage by breaking or altering
chemical bonds. It can inactivate enzymes, alter proteins, and cause
chromosomal breakage, trans locations , and point mutations. It can
also inhibit cell-mediated immunity. The ability of ionizing
radiation to induce neoplastic transformation appears to correlate
best its ability to induce genetic mutation within the cell.
Radiation induced mutations may serve as the initiating mechanism
and the immune inhibition as the promoter.
C. VIRUSES - although not numerous, both DNA and RNA viruses have been
found associated with human neoplasia. Through the process of
transduction and insertional mutagenesis, viruses may directly
rearrange the structure or alter the expression of the host cell
genome. In order to be transformed, however, the host cell must
survive the viral infection and be able to reproduce.
V. PROTO-ONCOGENES, ONCOGENES, AND. ANTI-ONCOGENES The specific
genetic information that was capable of transforming normal cells into
neoplastic cells was first identified in RNA tumor viruses (retroviruses)
that were known to produce cancer in animals. These genetic segments were
called viral oncogenes (v-one). It was discovered, however, that these
segments were not of viral origin but were very similar to genetic
sequences found in normal cellular DNA implying that they had at some
point in the past been incorporated into the ancestral viral genome while
the virus was replicating within an infected cell. This meant that a
normal cell must contain genetic information that could potentially trans-
form that cell into a neoplastic cell under appropriate conditions. These
naturally occurring cellular genetic segments were termed proto-oncogenes
and, surprisingly, were found in almost all life forms and coded for
almost identical protein products indicating that, under normal circum-
stances, they must play a fundamental role in normal cell physiology
(probably related to regulation of cell division and differentiation).
Since retroviruses, however, rarely cause human cancer, it was postulated
that something other than viral oncogenes could be responsible for
neoplastic transformation. Through transfection experiments, it was shown
that gen~tic material from a neoplastic cell could transform a normal cell
into a neoplastic cell. The genetic sequences capable of inducing trans-
formation were termed cellular oncogenes (c-onc) were found to be similar
in structure to normal gene sequences (proto-oncogenes) and may have
arisen through simple somatic mutation of the cell genome. Some were
similar to viral oncogenes but others did not seem to have viral
counterparts,.
A. PROTO-ONCOGENEFUNCTION cell division frequently involves the
interaction of a messenger (growth factor, etc) with cell membrane
receptors, transduction of that signal through the cell membrane to
a second messenger, transmission by the second messenger to the
nucleus, and initiation of DNA transcription and replication. This
complex biochemical pathway is in part mediated by the protein
products encoded by cellular proto-oncogenes. Classes of proto-
oncogenes functioning in this regard include the protein tyrosine
kinases (src, abl, erbB, etc); the GTP-binding proteins (ras

23
family); growth factor proteins (sis, int-2, etc); and DNA-binding
proteins (myc family, fos, etc). Proto-oncogenes, therefore, have
the. potential of being converted to oncogenes (through mutation,
retroviral transduction, increased expression, etc) that can promote
excessive or inappropriate cell proliferation.
B. ONCOGENE ACTIVATION - like all other genes, each proto-oncogene is
composed of a regulatory and a structural region. Changes in either
region could produce an active oncogene. Structural mutations could
lead to synthesis of a protein that has aberrant structure and
function whereas regulatory changes could lead to inappropriately
high levels of a growth-inducing protein.
1. STRUCTURAL CHANGES -are often due to alteration of a single base
pair (point mutation) that alters the protein product of the
proto-oncogene. Point mutations probably occur randomly
throughout the proto-oncogenes, but only those occurring at
certain critical "hot spots" produce oncogenic activation.
Point mutations are best exemplified by the ras oncogenes. 10-
15% of oncogenes isolated from human tumors have been shown to
be "hot spot" ras mutants. In addition to point mutations,
insertions or deletions of sequences of base pairs is also
possible. Point mutations may not be identifiable on karyotype
but, if large enough, insertions or deletions may be seen.
2. REGULATORY CHANGES - with regulatory changes, mutations affect
the amount of protein product rather than the structure or
function. This can occur through chromosomal translocation or
gene amplification.
a. Chromosomal trans locations (myc, abl) - proto-oncogenes
may become activated either because they are placed next
to strong promoter/enhancer sequences or because trans-
location removes them from the influence of normal
regulatory control sequences. Translocation can also
affect the biochemical functions of proto-oncogenes by
fusion with new genetic sequences. The abl oncogene in
chronic myelogenous leukemia (Philadelphia chromosome)
resul ts from a trans location that encodes a protein
different from the normal protein. Major gene rearrange-
ments such as trans locations can be observed in
karyotypic studies.
b. Gene amplification - produces many copies of an oncogene
and therefore increases the amount of protein products.
In karyotypic studies, gene amplification can be
observed as homogeneously staining regions (RSH) i f they
stay associated with the chromosome or as double minutes
(DN) if they break loose and replicate as extrachromo-
somal material.
C. EFFECTSOF ONCOGENE ACTIVATION - the protein products of oncogenes are
involved in the regulatory pathways that control cellular division
and differentiation and are capable of transforming normal cells
into neoplastic ones. They may impart growth autonomy by deregulat-
ing genes that encode growth factors (c-sis) or by coding for large
amounts of growth-promoting factors to which the cell can respond
(autocrine stimulation). They may encode for defective receptors (v-
erbB) that transmits stimulatory signals in the absence of a growth
factor, or through amplification and overexpression of growth factor
receptor genes (c-neu) , they may render tumor cells excessively
sensitive to low levels of growth factors that are below the thresh-
old for stimulating normal cells. They may also encode proteins that
enter the nucleus and directly stimulate cellular growth. The ras
family of oncogenes appear to influence cell shape, growth factor
dependence, and motility. They can also induce cells to secrete a
number of growth-stimulating factors. The myc oncogene seems to make
cells more responsive to growth factors controlling cell prolifera-
tion and appears to confer immortality to the cell line.

24
D. althou*h it is felt that most neoplasms arise from
AHrI-ONCQGENES -
multiple "spontaneous somatic mutations of cellular DNA, it is well
established that the development of certain tumors (retinoblastoma,
Wilms' tumor, etc) are in some instances related to heritable
factors. Since activated oncogenes are not transmitted in the germ
line, this implies that some form of cancer susceptibility genes
must exist which, in and of themselves, do not produce neoplasia but
that could affect the regulation of cell growth, the effectiveness
of the immune surveillance mechanisms, the ability to repair DNA
damage, or the manner in which potentially carcinogenic compounds
are metabolized. Since the expression of these genes (particularly
the growth suppressor genes) serve to protect the cell from the
events leading to neoplastic transformation, they are referred to as
anti-oncogenes. A heterozygous allele for one or more of these anti-
oncogenes, carried through the germ line, could be a genetic
predisposing risk factor. Loss or inactivation of the corresponding
locus (loss of heterozygosity) by somatic mutation may then result
in neoplastic development. Since it is much easier to destroy or
inactivate a gene than it is to amplify or potentiate gene
expression, it is conceivable that these anti-oncogenes may play an
even more important role in neoplastic transformation than the
oncogenes.
VI. PAlHOGENESISOFNEOPLASTICTRANSFORMATION - as previously mentioned,
it is generally accepted that neoplastic transformation is a multistep
process involving multiple genetic alterations. DNA transfection experi-
ments have shown that no single oncogene will transform normal cells into
neoplastic cells. To become neoplastic, cells must acquire a variety of
traits that are not present in their normal state. It is not surprising,
therefore, that more than one oncogene may be involved, with each oncogene
supplying some of the functions required to convert a normal cell into a
neoplastic one. Sequential activation of oncogenes (or suppression of
anti-oncogenes) may be reflected in the gradual transition of normal cells
to neoplastic cells through stages of dysplasia or preneoplasia. Since
cell replication is central to neoplastic transformation, it is also not
surprising that preexisting regenerative and/or hyperplastic conditions
are occasionally associated with subsequent neoplasia.
VII. TUMOR GROWTH - the propensity of tumor cel1s to reproduce is a
distinguishing feature of neoplasia and may be the result of genetic
alterations (oncogene activation, anti-oncogene suppression, etc) that
direct the cell to replicate rather than differentiate further.
A. IN VITRO - in tissue culture, neoplastic cells appear to be freed
from normal regulatory controls and have an increased rate of stem
cell renewal. There is reduced cohesiveness between cells and their
growth is no longer inhibited by the presence of neighboring cells
(loss of contact inhibition). Unlike normal cells, they do not
require attachment to a hard surface to proliferate (anchorage
independent growth). They require fewer, if any, exogenous growth
factors and in general are immortal (i.e. cell lines can be kept
alive indefinitely). They can develop invasive properties (probably
related to cytoskeletal changes or production of enzymes induced by
oncogene activation). When injected into other animals, these cells
will produce neoplasms (transplantibility).
B. IN VIVO - evidence suggests that most tumors are of monoclonal origin
i. e. a single cell from normal or preneoplastic tissue becomes
neoplastic at a specific level of differentiation and the clonal
derivatives of that cell produces the neoplasm. In most tumors there
appears to an increase in the proportion of stem cells undergoing
replication and a corresponding decrease in the proportion
progressing to full maturation. Tumor cells, howev·er, do not appear
to divide more rapidly than normal cells.

25
1. GROHTH FRACTION (GF) - refers to the proportion of cells within
a tumor population that are in the proliferating pool. The
rate of tumor growth depends upon the growth fraction and the
degree of imbalance between cell production and cell loss.
Most tumors have a low GF and proliferation only slightly
exceeds cell loss. The growth fraction of a tumor has a
profound effect on its susceptibility to chemotherapy. Cancer
drugs act primarily on dividing cells, therefore those tumors
which have a high growth fraction are most vulnerable. Slow
growing tumors which have a high proportion of cells outside
the cycle respond less favorably and harbor cells which can
potentially reenter the cycle at a later time.
2. ANGIOGENESIS - tumor growth is dependent upon vascularization.
Without it, neoplastic growth will stop at approximately one
millimeter diameter due to limitation of diffusing metabo-
lites. Angiogenic factors which include fibrinogen and various
substances produced by the tumor cells (angiogenin, fibroblast
growth factor, etc) promote and control the neovascularization
of the expanding cell mass. The neovasculature, however, tends
to be abnormal due to loose endothelial junctions. Hemorrhage
into tumors is not· uncommon and, in the case of malignant
tumors, this may even help assist the tumor cells to spread to
other sites. Hypoxia in the tumor environment also
significantly impairs the biologic effect of radiation therapy
and some forms of chemotherapy which are oxygen dependent.
Hormones can also influence the growth rate of hormonally
responsive tissues.
3. DOUBLING TIME - as tumor size increas es, the time it takes for
a tumor to double in volume also increases (i.e. growth rate
slows down) due to diminished blood supply, competition for
metabolites, and other factors that lead to a decrease in the
growth fraction and an increase in cell loss. As tumors
enlarge, an increasing proportion of tumor cells drop out of
the mitotic cycle either because of necrosis or by entering
into prolonged Gl periods or the GO phase of the cell cycle.
Even though the doubling time slows down as tumors enlarge, by
the time a solid tumor is clinically detected, it has already
completed a major portion of its life cycle. The earlier a
tumor is identified, the greater the chance of successful
radiation and/or chemotherapy.
VIII. CI.ONAL EVOLUTION AND HETEROGENEITY - although tumors probably arise
monoclonally, by the time they have reached clinical detection, the cell
population is heterogeneous in respect to morphologic appearance,
karyotype, degree of differentiation, invasiveness, metastatic
capabilities, etc implying that tumors can undergo clonal evolution as
they develop, probably related to DNA instability and the high rate of
random mutation in neoplastic cells. Dedifferentiated cells are more
likely to have characteristics that enable them to spread (i.e. increased
motility, decreased adhesiveness, decreased anchorage dependence, etc).
Clonal evolution, like the progression of preneoplasia to neoplasia,
appears to involve a sequence of oncogene activations that, over time,
leads to more highly malignant ce11s - those that have a survival
advantage due to their growth rate, invasiveness, drug resistance, etc. By
the time most malignant neoplasms have become clinically detectable, it is
likely that they have evolved several subclones with metastatic potential.
This implies that the earlier a cancer can be identified, the less likely
it is that more aggressive subclones have developed.
IX. MECHANISMS OF TUMOR INVASION AND METASTASIS - an important feature
of malignant ce11s is their ability to invade surrounding tissue and
ultimately to spread to distant sites. Only certain cells within a tumor
develop a high invasive and/or metastatic potential and these tend to be
cell clones with a high mutation rate. There is emerging evidence that
oncogene activation is involved in conferring metastatic potential on
tUlllor cells by enabling the ce11 to attach to, degrade, and penetrate
basement membranes and interstitial connective tissue.

26
A. tumor cells appear to
BASEMENT MEMBRANE DISSOLlIfION AND STROtIAL INVASION -
bind to structural glycoproteins such as the laminin and fibronec-
tins which are embedded in basement membranes and connective tissue.
Since laminin adheres tightly to the type IV collagen of the base-
ment membrane, neoplastic cells may evolve mechanisms to secrete
laminin or, alternatively, to produce tumor membrane receptors that
binds native laminin. Once secured to the basement membrane, tumor
cells then secrete proteases, such as type IV collagenase, to dis-
solve the basement membrane. They are then free to attach to stromal
fibronectin with additional release of proteases. The ability of
tumor cells to digest ground substance (collagen, glycoproteins,
proteoglycans, and elastin) by the production of proteases is
correlated with invasive potential. It is also likely that cancer
cells interact with host fibroblasts and other mesenchymal cells and
stimulate them to secrete additional collagenases. As previously
mentioned, tumor derived growth factors as well as breakdown
products of the connective tissue ground substance have angiogenic
and tumor chemotactic activities which promote neovascularization
and recruitment of additional tumor cells. The loose endothelial
junctions of these new vessels make it relatively easy for tumor
cell to gain access to the circulation.
B. LYtIPHATIC/VASCULAR INVASION AND DISSEMINATION - invasive properties also
allow tumor cells to penetrate existing vessels. Since lymphatic
channels lack a basement membrane, they are easily invaded while the
walls of venules and veins put up little resistance as well.
Hemorrhage and necrosis also promote the release of tumor cells into
the circulation. Once in the circulation, however, tumor cells are
vulnerable to hemodynamic factors that cause cell trauma, to
immunologic recognition and attack (particularly by natural killer
cells), and to other adverse conditions. Fewer than one in a
thousand tumors cells released into the blood stream will ultimately
metastasize. In the circulation, tumor cells tend to aggregate in
clumps. Clumps of five to ten tumor cells appear more effective in
producing metastases than either single cells or larger clumps. Some
clumps adhere to platelets or produce a procoagulant which may form
"shields" of precipitated fibrin to ward off immunologic
recognition. These procoagulants may partly explain some of the
hypercoagulable states seen in patients with disseminated cancer.
C. SITES OF METASTASIS - regional spread of tuniors is primarily influenced
by the anatomy of regional lymphatics. Since the capillary and
lymphatic systems are interconnected, however, hematogenous spread
can be expected in the face of demonstrated lymph node metastases.
The distribution of distant metastasis, however, is not indiscrim-
inate. Metastases from a variety of malignant tumors display
preferences for some tissue sites over others. This may reflect a
given tumors' ability to adhere to tissue specific endothelial cells
and invade tissue specific stroma. Metastatic deposits, like primary
tumors, are clonal in origin and the survival and growth of the
metastatic lesion generally depends on the same factors that enable
growth of the primary lesion.
x. TUMOR ANTIGENS

A. TUMOR SPECIFIC ANTIGENS - are those found on neoplastic cells and not on
normal cells. In animals, a tumor induced by a carcinogen generally
carries its own unique tumor antigen while all tumors produced by a
given virus share a common antigen. These tumor-specific antigens
can activate host immunologic destruction of cancer cells in animals
but whether this occurs in humans to any significant extent is not
known.
B. TUMOR ASSOCIATED ANTIGENS - are those found in normal cells but are in
higher concentration in tumor cells. Examples of tumor associated
antigen are differentiation antigens (beta HCG) and oncofetal
antigens which are expressed during embryonic development but
normally repressed during adult life (carcinoembryonic antigen).

27
XI. GRADING AND STAGING OF MAUGNANT NEOPLASMS

A. GRADIHG - is a pathologic determination based on the degree of


differentiation of the neoplastic cells and the number of mitoses.
Generally graded as I (low grade, well differentiated) through IV
(high grade, undifferentiated). Many malignant neoplasms progress to
a higher grade over time as less differentiated clones of cells
become dominant. In general, the lower the grade, the better the
prognosis.
B. STAGING - is a clinical and pathologic determination based on the
size of the neoplasm, the presence of or absence of regional lymph
node involvement and the presence or absence of distant metastases.
This is the basis of the TNM (tumor size, node involvement,
metastasis) staging system. In general, the smaller the tumor and
the more localized it is, the better the prognosis.
XII. LABORATORY METHODS OF DIAGNOSING CANCER - standard methods of
diagnosis of neoplasia include light microscopy and electron microscopy of
tissue biopsies. However, newer techniques (gene amplification by the
polymerase chain reaction with DNA probes, monoclonal antibodies, flow
cytometry, etc) make it possible to detect a variety of activated
oncogenes in biopsy specimens and allow much greater precision in
diagnosis with even smaller amounts of tissue.

28
CARDZOVASCULAR SYSTEM

CONGESTIVE HEART FAILURE


Congestive heart failure refers to a clinical condition manifested by numerous
signs and symptoms that arise when the heart is no longer able to supply enough
blood to the body tissues to maintain normal metabolic activity. The end result
of a variety of disorders, heart failure is generally related to loss of
myocardial contractility, increased resistance to cardiac outflow, inadequate
ventricular filling, or increased ventricular blood volume. Compensatory
mechanisms may involve increased heart rate, i.ncreased contractility of muscle
fibers, hypertrophy of muscle fibers, and/or dilatation of ventricles. These
compensatory mechanisms, however, constitute an additional burden on the already
compromised heart, and ultimately, the heart will decompensate. It is important
to remember that clinical manifestations of heart failure do not develop until
there is failure of the compensatory mechanisms and that the signs and symptoms
of heart failure are generally due to hypoxic and congestive effects on organs
and tissues other than the heart itself.
I. LEFT-SIDED HEART FAILURE - occurs when the left ventricle is unable to
maintain adequate cardiac o~tput and is generally due to ischemic heart
disease, systemic hypertension, or valvular disease.
A. PADIDGENESIS/IIDRPIIOLOGX -The inability to maintain adequate cardiac
output results in gross hypertrophy and/or dilatation of the left
ventricle and ultimately increased hydrostatic pressure in the
pulmonary venous circulation. Acutely, transudative fluid escapes
the septal capillaries and when the capacity of the interstitial
lymphatics to drain the excess fluid is surpassed, the transudate
subsequently seeps into the alveolar spaces producing pulmonary
edema. Additionally, renal hypoperfusion resulting from the
decreased cardiac output, causes sodium and water to be retained via
activation of the renin-angiotensin ~ystem and the action of
aldosterone on the renal tubules. This added fluid burden
intensifies the pulmonary edema and pleural effusions may develop.
Chronically, there are small capillary hemorrhages and leakage of
red cells into the alveolar spaces. Red cells and hemosiderin are
phagocytized bX alveolar macrophages to form the hemosiderin-laden
macrophages (beart failure" cells) characteristic of chronic
pulmonary congestion. Over time, there is irreversible fibrous
thickening of the alveolar walls leading to the grossly appreciated
"brown induration" of the lungs.
B. CLINICAL MANIFESTATIONS - although a decreased cardiac output may cause
neurologic symptoms (restlessness, irritability) due to cerebral
hypoxia, the clinical manifestations of left heart failure are
primarily pulmonary in origin and include easy fatigability,
sbortness of breatb (SOB) or dyspnea on exertion (DOE), paroxysmal
nocturnal dyspnea (PND), ortbopnea, and cougb.
II. RIGIIT-SIDED HEART FADJURE - occurs when the right side of the heart is
unable to overcome an increase in pulmonary arterial pressures. Pulmonary
arterial bypertension may be the result of interference with pulmonary
venous outflow, chronically increased pulmonary blood flow, a decrease of
the cross-sectional area of the pulmonary vascular bed, pulmonary
parenchymal disease, or interference with respiratory mechanics.
A. COR ~ - refers to right ventricular failure due to pulmonary
arterial hypertension resulting from primary parenchymal or vascular
disease of the lungs. This is usually reflected structurally by
hypertrophy and/or dilatation of the right ventricle.
1. A~ COR ~ (a major cause of sudden death) - refers to
sudden right ventricular failure usually resulting from large
pul~onary emboli. In addition to physical obstruction of the

29
right ventricular outflow, other factors (bronchoconstriction,
hypoxemia, neural reflexes, etc.) contribute to the clinical
manifestations of wheezing, dyspnea, tachypnea.
2. CHRONIC COR ~LE - generally arises in the setting of primary
pulmonary hypertension (an unusual progressive disease of
unknown etiology which generally appears in young women) or,
more commonly, chronic pulmonary parenchymal disease.
B. CLINICAL tlAHIfESTATIQNS

1. ELEVATED SYSTEtIIC VENOUS PRESSURE - resul ts in engorgement and


distention of neck veins, passive congestion of the liver
(chronic passive congestion gives a "nutmeg" appearance to the
cut surface of the liver and, in severe cases, results in
hepatic cardiac sclerosis), dependent pitting edema, and
increased body weight.
2. ELEVATED PORTAL VENOUS PRESSURE results in congestive
splenomegaly and ascites.

CONGENITAL HEART DISORDERS

I. EMBRYOGENESIS - the development of the cardiovascular system is


mandatory in early embryogenesis in order to supply oxygen and essential
nutrients .to, and remove waste products from, the rapidly developing
tissues. During gestation, when oxygenated blood is being supplied via the
umbilical veins, maintenance of a "right-to-Ieft" shunt is imperative.
After birth however, these shunts should be closed in order to establish
the normal adult circulatory blood flow. Abnormalities in the development
of the heart and great vessels occur in approximately 1% of infants.
Environmental as well as genetic factors are contributory influences.
Congenital heart defects may result in:
A. RIGHI-TO-LEfT SHUNTS (truncus arterio.sus, transposition of the great
vessels tetralogy of Fallot, tricuspid atresia) - these shunts
divert blood containing reduced hemoglobin away from the pulmonary
circuit and into the systemic circulation. In addition to reducing
the oxygen saturation of the arterial blood, this also results in
cyanosis (a bluish discoloration of the skin seen most readily
around the lips and nailbeds) at or soon after birth. Chronic
cyanotic conditions may be reflected by polycythemia and digital
clubbing.
B. LEfT-TO-RIGHT SHUNTS (atrial septal defect, ventricular septal defect,
patent ductus arteriosus) - these shunts divert oxygenated blood
from the systemic circulation into the pulmonary circulation,
depriving the systemic tissues of oxygen. The additional volume
demands on the right side of the heart and the pulmonary circulation
result in smooth muscle hypertrophy of the pUlmonary arterioles and
increased pulmonary circulatory resistance (pulmonary hypertension) .
The increased pulmonary resistance is transmitted back to the right
ventricle which responds by hypertrophy. Eventually, right sided
pressures may exceed left sided pressures and a "reversal" of the
shunt to a right-to-Ieft shunt occurs (Eisenmenger's syndrome)
favoring the late development of cyanosis (cyanose tardive).
C. OBSTRUCTIVE AHCIIALIES (aortic coarctation, valvular stenosis or atresia)
- generally do not cause cyanosis.
II. DEVELOPMENTAL SEQUENCF..8

A. OF RIGHI All) LEfT ATRIoyENDtICULAR (A-V) CANALS - the partitioning


DmLOPllENJ'
of the original tubular heart . into a common atrium and a common
ventricle is effected by the inward growth of tissue (endocardial
cushions) from the anterior and posterior aspects of the tube. The
endocardial cushions eventually meet and fuse leaving a
communication on either side between the common atrium and common
ventricle. These communications (the right and left A-V canals) will

30
eventually become the tricuspid and mitral valve. If the endocardial
cushions do not fuse, a common A-V canal results. If the endocardial
cushions are displaced to one side, there will be a structural
narrowing and functional stenosis of one valve in addition to the
possible structural enlargement and functional incompetence of the
other.
B. FORMATION OF THE INTERATRIAL SEPTUtI (lAS) - the septum primum is a thin,
crescent shaped membrane which grows downward from the dorsocranial
portion of the atrium. The opening between the leading edge of this
membrane and the endocardial cushion is the ostium primum, which
allows a right-to-left shunt. As the septum primum grows downward to
eventually fuse with the endocardial cushions, the ostium primum
grows gradually smaller and a new right-to-left shunt (ostium
secundum) is formed by the degeneration of the superior portion of
the septum primum. At this time, a second membrane (septum secundum)
develops to the right of the septum primum and grows downward to
also fuse with the endocardial cushions. This septum does not
develop completely but leaves an opening (foramen ovale) in its
center. In the fetus, blood entering the right atrium can then flow
through the foramen ovale and ostium secundum directly into the left
atrium. After birth, when the left atrial pressures become greater
than the right atrial pressures, the lower portion of the septum
primum acts as a flap valve and seals off the foramen ovale to
completely separate the right and left atria. Any abnormalities in
this complex developmental sequence may lead to the formation of an
atrial septal defect (ASDJ. These occur more frequently in females
and morphologically can be categorized as an ostium primum defect
(5%), an ostium secundum defect (90%), or sinus venosus (5%). Early,
these may be clinically asymptomatic although there may be a
pulmonic murmur dug to increased flow. With time, they may lead to
right atrial hypertrophy, right ventricular hypertrophy, and
pulmonary hypertension.
C. DEVELOPHEHI' OF AORTA AND PULMONARY ARTERY - the aorta and pulmonary artery
both arise from a common channel, the truncus arteriosus. Ridges
develop from either side of the truncus and fuse to form an
aorticopulmonary septum. As it develops, it spirals as it approaches
the endocardial cushions in order to direct blood from the left
ventricle into the aorta and blood from the right ventricle into the
pulmonary artery. If the septum fails to develop, there is a single
artery leaving the heart - a persistent truncus arteriosus. If the
septum develops but does not spiral, a transposition of the great
vessels occurs where blood from the left ventricle flows into the
pulmonary artery and blood from the right ventricle flows into the
aorta. Transposition is a form of congenital cyanotic heart disease
and is a major cause of congestive heart failure and death in early
infancy.
D. FORMATION OF THE INTERVENI'RICULAR SEPTUtI - the interventricular septum is
formed by two components. One is a muscular septum which begins at
the apex of the heart and grows upward to fuse with the second
component, or membranous septum, which consists of the endocardial
cushions and proximal portion of the spiral (aorticopulmonary)
septum. Any disturbance of the proper fusion of any or all of these
elements may lead to a ventricular septal defect (VSD), the most
common congenital anomaly of the heart. Most VSDs involve the
membranous septum (90%). Small defects « 0.5 cm.) may be
asymptomatic and may spontaneously close. In moderate sized defects,
the left-to-right shunt will produce pulmonary hypertension and
right ventricular hypertrophy. Ultimately there will be reversal to
a right-to-left shunt with cyanosis, digital clubbing, and
polycythemia. Small and moderate size defects predispose patients to
the development of infective endocarditis. Large defects may cause
problems from birth culminating rapidly in cardiac failure and
death.
III. BICUSPID AORTIC VA I,VE - is a relatively common defect and is generally
asymptomatic a,lthough it does predispose to infective endocarditis and
calcification.

31
IV. PATENT DUCTUS ARTERIOSUS (PDA) - functional closure of the ductus
arteriosus normally occurs within 24 hours after birth. A high incidence
of patent ductus arteriosus in premature infants and those with
respiratory distress syndrome suggest that increasing 02 tension may be the
trigger that initiates closing. The incidence of PDA is greater in females
and clinically causes a "machine murmur" and possible systolic thrill. The
left-to-right shunt results in pulmonary hypertenSion and right
ventricular hypertrophy. Subsequent right-to-Ieft shunts cause cyanosis
(usually of lower extremities due to bypassing aortic arch branches).
V. COARCTATION (stenosis of the aorta) - coarctation is relatively common,
more frequent in males, and often accompanied by other developmental
abnormalities.
A. POSTDUCfAL (ADULT FORM) - is the more common form. Most patients are
asymptomatic until adulthood. Upper extremities exhibit increased
blood pressure but lower extremities show decreased blood pressure
and diminished peripheral pulses predisposing to intermittent
claudication of the calf muscles. Substantial collaterals (internal
mammaries,intercostals) may develop and be picked up on physical
examination or x-ray (notching of lower rib margins). Eventually,
left ventricular hypertrophy and congestive heart failure will
intervene and there is an increased risk of hypertensive CVA and
dissection or rupture of the aorta proximal to the coarctation.
B. PREDUCfAL (INFANTILE FORtI) - these infants manifest evidence of heart
failure at or soon after birth. The ductus is usually patent and the
prognosis depends on ability of PDA to supply blood to the
postcoarctation aorta. The lower body tends to be cyanotic and the
upper body pink. The hypoplastic left heart syndrome may be an
exaggerated form in which there is stenosis of the proximal aorta,
atresia or stenosis of the left heart valves, and hypoplasia of the
left ventricle.
VI. TETRALOGY OF FALI~OT - is characterized by 1) right ventricular outflow
obstruction, 2) right ventricular hypertrophy, 3) ventricular septal
defect, and 4) dextroposition (overriding) of aorta. The incidence is
slightly greater in males and the clinical symptoms depend on degree of
outflow obstruction. Blood is shunted to aorta via the VSD and cyanosis
usually develops within short time after birth. Murmurs occur secondary to
the outflow obstruction, and the clinical manifestations includes dyspnea,
growth retardation, digital clubbing, etc.

ATHEROSCLEROSIS
Atherosclerosis is a slow progressive disease of the large elastic and large and
medium sized muscular arteries characterized by the formation of intimal
fibrofatty atherosclerotic plaques. It is one of, and by far the most common of,
a group of vascular disorders termed arteriosclerosis which is characterized by
thickening and loss of elasticity of arterial walls. Other disorders in this
group include arteriolosclerosis (hyaline thickening and/or hyperplastic
proliferation of the walls of small arteries and arterioles) and l1onckeberg's
medial calcific sclerosis (medial calcification of medium and small muscular
arteries) .
I. RISK FACTORS the etiology of atherosclerosis is not completely
understood, however there are definite risk factors associated with the
development of clinically significant disease. These risk factors tend to
have a cumulative rather than additive effect, but even some individuals
with no history of risk factors also develop significant disease. Major
risk factors include hyperlipidemia, hypertension, cigarette smoking, and
diabetes mellitus. Minor risk factors are physical inactivity, stress and
behavior patterns, and obesity. Other non-alterable factors include age,
sex, genetic predisposition, etc.

32
II. ROLE OF HYPERLIPIDEMIA
A. P~ LIPms are primarily derived from exogenous dietary
cholesterol and triglycerides that are absorbed across the
intestinal mucosa, endogenous cholesterol and triglycerides that are
synthesized in the liver, and free fatty acids derived from adipose
tissue. Cholesterol and triglycerides are insoluble in blood and
are, therefore, complexed with a variety of specific proteins
(apoprote.in) and polar lipids (phospholipids) to form soluble
lipoproteins. The major apoproteins are A (AI, All), B (B48, BlOO),
C (CI, CII, CIII), and E. It is the specific apoprotein composition
of the lipoprotein that directs the ultimate metabolic fate of its
associated lipid.
B• LIPOPROTEIN ELECTROPHORESIS PAmRN

Beta
Lipoprotein (LOL)
Pre-Beta
(-) Lipoprotein (VLOL) (+)

Alpha
Chylomicron Lipoprotein
(HDL)

1. CHYLOtlICRONS(apo E, A, CII, B48), are normally found in the


blood only after eating and primarily transport dietary
triglycerides and, to a lesser extent, dietary cholesterol.
2. BETA LIPOPROTEINS (apo B100), also known as "low dens.ity
l.ipoprote.in" (LDL), primarily transports endogenous
cholesterol and is the major plasma cholesterol carrier.
3. PRE-BETA LIPOPROTEINS ~apo E, C, B100), also known as "very low
dens.ity l.ipoprotein' (VLDL), are manufactured by the liver and
primarily transport endogenously produced hepatic
triglycerides to the peripheral adipose and muscle tissue.
4. ALPHA LIPOPROTEINS (apo A, E, C), also known as "high dens.ity
l.ipoprotein" (HDL) , primarily transports endogenous
cholesterol acquired from extrahepatic peripheral tissues and
returns it to the liver.
C. among other metabolic
REGULATION OF PLASMA AND CELLULAR CHOLESTEROL LEVELS -
functions, cholesterol is vital for the synthesis of cellular mem-
branes, steroid hormones, and bile acids. 93% of the body cholester-
ol is therefore intracellularly located to provide substrate for
metabolic functions while 7% is circulating as plasma cholesterol.
1. PLAStIA TRANSPORT - chylomicrons and VLDL are transported to
peripheral adipose and muscle tissue where they are' stripped"
of triglycerides by the lipoprotein lipase (LPL) of capillary
endothelial cells.
a. Chylomicron "remnants", now relatively cholesterol rich,
are returned to the liver where the dietary derived
cholesterol can be used for bile acid synthesis,
excreted as free cholesterol, or be used for other
metabolic functions.
b. YLDL "remnants", also known as intermed.iate dens.ity
lipoproteins (IDL) (apo BIOO, E) may be cleared by LDL
receptors on the hepatocytes that recognize the apo BIOO
and apo E subunits, or they may be directly converted to
LDL by alteration of the apoprotein on the lipoprotein
coat (removal of apo E).
33
2. PLAStIA CLEARANCE OF LDL

a. Receptor pathway (major clearance mechanism)


Hepatocytes, as well as other cell types, have specific
cell surface receptors for apo B100 and apo E to bind
LOL. The LOL is cleared from the plasma by incorporation
into the ce11 and subsequent hydrolysis by lysosomal
enzymes into amino acids and free cholesterol. The free
cholesterol will then be used for membrane synthesis or
other metabolic functions, regulate further uptake of
plasma cholesterol by decreasing the number of cell
surface LOL receptors, and inhibit endogenous cellular
production of cholesterol. From hepatocytes, excess
cholesterol can be excreted into the bile. In
extrahepatic cells, however, excess free cholesterol is
re-esterified and stored, but there is a limited storage
capacity. HDL may act as an acceptor of cellular
cholesterol, transfer it to LOL via the IDL
intermediary, and clear i t through the liver thereby
preventing excessive accumulations within the
extrahepatic cells.
b. Phagocytic (receptor-independent) pathway - cells of the
monocyte/phagocyte/reticuloendothelial system have
receptors for "modified" LOL and normally clears
approximately one third of the plasma LOL but will
increase in activity as the plasma LOL levels increase.
Unlike the cells of the receptor pathway, however, these
cells do not have the regulatory mechanisms to control
the accumulation of excessive intracellular cholestero,l.
High levels of plasma LOL from whatever cause may
therefore lead to the development of subcutaneous
xanthomas.
D. RISK ASSESSMENT

1. PLAStIA CHOLESTEROL- the incidence of coronary heart disease


increases when the plasma cholesterol levels exceed 200 mg/dl.
There is a linear relationship between the serum cholesterol
concentration and the amount of intimal surface covered by
raised atherosclerotic plaque. Once 60% of the intimal surface
is involved, the risk of coronary heart disease is greatly
increased. The higher the plasma cholesterol levels,
therefore, the younger the age when coronary heart disease
becomes symptomatic.
2. LDL/HDL RATIO - elevated levels of plasma LDL-C and to a lesser
extent VLOL are directly related to the development of
clinically significant atherosclerosis while elevated levels
of HDL-C seem to have a protective effect. Therefore, the
ratio of LDL-C/HDL-C can give a rough estimate of the risk of
coronary heart disease. Favorable ratios are 4.5:1 or lower.
E. Certain types of familial hyperlipidemias occur as
DIETARY CONTROL -
a result of genetic defects in the control of cholesterol synthesis.
Genetic defects in the production of specific apoproteins have also
been identified. The vast majority of cases of hypercholesterolemia,
however, are due to dietary excess or a combination of dietary and
genetic factors. The major contributors to hypercholesterolemia are
excesses in caloric intake, dietary cholesterol, and saturated fatty
acids.
III. PATHOGENESIS - Although some feel that the primary dysfunction relates
to the "neoplastic" proliferation of smooth muscle cells in the intima,
many others feel that the atherogenic process is a response to vascular
endothelial damage. Injury to the endothelium results in an increased
permeability of the vessel to plasma constituents at the site of injury.
Platelets adhere to the areas of injury, aggregate, and release a growth
stimulating substance (platelet derived growth factor) which stimulates
migration of smooth muscle cells from the media to the intima where they
34
undergo monoclonal proliferation. The smooth muscle cells elaborate the
extracellular components of the plaques. LDL and VLDL enter the intima
from the serum. Some are precipitated as extracellular lipid while others
attach to surface receptors of smooth muscle cells, blood borne
monocyte/macrophages, and fixed macrophages. These become incorporated as
intracellular lipids and with uncontrolled intracytoplasmic accumulation
eventually cause cell death and rupture. Transient injury may lead to
resolution as smooth muscle cell proliferation ceases when the endothelial
damage is repaired. Chronic injury, however, may lead to the formation of
non-reversible atherosclerotic plaques.
IV. MORPHOLOGY
A. FAm STREAKS - are probably reversible lesions and mayor may not
represent precursor lesions of adult plaques. Grossly these appear
as multiple, essentially flat yellow streaks on the intimal surface.
Microscopically, within the intima, there are elongated smooth
muscle cells and ovoid macrop,hages both of which contain
intracytoplasmic lipid droplets ('foam cells") and are embedded in
variable amounts of extracellular lipid, collagen, and elastic
fibers.
B. ATHEROSCLEROTIC PLAQUES - appear as multifocal asymmetric elevations of
the intimal surface. The firmness of a plaque varies from hard
(fibrous plaques which contain a large amount of collagen) to soft
(atheromatous plaques which contain abundant lipid material), but
most have a firm fibrous cap overlying a softer core of atheromatous
and necrotic debris. They tend to be most severe in the abdominal
aorta, particularly in areas of vascular branching. Microscopically,
within the intima, a typical plaque will have a fibrous, collagenous
cap overlying a necrotic central area containing free cholesterol,
cholesterol clefts, fibrin, plasma protein, and cellular debris. As
plaques increase in size, a neovascularization develops which allows
access of additional plasma lipids to the pla~ue as well as serves
as a potential source of hemorrhage. "Routine' plaques ·may evolve
into clinically significant "complicated" plaques when there is
thrombus formation, ulceration of the fibrous cap, hemorrhage into
the plaque, dystrophic calcification, or weakening of the media.
V. CLINICAL SIGNIFICANCE - although atherosclerosis can occur in any of the
arteries, clinical symptoms are generally related to the hemodynamic
effects on the heart, brain, kidney, small bowel, and lower extremities.

ISCHEMIC HEART DISEASE (IHD)/CORONARY HEART DISEASE (CHD)


HID results from an imbalance between the availability of oxygen and the
metabolic demand of the heart. Clinically, this imbalance is manifested as angina
pectoris or, when excessive, as myocardial infarct.
I. FACTORS AFFECTING OXYGENATION OF mE MYOCARDIUM

A. REDUCED CORONARY FLOW

1. ATIlEROSCLEROTIC NARROWINGof the subepicardial coronary trunks is


responsible for >90% of the cases. The presence of plaques
predisposes to the development of platelet microthrombi. In
addition to physical obstruction of the lumen, platelets
release vasoactive substances (serotonin, thromboxane Az ,
etc.) that cause vasoconstriction. Platelet aggregation also
predisposes to further, possibly occlusive, thrombus
formation. Soft atheromatous plaques are prone to ulceration
or hemorrhage into the plaque resulting in sudden expansion in
size and/or rupture.
2. HEMODYNAMIC ALTERATIONS - normally, the perfusion of the coronary
arteries is dependent on the pressure differential between the
coronary ostia and the coronary sinus. In systole, there is
compression of intramural arteries and increased resistance to
blood flow so that coronary blood flow is maximal during

35
diastole and depends on aortic diastolic pressure. During
diastole, any condition which decreases pressure at the
coronary ostia (hypotension, aortic regurgitation, etc.) or
increases pressure at the coronary sinus (tricuspid
regurgitation, right heart failure, etc.) will reduce coronary
flow.
3. SHALL VESSEL DISEASE of the intramural vessels (SLE,
polyarteritis, radiation, diabetes, emboli, etc).
B. INCREASED HETABOLIC DEIWI) - conditions which result in tachycardia
and/or hypermetabolic states (infection, exercise, pregnancy,
hyperthyroidism, etc.) increase metabolic demands and decrease the
time of diastole. Myocardial hypertrophy (hypertension, valvular
disease) increases metabolic demand as well as the compressive
forces on the intramural vessels.
c. DECREASED SATURATED IIEtfOGL(BIN AVAILABILITY - anemia, pulmonary disease,
right-to-left shunts, carboxyhemoglobin, etc.
II. ACUTE ISCHEMIC .HEART DISEASE

A. _INA PECTORIS - is a clinical syndrome resulting from transient


ischemic myocardial injury and manifested by paroxysmal attacks of
substernal or precordial chest discomfort which may extend into the
neck, left jaw, left shoulder, and left arm.
1. STABLE ANGINA (most common) - refers to exertional chest
discomfort relieved by rest and/or nitroglycerin. It may also
be precipitated by emotional stress, cold weather, and heavy
meals. These patients usually have severe atherosclerotic
coronary disease which prevents an increase in coronary blood
flow as the metabolic demand of the myocardium is increased.
2. VARIANT (PRDlDtETAL) ANGINA - refers to paroxysmal chest discomfort
AT REST due to vasospasm of coronary vessels which mayor may
not also have significant atherosclerotic disease. The
ischemia, therefore, result~ from decreased coronary flow
rather than increased myocardial demand.
3. UNSTABLE ANGINA (most ominous) - refers to prolonged discomfort,
discomfort at rest in a patient with previous stable angina,
or more severe discomfort than usual on exertion. Most
patients have severe atherosclerotic coronary disease with
possible superimposed vasospasm or platelet microthrombi.
Any of the forms of angina pectoris may show patchy myocardial
necrosis or fibrosis, atrophic myofibers, subendocardial
myocytolysis, or old scars indicative of previous myocardial
infarcts.
B. ACUTE IlXOCARDIAL· INFARCTION - is due to irreversible myocardial damage
resul ting from inadequate oxygenation of myocardial fibers. The
majority of cases show underlying severe coronary artery disease and
(in most cases) injury is due to occluded or markedly reduced blood
flow rather than increased metabolic demand. Although the clinical
symptoms are often dependant on the site of the infarct and its
severity, typically there is a severe persistent chest RAin
unrelieved by rest or nitroglycerin and accompanied by nausea,
vomiting, and diaphoresis.
1. TRANStlURAL
INFARCT involves the full thickness of the
ventricular wall and is greater than 2.5 cm in diameter. In
most cases, there is thrombosis over a complicated plaque in
a vessel supplying the area of myocardial damage. The vast
majority affect the left ventricle (with or without
involvement of the interventricular septum). The size is
dependent on the severity and duration of ischemia and the
degree of collateral circulation to the ischemic area.

36
a. Hicroscopic appearance - the initial changes to the
irreversibly injured myocardial cells (nuclear chromatin
condensation, cytoplasmic eosinophilia) begin around 6-
12 hours after the event. Paradoxically, if the region
is reperfused, the influx of Ca" and free radicals may
increase tissue damage (reperfusion injury). Dying
myofibers may develop contraction bands due to
hypercontraction of myofibrils. Nuclear pyknosis,
blurring of the cross striations, myocytolysis, and an
increasing inflammatory infiltrate develop over the next
several days. Neutrophils begin to subside after about
5-7 days and, by 10-14 days, prominent granulation
tissue is present at the margins of the infarct.
Progressive collagenization occurs over the next several
months.
b. Gross appearance - no gross abnormality appears until
12-24 hrs when the infarcted area begins to show a grey-
brown pallor which progresses to a soft, yellow-brown
region of necrosis with an irregular hyperemic border.
Maximum softening is present at 7-10 days. Granulation
tissue appears at the edges of the infarct within 10-14
days and the lesion becomes grey-white in color as
scarring develops.
c. Complications
(1) Cardiac arrhythmias (90%) - are the most frequent
complications within the first week post-infarct.
(2) Left ventricular dysfunction (> 50%) - leading to
congestive failure or cardiogenic shock.
(3) Hural thrombosis and embolization (15-20%).
(4) Hyocardial rupture « 5%) generally occur
around 10-14 days post-MI and may involve the
left ventricular wall (leading to a rapidly
developing cardiac tamponade or a slower
developing pseudoaneurysm), the interventricular
septum (leading to 1eft-to-right shunting), or
the papillary muscles (leading to mitral valve
dysfunction) .
(5) Ventricular aneurysm (10-15%)
2. due to increased contractility, increased
SUBENDOCARDIAL INFARCT -
metabolic demand, and decreased vascular tone, the
subendocardial myofibers are more sensitive to perfusion
deficits. These are less common than transmural infarct and
often result from myocardial hypoperfusion secondary to severe
coronary atherosclerosis. Patients who otherwise have marginal
blood perfusion of the subendocardial area, however, might
precipitate an infarction with any process that would decrease
coronary blood flow or increase metabolic demand. Myocardial
necrosis may be patchy or circumferential. Arrhythmias or
mural thrombi may develop but rupture or aneurysm formation is
rare.
C. the highest predictive value
LABORATORY ASSESSHENT OF MXOCARDIAL INFARCT -
for the diagnosis of myocardial infarction is obtained with serial
studies of isoenzymes of CPK and LDH performed at the onset of chest
pain (for baseline comparative levels) followed by measurements at
6-13 hours and 24-36 hours after the chest pain.
1. CPK - The three isoenzymes of CPK are dimeric in structure,
consisting of dimers of peptide chains "M" (found primarily in
muscle) and "B" (found primarily in the brain). Accordingly
these isoenzymes are called CPK...,. CPK,.,b' and CPKbb or CPKp CP~
or CPK1 respectively. In the normal population, CP~ is absent
in the serum. However, at 6 to 13 hours after a mild to
moderate myocardial infarct, CP~ is detectable in the serum.
This appearance of CP~ is not surprising since 40 percent of
the total CPK in heart tissue is CP~. The remaining CPK

37
activity in heart tissue is CPK3 • With infarction, both the
CP~ and CPK3 isoenzymes are elevated. CPK is cleared rapidly
however and after 24 hours , CP~ has returned to normal
levels.
2. LDH - Isoenzymes of LOH consist of tetrameres of two peptide
chains. "H" chains are found primarily in heart muscle; "M"
chains are found primarily in skeletal muscle. These two
distinct peptide chains are then linked together into
tetrameres giving rise to five distinct LOH isoenzymes. In the
normal population, the serum levels of LO~ (35% of total LOH)
is greater than LOH I (25% of total LDH). However, in heart
tissue LDHI is 40% of the total LDH while LO~ is 35%. With
injury to myocardial cells a proportionately greater amount of
LDH, is released and there is reversal of the serum LDHl/LO~
ratio (a LDH,/LD~ "flip"). It is important to note that this
LOH "flip" occurs in 90 percent of patients with myocardial
infarction only if the blood sample is collected 24 hours
after the onset of chest pain.
III. SUDDEN CARDIAC DEAlH « 1 hour after onset of symptoms) - is usually
associated with fatal arrhythmias precipitated by an acute ischemic event.
Most patients have severe coronary disease and 50% have evidence of old
infarcts but only rarely acute infarcts. Sudden cardiac death, however,
can occur without clinical or morphologic evidence of coronary disease.
IV. CHRONIC ISCHEMIC HEART DISEASE (40% of deaths from IHD) - asymptomatic,
slow, progressive atherosclerotic coronary disease may become manifested
by the insidious onset of congestive heart failure as the cardiac reserve
is slowly depleted. Most patients, however, have a past history of angina
or myocardial infarction, and the heart failure may follow a precipitating
illness such as pneumonia. Diffuse myofiber atrophy is associated with
interstitial fibrosis and spotty loss of myocytes (myocytolysis).

RHEUMATIC FEVER AND RHEUMATIC HEART DISEASE


Rheumatic fever is a systemic immunologically mediated disease clinically
characterized by one or more of the following Jones criteria: migratory
polyarthritis, carditis, erythema marginatum, subcutaneous nodules, and
Sydenham's chorea. Acute attacks of occur a few days to several weeks after (with
rare exceptions) pharyngeal infections by Group A beta-hemolytic streptococci and
are more common in children than adults. The onset may be abrupt with fever,
tachycardia, and painful swollen joints or it may be insidious with malaise and
low fever. After the initial episode, recurrent attacks are likely with
subsequent Streptococcal pharyngitis and usually show similar manifestations as
were apparent with the initial attack. Evidence indicates symptoms are due to
immunologic cross reactivity between streptococcal antigens and host tissue
antigens - particularly cardiac myofiber/ smooth muscle antigens. Although
patients with acute rheumatic fever may have elevated serum titers to
streptococcal antigens (ASO, antistreptokinase, etc.), there is no single
confirmatory diagnostic laboratory test available.

A. HEART rheumatic fever produces a pancarditis which is more


frequently seen in children than in adults. Adults, however, are
more prone to recurrent attacks.
1. may cause pericardial friction rub in the
FIBRINOUS PERICARDITIS -
acute phase but generally resolves without sequelae.
2. MYOCARDITIS - although rare, it may be severe enough to cause
death during the acute illness. Foci of fibrinoid necrosis
develop in the myocardial interstitium (usually around
vessels) and subendocardium accompanied at first by a scanty
mononuclear infiltrate followed by the appearance of plump
modified macrophages (Anitschkow myocytes) some of which may
be multinucleated (Aschoff giant cells). These proliferative
Aschoff bodies are pathognomonic for rheumatic heart disease.

38
Progressive fibrosis of the Aschoff bodies occurs as the acute
attack subsides and focal thickening of the left atrial
endocardium (McCallum's plaques) may appear.
3. ENDOCARDITIS - is the most consistent and the most crippling
aspect of rheumatic heart disease and tends to involve the
valves of the left side of the heart. Acutely, small fibrin
verrucae overlie foci of fibrinoid necrosis that develop along
the closure lines of the valves and occasionally on chordae.
As the acute attack subsides, valves heal with fibrous
scarring and thickened, shortened, blunted leaflets having
varying degrees of commissural fusion. The mitral valve shows
a characteristic "fish-mouth" deformity as well as shortening,
thickening, and fusion of the chordae. In addition to
commissural fusion, nodular calcifications may form behind the
aortic valve cusps leading to variable degrees of stenosis and
insufficiency.
B. changes in the joints, skin, and blood vessels include
OTHER ORGANS -
foci of fibrinoid necrosis with varying degrees of inflammatory
infiltrates and often resemble the Aschoff bodies found in the
heart.
II. COMPLICATIONS - include cardiac failure (secondary to valvular disease),
thrombosis/embolization; and infective endocarditis.

VALVULAR HEART DISEASE


Stenosis implies failure of a valve to open properly thereby creating obstruction
to the forward flow of blood. Acquired stenosis is almost always due to a primary
abnormality of the leaflets or cusps. Insufficiency implies an inability of a
valve to close properly and thereby allows for the backward flow of blood. It can
be due to intrinsic valve disease or damage to the valve's supporting structures
(chordae tendineae, papillary muscles, skeletal annulus, etc.). Stenosis and
insufficiency may also be coexistent. -
I. CALCIFIC AORTIC STENOSIS - develops when calcific excrescences build up
in the Sinus of Valsalva and encroach on the cusps with subsequent
stenosis and resultant left ventricular hypertrophy. Unlike rheumatic
valves, there is little or no commissural fusion. The frequency of
clinically significant disease increases with advancing age but it may
also be seen in younger patients with congenitally abnormal valve (usually
bicuspid valve). Symptoms generally are referable to left heart failure or
inadequate cardiac output.
II. MITRAI__ ANNUI-,US CALCIFICA1l0N - Nodular calcification of the mitral
annulus also occurs with increasing age but generally does not
significantly alter valve function.
III. MITRAI-, VAI-,VE PROLAPSE - is a common condition (especially young women)
which may in some cases be congenital in origin. In some cases, it may be
related to a metabolic defect in connective tissue metabolism. It is
characterized by enlarged mitral leaflets (especially posterior leaflet)
and/or elongated chordae which allows for ballooning of leaflets during
systole and may produce a midsystolic click corresponding to the snapping
of an everted leaflet. There may also be some degree of insufficiency with
late systolic murmur. Eventually, the leaflets and chordae thicken. The
valve may ultimately resemble a rheumatic valve except that there is no
commissural fusion. The majority of patients are asymptomatic but death
can result from infective endocarditis, chronic congestive failure,
chordal rupture, or arrhythmia.
IV. NONBACTERIAI. THROMBOTIC (MARANTIC) ENDOCARDITIS - most frequently
seen in patients dying of chronic debilitating disease, it also occurs in
hypercoagulable states or with valve trauma. Small platelet-fibrin thrombi
develop along lines of valve closure and may, on occasion, engender
systemic-arterial emboli.

39
V. INFECTIVE ENDOCARDITIS - is a potentially lethal condition characterized
by the formation of friable septic vegetations on heart valves or
endocardial surfaces. The disease may affect either normal hearts (25-50%)
or those with preexisting abnormalities (50-70%), involve highly virulent
or relatively innocuous organisms, have abrupt onset with fulminant course
and rapid death or insidious onset with protracted course and resolution.
The causative organisms are predominantly bacterial (95%) but with
immunosuppression, increasing IV drug usage, and immunodeficiency
diseases, more exotic organisms are becoming involved.
A. AC1ll'E EIDJCARDITIS (ABE)

1. ABE is usually caused by virulent and aggressive


PATHOGENESIS -
organisms (staph. aureus) which directly damage the cardiac
valves and induce septic thrombus formation. It is frequently
seen in I.V. drug users or chronic alcoholics whose hearts
usually do not have underlying abnormality. Some patients have
preexisting pyogenic infections.
2. MORPHOLOGY - large, bulky, friable vegetations develop on the
valve surfaces and are prone to perforate the underlying
valve, erode into the myocardium, or fragment and embolize.
3. CLINICAL - characteristically, there is an abrupt onset of high
fever, shaking chills, and profound weakness with skin and
mucosal hemorrhage (which may have infective centers from
which organism can be cultured). Embolic phenomena occur early
and involve the brain, spleen, myocardium, and kidneys. The
resultant septic infarcts frequently develop into abscesses.
Septic arthritis and small painless nodular hemorrhages on the
palms or soles (Janeway lesions) may develop. If heart murmurs
are present, they have a tendency to change in character and
intensity as portions of vegetations break off and alter the
hemodynamic flow. Cardiac decompensation occurs early as
valves are eroded or chordae rupture. Aortic ring abscesses
may cause arrhythmia. There is a high mortality rate and death
is usually due to CHF, fatal embolism, arrhythmia, rupture of
mycotic aneurysm, or uncontrolled sepsis.
B. SUBACUTE ENDOCARDITIS (SBE) - (duration of more than six weeks)
1. PATHOGENESIS - characteristically, SBE involves hearts that have
underlying deformities that alter normal hemodynamic blood
flow patterns. Regurgitation and/or jet streams promote the
formation of sterile platelet-fibrin deposits. These deposits
may then be seeded by blood borne organisms. Seeding may be
promoted by agglutinating antibodies which produce clumps of
organisms that tend to precipitate out in areas of "low
pressure sink" (along the free margins of the atrial surface
of mitral valve and ventricular surface of aortic valve).
Because of these deranged hemodynamic factors, low virulence
organisms (often indigenous bacteria) have potential for
implanting on valve or endocardial surfaces. Strep. yiridans,
normally found in the gingival sulci, is the predominant
organism.
2. MORPHOLOGY - valvular vegetations form along the free margins of
the valves (predominantly left-sided) as friable, irregular
masses but tend to be less bulky than in ABE. They may undergo
progressive fibrosis, organization, and calcification; and
extend to adjacent endocardium or chordae; but are less likely
burrow into ventricular wall.
3. CLINICAL - usually there is insidious onset with progressive
weakness, anorexia and weight loss, anemia with skin pallor,
fever, and occasional night sweats. Initial manifestation may
also be due to embolic event to brain, heart, spleen, or
kidneys. Other manifestations may include linear subungual
splinter hemorrhages; skin or mucosal petechial hemorrhages;
digital clubbing; or tender, purplish subcutaneous nodules in
palms, soles, or fingertips (Osler's nodes) which probably due
represent immune complex mediated vasculitis. On physical
40
exam, cardiac murmur is usually present but is mostly a
function of the underlying valvular disorder and infrequently
changes in character. Mortality rates are much lower that with
ABE. When death occurs, it is usually due to CIIF, embolic
events, arrhythmias, and/or renal failure.
C. LABORATORY EVALUATION - in ABE, the leukocyte count is usually increased
but with SBE the leukocyte count is often normal. A normochromic,
normocytic anemia is generally present and rheumatoid factor may be
found in up to 50% of patients but will disappear with treatment.
Circulating immune complexes may be present and
hypergammaglobulinemia can generally be demonstrated. Diagnosis
ultimately rests with identification of the organism in blood.
Cultures (when taken at regular intervals) will be positive in 90%.

MYOCARDIAL HEART DISEASE


I. MVOCARDmS - refers to any inflammatory condition that involves the
myocardium. It can ranges from fulminant disease with abrupt onset and
acute cardiac failure to asymptomatic disease. It may be due to
microbiologic agents, hypersensitivity reactions, radiation, or unknown
causes. Viral myocarditis (Coxsackie, ECHO, polio, and influenza are most
common) comprises more than 50% of the cases and is most frequently seen
in infants, pregnant women, and immunosuppressed patients.
Morphologically, there is four chamber dilatation and the myocardium tends
to be flabby. Acute infections show interstitial edema with an
inflammatory infiltrate characteristic of the inciting cause. The
inflammation usually resolves in 6-8 weeks but may progress to chronic
disease associated with myocardial fibrosis. When symptomatic, the
clinical manifestations include tachycardia, AV conduction blocks, low
grade fever, dyspnea and malaise. .
II. HYPERTENSIVE HEART DISEASE - After IHD, HHD is the most common cause
of cardiac failure and death. Systemic hypert~nsion increases resistance
to left ventricular outflow and creates a pressure overload on the left
ventricle. The heart responds to a pressure overload by concentric
hypertrophy of the left ventricle which thickens the wall, increases heart
weight (without significant increase in size), and decreases the left
ventricular volume. In the absence of any other abnormality that might
produce left ventricular hlPertrQphy (i.e. valve disease), this is the
identifying hallmark of h'rertension. As decompensation occurs, however,
the ventricle dilates increasing heart size) and obscures the
hypertrophic changes. Microscopic changes consist mainly of increased
myofiber diameter with blunted, prominent, "box-csr" nuclei. Hypertrophied
myofibers demand more oxygen but hypertension accelerates atherosclerosis
of extramural coronary arteries and arteriolosclerosis of intramural
arterioles, both of which decrease 0 availability to the myocardium.
Eventually focal atrophy and degeneration of myocytes occur and a diffuse
myocardial fibrosis develops. Except for the symptoms associated with
severe hypertension (tinnitus, epistaxis, dizziness, etc.), hypertension
is asymptomatic. As cardiac decompensation intervenes, however, there is
an insidious onset of left heart failure. Untreated hypertension will
result in death due to heart failure, stroke, renovascular disease and/or
vascular complications.
III. CARDIOMVOPAmV - refers to non-inflammatory disorders of the
myocardium. The clinical and pathologic patterns generally fall into one
of three categories.
A. may simply represent a common end
DILATED (CONGESTIVE) CAJ!DIClIIXOPADlY -
point of a variety of previously undiagnosed cardiac diseases and
can occur at any age. Some are of known cause (alcoholic, familial,
peripartum, nutritional, and post-infectious cardiomyopathies), but
the large majority must be classified as idiopathic. These are
characterized by dilatation and hypertrophy of all chambers of the
heart with impairment of ventricular contraction, low ejection
fractions, high end-systolic volumes, and congestive heart failure.
There is always. increased heart weight due to ventricular

41
hypertrophy, but the hypertrophy may be grossly obscured by the
ventricular dilatation. The myocardium shows diffuse interstitial
fibrosis without evidence of inflammation or severe coronary
atherosclerosis. Due to poor contractility, mural thrombi are prone
to develop (most frequently in the left ventricle). Patients
generally present with signs and symptoms of congestive heart
failure. Progressive heart failure usually culminates in death
unless patients first succumb to arrhythmias or the effects of
systemic emboli.
B. HYPERTROPHIC ((BS'fBUCTIVE) CARDmtYOPA'DIY (ASH. IHSS) - in most instances,
this represents an inherited condition which is characterized by
hypertrophied, hypercontracting myocardium which mayor may not
cause symptoms of decreased left ventricular outflow. Prominent
systolic murmur is due to the LV outflow obstruction and mitral
insufficiency. Morphology is characterized by dilated atria,
disprcmortional ~ertrcmhy of IVS with myofiber disarray, decreased
ventricular volume, mitral valve thickening, and endocardial
thickening of LV outflow tract. If there is also thickening of the
ventricular wall behind the posterior leaflet of the mitral valve,
obstructive symptoms may be present due to abnormal motion of the
anterior mitral valve leaflet. This also allows mitral regurgitation
which further compromises outflow from a ventricular chamber already
decreased in volume. Clinical course is variable with some patients
developing progressive heart failure complicated by embolization
from atrial thrombi or infective endocarditis. Ironically, in those
patients with obstructive symptoms, as the heart begins to
decompensate and dilate, the obstruction is relieved and symptoms
improve.
,C. RESTRICTIVE/INFILTRATIVE CARPI(It'{OPATIlIES are rare conditions
characterized by restriction of ventricular filling. Although the
pathologic process is different, the clinical signs and symptoms are
essentially the same as dilated cardiomyopathy. High venous filling
pressures lead to right and left heart failure.
1. AtIYLOmOSIS, SARCOmosIS, HEKOCHRCIIATOSIS, All) 5mIE IHBORN ERRORS OF
HETMKLUM result in diffuse infiltration of the myocardium by
abnomal substances thereby restricting normal myocardial
compliance and elevating the ventricular filling pressures.
2. EJIDCItYOCARDIAL FIBROSIS, ENDOCARDIAL FIBROELASTOSIS, All) LOEFFLER'S
ENDOCARDITIS -although of separate etiologies, these have in
common a thickening and fibrosis of the
endocardial/subendocardial tissues and also restrict
comp 1iance.

PERICARDIAL HEART DISEASE


The normal pericardium consists of a thin, delicate membrane (visceral
pericardium) adherent to the epicardial fat and a thicker fibroelastic membrane
(parietal pericardium) that is adherent to surrounding mediastinal structures.
Between'the two membranes is a potential space which normally contains less than
50 ml. of clear yellow fluid. The visceral and parietal pericardium are in
continuity at the base of the heart and the proximal portions of the great
vessels lie within the pericardia 1 space. Normally, the pericardia 1 pressures are
equal to the right ventricular diastolic pressure.
I. ACUTE PERICARDmS
A. ~ is usually due to an early bacterial, viral, or
immunologically mediated inflammation of the epicardium and
pericardium resulting in a serous effusion into pericardia 1 space.
Volume generally is less than 200 mI., contains few inflammatory
cells, and is usually reabsorbed without sequelae when the inciting
cause is removed.

42
B. FJBRDINS'(most common) - is usually non-infectious in origin and is
seen after a myocardial infarct, after trauma to the pericardium,
and in various systemic disorders. May be clinically manifested by
malaise and fever associated with a variable degree of chest pain.
Produces a loud pericardial friction rub unless serous fluid
intervenes and separates parietal and visceral pericardium. Usually
resolves without sequelae but asymptomatic delicate fibrinous
adhesions between the visceral and parietal pericardium may persist.
C. ~ - is seen most frequently in young males (10-40 years old)
and associated with infectious agents (primarily bacteria and
fungi). A thick creamy exudate up to 500 mI. in volume accumulates
within the pericardial cavity. A friction rub is less prominent than
in fibrinous pericarditis. This often organizes to produce chronic
adhesive or constrictive pericarditis.
D. HBlHmHASlC - connotes fibrinous or suppurative effusion admixed with
blood. Most frequently due to TB or neoplastic involvement of
pericardium.
II. CLINICAL MANIFESTATIONS OF PERICARDIAL FLUID ACCUMULATION
A. ~ (CUOIAC TAN~E) - due to the relative noncompliance of the
normal parietal pericardium, a rapid accumulation of even a modest
volume (150-200 mI.) of fluid in the pericardial space can lead to
equalization of the pericardial pressure and the left and right
ventricular diastolic pressure. This will result in decreased
filling of the ventricles during diastole and therefore decreased
cardiac output during systole. This can be clinically expressed as
jugular vein distension and sl.stemic hypotension. On auscultation,
the heart sounds may sound 'distant" due to the muffling of the
heart sounds by the fluid surrounding the heart.
B. atBONIC - since the parietal pericardium, over time, can stretch,
chronic accumulations of fluid can reach much greater volumes before
clinical manifestations become apparent. When symptoms do occur,
however, they tend to be similar to the symptoms of acute tamponade
but have a more gradual onset. -
III. CHRONIC (HEALED) PERICARDmS
A. ADHESIVE PERICARDITIS - is often secondary to healed suppurative
pericarditis or to radiation or cardiac surgery. The pericardia1 sac
is obliterated with adherence of parietal pericardium to surrounding
thoracic and mediastinal structures. This inhibits cardiac
contraction resulting in increased workload, hypertrophy, and
dilatation of heart.
B. CONSIRICfIVE PERICARDITIS - often consists of idiopathic fibrosis or
fibrocalcific scarring of the pericardium which inhibits cardiac
filling during diastole and results in decreased cardiac output.
Although the signs and symptoms are similar to adhesive
pericarditis, hypertrophy and dilatation cannot occur.

CARDIAC NEOPLASMS
Primary tumors of the heart are exceedingly rare and most often (80%) benign.
Myxomas are the most frequent and are most likely to arise in the left atrium in
the region of the foramen ovale. They may be sessile or pedunculated and
depending on their size may cause valve obstruction or atrial filling deficits.
Rhabdomyomas are most common in infants and children and often associated with
tuberous sclerosis. Angiosarcoma is the most common of the malignant tumors, but
it, too, is rare. Metastatic disease occurs but also is uncommon.

43
VASCULAR DISEASE
I. ANEURYSMS

A. ATHEROSCLEROTIC -are the most commonly encountered aortic aneurysms


and generally occur in middle-aged to elderly males. The vast
majority occur in the distal abdominal aorta and lie between the
renal arteries and the bifurcation into the common iliacs. They
develop slowly over time and do not produce clinical symptoms until
they become large, give off thromboembo1i, or rupture (the risk of
rupture increasing with the size). On physical exam, a pulsatile
abdominal mass may be palpated or auscultated.
B. SYPHILITIC (LUETIC) - occur in the ascending aorta and aortic arch.
Tertiary syphilis causes inflammation and obliteration of vasa
vasorum leading to patchy medial necrosis. Contraction of the
resulting fibrous scar tissue leads to longitudinal "tree barking"
of the intimal surface. As the aorta dilates due to the weakening of
the elastic media, there is often secondary atherosclerotic
involvement of the intima. The aneurysmal dilatation of the
ascending aorta may dilate the aortic annulus resulting in aortic
insufficiency manifested by a diastolic murmur and a widened pulse
pressure. Over time, this aortic regurgitation may result in a
massively enlarged heart. Due to their location, clinical symptoms
are more common with thoracic aneurysms. Although these may rupture,
death most frequently the result of heart failure due to the aortic
valve incompetence.
C. t1Yconc - result from weakening of a vessel wall due to local
infection (usually by bacteria or fungi). These may occur at any
location and generally become symptomatic only when they rupture.
II. CYSTIC MEDIAL NECROSIS - is a chronic degenerative process of unknown
etiology that results in focal destruction of the media of the thoracic
aorta. It is most likely due to a metabolic defect in the synthesis of
collagen and elastin and is frequently associated with Marfan's syndrome.
It is also seen in hypertension as weIr as with normal aging. With the
weakening of the aortic wall, aneurysmal dilatation may ensue. With the
decline of syphilitic aneurysms, this disorder is now responsible for the
majority of thoracic aortic aneurysms. It has also been linked to the
development of aortic dissections.
III. AORTIC DISSECTION (DISSECTING ANEURYSl1) - is more common than rupture of
atherosclerotic aneurysms. Most occur in middle-aged men (40-60) although
there is also an increased incidence in pregnant women. Hypertension is
frequently a predisposing factor particularly in those patients without
preexisting damage to the aortic media. The dissection is characterized by
hemorrhage between the middle and outer 1/3 of the aortic media. Although
some feel that the precipitating event is hemorrhage into the media
secondary to rupture of diseased vessels of the vasa vasorum (hypertensive
damage), they usually arise secondarily to the development of spontaneous
transverse intimal tears in either the proximal ascending aorta or
immediately distal to the origin of the left subclavian artery. Areas of
medial necrosis may allow the intima to buckle into the lumen where
hypertensive pressures cause shearing and tearing. Blood is then forced
into the outer media where it may dissect both proximally and distally.
The clinical presentation usually involves severe, "tearing" pain in the
anterior chest which radiates into and down the back.
A. TYPE A DISSECTIONS -are usually seen in middle aged men, involve the
ascending aorta, and are often secondary to medial degeneration. A
major cause of death is retrograde dissection, rupture into the
pericardial cavity, and cardiac tamponade.
B. TYPE B DISSECTIO~ (descending aorta only) - are less common and are
usually seen in elderly patients with atherosclerosis and
hypertension. Anterograde dissection occurs with possible rupture
into pleural or peritoneal cavities but rupture is less likely than
in patients with Type A.

44
IV. NON-INFECTIOUS NECROTIZING VASCULITIDF-..4) are characterized by
vascular necrosis and inflammation and are probably mediated by
immunologic mechanisms. Many exhibit multi-system involvement with highly
variable clinical presentations.
A. - Primarily affecting young to middle-aged
PQLYAmBJJ'lS IIIJOSA (PAN)
males, the classic form of the disease causes segmental necrosis of
medium and small arteries within all viscera except lungs. Kidney,
heart, liver, and GI tract are most commonly involved.
1. - three stages of vascular involvement can be
IIORPHOLOGY
identified which may exist simultaneously within the same
artery or different arteries.
a. ~ - segmental fibrinoid necrosis of the intima may
expand to involve the entire vessel wall thickness and
elicit an acute inflammatory infiltrate. This tends to
occur at vascular branching points and predisposes to
thrombus and/or aneurysm formation.
b. Healing - development of granulation tissue results in
thickening and narrowing of the lumen, and perivascular
fibrosis may produce clinically palpable nodules.
c. Healed - characterized by marked fibrous thickening of
the vascular wall.
2. ~ICAL ~ - complaints are often non-specific and tend to
come and go. Lab changes (increased ESR, anemia, leukocytosis)
are likewise non-specific. Renal involvement occurs in 75% of
patients and, if untreated, is the most common cause of death
although eVA, bowel infarcts, etc. may supervene. Fortunately,
however, the prognosis has greatly improved with advent of
steroid and cyclophosphamide therapy.
B. HYPERSENSlTIYm VASCULITIS

1. LEUKOCYTOCLASTIC VASCULITIS - is a hypersensitivity immune complex


vasculitis affecting small arterioles, capillary, and venules.
Although most frequently confined to the skin, it may also
involve lung, brain, heart, GI, kidney, muscle. In contrast to
PAN, all lesions are at the same stage of development and
consist of polys infiltrating the vessel wall. Although
fibrinoid necrosis mayor may not be present, immunoglobulins
and complement can be found in the lesions. Extravasation of
erythrocytes into surrounding tissue and karyorrhexis of the
neutrophil nuclei (leukocytoclasis) are characteristic. By
eliminating the antigen, most patients improve, although some
die of crescentic (rapidly progressive) glomerulonephritis.
2. H~-SCH~lN ~ is a hypersensitivity vasculitis
affecting the skin, joints, kidneys and GI tract of children.
IgA and complement are deposited in the vascular walls. There
is abrupt onset of fever, arthralgias, and rash. It is usually
self limited but renal failure can occur.
C. GRAHULCIIATOUS (GIANt CELL) ARTERITmES

1. TAKAYASU'S ARTERITIS(pulseless disease) - is a disease of unknown


etiology, most frequentll seen in young females, which results
in progressive segmenta stenosis of the aorta (particularly
aortic arch) and the major arch arteries. The pulmonary artery
may also be involved. Granulomatous inflammation and
subsequent fibrosis of the media and intima causes marked
thickening of the vessel especially at the origins of the arch
vessels. Clinical signs and symptoms are frequently referable
to decreased upper body perfusion.
2. . TBIPORAL ARTERITIS - probably most common of the vasculitides,
this tends to be a disease of the elderly characterized by
focal segmental granulomatous lesions of small, medium
(especially temporal), and occasionally large arteries. The
45
characteristic microscopic findings include medial necrosis
with disruption of the internal elastic lamina; non-specific
acute and chronic inflammatory infiltrate involving entire
wall thickness; and intimal fibrosis with reduction of lumenal
caliber. The hallmark is the presence of granulomas (complete
with giant cells) in the vessel wall but these are frequently
absent. The clinical presentation may be insidious and non-
specific, or i t may be abrupt with severe throbbing headache
and tenderness and nodularity along the course of the vessel.
Visual symptoms are present in 40% of patients and may
progress to blindness due to ophthalmic artery involvement and
ischemia to optic nerve. Lab tests are non-specific (increased
ESR) and the disorder usually responds rapidly to steroids.
D. BUERGER'S DISEASE (thromboangiitis obliterans) is a disease of
unknown etiology, almost exclusively limited to cigarette smokers,
which is characterized by segmental inflammation and thrombosis of
the small and medium sized arteries of the extremities. Neutrophils
are present throughout the thickness of the wall and microabcesses
may be found in the occlusive thrombi. The clinical course will wax
and wane in relationship to cigarette use. Acute episodes cause
excruciating pain and eventually gangrenous necrosis may develop.
E. WEGENER'S GRAHYLOtIATOSIS - is a rare disorder of unknown etiology but
which may be related to a hypersensitivity to a foreign substance.
Most frequently seen in middle-aged males, it is manifested by 1)
necrotizing granulomas of upper and lower respiratory tract which
sometimes cavitate, 2) focal necrotizing vasculitis of small
arteries and veins, and 3) focal necrotizing glomerulonephritis. The
most frequent clinical presentation involves persistent pneumonitis
with bilateral cavitary lesions, chronic sinusitis, nasopharyngeal
mucosal ulcerations, and renal disease. At one time, it was rapidly
fatal but long term remission can now be expected with the use of
steroids and cyclophosphamide.
F. RAYNAUD' S DISEASE - refers to vasospasm, often bilateral, of small
arteries and arterioles in the fingers and toes of young healthy
women often triggered by cold temperatures or emotional stress.
Usually the disorder follows a benign course but with chronic
disease, ulceration and necrosis of the skin may develop.
G. RAYNAUD'S PHENa1ENON - refers to conditions which cause the same
symptoms as Raynaud's disease but which are secondary to some
underlying disorder which affects structural integrity of vessels.
V. VARICOSE VEINS - refers to any venous dilatation that results from
chronic elevation of intravascular hydrostatic pressure. This is most
commonly seen in the superficial leg veins and becomes more clinically
apparent with increasing age, especially in females. As the vessels
dilate, they become elongated and tortuous. The overall effect is to
create chronic soft tissue edema, venous stasis, and thrombosis. The skin
may develop stasis dermatitis and chronic ulcerations.
VI. NEOPLASTIC DISEASE

A. BENIGN

1. CAPILLARY HEMANGIOMA -is composed of non-encarsulated aggregates


of thin walled capillary-like vessels ined by flattened
endothelium and separated by a scant connective tissue stroma.
They range in size up to several centimeters in diameter.
2. CAVERNOUS HEMANGIOMA - is comprised of large cavernous vessels
that form a spongy mass that can be quite large but is usually
under 2-3 cm. in diameter. Occasionally, these may occur as
part of a congenital syndrome (von Hippel-Lindau disease).
3. G~ nmOR - arises from a glomus body and usually appear on
the distal portions of the fingers or subungually as small,
firm, purplish, extremely painful nodules. The tumors contain
vascular channels and the characteristic glomus cells in the
connective tissue stroma.

46
B. MALIGNANT

1. ANGIOSARCOtIA - appears in the skin or subcutaneous tissues as an


ill defined fleshy mass with central necrosis and hemorrhage.
There is a wide range of histologic appearance depending on
the degree of differentiation. These are relatively rare
lesions except for those arising on the scalp or face of
elderly persons and those arising secondary to chronic
lymphedema (Stewart-Treves syndrome).
2. HEHANGIOPERICYTOtfA - arising from the vascular pericytes, these
are generally small tumors which may metastasize widely.
3. KAPOSI'S SARCOtIA - originally a slowly progressive, multifocal,
cutaneous neoplasm of the legs in elderly men of Mediterranean
descent, Kaposi's sarcoma is now seen most frequently as an
aggressive. although treatable, disseminated disease in
patients with AIDS (particularly those infected by homosexual
. contact as opposed to IV drug contact). It also may appear as
a complication of prolonged immunosuppression in patients with
allograft transplants. Early lesions resemble granulation
tissue but the vascular spaces eventually become angular with
proliferating spindle cells ultimately producing inconspicuous
vascular slits.

47
RESPIRATORY TRACT

I. REVIEW OF NORMAL
A. ~ - the trachea (cartilaginous plates) branches into main bronchi
which branch into lobar bronchi (discontinuous cartilaginous plates)
which branch to supply the intralobar bronchopulmonary segments.
Further branchin~ produces bronchioles ( loss of cartilage and
submucosal glands), terminal bronchioles, respiratory bronchioles,
alveolar ducts, alveolar sacs, and alveoli. A lobule is the smallest
discrete portion of lung bounded by fibrous septa and consists of a
terminal bronchiole and its branching structures while the smaller
acinus comprises a respiratory bronchiole and its branching
structures.
B. "IC~PIC - the tracheobronchial tree is lined by pseudostratified
columnar ciliated epithelium with interspersed mucus-secreting
goblet cells and neuroendocrine cells and submucosal mucus-secreting
glands. The submucosal glands and goblet cells are lost at the level
of the bronchioles which are lined by ciliated epithelium and Clara
cells which secrete a non-mucoid watery substance that contains
lysozyme and immunoglobulins. The alveolar walls contain capillaries
lined by endothelial cells. The capillary basement membrane fuses
with the basement membrane of the alveolar epithelial cells on one
side (the "thin" side) and is separated from the basement membrane
of the alveolar epithelial cells by the pulmonary interstitium on
the other side (the "thick" side). The interstitium contains
collagen, elastin, mast cells, occasional inflammatory cells, and
connective tissue (smooth muscle, fibroblast) cells. The majority of
the alveolar surface is lined by the Type I (membranous) pneumocytes
which are interspersed with the surfactant-producing Type II
(granular) pneumocytes which contain the lamellar bodies. Alveolar
macrophages, derived from blood monocytes, are loosely attached to
the alveolar wall or lie free within the alveolar space.
II. CONGENITAL ABNORMALmES

A. HYPOPLASIA - any condition which impedes the development of the


thoracic cavity (polyhydramnios, cystic kidneys, etc) may lead to
unilateral or bilateral underdevelopment of the lungs.
B. DIAPHRAGtlATIC HERNIA - partial or total absence of the diaphragm may
rQsult in herniation of the abdominal contents into the thoracic
cavity. In utero, diaphragmatic defects will result in pulmonary
hypoplasia due to compression of the lungs by the herniated viscera
and these infants will have respiratory difficulties at birth.
C. BRO~IC ~ - peribronchial cysts (single or mUltiple) lined by
bronchial epithelium may contain air or mucoid material. They may
predispose to abscess formation or may rupture into bronchi
(hemorrhage and hemoptysis) or pleural cavity (pneumothorax).
D. BRONCHOPULMONARY SEQUESTRATION - is the presence of lung tissue that has
no connection with the tracheobronchial tree and receives blood
supply from the aorta. It may occur in either an intralobar or
extralobar location. Primary significance is that it may serve as a
focus of recurrent infection.
III. ATELECTASIS - refers to a condition of incomplete expansion of alveoli.

A. PRIMARY (ATELECTASIS NEONATORUIt) - results from the failure of lungs to


ventilate at time of birth. This may be due to birth trauma,
bronchial obstruction, drugs, immaturity, CNS disorders, etc.
B. SECONDARY (ACQUIRED ATELECTASIS) - may be due to deficiency of surfactant
(respiratory distress syndromes), loss of negative intrapleural
pressure (chest trauma, pneumothorax), complete obstruction of an
airway (by secretions, exudates, neoplasms, or foreign bodies),
direct pressure on lungs (usually by accumulation of material in the
pleural cavitie.s), or contraction (parenchymal fibrosis). Depending

48
on the etiology, the distribution may be focal, segmental, or
massive. Infections may develop in areas of collapse and "carnifi-
cation" (organization by fibrous tissue) can occur if lungs remain
collapsed.
IV. RESPIRATORY DISTRESS OF THE NEWBORN (RDS Type I ~ Hyaline Membrane
Disease)
A. ETIOLQSY - immature development of the lung results in a deficiency
of surfactant, the material synthesized by the Type II pneumocytes
and needed to lower the surface tension of the alveoli and help
maintain patency. Infants at risk include those born prematurely,
those delivered by Caesarean section, and those whose mothers are
diabetic. An amniotic fluid lecithin:sphingomyelin ratio of less
than 2: I indicates a high probability of developing hyaline membrane
disease.
B. PAllKIGENESIS - a deficiency of surfactant requires greater inspiratory
effort to expand collapsed airways (clinically manifested as
retraction of ribs and sternum during inspiration) . Decreased
ventilation of an already immature lung produces hypoxia, cyanosis,
and metabolic acidosis. The latter triggers vasoconstriction and
further hypoxia causing endothelial and epithelial injury, exudation
of fibrin rich fluid into the interstitium and alveoli, and the
formation of hyaline membranes (fibrinous exudate admixed with
necrotic epithelial cell debris) along the respiratory bronchioles,
alveolar ducts and alveoli. This further interferes with gas
exchange, perpetuates the hypoxia, and hinders the ability of the
Type II pneumocytes to produce surfactant.
V. ADULT RESPIRATORY DISTRESSSYNDROME ( ARDS~ diffuse alveolar damage
(D.AJ))~ "shock" lung~etc) - is a life-threatening disorder characterized
by the acute onset of dyspnea and tachypnea, hypoxemia refractory to
therapy, and cyanosis.
A. may involve a wide variety of mechanisms all of which have
ETIOLOGY -
the common denominator of widespread microvascular injury.
B. PA~IS - diffuse injury to alveolar capillary endothelium (and
subsequent involvement of the alveolar epithelium) may be produced
by direct damage to endothelial cells or mediated by leukocyte
aggregation and activation. Leukocyte generated free radicals,
lysozymes, and arachidonic acid metabolites may be responsible for
epithelial and endothelial cell damage, vasoconstriction, and
destruction of interstitial elastin and collagen. The inflammatory
response results in the escape of fibrin rich exudate into alveoli
which, admixed with necrotic epithelial debris, forms hyaline
membranes along septal walls. Loss of surfactant co~licates the
problem and atelectasis produces a noncompliant, "stiff" lung.
Enlarged, regenerating Type II cells may become prominent along the
alveolar membrane and ultimately fibrosis of the interstitium may
occur.
VI. HEMODYNAMIC DISTURBANCES
A. CRmMIe PMSPi omIESTn. - is associated with chronic failure of the
left side of the heart leading to brown induration of the lung.
B. ~ ~ - extravascular fluid in the lung, initially in the
interstitial space and subsequently spilling over into the alveoli,
due to disturbances of the normal hemodynamic equilibrium or
microvascular injury.
C. PUlJllllARX EI!BOLISII - results from the impaction of a previously
detached thrombus or a foreign substance in the pulmonary vascular
bed. The etiology includes multiple entities which would predispose
to venous thrombosis or stasis (such as congestive heart failure,
ilDQlobi liz at ion , phlebitis, etc) . The pathologic change and the
clinical manifestations of pulmonary embolism is dependent on the
size of the vessels in which the embolus impacts and on the
preexistent disease state of the lung. Small emboli impacting in

49
peripheral vessels may be clinically asymptomatic. Pathologically,
one may find only a few pulmonary hemorrhages and/or congestion and
edema. However, recurrent showers of small emboli will result in
progressive reduction of pulmonary vascular bed and give rise to
pulmonary hypertension. Occlusion of medium size pulmonary arteries
can give rise to infarction and the development of pleuritic
symptoms, however, infarction from an embolus is unusual (5-10% of
the cases). These patients may present with hyperventilation, a
history of sudden onset of dyspnea, and a tachycardia of over
100/minute. Blood gases would reveal a decreased p02 and decreased
PC02 secondary to the hyperventilation (respiratory alkalosis).
Massive embolization, particularly of the saddle type, may induce
the immediate catastrophic syndrome of acute cor pulmonale and
sudden death. If not immediately fatal, it may result in chest pain,
severe dyspnea, cyanosis, tachycardia, diaphoresis, and shock.
D. PULMONARY INFARCTION - signifies ischemic necrosis of lung parenchyma
and is not synonymous with pulmonary embolization. Infarcts are more
likely to be seen following embolization in those patients in whom
there is already impairment of the dual blood supply. Grossly, the
lesions will vary in size from those barely visible to wedge-shaped
involvement of a large part of an entire lobe. Classically, they
abut on the visceral pleura with the apex of the wedge-shaped
infarct pointing toward the hilus. The majority of infarcts are
Within the lower lobes, and more frequently seen on the right.
Histologically, the infarct shows hemorrhagic coagulation necrosis.
Clinically, patients tend to develop pleuritic chest pain and
pleural friction rub, hemoptysis, and dyspnea. A low grade fever and
a leucocytosis may be additional findings.
E. PULtlONARY HYPERTENSION - refers to increased pressures within the
pulmonary vasculature and is due to increased vascular resistance.
1. PRDURY - is mostly a disease of young women that results from
prolonged vasoconstriction induced by hypersensitivity to
unknown agents. There is marked thickening of the arterioles
and small arteries with hypertrophy of the media and
reduplication of the elastic membranes. Medium sized vessels
also show medial hypertrophy and the large arteries may
develop uncomplicated atherosclerotic plaques. Insidious
development of pulmonary symptomatology culminates with death
usually due to cor pulmonale.
2. SECOtilARY occurs with known underlying conditions that
increase pulmonary vascular pressures or resistance
(congestive heart failure, primary pulmonary disease, or
recurrent emboli).
VII. INFLAMMATORY LESIONS OF mE UPPER RESPIRATORY TRACT (URI) - are
generally due to a wide variety of viruses. Clinical presentation
typically relates to nasal, pharyngeal, and/or conjunctival involvement.
Systemic effects may include chills, malaise, myalgias, headache and
fever.
VIII. INFLAMMATORY I,ESIONS OF THE LOWER RESPIRATORY TRACT
A. ACUTE BRONCHITIS - is usually the result of irritant gases, viruses, or
less commonly bacteria.
B. VIRAL/MYCOPLASMAL PNEUMONIA - the pathologic lesions are peribronchiolar
and predominantly interstitial (i.e. within the alveolar walls). The
alveolar walls are widened by edema and a predominately mononuclear
infiltrate. The alveolar spaces are generally free of significant
cellular exudate but focal hyaline membranes may be present
reflecting alveolar epithelial damage. Often there is a history of
recent URI and non-specific systemic symptoms. A persistent sparsely
productive cough is the hallmark of the disease. Chest pain is
substernal, pleuritic pain and effusions are infrequent. Dyspnea and
cyanosis are rare.

50
C. the pathologic changes depend in part on the agent
BACTERIAL PNEUtIONIA -
and the host response to that agent. Clinical signs and symptoms
also depend on the agent but also on the extent of involvement.
1. EKrEKr OF INVOLVEMENT - two anatomic and radiologic patterns of
bacterial pneumonia are classically described but there may be
overlap between the two.
a. Bronchopneumonia refers to an inflammatory
consolidation which occurs in a patchy distribution
throughout a lobe (most frequently lower lobes) or lung.
There is a predilection for infants and the elderly. In
adults, predisposing factors include acute viral
bronchitis or bronchiolitis, chronic bronchitis,
alcoholism, malnutrition, pulmonary edema, and cardiac
failure. The clinical picture frequently overshadowed by
the predisposing condition.
b. Lobar pnewnonia refers to a more extensive
inflammatory consolidation involving an entire lobe or
large portion thereof. With the advent of antibiotics,
however, well developed lobar pneumonia is encountered
less frequently. A lobar distribution tends to reflect
the virulence of the organism and the effectiveness of
the patient's defense mechanisms. The vast majority of
lobar pneumonias are due to streptococcus pneumoniae.
2. ETIOLOGIC AGENTS

a. Pneumococcal (streptococcus pneumoniae) pneumonia


four stages have classically been described in untreated
lobar pneumonia.
(1) Congestion - involves rapid proliferation of the
bacteria and the early stages of the inflammatory
response (vascular engorgement and serous
exudation into alveolar spaces).
(2) Red hepatization - involves the outpouring of
neutrophils, extravasated erythrocytes, and
precipitated fibrin into alveolar spaces.
Gross ly, this produces an appearance and
consistency similar to that of the liver.
Microscopically, the alveolar infiltrate obscures
the underlying pulmonary architecture.
(3) Grey hepatization - involves disintegration of
neutrophils and lysis of erythrocytes in
combination with continued accumulation of
fibrin. Grossly, this imparts a grayish
appearance to the still firm parenchyma.
Microscopically, contraction of the fibrin away
from alveolar walls again discloses the preserved
native pulmonary architecture.
(4) Resolution - in uncomplicated cases, the exudate
in the alveolar spaces is digested and reabsorbed
or removed restoring the pulmonary parenchyma to
its normal state.
Clinically, there is a sudden onset, often with shaking
chills, followed by high fever. Cough at first is dry or
productive of watery sputum. At the stage of red
hepatization, sputum becomes thick, purulent, and
hemorrhagic (rusty sputum). The associated pleuritis
manifests itself by pleuritic pain and a friction rub.
There may be a pleural effusion. Leucocytosis of 15,000
to 40,000 is usually present.
b. Staphylococcal pneumonia occurs in infants and
children under two years of age, postoperative patients,
patients with staphylococcal wound infections, patients
with chronic pulmonary disease, and persons who have

51
influenza. Pseudocysts (pneumatocele) and abscess
cavities, as well as effusions or empyema occur. Other
complications include metastatic abscesses and
pneumothorax, especially in infants and children.
Mortality varies but is generally higher than
pneumococcal pneumonia.
c. Gram negative bacteria - most gram negative pneumonias
represent hospital acquired infections. They occur in
patients who have received previous extended broad spec-
trum antibiotic therapy or assisted ventilation and
those with dissemination to the lungs from another site
(urinary tract, etc). These organisms tend to cause
necrotizing pneumonias with permanent damage to the
pulmonary architecture. Mortality rate of gram negative
pneumonias is much higher than with gram positive
pneumonias.
(1) Klebsiella pneumonia - often accompanies other
underlying diseases, chiefly alcoholism. Dyspnea
and cyanosis may be quite prominent. Sputum is
characteristically thick, gelatinous, and bloody.
Abscesses are quite common. Chronic lung abscess
and pleural empyema are frequent complications.
(2) Hemophilus pneumonia - is often found in the
pediatric age group, usually in children less
than 2 years of age and rarely occurs in adults.
(3) Pseudomonas aeruginosa - will grow in the water
of humidification devices so patients who require
assisted ventilation are especially prone to
pseudomonas pneumonia.
(4) Enteric gram negatives - are often seeded to the
lungs from urinary tract infections. may colonize
in the pulmonary vasculature, and form abscesses.
D. LUNS ABSCESS - refers to localized suppuration and liquefaction
necrosis of lung parenchyma, which in chronic cases (> 6 weeks
duration) may elicit considerable fibroblastic proliferation in the
wall. Aspiration is the most common cause especially when the gag
reflex has been suppressed. This often introduces anaerobic
organisms from the oral cavity. These abscesses tend to be solitary
and occur most frequently on the right side due to the shallow angle
of the right mainstem bronchus. Other predisposing factors include
bacterial pneumonia (especially Staph. and Klebsiella), bronchial
obstruction, septic emboli, cysts or bullae, and penetrating chest
trauma. If no underlying etiology can be identified, they are
referred to as primary idiopathic (cryptogenic) lung abscesses.
Generally, there is a prominent cough, usually with copious amounts
of foul smelling or bloody sputum. X-rays may show a homogenous
density which, if there is communication with an airway, may reveal
an air-fluid level. If the patient does not have sputum production,
bronchial obstruction should be suspected as the etiology.
E. PNEUHOCYSI'is PNEUHONIA - results from reactivation of a latent l!..,...
.carinii infection in immunosuppressed or immunodeficient patients.
The organisms do not invade tissue but do cause necrosis of alveolar
epithelial cells which creates a characteristic "frothy" edematous
exudate in alveolar spaces.
F. CHEMICAL PNEUHONIA

1. ASPIRATIONPNEUHONIA - occurs most frequently in unconscious


patients or those whose gag reflexes are suppressed. The
clinical effects are dependent upon the volume and the nature
of the aspirate. Liquid gastric aspirate causes extensive
acute in.flammatory reaction, pulmonary edema, widespread
destruction of the epithelium, hemorrhage, and hyaline
membranes. With extensive involvement, lung parenchyma may be

52
almost completely destroyed. Chronic or recurrent aspiration
will cause repeated bouts of tracheobronchitis and
pneumonitis. With time, chronic fibrosis will ensue with
resultant dyspnea and digital clubbing.
2. LIPm PNEUMONIAS

a. Endogenous (golden pneumonia, cholesterol pneumonia) -


is usually seen as a complication of an obstructive
lesion of the bronchial tree. In the alveoli, there are
large numbers of "foamy" macrophages containing
surfactant and lipids from degenerating cells.
b. Exogenous - occurs when fatty or oily material is
inhaled or aspirated. Clinical symptoms may be
relatively few. Occasional cough with sputum production
and dyspnea may occur. Occasionally, there may be a
large localized mass suggestive of granuloma or
carcinoma.
G. PULMONARY TUBERCULOSIS

1. PRIMARY TUBERCULOSIS - the initial pulmonary infection often


occurs in childhood and is usually subpleural, most commonly
in the upper portion of lower lobe or the lower portion of the
upper lobe. This focus (the ghon focus) shows central
caseation and is associated with caseous lymphadenitis in the
hilar lymph nodes. The combination of the peripheral lung
lesion and the hilar node involvement is called the primary
ghon complex. Usually the only residuum of the primary
infection is the presence of a fibrotic or sometimes calcified
ghon complex. Occasionally, particularly in pre-school
children, a primary infection does not run a benign course.
Possible complications include tuberculous pneumonia,
bronchogenic dissemination, or lymphatic and hematogenous
dissemination.
2. SECONDARY (REACTIVATION) TUBERCULOSIS - develops from reactivation of
dormant lesions or, less frequently, from reexposure of a
sensitized individual. Since the organisms prefer areas of
high oxygen tension, reactivation is almost invariably
localized to the apices of one or both upper lobes and is
called the assmann focus. Because of previous exposure, there
is a prompt granulomatous tissue response often with caseous
necrosis. Granulomas may coalesce to form larger foci of
destruction but most eventually become encapsulated by fibrous
tissue and develop areas of dystrophic calcification. In some
cases, the lesions may remain active, erode into the
tracheobronchial tree and cavitate. Cavitation is considered
the anatomic hallmark of secondary tuberculosis. Apical
cavitary fibrocaseous tuberculosis may heal, spread by direct
extension, or be disseminated through the tracheobronchial
tree, the lymphatics, or the blood. The pattern of
dissemination is determined primarily by the route.
Complications may include massive hemoptysis due to erosion of
a pulmonary vessel, parenchymal fibrosis, and involvement of
the pleura which may lead to a tuberculous empyema.
H. MYCOSESOF THE LUNG - in general, the fungi are weak antigens and cause
tissue damage primarily by virtue of the hypersensitivity reaction
of the host against the parasite proteins. Most of the deep mycoses
(histoplasmosis, coccidioidomycosis, North American blastomycosis,
cryptococcosis) produce chronic granulomatous disease. Candida,
mucor and aspergillus are the classic opportunistic fungal organisms
and elicit an acute pneumonitis. Nocardia characteristically elicits
purulent abscess formation and actinomycosis elicits both chronic
granulomatous formation and purulent exudation.

53
I. SARCOIDOSIS- is a systemic disease of unknown etiology but may
represent an abnormal immunologic response to a variety of non-
specific agents or antigens. The classical syndrome includes
bilateral hilar lymphadenopathy, uveoparotitis, lesions in the short
bones of the hands and feet, erythema nodosum,
hypergammaglobulinemia, hypercalcemia and llypercalciuria. The lung
is the most frequently affected organ but the pulmonary
manifestations are variable. Presenting symptoms include cough,
dyspnea, chest pain, loss of weight, and malaise or excessive
fatigue. The lesions may have a miliary distribution throughout the
lungs and histologically there are non-caseating, or so called
"hard" granulomas. No organisms are present on acid-fast stains nor
can organisms be cultured from the fresh tissue. Schaumann bodies
and asteroid bodies are characteristic but not pathognomonic. The
lesions become fibrotic and depending on extent of spread may lead
to symptoms of restrictive lung disease. "Honeycombing" (end stage
pulmonary disease) occurs as a late result of fibrosis and
destruction of the pulmonary parenchyma.
IX. CHRONIC OBSlRUCTIVE PULMONARY DISEASE (COPD) - is characterized by
decreased forced expiratory volumes as the result of chronic or recurrent
narrowing of the tracheobronchial tree or loss of elastic recoil of the
pulmonary parenchyma.
A. CH~IC B~ITIS - is a clinical disorder characterized by excessive
mucous secretion of the bronchial tree than cannot be explained by
either specific infection or infiltrative disease. It is defined as
a chronic cough, with sputum production for at least three months of
the year in at least two consecutive years. The etiology relates
primarily to cigarette smoking but can be exacerbated by air
pollution, occupational irritants, etc. Through unknown mechanisms,
chronic irritation to the tracheobronchial tree induces hypertrophy
and hyperplasia of the submucosal mucous glands in the large airways
and goblet cell metaplasia (reducing the number of normal ciliated
cells) of small bronchi .mm- bronchioles. The resultant
hypersecretion of mucus accounts for the increased expectoration and
also leads to obstruction of small airways. Blood gases generally
show hypercapnia and hypoxemia and the patients may be cyanotic.
Squamous metaplasia and dysplasia of the surface epithelium may also
be present. Excessive mucus production and interference with the
tracheobronchial toilet also predisposes to repeated bacterial
infection with damage to the bronchiolar walls and ultimate fibrous
obliteration (bronchiolitis fibrosa obliterans). Cor pulmonale,
peptic ulcers, polycythemia, and respiratory failure may also ensue.
B. ~y~ - refers to an abnormal, permanent increase in the size of
air spaces distal to the terminal bronchiole and accompanying
destructive changes. The septa and alveolar walls ap,pear attenuated
and broken with fragments of the wall appearing to 'float" in large
distorted air spaces.
1. CEKrRIL«mULAR is characterized by destructive changes
primarily to the respiratory bronchioles with sparing of the
more distal segments of the alveolus. Most striking in the
upper lobes, it is seen most frequently in cigarette smokers
and is often associated with chronic bronchitis. Particles of
cigarette smoke impact in the distal airways and stimulate
macrophages which, among other things, recruit and activate
neutrophils. Both macrophage and neutrophilic elastases are
released and unless they are inactivated, proteolytic
digestion of the alveolar walls will ensue. Additional damage
results from free radical release from neutrophils.
2. PANL«mULAR - most frequently seen in the lower lobes, it is
characterized by uniform involvement of the acinus and is seen
in patients with alpha-I-antitrypsin deficiency, especially i f
they also smoke. Alpha-I-antitrypsin is a naturally occurring
antiprotease that can inhibit the action of elastases that are
released by neutrophils during the inflammatory response.

54
3. PARASEPTAL - is characterized by destruction of the distal
portion of the. acinus directly underlying the pleura or
fibrous septa and usually is seen in areas of scarring and
fibrosis. It tends to occur more frequently in the upper lobes
and is a common cause of spontaneous pneumothorax occurring in
young adults.
4. IRREGULAR (SCAR) EMPHYSEMA - focal parenchymal loss found in almost
all adult lungs often in areas of old scars from tuberculosis,
histoplasmosis, etc. Generally asymptomatic.
Clinically, symptomatic patients usually present with a history of
progressive dyspnea and often weight loss. The A-P diameter of the
chest is increased (barrel chest). Due to loss of elastic recoil,
expiration is prolonged and patients exhale through pursed lips to
force air out. Blood gas values are often normal. On chest X-ray,
there is increased radiolucency of the lung fields with depression
and flattening of the diaphragm.
C. BRONCHIAL ASTHMA - is characterized by an increased sensitivity of the
tracheobronchial tree to various stimuli and is manifested by
recurrent widespread, but generally reversible, narrowing of the
airways.
1. is an IgE mediated hypersensit:i,vity reaction
ATOPIC (ALLERGIC) -
and can be triggered by a wide variety of environmental
allergens. Onset typically occurs in childhood and there is
often a family history of allergy. Serum IgE levels are
usually elevated. The immediate response of wheezing, edema,
and increased mucus secretion results from degranulation of
sensitized mast cells and from stimulation of subepithelial
vagal receptors to cause bronchoconstriction. Release of the
various chemical mediators of inflammation (leukotrienes,
prostaglandins, chemotactic factors, etc) potentiate the
bronchoconstriction. A delayed response results from the
recruitment of granulocytes which release further chemical
mediators (including major basic protein) that causes further
edema, bronchospasm, and epithelial necrosis.
2. NON-ATOPIC - is frequently triggered by upper respiratory
infections (primarily viral). The mechanism of action is
unknown. IgE levels are usually normal and a family history is
usually lacking.
Mucous plugs containing desquamated epithelial cells (Curschmann's
spirals) and eosinophiIs are present in small bronchi and
bronchioles. Charcot-Leyden crystals (crystalloids of eosinophil
membrane proteins) may be identified in sputum samples. Muscular
hypertrophy of bronchial walls results from repeated bronchospasms,
and there is submucosal inflammatory infiltrates composed chiefly of
eosinophils. Goblet cell metaplasia and hyperplasia of submucosal
glands may also be present.
D. BRONCHIEC1'ASIS refers to permanent dilatation of bronchi and
bronchioles caused by necrotizing inflammatory damage secondary to
bronchial obstruction, necrotizing pneumonia, or a variety of
congenital or inherited conditions. The inflammatory reaction
destroys the wall causing elongation, dilatation, and ultimately
fibrosis. Epithelial necrosis or metaplasia further inhibits normal
removal of bronchial secretions which become viscous and infected.
The lower lobes are most frequently affected, the left more often
than the right, but it may be bilateral. Clinically, it is usually
associated with cough and copious production of purulent sputum.
E. MUCOVISCIDOSIS (cyst ic fibrosis) is a hereditary, mendelian
recessive, generalized disturbance of exocrine glands manifested by
pancreatic insufficiency, chronic respiratory disease, electrolyte
disturbances, and occasionally, cirrhosis of the liver. The
submucous glands in the bronchial tree secrete an atypical viscous
mucus which is difficult to clear from the airways. These patients
develop repeated infections and mucopurulent material is frequently
present within the trachea and bronchi. With time, bronchiectasis
also usually develops.

55
x. RESTRICTIVE PULMONARY DISEASES - comprise a group of disorders
characterized by reduced total lung capacity, reduced oxygen diffusing
capacity, and reduced lung compliance ("stiff" lung). The initiating
events are widely varied but all probably activate alveolar macrophages
which then release fibroblast stimulating factors and recruit and activate
neutrophils resulting in inflammatory or immunologic damage to alveoli and
small airways. Early lesions generallr show diffuse inflammatory
infiltrates within the alveolar walls (a veolitis) and peribronchiolar
interstitium. Ultimately, without intervention, vascular and parenchymal
destruction produces a cystic, fibrotic "honeycomb" lung and respiratory
failure.
A. - are diseases caused by the inhalation of inorganic
PNEUHOCONIOSES
mineral dusts (coal dust, silica, asbestos, talc, kaolin, beryllium,
etc). These dusts will elicit, to a variable degree, pulmonary
fibrosis as a host response. In general, particles less than 2~ in
size are able to reach the terminal airways where the alveolar
macrophage is the primary defense mechanism.
1. COAL NORKERS LUNG(black lung) - coal dust is deposited in small
airways where it is ingested by macrophages that then migrate
into the interstitium and collect around the respiratory
bronchioles. Although there is relatively little fibrous
reaction, the respiratory bronchioles do dilate and in
actuality, this tends to induce an obstructive disease that
mayor may not have a restrictive component.
2. SILICOSIS - is due to the inhalation primarily of quartz dust.
The silica particles are ingested by alveolar macrophages
which secrete a fibroblast stimulating factor but are also
destroyed by the toxic effects of the silica. The silica is
released by the irreversibly injured macrophages only to be
reingested by others thereby initiating a repetitive cycle.
Eventually they are "walled off" by fibrotic nodules of
whorled collagen surrounded by lymphocytes and fibroblasts.
Silicosis and tuberculosis are often associated.
3. ASBESTOSIS - causes an interstitial fibrosis resulting from
alveolitis caused by the long, thin asbestos fibers. Asbestos
fibers cannot be completely engulfed by macrophages and may
become encrusted by iron (derived from the hemoglobin released
in microhemorrhages). These are called asbestos bodies and can
be seen around alveolar ducts and distal acinar structures
primarily in the lower lobes. Fibroblast stimulating factors
released by macrophages incite fibrosis. The pleura is
frequently thickened and there is an increased incidence of
malignant mesothelioma.
B. encompass a group of diseases which have
CHRONIC INTERSTITIAL PNEUtlOHIA§ -
in common alveolitis with subsequent fibrosis. The end result of
each of these is "honeycomb" lung characterized by multiple cystic
spaces separated by dense fibrous scars.
1. mIOPATIlIC PULMONARY FIBROSIS (Hamman-Rich syndrome, YIP, fibrosing
alveolitis) - the etiology of this disorder is unknown. It may
be immunologically mediated and mayor may not be associated
with coexistent collagen vascular disease. Within the same
lung, there are a variety of changes ranging from slight
inflammatory cell infiltrates of the alveolar wall with
minimal fibrosis to diffuse alveolar damage with extensive
fibrosis and alveolar collapse. It tends to occur in middle
aged males and generally is slowly progressive.
2. DESQUAMATIVE INTERSTITIAL PNEUMONITIS (DIP) - is characterized by the
accumulation of macrophages and desquamated epithelial cells
within the alveolar spaces. This possibly represents an early
stage of idiopathic pulmonary fibrosis however, these patients
are more likely to benefit from steroid therapy. Mononuclear
cells infiltrate the alveolar walls but there is little
fibrosis. Progressive dyspnea may lead to respiratory failure.

56
3. HYPERSENSITIVITY PNEUMONITIS (extrinsic allergic alveolitis)
results from immune-mediated alveolar damage caused by
inhalation of environmental organic dusts contaminated by
various antigens (animal protein, bacterial products, fungi,
etc). These include moldy hay (farmer's lung), cotton dust
(byssinosis), sugar cane dust (bagassosis), maple bark dust
(maple bark stripper's lung), etc. Acute attacks of dyspnea
and cough follow exposure in sensitized individuals and
continued exposure may lead to progressive respiratory
failure.
4. LYMPHOm INTERSTITIAL PNEUHONIA - is characterized by lymphocytic
infiltrates confined to the alveolar septa. It tends to be an
indolent or slowly progressive disease, but the incidence of
pulmonary lymphoma is increased in these patients.
5. BRONCHIOLITIS OBLITERANS AND ORGANIZING PNEUMONIA (BOOP) is
characterized by loose granulation tissue filling the
respiratory bronchioles, alveolar ducts, and alveolar sacs. It
is associated with diffuse alveolar damage and mild
interstitial fibrosis. The disease can have a fulminant
course, but patients may benefit from steroid therapy.
XI. RESPIRATORY TRACT NEOPLASMS

A. -comprises over 90% of the primary malignant


BRONCHOGENIC CARCIIDfA
tumors of the lung. Overall, bronchogenic carcinoma is the leading
cause of cancer deaths in the u.s. - both male and female. With the
exception of small cell undifferentiated carcinoma, if discovered
early, surgery may be effective but overall the 5-year survival is
poor. Although various morphologic subtypes of bronchogenic
carcinoma are recognized, it is not uncommon to see a mixture of
different elements within these neoplasms. Etiologies include
cigarette smoking (probably the single most important etiologic
factor), occupational exposure, environmental pollution, and
pulmonary scars. The danger from smoking is proportional to the
number of cigarettes smoked daily, duration of the habit, and the
tendency to inhale and is cumulative over time. Most bronchogenic
carcinomas begin in the central, hilar regions of the lung. Pre-
neoplastic changes create a small elevated plaque-like lesion on the
mucosal surface of the bronchus. Once neoplastic, the tumor may
protrude into the lumen of the bronchus (with possible obstruction),
penetrate rapidly into the underlying tissue, or grow slowly as an
endophytic mass. At the time of discovery, most have metastasized to
regional lymph nodes and may have infiltrated surrounding
mediastinal structures. They tend to metastasize to distant organs
early in their development by both lymphatic and hematogenous
routes.
1. is primarily a central lesion and is the
SQUAMOUS CELL CARCINOHA -
most closely associated with smoking and preceding dysplastic
squamous metaplasia of the bronchial epithelium. It varies
from well-differentiated to poorly differentiated and tends to
infiltrate locally before metastasizing. A small percentage
may cavitate. The majority of patients present with signs and
symptoms attributable to bronchial obstruction (atelectasis,
pneumonia and abscess formation).
2. ADENOCARCIIDfA - approximately 75% are peripheral lesions, grow
slowly, and remain clinically silent until signs and symptoms
of definite metastases appear. They occur in equal frequency
in men and women and are not as closely related to cigarette
smoking. The histologic patterns vary but most can be
demonstrated to contain mucus-secreting cells. Rarely, they
may be associated with preexisting fibrous pulmonary scars.
3. SHALL CELL UNDIFFERENTIATED (OAT CELL) CARCINOHA - occurring
predominantly in men, this is almost always a central lesion.
It too is associated with smoking and progresses rapidly with
wide dissemination. The cells are relatively small with little
cytoplasm and have no distinctive architectural growth

57
pattern. They may arise from the neuroendocrine Kulchitsky
cells, and these tumors are notorious for producing hormone-
like substances (ADR, ACTH, gonadotropins, etc).
4. LARGE CELL UHDIFFEREtrrIATED CARCINOHA - is comprised of large
pleomorphic undifferentiated cells and may represent
undifferentiated squamous cell carcinomas or adenocarcinomas.
Clinically, when symptomatic, the usual presentation consists of
chronic cough (often with hemoptysis), chest pain, anorexia and
weight loss, and dyspnea. X-ray almost always shows an abnormal
mass. Intrathoracic manifestations may include endobronchial
obstruction with secondary atelectasis, pneumonitis, abscess, or
bronchiectasis; superior vena cava obstruction; extension to the
pleura and pleuritis with or without effusion; extension to hilar
and mediastinal lymph nodes; and hilar extension with involvement of
the phrenic and recurrent laryngeal nerves causing paralyzed
diaphragm and a paralyzed vocal cord. Extrathoracic manifestations
are generally the result of widespread metastases or systemic
syndromes resulting from ectopic hormone production by the tumors.
B. BRONgtIOLOALVEOLAR CARCINOHA apparently arises from bronchiolar
epithelium and characteristically grows as cuboidal or columnar
cells growing along alveolar septa. They mayor may not be mucin
secreting. It may present as a peripheral mass lesion or it may
present as irregular nodules scattered throughout one or both lungs
simulating diffuse interstitial pneumonia. Patients typically
present with cough and chest pain and are often hypoxemic. Although
metastases are late occurrences, the overall survival is about 25%.
C. BRONCHIAL CARCINOm TUMOR - tends to occur at a younger age than other
malignant neoplasms of the lung, may arise from the neuroendocrine
Kulchitsky cells, resembles intestinal carcinoid tumors and, like
those tumors, can secrete serotonin. Though originally called
adenomas, they are not truly benign, but are slow growing and
locally invasive. Occurring centrally, they tend to extend into both
the lumen and the bronchial wall in a dumbbell shape. The surface of
the endobronchial part is usually covered by intact mucosa.
Presenting symptoms may be related to the secretion of vasoactive
amines or to bronchial obstruction due to the occluding mass.
Distant metastases are unusual and the overall prognosis is good.
D. METASTATIC TUMORS - are more common than primary neoplasms and tend to
present as multiple, bilateral peripheral lesions.
XII. DISORDERS OF THE PI.EURA AND PIJEURAL CAVITY

A. PNE~O~ - refers to the presence of air in the pleural cavity.


This may occur spontaneously as a complication of other pulmonary
disease (emphysema, abscess, tuberculosis, carcinoma) or as a result
of traumatic damage to the lung or chest wall. Clinically, it may be
asymptomatic or present as sudden onset of pleuritic pain, dyspnea,
and tachypnea.
1. CLOSED PNEUMOTHORAX - there is no free movement of air into or
out of the pleural cavity during respiration. The air in the
cavity will gradually be absorbed and the lung will re-expand.
2. OPEN PNEUMOTHORAX - air is able to move freely into and out of
the pleural space during respiration. The lung will remain
collapsed until a negative pressure can be re-established.
3. TENSION PNEUMOTHO~ air enters the pleural space during
inspiration but does not escape during expiration. As a
result, the increasing pressures in the pleural cavity will
force the mediastinum toward the opposite side causing
distortion and kinking of the great veins and resulting in
cardiac and respiratory dysfunction. This can rapidly lead to
death unless the increased pressure is released.

58
B. PLEURAL EFFUSIONS - predisposing factors include any general condition
with sodium or protein imbalance, increased pressure in pulmonary
capillaries, increased permeability of pleural capillaries, and
decreased pleural lymphatic drainage. The major causes of an exudate
in the pleural cavity include malignant disease, tuberculosis,
pulmonary infarction, and pneumonia. The most likely cause of an
exudative effusion in any patient over the age of 40 years (with no
history of febrile illness, no pain, and negative tuberculin test)
is cancer. The most likely cause for an exudative pleural effusion
in a person under the age of 40 years is tuberculosis.
c. I'fALIGNANI tiESQTHELItI1A - is a rare, aggressive tumor derived from
pleural mesothelial cells and associated with a history of prior
exposure to asbestos. The tumor tends to spread over the surface of
the lung but can invade the pulmonary parenchyma and metastasize
widely.

59
HEMATOPOXETXC/~YMPHORETXCULAR
SYSTEMS

BASIC LABORATORY HEMATOLOGY


1. AUTOMATED CELL COUNTERS Automated (as opposed to manual)
differential counts have been shown to be a reliable tool for detecting
abnormal hematologic results. All automated cell counters are basically
particle counters, using preprogrammed information to classify particles
as wbc, rbc, or platelets. They reliably count and report abnormal numbers
of normal cells, and very reliably flag the presence of abnormal cells
although it does not identify them.
A. BASICPARAMETERS - include directly measured WBC, RBC, Hgb, PIt, MCV
(rbc size), MPV (pIt size) and calculated HCT, MCH, MCHC, ROW, and
WBC parameters. Because these instruments are particle counters,
anything that alters size, shape, or refractive index can produce
spurious re.sults. Platelet clumps may result in false thrombocyto-
penia and false leukocytosis; very microcytic rbc or schistocytes
may indicate a false thrombocytosis; nucleated RBC, platelet clumps,
and red cells resistant to lysing (i.e. sickle cells) may simulate
leukocytosis; and hyperlipemia or the presence of a warm and/or cold
auto-agglutinin may cause an elevated MCV, MCH, and MCHC.
B• HINTPE IJI)ICES
1. IICV (tEAN CORPUSCULAR WLUtIE) = Hcr/1fBC - measures the average RBC
size. Increased MCV is generally due to folic acid deficiency,
Vitamin B12 deficiency, a substantial degree of
reticulocytosis, hepatic cirrhosis, and chronic alcoholism.
Decreased MCV is generally due to chronic iron deficiency,
alpha or beta -thalassemia (minor) , and anemia of chronic
disease.
2. ItCH (tIEAN CORPUSCULAR IIEKOGLClBIN) = HGB/1fBC - measures the quantity
of 11gb in the average RBC: Anemias may be classified as
hypocllromic, normochromic or hyperchromic.
3. tICIIC (tIEAN CORPUSCULAR HEIIOGUBIN CONCEN'I'RATION) = HGB/Hcr - estimates
the average concentration of Hgb in the average RBC.
4. IIDH (RED CELL DISI'RIBUTION HlD'I1I) - is the range about the mean red
cell size that correlates the degree of anisocytosis.
II. MANUAL DIFFERENnAL COUNTS (TERMS AND DEFINITIONS)

A. RED BJ.OCI) CELLS

1. HYPOCHIDIASIA - refers to less than normal amount of hemoglobin


staining. The central pallor is increased to greater than one
half of the cell diameter.
2. POLYCIIDIASIA - refers to presence of large, bluish staining
immature erythrocytes (reticulocytes). Regularly increased in
anemias with increased erythropoietic activity.
3. ANISOCYTOSIS - refers to a variation in size; I.e. microcytes «
6 microns diameter), macrocytes (> 9 microns diameter).
4. POIKILOCYTOSIS refers to variation in shape of the
erythrocytes. The size and hemoglobin content may vary
greatly. Poikilocytosis is non-specific, but is usually
associated with severe anemia with active erythroid
regeneration or extramedullary hematopoiesis.
5. ECHINDCYTE (BURR CELL) - refer to spiculated cells with short
equally spaced projections over the entire surface. Seen in
diseases associated with uremia, pyruvate-kinase deficiency,
post-transfusion, gastric carcinoma, bleeding peptic ulcers,
etc. They are produced by rupture of the cell membrane by
enlarged cytoplasmic vacuoles.

60
6. ACMnrn~ - refers to irregularly spiculated red cells with
projections of varying length and position. They are seen in
abetalipoproteinemia, alcoholic liver disease, post-
splenectomy, malabsorption states, etc.
7. SPHEROCYTE - refers to small spherical red cells with dense
hemoglobin content. They appear hyperchromatic because of
their spherical form and decreased surface to volume ratio.
They are found in hereditary spherocytosis, immune hemolytic
anemia, post-transfusion, fragmentation hemolysis, etc.
8. SOfISTOCYTE (fragmented cell) - refers to split red cells, often
half disk shape with two or three pointed extremities. They
are seen in microangiopathic hemolytic anemias (DIC, TIP),
vasculitis, renal graft rejection, prosthetic or pathological
heart valve hemolysis, severe burns, malignant hypertension,
etc. They are produced by transverse fracture of red cells
across filaments of fibrin.
9. ELLIPTOCYTE (OVAL~) - refers to red cells which vary from
slightly oval to pencil or cigar shapes in which the
hemoglobin appears concentrated at both ends of the cell. They
are especially prominent in thalassemia, iron deficiency
anemia, myelophthisic and megaloblastic anemias. These cells
are characteristic of in hereditary elliptocytosis, in which
elliptocytes may constitute as many as 95% of all
erythrocytes.
10. CODOCYTE (TARGET CELLS) refers to thin, flattened ce11s
revealing peripheral and central zones of hemoglobin with an
empty intermediate zone, giving the appearance of a target.
The target cell represents an excess of cell membrane in
relation to the amount of hemoglobin. They are seen in
obstructive liver disease, hemoglobinopathies (S,C),
thalassemia, iron deficiency, etc.
11. DACRYOCYTE ("TEAR DROP" CELLS) - refer to cells that have the shape
of a drop, usually microcytic. Frequently seen in
myelofibrosis and less frequently in myelophthisic anemia and
thalassemia.
12. ERYTHROCYTE INCLUSIONS

a. Basophilic stippling - pink cytoplasm stippled with


coarse blue granules resulting from pathologic
precipitation or aggregation of ribosomes. Seen in lead
poisoning, thalassemia, and states where biosynthesis of
hemoglobin is altered.
b. Howell-Jolly bodies - dark-violet spherical granules.
They represent nuclear fragments containing aberrant
chromosomes. They are pitted from the red cell (usually
reticulocyte) as they pass through the interendothelial
slits and sinuses of the spleen. They are
characteristically seen in splenectomized persons and in
those with hemolytic anemia, hyposplenism, or
megaloblastic anemia.
c. Pappenheimer bodies - on Wright's stain, coarse blue
granules (1-10) present in the periphery of the red cell
(whereas basophilic stippling is distributed
throughout). They stain positive for iron and EM shows
the iron to be present in lysosomes.
d. Heinz bodies - are precipitated, oxidized, and denatured
hemoglobin. They are seen with oxidant compound staining
and are rarely seen in Wright's stained smears. Normally
one per cell is seen. If more than five per cell are
seen, the patient has enzyme deficiencies, i.e. G-6-PD
or abnormal hemoglobin.
B. WHITE Buxm CELLS - granulocytic inclusions or variants include:
1.. TOXIC GRANULATION refers to large, dark purple staining
azurophilic granules seen in neutrophilic cells. Toxic
granules suggest infection and are often accompanied by other
cellular abnormalities.

61
2. DOHLE BOOIES - refer to pale blue staining areas in the
cytoplasm of neutrophils, occurring either singly or in
multiples, formed by parallel arrays of rough endoplasmic
reticulum. They can be found in a variety of infections,
burns, trauma, pregnancy, cancer and with certain cytotoxic
drugs. With infection, Dohle bodies are often accompanied by
toxic granulation and/or vacuolization, and are transient in
nature.
3. HYPERSEGtlENTATIOH - refers to neutrophils containing 6 or more
nuclear segments. Hypersegmented polys occur primarily in the
megaloblastic anemias.
4. HYPOSEGHENTATION - Pelger-Huet anomaly is an autosomal dominant
heterozygous condition in which nuclear segmentation of most
granulocytes is arrested at two lobes. Dumb-bell or spectacle-
shaped nuclei (pince-nez) with excessively clumped chromatin
are the two morphologic characteristics of Pelger-Huet granu-
locytes. Hyposegmented granulocytes can be acquired (pseudo-
Pelger-Huet-anomaly) in the course of chronic and acute
leukemias, myeloproliferative disorders, or following
chemotherapy.
5. ABNORMAL GRANULATION - refers to fusion of giant azurophilic
granules (Chediak-Higashi syndrome) or numerous, large
reddish-purple granulation in granulocytes, monocytes and
lymphocytes (Alder Reilly anomaly).
6. AUER RODS OR BOOIES - refer to reddish purple, elongated, needle-
like, rod-shaped or comma-shaped cytoplasmic inclusions that
occur singly or numerously in leukemic myeloblasts or
monoblasts. Occasionally these may be seen in cells beyond the
blast stage i.e. in progranulocytes of acute promyelocytic
leukemia.
III. HEMATOLOGIC STAINS FOR PERIPHERAL SMEARS AND/OR BONE MARROW

Routine stains include the standard Wrights or Giemsa stains. Other useful
stains include Prussian Blue (iron) for histiocytes and RBC precursors;
Sudan Black (SBB) for myelocytic serl.es; Myeloperoxidase (MPX) for
myelocytic series; Chloracetate Esterase (Leder) for neutrophilic series
and basophils; Alpha Napthol Acetate Esterase (ANAE) for monocytic series,
and T lymphocytes; Alpha Napthol Butyrate Esterase (NBE) for monocytic
series; Methyl Green Pyronine (MGP) for RNA:plasma cells).

COAGULATION DISORDERS

I. COAGUIJATION TESTS

A. BLEEDING TIME - is a gross in vivo test that measures the adequacy of


platelet function. An incision (10 mm. in length and 1 mm. in depth)
is made in the forearm and the blood is blotted with a piece of
filter paper every 30 seconds until the bleeding stops. Normal
bleeding time is approximately 6 minutes. Platelet count is usually
in the normal range.
B. MANUAL PLATELET COUNT - A blood sample is placed in a unipet device to
dissolve all the formed blood elements except platelets. The fluid
is then placed in a hemocytometer and the platelets are counted.
Normal platelet count is > 200,000/ml. Most platelet counts are now
automated and performed with the CBC and verified by the manual
technique when necessary.
C. ACfIVATED PARrIAL THRtI1BQPLASTIH TIME (APTI) - measures the intrins ic
coagulation pathway (factors XII, XI, IX, VIII, X, and V). This test
is useful monitoring heparin therapy and is a two stage test.
1. take patient's citrated platelet-poor
STAGE I (THE TEST SYSTEH~ -
plasma and add "dirt' (kaolin) which activates factor XII. Let
stand 3 - 5 minutes to allow the non-calcium dependent
coagulation factors (VIII, IX, XII) to be activated.

62
2. add calcium and cephalin (substitute
STAGE II (TIlE HARKER SYSTEH) -
for phospholipid of platelet membrane). Thrombin is produced
and a clot will form in about 35 seconds. If SDY of the stage
I factors are deficient, the clotting time will be increased.
D. 50/50 (MIXING) STUDIES FOR PIT - if PIT is prolonged, the patient I s serum
can be mixed 1:1 with normal plasma. If PIT (as measured above)
returns to normal, the patient is deficient in one of the factors of
the intrinsic pathway. If the PIT does not correct to normal, the
patient probably has circulating anticoagulants or inhibitors
(heparin, lupus, etc).
E. PROTHROttBIN TDIE (PT) - measures the extrinsic coagulation rathway
(factors II, VII, V, and X). Tissue thromboplastin (usual y from
lung or brain extract) is added to patient s citrated serum and
thrombin is formed with clot formation in about 13 seconds. This is
used to monitor coumarin and warfarin therapy since factor VII is
coumarin sensitive.
F. FIBRINOGEN LEVEL - a citrated plasma sample is treated with thrombin
until it coagulates. The coagulation time of the patient is compared
to a standard curve to determine the fibrinogen level. This is, at
best, only a semi-quantitative assay. Normal levels range between
200 - 500 mgjdl. If fibrinogen levels are extremely low « 100), PT
and PIT may also be prolonged.
G. ADP PLATELET AGGREGATION - ADP is added to citrated platelet-rich plasma
which is placed in a spectrophotometer and the platelet aggregation
is measured by the degree of change in turbidity over time.
H. RISTOCETIN AGGREGATION TEST - measures presence or activity of Von
Willebrand's Factor. Ristocetin (an antibiotic) uncovers the
platelet receptors for VWF and, in the presence of VWF, platelets
will aggregate.
I. THIOIBIN TII'IE (TI) - measures the third stage of coagulation. Exogenous
thrombin is added to citrated plasma and the time to clot formation
is measured. Normal ranges depend of the concentration of thrombin
used in the test but generally is about 20-30 seconds. It is
increased in deficiency or abnormality _of fibrinogen, presence of
fibrin split products (FSP), or presence of heparin.
II. BLEEDING DISORDERS

A. in these disorders, bleeding is often


COAGULATION FACTOR ABNORMALITIES -
severe with hematomas and ecchymoses developing after minor trauma.
1. CONGENITAL usually characterized by single factor
deficiencies.
a. Sex-linked - the defect resides on the X chromosome and
therefore is predominantly clinically expressed in
males. Depending on the degree of activity, they may be
classified as mild, moderate, or severe.
(1) Hemophilia A (Factor VIII deficiency)
approximately 25% of patients do not have a
family history which indicates a high rate of
spontaneous mutation. VIII-C is generally
decreased while VIII-VWF remains relatively
normal. The degree of deficiency varies but> 50%
have severe disease. Bleeding into joint spaces
(hemarthrosis) will, with time, lead to a
crippling arthropathy. Patients will have normal
bleeding time, normal platelet count, normal PT,
and increased APIT.
(2) Hemophilia B (Christmas disease. Factor IX
deficiency) - has same inheritance pattern and
similar symptoms but is about 20% as common. Like
hemophilia A, patients will have normal bleeding
time, normal platelet count, normal PT, and
increased APTT.

63
b. Autosomal Dominant
(1) Von Willebrand I s disease - incidence of 1 in
30,000 and usually diagnosed in children or young
adults. Characterized by easy bruisability and
bleeding with little or no bleeding into joints.
Deficiency in ability to release synthesized
VIII-VWF. For unknown reasons, VIII-C levels are
also decreased. Clinically, patients will have
increased bleeding time, normal platelet count,
normal PT, and normal or increased APTT.
c. Autosomal Recessive - deficiencies of each of the
coagulation factors have been described, but most are
relatively rare.
2. Aawlmm usually characterized by multiple factor
deficiencies and clotting abnormalities.
a. Vitamin K deficiency - hepatic synthesis of factors II,
VII, IX, and X are dependent on the presence of Vitamin
K - a fat soluble vitamin ingested in the diet and
synthesized by intestinal flora. Deficiencies may occur
in cases of malnutrition, malabsorption, biliary
obstruction, or drug therapy. Clinically, patients will
show normal bleeding time, normal platelet count,
increased PT, and normal or increased APTT.
b. Severe liver disease - impairs the hepatic synthesis of
II, V, VII, IX, X, and fibrinogen. With chronic disease,
patients will show normal bleeding time, normal platelet
count, and increased PT and APTT.
B. pu~~ ~omuLnIB

1. THIOIBOCYTOPENIA refers to a decrease in the number of


platelets and is generally - characterized by bleeding from
small vessels into the skin, GI tract, mucous membranes,
urinary tract, and brain. Platelet counts below 50,OOO/ml may
impede coagulation while levels below 20,000/ml may result in
spontaneous hemorrhage. Clinically patients will have normal
to increased bleeding time, decreased platelet count, normal
PT, and normal APTT.
a. Decreased production - diffuse bone marrow disease
(aplastic anemia, primary or metastatic tumor, etc.),
megakaryocyte disorder, etc.
b. Increased utilization (DIC)
c. Increased destruction
(1) Isoimmune thrombocytopenia
(a) Neonatal Similar pathogenesis as
hemolytic disease of the newborn. A
sensitized platelet antigen (PLAl) negative
mother will produce IgG antibody to the
platelet antigen of the fetus. IgG crosses
the placental membrane and destroys the
fetal platelets.
(b) Post-transfusion - A sensitized platelet
antigen negative patient given platelet
antigen positive platelets will destroy not
only the transfused platelets but also his
own.

64
(2) Idiopathic thrombocytopenic purpura (ITP)
(a) ~ - self limiting disease which usually
affects children following a viral
infection. Platelets are probably destroyed
as "innocent bystanders".
(b) Chronic a disease of adults (often
premenopausal women) which may be assoc-
iatedwith other autoimmune diseases. Auto-
antibodies (platelet associated immuno-
globulins) produced in the spleen are
directed against the patient's own
platelets. Opsonized platelets are then
removed by the reticuloendothelial system,
primarily the spleen. There are decreased
numbers of platelets in the peripheral
blood but an increased number of megakaryo-
cytes in the bone marrow. Patients usually
give a history of easy bruising and
bleeding after minor trauma. Treated with
steroids and often splenectomy.
(3) Thrombotic thrombocytopenic purpura (TTP) - This
disease results from abnormal aggregation of
platelets which obstruct the microcirculation
leading to a decreased platelet count, micro-
angiopathic hemolytic anemia, fever, transient
neurologic deficits, and renal failure.
(4) Drug reaction
(5) Mechanical destruction from cardiac or other
prostheses, roughened endothelium, etc.
(6) Hypersplenism
2. (increased bleeding time, normal platelet
FUNCTIONAL ABNORtlALITIES
count, normal PT, normal APTT)
a. Congenital
(1) Defective adhesion - Bernard-Soulier syndrome
(autosomal recessive) is a defect in the platelet
membrane glycoprotein, GPlb.
(2) Defective aggregation - Thrombasthenia (autosomal
recessive) is a defect in the platelet membrane
glycoproteins GPIIb and GPIIla so that no
fibrinogen linking of platelets can occur.
Incidence is about 1 in 100,000 and usually
diagnosed in childhood. Characterized by easy
bleeding and no clot retraction.
b. Acquired
(1) Aspirin ingestion - aspirin inhibits cyclooxy-
genase which suppresses prostaglandin (throm-
boxane A2) synthesis. 72 hour effect.
(2) Thrombocythemia occasionally seen with
myeloproliferative disorders. Platelet count may
exceed 3,000,000/ml but they may be functionally
abnormal. These patients have problems with both
thrombosis and hemorrhage.
(3) Uremia
C. (increased vascular fragility) - these disorders
Y§SJiL ABHOI!IIALITIES
are generally manifested by petechial hemorrhages of the skin or
mucous membranes; characterized by normal bleeding times, normal
pl~telet counts, normal PT, normal APTT; and usually are not severe
life threatening situations.

65
1. CONGENITAL

a. Ehlers-Danlos syndrome (autosomal dominant) - causes


impaired synthesis of the collagenous vascular support.
b. Hereditary hemorrhagic telangiectasia (autosomal
dominant) - produces variceal dilatation of small
vessels which are fragile. Patients usually present with
epistaxis or GI bleeding.
2. ACQUIRED

a. Hypersensitivity vasculitis
(1) Drug reactions - antibodies produced against drug
antigens result in immune complex deposits in
vessel walls.
(2) Henoch-Schonlein purpura is a generalized
hypersensitivity vasculitis of unknown cause
which results in purpura, colicky abdominal pain,
polyarthralgias, and acute glomerulonephritis.
b. Scurvy (Vitamin C deficiency) causes impaired
synthesis of the vascular collagenous support.

ANEMIA
1. ETIOLOGIC CLASSIFICATION - anemia is a decrease in hemoglobin concentra-
tion. It is a symptom of an underlying disease and, as such, requires
further investigation as to its etiology.
A. IMPAIRED RBC PRODUCfION (ineffective erythropoiesis)
1. ALTERATION OF STEH CELLS (BONE tlARRON FAILURE)

a. Myelophthisic infiltrations
b. Hypoplasia due to aging, myelofibrosis.
c. Toxic suppression/aplasia - due to benzene, radio-
activity, chemothera~y.
d. Congenital - Fanconi s (autosomal recessive), Diamond-
B1ackfan (autosomal recessive)
e. Systemic disease - renal failure, neoplasia, septicemia,
rheumatoid/collagen disease, chronic liver disease,
hypothyroidism, etc.
2. ALTERATION OF ERYTHROCYTE MATURATION

a. Defective DNA synthesis - characterized by macro-ovalo-


cytes (MeV > 100), hypersegmented neutrophils, and
elevated LDH and bilirubin.
(1) B12 deficiency - B12 combines with intrinsic factor
in the stomach, is absorbed in ileum, and is
necessary for nucleic acid synthesis. May be
caused by decreased intake (rare) or decreased
absorption. Symptoms may include sallow skin
(grayish green/yellow), enlarged spleen,
neurologic changes (personality, tingling,
incoordination, etc).
(2) Folate deficiency - folate is absorbed in small
bowel and is also necessary for nucleic acid
synthesis. Deficiencies may be caused by
decreased intake (common), increased demand,
impaired absorption, or inhibition of folate
metabolism (drugs, alcohol). Symptoms are the
same as B12 but without the neurologic symptoms.

66
b. Defectiye Hgb synthesis
(1) Iron deficiency iron is absorbed in the
duodenum and used in heme synthesis. Deficiencies
result from chronic blood loss, decreased intake,
increased demand, or decreased absorption. The
serum concentration of iron is decreased while
the concentration of transferrin (total iron
binding capacity or TIBC) is increased. The
percent saturation (ratio of [Fe)/[TIBC) x 100;
normal 20-50%) is decreased. Symptoms include
3eneral fatigue, shortness of breath, spoon nails
(koilonychia), smooth sore tongue.
(2) Thalassemia - genetic defect in production of the
globin portion of Hgb. A deficiency of beta
chains characterizes beta thalassemia and a
deficiency of alpha chains produces alpha
thalassemia. The disease presents as a spectrum
of disorders ranging from asymptomatic changes in
the CBC to severe anemia in beta thalassemia
(Cooley's anemia) with early death. The test of
choice for identifying the anemia as thalassemic
is hemoglobin electrophoresis.
B. INCREASED DESTRUCTION OF RBC (hemolytic anemia)
1. Itn'RACORPUSCULAR (mostly genetic causes)
a. Membrane disorders
(1) Hereditary spherocytosis (autosomal dominant) -
is a disorder with defective synthesis of RBC
membrane that leads to spherical rather than
biconcave red cells. This structural abnormality
is detected by the spleen which enlarges as it
removes the abnormal cells from the circulation.
Splenectomy will alleviate the symptoms but does
not correct the underlying problem. Osmotic
fragility test is useful diagnostic test.
(2) Hereditary elliptocytosis (autosomal dominant)
b. Enzyme deficiencies
(1) Glucose-6-phosphate dehydrogenase (G6PD) (X-
linked inheritance) - results in loss of protec-
tion against chemical oxidation and leads to
hemolysis when certain drugs are used. Heinz
bodies (denatured Hgb) may be present in the red
cells.
(2) Pyruvate kinase deficiency (autosomal recessive)
- inability to maintain normal ATP levels results
in membrane abnormalities.
c. Abnormal hemoglobin (hemoglobinopathies) - abnormal Hgb
structure due to altered amino acid sequence in globin
molecule.
(1) Sickle cell anemia (autosomal recessive) - under
conditions of low oxygen tension, the abnormal
structure of the Hgb causes the cell to "sickle".
These cells can obstruct small vessels (esp. bone
and spleen) to produce abdominal and bone pain.
They are also more susceptible to destruction
because of their abnormal shape.
(2) Other hemoglobinopathies - Hgb C, Hgb SC, etc.
May see anisocytosis and Howell-Jolly bodies.

67
2. (mostly acquired causes) - because the mechanism
EXTRACORPUSCULAR
of production of these anemias is increased destruction of
circulating red cells, they will almost always show polychrom-
asia and basophilic stippling, both signs of reticulocytosis.
a. Antibody mediated - transfusion reactions, drugs,
neoplasia, autoimmune diseases, etc. Direct Coomb's
positivity.
b. Mechanical destruction (microangiopathic) DIC,
prosthetic valves. Produces schistocytes .
. c. Infections - malaria, babesia, etc.
d. Hypersplenism - increased sequestration and destruction
of RBC by the spleen.
C. ABNORtlAL BUXIJ LOSS - acute or chronic hemorrhage.
II. MORPHOLOGIC CLASSIFICATION (according to size and Hgb content)

MCV MCH MCHC


Size Hgb Content um3 uug % Conditions

Macrocytic Normochromic 103-\60 37-55 31-36 Pernicious anemia and other megaloblastic anemias, chronic
liver disease

Normocytic Normochromic 82-102 27-34 31-36 Acute blood loss, hemolytic anemias, aplastic anemias,
myelophthisic anemias, hypo proliferative anemias

Microcytic Normochromic 70-81 27 -34 31-36 I nflammatory and neoplastic diseases

Microcytic Hypochromic 50-8\ \5-26 25-30 Iron deficiency, thalassemia, lead poisoning, sideroblastic
anemias

ACUTE LEUKEMIA
Acute leukemias are characterized by monoclonal proliferation of immature "blast"
cells (large cells with high N/C ratio and variable numbers of nucleoli) that
fail to participate in the normal maturational process. As the cells accumulate
in number (more a function of their inability to mature rather than uncontrolled
proliferation), they spillover into the peripheral blood and may extensively
infiltrate many organ systems including the lymph nodes, liver, kidneys, spleen
and heart. In spite of this, morbidity and death is rarely caused directly by the
accumulation and dissemination of leukemic cells but rather by the secondary
"suppression" and decrease in the numbers of mature granulocytes, erythrocytes
and platelets which leads to the classic symptoms of acute leukemia - anemia,
infections, and hemorrhage. Acute leukemias often have an abrupt onset and are
rapidly fatal if untreated. They may initially be recognized by the presence of
blast cells in the peripheral blood, but the diagnosis rests on increased numbers
of blast cells (>30%) in the bone marrow.
I. ACUTE NON-LYMPHOBLASTIC LElJKEMIAS/ACUfE MYEI~OGENOUS LEUKEMIAS -
make up approximately 60% of the acute leukemias. They arise from the
myeloid stem cell line (and include the erythrocytic and megakaryocytic
lines) and tend to arise most frequently in young to middle aged adults.
Patients may initially present with pallor and petechiae. Lymphadenopathy
and splenomegaly mayor may not be present and there usually is no fever
unless secondary infection is present.
A. based on the cell line and the degree
MORPHOLOGIC (FAB) CLASSIFICATION -
of maturation, ANLL/AML has been subdivided into seven SUbtypes.
With two exceptions (M6 and M7), these cells stain positively with
myeloperoxidase (HP) and Sudan Black B (SBB). Further
differentiation can be made with the use of Nonspecific Esterase
(NSE) , Chloracetate Esterase (CLE), and periodic acid Schiff (PAS)
stains.

68
1. HI (ACUTE MYELOBLASTIC LEUKEHIA HInlotrr MATURATION) - 90% or more of
nonerythroid cells in the bone marrow are blasts. (MP+, SBB+,
NSE-, CLE+).
2. H2 (ACUTE MYELOBLASTIC LEUKEHIA HInI MATURATION) - 30 to 89% of
nonerythroid cells in the bone marrow are blasts. (MP+, SBB+,
NSE-, CLE+).
3. H3 (ACUTE PROtIYELOCYTIC LEUJ<EHIA) - abnormal promyelocytes with
heavy primary granulation of the cytoplasm are present in the
bone marrow. Some may contain bundles of Auer rods (faggot
cells). This is frequently associated with DIC. (MP+, SBB+,
NSE-, CLE+).
4. H4 (ACUTE MYELOtIONOCYTIC LEUKEHIA) - 20 to 80% of nonerythroid cells
in the bone marrow must be of monocytic lineage, usually
promonocytes and monocytes. (MP+, SBB+, NSE+, CLE+).
5. H5 (ACUTE HONOCYTIC LEUKEMIA) - 80% or more of nonerythroid cells
in the bone marrow must be monoblasts, promonocytes, or mono-
cytes and there may be erythrophagocytosis by the leukemic
monocytes. This form differs clinically from other forms in
that the gums are hypertrophied; oral and anorectal ulcers are
frequently noted; infiltrations of the skin are common (chlor-
oma); and lymphadenopathy and splenomegaly is common. (MP+,
SBB+, NSE+, CLE-).
6. H6 (ERYTIfROLEUKEHIA, 01 GUGLIELMO IS SYNDIKl'IE) - these derive from the
erythrocytic line and do not stain with MP and SBB. Over half
of all nucleated cells in the bone marrow are erythroblasts
and greater than 30% of nonerythroid cells are blasts. The
erythrocytic series shows evidence of dyserythropoiesis
(irregular, lobated, or fragmented nuclei) and are PAS
(periodic acid Schiff) positive.
7. H7 (ACUTE HEGAKARYOBLASTIC LEUJ<EHIA) these derive from the
megakaryocytic line and do not stain with MP and SBB.
Monoclonal platelet antibodies and antibodies to Factor VIII
will stain megakaryoblasts and their abnormal platelet
derivatives. This has an association with trisomy 21.
B. shows a normocytic, normochromic anemia (often
PERIPHERAL BLOOO SHEAR -
severe) with a decreased reticulocyte count. Although the white
blood count may be elevated, it is usually normal or depressed.
Myeloblasts are generally present and they may contain abnormal
intracytoplasmic lysosomal structures having a rod-like appearance
(Auer rods). Platelets usually low.
C. PROGNOSIS - remission can be achieved in the majority of patients, but
most will relapse within a relatively short time. The prognosis is
least favorable in infants and the elderly or when the disease
arises as a complication of chemotherapy/irradiation for other
neoplasms.
II. ACUTE LYMPHOCYTIC (ISMPHOBLASTIC) I,EUKEMIAS - comprise about 40% of
the acute leukemias. They arise from the lymphocytic cell series and are
the most frequent malignancy of childhood with a peak incidence at 3-4
years of age, although there is also another rise in frequency from middle
age onward. These account for 50% of cancer deaths in children under the
age of 15. Patients may present with symptoms of bone marrow failure
(pallor, lethargy, abnormal bleeding), bone and joint pain, peripheral
lymphadenopathy and splenomegaly. Although leukemic involvement of the CNS
can occur with any leukemia, it is more frequently seen with ALL. CNS
manifestations are not usually present initially, but since most
chemotherapeutic drugs cannot cross the blood-brain barrier, CNS symptoms
may eventuate even though the disease is controlled in the rest of the
body. CSF usually shows increased pressure and cellularity with decreased
glucose and normal protein. Symptoms may be due to increased pressure
(headache, vomiting, papilledema, lethargy), ocular disturbances (blurred
vision, strabismus, diplopia), cranial and peripheral nerve palsies,
psychic.and auditory disturbances, and diabetes insipidus.

69
A. tIORPHOLOGIC (FAa) CLASSIFICATION
1. L1 - the majority of cells are small with cytoplasm comprising
less than 10% of cell area, regular nuclear outline, and small
or inconspicuous nucleoli.
2. L2 - heterogeneous population with larger cells, nuclear
clefts, and conspicuous nucleoli.
3. L3 large, homogeneous cell size, prominent vesicular
nucleoli, moderately abundant cytoplasm with deep basophilia;
and prominent vacuolization.
B. DllUNQLOGIC CLASSIFICATION - most forms of ALL are comprised of cells
that stain positively for terminal deoxynucleotidyl transferase
(TdT) , a marker for primitive lymphoid cells. A majority also have
the common acute lymphoblastic leukemia antigen (CALLA) but lack T-
cell antigens (T) or surface Ig (SIg). Some may, however, contain
cytoplasmic Ig (CIg) or immunoglobulin gene rearrangements which
identify them as being derived from B-cell lineage. 10% will consist
of cells that have definite T-cell markers while a lesser number
show surface 19 characteristic of more mature B-cells.
1. cotItIONALL (TdT+, CALLA+, Cig-) - accounts for 50-70% of
childhood ALL cases. Approximately 75% of cases demonstrate L1
morphology. Significant lymphadenopathy or hepatosplenomegaly
is usually not seen at presentation. This form responds
favorably to chemotherapy. Typical sites of relapse include
CNS and gonads.
2. PRE-T-CELL ALL AND T-CELL ALL (TdT+, cALLA-, T+) - approximately
15-20% of all ALL cases. Usually occurs in the adolescent or
young adult with male predominance; presents as high WBC count
> 100,000, mediastinal mass, and marked hepatosplenomegaly.
CNS is usually involved at time of diagnosis. Resistant to
chemotherapy and carries a poor prognosis.
3. UNDIFFERENTIATED ALL (TdT+, cALLA-, T-) - is more common in adults
and shows a poor response to_chemotherapy.
4. PRE B-CELL ALL (TdT+, cALLA+, C1g+) - approximately 15-20% of
ALL cases, these also responds favorably to chemotherapy.
5. B-CELL ALL (TdT-, S1g+) ~ make up < 5% of all ALL cases. They
have L3 morphology (Burkitt type) and are thought to represent
acute leukemic presentation of Burkitt's lymphoma.
C. PERIPHERAL BLOCI) shows normocytic, normochromic anemia and
thrombocytopenia. WBC may be low, normal, or high. I f high, immature
lymphoid cells (lymphoblasts) will be present.
D. P~OSIS - overall, remissions can be achieved in large majority of
children. Disease free states are maintained in about 50% at five
years and cures are possible. Prognosis is poorer for adults and
patients with ALL showing T-cell markers.

MYELODYSPLASTIC SYNDROMES (MDS)


These are a group of related diseases characterized by unexplained failure of
marrow to produce myeloid cells at rate appropriate to meet needs, despite a
normocellular or hypercellular marrow. The diagnosis begins with recognition of
unexplained single or multiple cytopenias (iron deficiency, B12 or folate defic-
iency, pyridoxine deficiency, etc. must be excluded). Morphologic evidence of
dyspoiesis may be helpful in confirming the diagnosis of MDS, but these changes
may be very subtle. Increased numbers of blasts may be seen in the bone marrow
but is always less than the 30% required for a diagnosis of acute leukemia.
Confusion frequently arises due to the myriad synonyms, partial synonyms, and
related terms for myelodysplastic syndrome and/or its subtypes. These include
pre-leukemia; hemopoietic dysplasia; smoldering (myeloid) leukemia; oligoleuk-
emia; subacute myeloid leukemia; panmyelosis; dysmyelopoietic syndrome; etc.

70
I. CUNICAL FEATIJRES - The incidence of MDS approximates that of acute
leukemia. It is most frequently seen in elderly males and is often
underdiagnosed. Younger patients, however, may also develop MDS especially
after chemotherapy/radiation therapy for other malignancies. The disorder
often presents as unexplained cytopenias with paradoxical normoce11u1ar or
hypercellu1ar marrow. It frequently progresses to another form of MDS
and/or acute leukemia and is often fatal.
II. PATHOLOGIC FEATURES - MDS is generally characterized by dyspoiesis
(abnormalities of erythropoiesis, granulopoiesis, and megakaryopoiesis).
A. DYSERYTHROPOIESIS

1. PERIPHERAL BLOOD - normochromic, normocytic anemia; decreased


absolute reticulocyte count; nucleated RBCs and dual RBC
population; anisocytosis; poikilocytosis; macro-ova10cytes,
polychromasia, and basophilic stippling.
2. BONE HARROW - normocellular or hypercellular; erythroblasts may
demonstrate megaloblastic or mega10b1astoid changes, nuclear
budding, karyorrhexis , multiple nuclei, internuclear bridging;
vacuolated basophilic/polychromatophilic normoblasts; ring
sideroblasts; and PAS positivity in RBC precursors.
B• DYSHYELOPOIESIS

1. PERIPHERAL BLOOD - neutropenia; monocytosis; pseudo-Pe1ger-Huet;


hypogranular neutrophils; immature granulocytes; and decreased
myeloperoxidase and alkaline phosphatase activity.
2. BONE HARROW - myeloid hyperplasia with partial maturation
arrest; diminished or absent secondary granules; mye1operoxi-
dase deficient neutrophi1s; abnormal granulation; irregular
cytoplasmic basophilia; and increased blasts.
C. DYSMEGAKARYOCYTOPOIESIS

1. PERIPHERAL BLOOD -thrombocytopenia;- micromegakaryocytes; large


atypical platelets, abnormal granulation; and vacuolization of
platelets.
2. BONE HARROW - micromegakaryocytes; multiple small nuclei; large
mononuclear megakaryocytes; abnormal granulation; and nuclear
dysplasia.
III. CIJASSIFICATION (FAB)

PB Blasts BM Blasts
Refractory Anemia (RA) < 1% < 5%

RA with Ring Slderob1asts (RARS) < 1% < 5%


(siderob1astic anemia)
RA with Excess Blasts (RAEB) 1-5% 5-20%
Chronic Mye1omonocytic Leukemia (CMML) < 5% < 20%

RAEB in Transformation (RAEB-T) > 5% 20-30%

71
MYELOPROLIFERATIVE DISORDERS
These represent a group of diseases characterized by overgrowth of one or more
hematologic cell lines in the bone marrow and involve granulocytes (chronic
myelogenous leukemia), erythrocytes (polycythemiavera), megakaryocytes/platelets
(essential thrombocythemia), or fibroblasts (myelofibrosis).
I. CHRONIC MYELOGENOUS (GRANULOCYTIC) LEUKEMIA (CML) - is the second
most common leukemia and generally arises in middle aged adults. There
appears to be an increased proliferation of stem cells which, unlike acute
leukemias, proceed in the maturational process to produce excessive
numbers of "mature" granulocytes (as well as metamyelocytes and occasional
myeloblasts) in the peripheral blood. All stages of maturation are present
in the bone marrow. This proliferation also involves basophils and
eosinophils and increased numbers of these cells may be present in the
peripheral blood. In 90% of patients, an acquired chromosomal abnormality
(Philadelphia chromosome - PhI) can be identified in erythroid, megakaryo-
cytic, and granulocytic cells suggesting a multipotent myeloid stem cell
or perhaps a pluripotent stem cell origin. The chromosomal abnormality is
usually a reciprocal translocation from the long arm of chromosome 22 to
the long arm of chromosome 9. Clinically, patients present with non-
specific constitutional symptoms (weakness, weight loss, fatigue, etc) or
excessive bleeding or bruising after minor trauma. WBC usually exceeds
100,000 and thrombocytosis is often present. Although the total number of
granulocytes is increased, they are functionally impaired as is evidenced
by the lack of leukocyte alkaline phosphatase (LAP). The disease is slowly
progressive and may cause massive splenomegaly with the spleen reaching
weights in excess of 5000 gm. Hepatomegaly is generally not as striking
but leukemic cells may be seen in the sinusoids. The median survival is 3-
4 years from the time of diaP,0sis and the terminal course of many
patients is the development of blast crisis" in which there is a trans-
formation into an acute leukemia phase.
II. POLYCYTHEMIA VERA - also takes its origin from the multipotent myeloid
stem cell but instead of granulocytic dominance, there is erythrocytic
dominance resulting in an increase in the red cell mass and causing an
increase in the blood volume and viscosity. The tissues subsequently
become congested and plethoric with complications due to hypertension
(headache, dizziness, heart failure), thrombosis (infarction of spleen,
kidneys, heart, brain), or hemorrhage (hematemesis, melena). The bone
marrow is hypercellular with erythroid and megakaryocytic hyperplasia and,
though less striking, granulocytic hyperplasia. This is reflected in the
peripheral blood by increased numbers of red cells, white cells, and
platelets. Repeated phlebotomy may prolong survival and, if patients live
long enough, some will progress to develop bone marrow fibrosis and extra-
medullary hematopoiesis (myeloid metaplasia) resulting in splenomegaly.
III. ESSENTIAL THROMBOCYTHEMIA - is a rare disorder characterized by
proliferation of megakaryocytes with tremendous increases in circulating
platelets that are morphologically abnormal. Peripheral platelet count is
usually over 1 million. Splenomegaly, hemorrhage (especially mucosal) and
thromboses are common.
IV. MYELOFmROSIS (myeloid metaplasia with myelofibrosis) - occurs primarily
in middle-aged adults (50-60 years old). It is characterized by fibrosis
of the bone marrow and shifting of the proliferation of myeloid elements
to the spleen. This may develop as an extension of CML or PV, but it may
also occur insidiously (agnogenic myeloid metaplasia). The spleen is
enlarged and may show areas of infarct. The liver may also be enlarged and
show foci of hematopoiesis. The marrow is fibrotic and h¥'pocellular and
the peripheral blood shows abnormal red cells ("tear-drop' and nucleated
RBC), immature white cells, and abnormal platelets. Major complications
are due to infection, thrombosis or hemorrhage, but a minority of patients
may develop a picture similar to AML.

72
LYMPHOPROLIFERATIVE DISORDERS
These disorders are primarily of B cell origin and represent clonal expansions
from the pathway for antifen stimulated lymphocyte differentiation. They appear
to arise as a maturationa arrest due to molecular derangements and give rise to
the neoplastic process. This may be induced by 'chromosome trans locations ,
retroviruses (HTLV-l) or activation of oncogenes. T cells as well can give rise
to such chronic leukemias, but are rare.
1. CHRONIC LYMPHOCYTE IJEUKEMIA (CLL) - is the most common form of
leukemia; Most patients are over 60 years old and asymptomatic. If
symptomatic, complaints are often non-specific but generalized
lymphadenopathy may be present. Leukemic infiltration of the hepatic
portal areas may cause hepatomegaly, and splenomegaly may be present
although not to the extent seen with CML. Lymphocytosis of peripheral
blood is generally present and may reach striking levels (> 200,000).
Patients are somewhat more susceptible to bacterial infection due to low
immunoglobulin levels. Autoimmune hemolytic anemia may be associated. The
course of the disease is usually indolent with median survival 4-6 years
after diagnosis. Occasionally, however, there can be a transformation to
a large cell lymphoma ("Richter's syndrome") which is a poor prognostic
sign.
II. HAIRY CELL LEUKEMIA - primarily affects males, 40-60 years old, and
frequently presents with splenomegaly. Peripheral smears show cells with
fine hair-like cytoplasmic projections which show tartrate resistant acid
phosphatase (TRAP) positivity. The bone marrow is packed with "fried egg"
appearance of cells. Splenic red pulp is expanded with pseudosinuses
(lakes) lined by hairy cells. Treatment consists of splenectomy and alpha-
interferon.

IMMUNOSECRETORY DISORDERS
These encompass a group of disorders that tend to occur in middle aged to elderly
people and are characterized by a monoclonal proliferation of B-cells which
secrete immunoglobulins or portions of immunoglobulins (paraproteins~ N compon-
ent) into the serum and generally behave in a malignant fashion. The amount of
paraprotein produced is roughly proportional to the size of the expanded
population of cells. Clinical symptoms are related to involvement of the various
tissues of the body and the hyperproteinemia that is produced.
1. MONOCIJONAL GAMMOPATIllF.s OF UNKNOWN SIGNIFICANCE ("benign" mono-
clonal gammopathies) - occur in 1-2% of adults over 30 years of age.
Ninety percent of these gammopathies are of the IgG class and remain
stable for more than 3 years. In the bone marrow, plasma cells constitute
less than 15% of the cellularity and never occur in large groups. They
rarely, if ever, evolve into overt myeloma. Monoclonal gammopathies are
occasionally associated with carcinomas, lymphomas, or leukemia. Occasion-
ally, they are also associated with or a complication of immune disorders.
II. MALIGNANT MONOCIJONAI~ GAMMOPATHIES

A. (PLAStIA CELL tlYEUItA) - is the most common form of


MULTIPLE tlYEUItA
malignant disease of plasma cells. It involves the bone marrow
extensively but often involves other tissues as well, especially
late in its course. Typically, the disease occurs in middle aged to
elderly males. Patients generally present with bone pain and are
found to have lytic bone lesions, fractures, and severe osteoporosis
due to the production of osteoclast activating factor by the tumor.
Diagnosis is based on clinical features, monoclonal protein spikes,
and the presence of plasmacytosis (> 30%) or multiple plasma cell
accumulations in the bone marrow.

73
1. PATrERNS OFIG SECRETION - most cases show only monoclonal Ig
secretion (52% IgG, 18% IgA, 11% IgM, 1% IgD, rare IgE) or
monoclonal Ig and free monoclonal light chains. Less
frequently there may be monoclonal light chains only, and
rarely biclonal or nonsecretory forms occur. Bence-Jones
proteins (immunoglobulin light chains) are excreted through
the kidney and are toxic to the tubular epithelium.
2. P~m6IS - myeloma usually follows a progressive course with
death occurring in 2-5 years after diagnosis. Most patients
succumb to infection, renal insufficiency, or hemorrhage.
3. VARIAtrrS

a. plasma cell leukemia - occurs in 2% of myeloma patients.


Patients are usually younger and are more likely to have
organomegaly, less bone pain, fewer bone lesions and
less paraprotein. Survival time is also somewhat shorter
than in myeloma. 20% or more of the circulating
peripheral blood cells are plasma cells.
b. asymptomatic. indolent myeloma - 2-5% of patients who
have typical features of myeloma have a longer clinical
course with a mean survival of 4-6 years. These
patients, however, occasionally lack bony lesions.
B. SOLITARY PLASI1ACY'lJIfA OF BONE- this lesion consists of a bone tumor
histologically identical to myeloma in the absence of other bone
lesions and no evidence of paraprotein in the serum or urine. This
slow growing tumor is usually located in the vertebrae, pelVis,
femur or humerus and remains solitary for an average of 5 years
before becoming disseminated.
C. EXTIWIEDULLARY PLAStlACYTOIA - these are plasma cell tumors that arise
outside of the skeleton and occur in one of three clinical settings:
(1) as an extraosseous extension of multiple myeloma; (2) as an
indolent, localized primary tumor; or (3) as a primary tumor that
disseminates to produce widespread disease. Most extraskeletal
plasmacytomas occur in the gastrointestinal tract, periorbital soft
tissue and viscera. -
D. PRDlARY 1'IA~r.cmULINEtlIA (HALDENS'fRQtI' S I'IAC~UlWLlNEtlIA) - represents a
proliferation of B-cells that are differentiated enough to secrete
immunoglobulins but not differentiated enough to look like plasma
cells. In essence, it is a hybrid of both well differentiated
lymphocytic lymphoma (in terms of its capacity to infiltrate
lymphoid organs creating lymphadenopathy and hepatosplenomegaly) and
multiple myeloma (in terms of secretion of an M component which is
generally a monoclonal IgM). Although diffuse infiltration of bone
marrow may occur, destructive skeletal lesions are not present. The
increased IgM, however, causes a hyperviscosity of the serum leading
to a hyperviscosity syndrome (retinal hemorrhage, transient paresis,
mental confusion, congestive heart failure, and bleeding). Patients
usually live only 5-6 years from the onset of symptoms.
E. HEAVY-CHAIN DISEASE (HCD) - these malignant disorders are characterized
by the production of structurally altered monoclonal proteins. These
Ig' s consist of only a fragment of the Ig heavy chain. HCD's
apparently result from the proliferation of mutant B lymphocytes.
1. ALPHA-HCD - commonly affects young adults who live in the
Mediterranean region. Histologically, the lesion is
characterized as a diffuse infiltrate of plasma cells
throughout the small bowel that is associated with
malabsorption.
2. G~-HCD - affects elderly individuals and usually manifests
itself as a well differentiated lymphocytic lymphoma with
plasmacytic features. This tumor has a predilection for
involvement of cervical lymph nodes and Waldeyer's ring.
Patients often present with palatal edema and die of
infection.
3. MU-HCD - most of these patients present the clinical picture of
a chronic lymphocytic leukemia-like illness.

74
REACTIVE LYMPHOID HYPERPLASIA
Lymphoid tissue undergoes reactive changes to microbiologic agents or their
products, foreign material introduced into the body, and cell debris from tissue
injury, systemic diseases or neoplasia. These reactive changes inevitably produce
an enlargement of the lymphoid tissue mass. Lymphoid hyperplasia is the generic
term used to describe the histology observed and can be categorized according to
microscopic architectural patterns (diffuse, follicular, sinus, or mixed). These
patterns may occasionally simulate very closely certain architectural forms of
lymphoma. In addition to histologic similarities, lymphoid hyperplasia may
simulate lymphoma both clinically and radiographically.

MALIGNANT LYMPHOMA
Lymphomas represent neoElastic proliferation of lymphoreticular tissue generally
arising in lymph nodes (but capable of arising in any lymphoreticular tissue) and
secondarily involving extranodal sites. The vast majority of these tumors are
lymphocytic in derivation (60% B lymphocyte, 35% T lymphocyte) but rarely may
arise from mononuclear-phagocytic cells or reticulum cells. They typically
present clinically as a painless enlargement of one or more lymph node groups
and/or as hepatosplenomegaly or, less frequently, as an ext ranoda I mass.
Lymphomas are divided into two major categories: Hodgkin's disease (HD) which
comprise about 40% of cases and non-Hodgkin's lymphomas (NHL).
1. STAGING OF IJYMPHOMAS - the clinical stage of the disease is an
important predictor of outcome when combined with the histologic
sUbtyping.
Stage I Disease limited to one lymph node region.
Stage II Disease limited to two lymph node regions, on
same side of diaphragm.
-
Stage III Disease limited to lymph nodes but on both sides
of the diaphragm.
Stage IV Disease involving extranodal tissue.
A: No systemic symptoms
B: Systemic symptoms present (fever, night sweats, weight loss, etc.)
II. NON-HODGKIN'S LYMPHOMA (NHL) - are primarily derived from B-cells. T-
cell lymphomas are much less common and include mycosis fungoides (and its
derivative Sezary syndrome), most lymphoblastic lymphomas, and adult T-
cell leukemia/lymphoma. Clinically, NHL tends to present as painless
lymphadenopathy or with symptoms referable to hepatosplenomegaly and, less
frequently, with systemic symptoms of fever, weight loss, night sweats,
etc. Cervical and axillary lymph nodes are more frequently the initial
nodes at presentation followed by inguinal, femoral, iliac, and
mediastinal. On cut surface, the involved nodes have a yellow-white color
and with the more aggressive forms, the nodes tend to be matted.
A. - like other neoplasms, lymphomas are felt to
CLASSIFICATION SCHEMES
arise from a single, transformed ancestral cell from some point
along the differentiation pathway. During the course of hemopoietic
differentiation, the cells undergo both morphologic and immunologic
changes. The Rappaport classification (the oldest and most ingrained
of the currently used classifications) is based principally on the
morphologic changes while the Lukes-Collins classification takes
into account not only morphology but immunological differentiation
as well. This, in part, has led to confusion in the nomenclature of
NHL and, for the clinician, confusion about the impact a specific
diagnosis will have on the patient. In an attempt at clarification,
the International Working Formulation was developed in 1982 which
also took into consideration the biologic behavior of the various
types of NHL.
75
WORKING FORMULATION RAPPAPORT COUNTERPART

Low Grade

Small lymphocytic Well differentiated lymphocytic (WDLL)


Follicular, predominantly small cleaved cell Nodular poorly differentiated lymphocytic (POLL)
Follicular, mixed, small cleaved and large cell Nodular Mixed Lymphocytic-Histiocytic

Intermediate Grade

Follicular, predominantly large cell Nodular Histiocytic


Diffuse, small cleaved cell Diffuse poorly differentiated lymphocytic (POLL)
Diffuse, mixed small and large cell Diffuse Mixed Lymphocytic-Histiocytic
Diffuse, large cell (cleaved/non-cleaved) Diffuse Histiocytic (DHL)

High Grade

Diffuse large cell immunoblastic Diffuse Histiocytic (DHL)


Lymphoblastic (con voluted/non-convoluted) Lymphoblastic
Small non-cleaved cell (Burkitt's/non-Burkiu's) Diffuse Undifferentiated (Burkitt's/non-Burkitt's)

B. The neoplastic infiltrate most often completely effaces


MORPHOLOGY -
the usual lymph node architecture and may involve the perinodal
tissue. Partial involvement sometimes occurs. The infiltrate can
grow as uniform nodules or as diffuse, monotonous sheets of
cytologically abnormal cells. The intiltrate can be composed of a
mixture of both normal cells and different cytologic varieties of
malignant lymphoid cells representing different stages of
differentiation, activation, and/or transformation. The histologic
subtype of NHL, then, is determined by the presence of one or more
of these cytologic varieties and their pattern of growth.
C. CELL TYPES (WORKING FORtlULATION tDlENCLATURE"/RApPAPORT JDlENCLATUREnl

1. normal appearing,
SHALL LYMPHOC'.iTE" /HELL DIFFERENTIATED LYMPHOC'.iTE"· -
but slightly enlarged, small lymphocytes; identical to the
cells of chronic lymphocyt1c leukemia; produces a diffuse
tumor.
2. LARGE CELL • /HISTIOC'.ITEw• - large cells with an open chromatin
pattern in an oval or folded nucleus which may (large non-
cleaved cell) or may not (large cleaved cell) have several
distinct nucleoli; narrow rim of cytoplasm; produces diffuse
and/or nodular tumors.
3. DI1UNOBLAST" /HISTIOC'.ITE"" - large cells which have a uniform or very
pleomorphic vesicular nucleus with an open chromatin pattern
and a single, usually centrally-located, prominent nucleolus;
moderate amount of cytoplasm; produces a diffuse tumor.
4. LYMPHOBLAST" /LYMPHOBLAST" - cells of intermediate size which are
cytologically identical to cells of acute lymphoblastic
leukemia; the nuclei have finely, dispersed "blastic"
chromatin with inconspicuous nucleoli and little discernible
cytoplasDl; in about half of these tumors, the nuclear contour
is conspicuously convoluted; mitoses are frequent; produces a
diffuse tumor growth pattern.
5. SHALL NONCLEAVED CELL· /UNDIFFERENTIATED CELL"· cells which are
intermediate in size with nuclei that may be more or less
monotonously uniform and contain finely dispersed chromatin
containing I to 4 distinct nucleoli usually along the nuclear
rim; the cytoplasm is moderate and in tissue section abuts
sharply a.gainst that of adjacent ce11s in a "molded" fashion;
it produces a diffuse tumor although germinal centers may be
infiltrated by or formed by the tumor cells. Benign
histiocytes (tingible body macrophaKes) characteristically
stand out against the ce11s to give a starry sky" appearance.

76
D. despite the differences in
SUBTYPES OF NHL WITH PROGNOSTIC SIGNIFICANCE -
concepts and terminology employed by the major classification
systems, histological subtypes that are defined by the identifi-
cation of cell types in the lymphomatous infiltrates are often
predictive of distinct clinicopathologic behavior.
1. L'iHPllmtASWITH FOLLICULAR AIm/OR DIFFUSE PATI'ERNS - the fact that
certain NHL's can have a nodular (follicular) or diffuse
pattern of growth is recognized in all of the major
classification systems. The cell infiltrate may be
predominantly small cleaved cell" /poorly differentiated
lymphocytic" or predominantly large cell" /histiocytic" or a
mixture of both. Both the architectural pattern and the cell
type are important in predicting survival. In general.
patients with nodular tumors live longer than those with
diffuse tumors for a given cell type. and patients with small
cell tumors live longer than those with large cell tumors.
2. SMALL LYMPHOCYTIC· /HELL DIFFERENTIATED LYMPHOCYTIC" L'iHPllmA (5%) - is
characterized by a diffuse effacement of the lymph node
architecture. A true nodular (follicular) pattern is never
present although ill-defined proliferation centers can be
found. This tumor usually presents at a high stage (70-80% are
stage IV) due to involvement of the bone marrow and often
evolves into leukemia indistinguishable from chronic
lymphocytic leukemia. In spite of the advanced stage at
presentation, the prognosis for these patients is good.
3. DIFFUSE LARGE CELL LYMPHmtAS - there are at least two major
subtypes of large cell lymphomas: lymphomas derived from
follicular center cells and lymphomas not derived from
follicular center cells. These two groups are designated as
large cell" /histiocytic·· lymphoma and immunoblastic' /-
histiocytic·· lymphoma, respectively. The former comprises
about 70% of the diffuse large cell lymphomas. The immunoblast
type affects slightly younger individuals (median age 51) and
presents more often with low stage disease.
4. LYMPHmLASTIc·/LYMPlimLASTIc" LYMPHmA - the majority of the patients
are young adolescent males (10-16 years) who present with
large anterior mediastinal masses. Other age groups and other
nodal and extranodal sites, however, may also be affected.
Most patients either present with bone marrow involvement
(stage IV disease), or it rapidly becomes involved. This tumor
typically evolves into a leukemic phase, and it responds
better to acute lymphoblastic leukemia therapy than to
therapies given for other high-grade lymphomas. Nevertheless,
prognosis is poor.
5. SMALL NONCLEAVED CELL" /UNDIFFERENTIATED·· LYMPHmA - this group actually
includes three different clinicopathological entities:
Burkitt's tumor; Burkitt's type; and non-Burkitt' s (pleo-
morphic) type. Burkitt's tumor is endemic to subtropical West
Africa and New Guinea and has a predilection for involvement
of the jaws and other extranodal sites of young children who
apparently have had an early exposure to Epstein-Barr virus
(EBV) in the setting of endemic malaria that produces an
immunosuppressant effect. The nuclear antigen of EBV is
identified in greater than 90% of these tumors whereas it is
found in less than 10% of the nonendemic Burki tt' s type.
Patients with Burkitt's type lymphomas are younger than those
with non - Burki tt' s type. While there are apparent clinical
differences bettl1een these varieties of undifferentiated
lymphomas, they are grouped together because their histology
is remarkably similar and their response-to-treatment and
survival are about identical. Even though bone marrow
involvement is infrequent, the majority of patients have stage
IV disease often because of gastrointestinal tract (ileum)
involvement or unusual extranodal sites of disease. The median
survival of this group is the poorest observed in all NHL's.

77
E• GENERALITIES

1. NHL often presents in peripheral (50%) or


CLINICAL PRESENTATION -
abdominal (25%) lymph node groups. Nodular lymphomas and
diffuse small cell lymphomas (Le., WDLL subtype) often
present as generalized, widespread disease while diffuse large
cell lymphomas (i.e., diffuse histiocytic and immunoblastic)
often present as localized disease and may involve extranodal
sites (30% of cases) as a primary manifestation. The relative
frequency of involvement of extranodal primary sites in NHL
(in descending order) is stomach, small bowel, soft tissue and
periorbital tissue, skin (excluding mycosis fungoides), colon,
salivary glands, lung and bone.
2. G~ APPEARMKE - lymph nodes tend to be enlarged with a fleshy
white, grainy cut surface which may appear as small white
nodules when the follicular growth pattern occurs. Occasion-
ally, foci of necrosis and hemorrhage are present. Extranodal
infiltrates tend to form similar, but larger, discrete tumor
masses in the spleen, liver or soft tissues. With NHL of small
cell types (WDLL, PDLL), the spleen and liver are infiltrated
by an expansion of the white pulp and portal spaces. PDLL
often infiltrates the bone marrow in a paratrabecular location
while diffuse lymphomas involve the marrow space in a random
fashion.
3. PROGNOSTIC CLUES - cell types with more favorable prognosis
present more often in a nodular growth pattern while cell
types with a poorer prognosis are more likely to present in a
diffuse pattern. It is not uncommon for tumors to evolve from
a nodular to a diffuse pattern often with a simultaneous
change to a large cell type. When this occurs, the prognosis
becomes worse than for a diffuse large cell tumor arising de
novo. Generally, lymphomas of small cell types have an
associated lymphocytosis and bone marrow involvement more
often than those of large cell types because of the increased
mobility of the small cel'ls. Stage IV involvement by small
cell types, however, has a- less ominous prognostic meaning
than Stage IV involvement by large cell types.
I I I. HODGKIN'S DISEASE

A. the neoplastic cell is the diagnostic Reed-Sternberg


CLASSIFICATION -
cell. They are large cells with abundant eosinophilic cytoplasm,
"mirror image" or multilobed nuclei with prominent eosinophilic
nucleoli, and distinct irregularly thickened nuclear membranes; In
addition, Hodgkin's lymphoma demonstrates a pleomorphic, non-
neoplastic inflammatory infiltrate consisting of a variable number
of lymphocytes, histiocytes, plasma cells, and eosinophils which may
be a host reaction to the presence of tumor. Although Reed-
Sternberg-like cells can be seen in other conditions, true Reed-
Sternberg cells are reguired for the diagnosis of Hodgkin's
lymphoma. The varying nature of the non-neoplastic, pleomorphic
infiltrate, however, serves as the basis for the histological
classification of Hodgkin's disease.
1. LYMPHOCYTE PRED(JfINANCE (15%) - characterized by a large number of
mature lymphocytes admixed with variable numbers of benign
histiocytes. Although there may be poorly defined nodules,
more often there is effacement of the lymph node architecture.
RS cells are scarce as are other types of inflammatory cells,
areas of necrosis, or areas of fibrosis. Usually affects males
under 35 years of age, generally presents as localized
cervical lymphadenopathy at an early stage (I or II), and
carries an excellent prognosis.
2. NODULAR SCLEROSIS (30-50%) - is characterized by lacunar cells
(variants of the RS cell which are large cells with
hyper lobated nuclei containing multiple small nucleoli and
abundant pale staining cytoplasm. When fixed in formalin, they
tend to artificially retract from surrounding cells giving the
78
appearance of a lacuna). Also present are birefringent
collagen bands (which may be scanty) that arise from the
capsule and tend to divide the node into circumscribed
compartments. Classic RS cells are infrequent. This form is
more frequent in females (teenage and young adult) and has a
predilection for involvement of the mediastinal,
supraclavicular, and lower cervical lymph nodes. Prognosis is
generally excellent.
3. nIXED CELLULARITY (20-40%) - contains fewer lymphocytes but there
is diffuse infiltration of eosinophils, neutrophils, plasma
cells, and histiocytes. RS cells are generally plentiful and
small foci of necrosis and fibrosis (non-collagenous) may be
present. Often presents as Stage III disease with
retroperitoneal involvement, and carries an intermediate
prognosis.
4. LYllPHOCYTE DEPLETION (10-15%) - these tend to arise in older males
and often present as disseminated disease (Stage III and IV)
with systemic symptoms and a more aggressive course.
B. CLINICALPBESEHIATION - generally presents in supradiaphragmatic LN
groups (cervical, supraclavicular, mediastinal, axillary) and rarely
presents in abdominal groups (most often with lymphocyte depletion
subtype) or in extranodal soft tissue. Patients may have systemic
manifestations such as fever, night sweats, weakness, weight loss,
anemia, etc, especially with disseminated disease. All forms may
eventually spread to extranodal sites.
C. P~IS - with treatment (radiation therapy and/or chemotherapy),
the overall 10 year survival is about 65%. In general, the younger
the patient and the lower the Stage at presentation, the better the
prognosis.

HISTIOCYTOSES
(neoplasms of tissue-based cells)
I. mSTIOCYTOSIS-X - characteristic histology usually shows many eosinophils
admixed with pale histiocytes having "grooved" or cleaved nucleus, and on
E.M., Birbeck granules are present. The disseminated form is called
Letterer-Siwe disease and affects young infants «1 yo); the multifocal
form is Hand-Schuller-Christian disease and affects young children; and
the focal form is called eosinophilic granuloma and affects older children
and adults.
II. MALIGNANT HISTIOCYTOSIS - lacks Birbeck granules. A related disorder is
histiocytic medullary reticulosis which is probably the same disease. This
affects any age group with a predilection for children and young adults.
Patients are acutely ill with fever, weight-loss, hepatosplenomegaly,
lymph node enlargement", skin rash, and pancytopenia. Histologically, there
is erythrqphasoGYtosis and proliferation of neoplastic histiocytes within
organ sinuses and .in:t2 organ parenchyma. Usually, patients die with a
rapid course which is fatal within one year, but a chronic variant form
has been described. Differential diagnosis includes disseminated histio-
cytosis-X, atypical (viral) reactive hyperplasia, sinus histiocytosis with
massive lymphadenopathy, and hairy cell leukemia.

DISORDERS OF THE THYMUS GLAND


I. THYMIC CYSTS - may be found anywhere along the lines of thymic descent,
from the angle of the mandible to the body of the sternum. Thymic cysts
are lined by single layered epithelium, contain accumulated fluid,
cellular debris and hemorrhagic extravasation, and are characterized by
finding thymic tissue in their walls. Although some may be developmental
in origin derived from the third branchial pouch, the origin of most
thymic cysts, whether mediastinal or cervical in location, appear to be
from degenerating Hassall's corpuscles or secondarily formed spaces
(radiation induced, cystic degeneration of thymoma, etc).

79
II. 1HYMIC HYPERPLASIA

A. GIANT (1'IASSIVE) IIMIIC HYPERPLASIA - is considered true hyperplasia and is


a dramatic enlargement of the gland through an increase in the
cellular elements which remain normally organized. Thymus glands
which are histologically unremarkable can weigh in excess of 200
grams. The pathogenesis and significance of enlargement to this
degree is unknown. Lymphocytosis, presumably antigen-responsive T
lymphocytes, and an expansion of the thymus-dependent regions of
lymph nodes often accompany thymic hyperplasia and resolve following
thymectomy.
B. FOLLICULAR HYPERPLASIA - is characterized by the presence of lymphoid
follicles with active germinal centers in thymus glands which are
infrequently increased in size or weight. The presence of lymphoid
follicles in the thymus gland is associated with a large number of
diseases, most of which are believed to have an autoimmune basis or
secondary immune complications (myasthenia gravis, rheumatoid
arthritis, scleroderma, etc). Myasthenia gravis is the disease most
commonly associated with follicular hyperplasia, occurring in a
large majority of patients with this disease.
III. THYMOMA - this designation has been restricted to neoplasms of the
thymic epithelial cells. The lymphocytic component of thymomas is not
thought to be neoplastic.
A. CLINICAL PRESENTATION - in over half of the patients, thymomas are
asymptomatic and are discovered by routine radiologic examinations.
In the remaining patients, symptoms from systemic diseases
associated with thymoma (primarily myasthenia gravis) or from the
presence of the tumor itself, such as cough, dyspnea, dysphagia,
chest pain or superior vena cava compression, precede the tumor's
discovery.
B. GROSS APPEARANCE typically, thymomas occur in the anterior
mediastinum as well-encapsulated masses although they can also arise
in ectopic sites such as in the neck along the route of embryonic
migration. In about 20-25% of cases, the tumor will be adherent to
surrounding tissues, and most often microscopic invasion of these
tissues can be demonstrated. The cut surface of the neoplasm usually
reveals a thick fibrous capsule surrounding tumor lobules of varying
sizes that are separated by prominent, fibrous trabeculae.
C. MICROSCOPIC APPEARANCE - most thymomas are composed of an admixture of
epithelial cells and lymphocytes, and they have been classified
based on the ratio of these cell types (predominantly lymphocytic,
mixed, or predominantly epithelial), but only rarely are thymomas
purely epithelial. Microcystic degeneration may be extensive in some
tumors. Well-formed Hassa11's corpuscles are rarely found.
D. BEHAVIOR - thymomas are extremely slow growing neoplasms which usually
remain localized for many years before invading adjacent structures.
Distant metastases occur very rarely. If complete resection is not
accomplished, death from cardiorespiratory complications of tumor
invasion will result.

80
ENDOCRINE SYSTEM
GENERAL PRINCIPLES
The endocrine system consists of a variety of glandular tissues and individual
non-glandular cells (neuroendocrine system) scattered throughout the body whose
primary function is to maintain body homeostasis through the manufacture and
secretion of a variety of chemical mediators (hormones). These can exert their
effects on distant sites by secretion into the circulation (endocrine effects),
adjacent sites by diffusion through interstitial tissue (paracrine effects) or
locally (autocrine effects). In general, hormones are either amino acid deriv-
atives (polypeptides) or cholesterol derivatives (steroids). Depending on the
target cell, the same hormone can exert biologically different effects. The
steroids and thyroid hormones are hydrophobic, requiring transport proteins, but
can easily diffuse through the plasma membranes of target cells to bind rever-
sibly to intracytoplasmic receptors. Ultimately, they are bound to nuclear DNA
where they act to regulate transcription of specific genes. The remainder of the
hormones are hydrophilic, transported in the circulation in low concentrations,
and bind to specific receptors on the target cell plasma membrane. They effect
their action either by induction of secondary messengers (cyclic AMP, etc) or by
altering the plasma membrane permeability to specific ions, etc. There are deli-
cate, sometimes complex, regulatory mechanisms that control hormone secretion by
the endocrine system. Abnormalities of endocrine organs are usually manifested
by hyperfunction (abnormal stimulation, autonomous function, ectopic production) ,
hypofunction (congenital defect, acquired defect, end organ unresponsiveness),
or the local effects of an enlarging mass. The differentiation between benign and
malignant neoplasms is often difficult since the normal rules (mitoses, ana-
plasia, etc.) don't tend to apply. In many cases the best evidence of malignancy
is the existence of metastases.
PITUITARY
I. REVIEW OF NORMAl,

A. AHrERIOR LgBE (ADENOHYPOPHYSIS) - is derived from an upward extension of


Rathke's pouch located in the roof of the mouth. The developing
sella turcica of the sphenoid bone separates the anterior lobe from
its embryonic origins but small cellular rests may remain along the
line of embryologic ascent.
1. HISTOLOGY- the cells are arranged in nests, cords, and
pseudoacinar· formations separated by a rich fibrovascular
network. Originally three cell types were identified
(acidophil, basophil, and chromophobe) based on staining
characteristics with routine H&E stains. The development of
immunohistochemistry techniques and monoclonal antibodies,
however, has further divided these into cell types responsible
for secretion of specific hormones.

{
(1) Somatotroph (40-50%) secretes growth hormone
(GO), a polypeptide.
Acidophil
(2) Lactotroph (15-20%) secretes prolactin (PRL),
a polypeptide.
(3) Corticotroph (15-20%) secretes adrenocortico-
tropic hormone (ACTO), a polypeptide.
Basophil (4) Thyrotroph (5%) secretes thyroid-stimulating
hormone (TSK), a glycoprotein.
(5) Gonadotroph (5%) secretes follicle-stimulating
hormone (FSK), and luteinizing hormone (LK),
glycoproteins.
Chromophob
(6) Non-secretory (15-20%) probably represent
degranulated cells.

81
2. REGULATION

a. Hypothalamic releasing factors - GHRF, CRF, TRF (also


stimulates prolactin release), and LHRF-FSHRF ..
b. Hypothalamic inhibitory factors - GHIF (somatostatin)
inhibits GH and TSH release, and PIF (dopamine) inhibits
prolactin.
c. Negatiye feedback loops - by target organ hormone.
B. is derived from a downward growth from
POSTERIOR LOBE (NEUROHYPOPHYSIS) -
the median eminence of the hypothalamus to lie posterior to anterior
lobe. A direct connection between hypothalamus and posterior lobe is
maintained via the pituitary stalk which penetrates the diaphragma
sellae, an extension of the dura which covers the roof of the sella.
Histologically, it consists of scattered pituicytes and tangle of
unmyelinated nerve fibers (axons). Posterior pituitary hormones,
antidiuretic hormone (ADH, vasopressin) and oxytocin, are formed in
hypothalamus and conveyed to the posterior lobe via axonal transport
where they are stored prior to release.
II. HYPERPITUITARISM - although hypothalamic hypersecretion and disruption
of negative feedback loops are occasionally at fault, autonomous pituitary
hypersecretion is the most common basis and is usually due to functional,
benign, monoclonal neoElasm (pituitary adenoma) which secretes one of the
polypeptide hormones (PRL, GH, ACTH). Rarely one neoplasm may secrete 2
hormones (usually PRL and GH). Adenomas tend to appear in young adults and
are slightly more frequent in males.
A. ANATOMIC FEATURES - adenomas vary in size, but in general, are small.
When the tumors enlarge, they compress the surrounding normal pitui-
tary tissue, expand the sella, and may become locally aggressive.
They also tend to compress their own blood supply to become
centrally necrotic. Hemorrhage into areas of necrosis may lead to
rapid expansion in size (pituitary apoplexy). Since the tumor cells
are derived from a monoclonal expansion, they usually are similar in
appearance and generally tend to closely resemble one of the normal
pituitary cell types. Some adenomas, however, may show considerable
cellular pleomorphism but overt malignant neoplasms are rare.
B. CLINICAL FEATURES

1. MASS EFFECTS- visual impairment occurs when there is growth


outside of the sella and impingement on the optic chiasm
(usually bitemporal hemianopsia) or compression of the oculo-
motor nerves. Although less likely, the mass may compress the
floor of the 3rd ventricle, increase intracranial pressure and
produce headaches and nausea and vomiting.
2. EtmOCRINOLOGIC EFFECTS

a. PRL - is the most common secretory product. Pathologic


hyperprolactinemia is defined as a consistently elevated
serum prolactin (greater than 20 ng/ml) in the absence
of pregnancy or rostpartum lactation. In women this
often results in 2 amenorrhea and galactorrhea. In men,
it is usually asymptomatic and not discovered until the
tumor grows to a size that produces symptoms due to mass
effect.
b. GH - before puberty, this is reflected by exaggerated
skeletal growth (giantism) which is often accompanied by
nerve, muscle, and joint problems. After puberty,
hypersecretion produces acromegaly (2nd most common
functional disorder of pituitary) which involves
insidious enlargement and coarsening of the face, hands,
and feet. Facial features include prominent forehead,
protruding jaw, thick lips, and enlarged tongue. Metabo-
lic problems include hypertension, glucose intolerance,
and osteoporosis. The diagnosis of acromegaly should be
based upon the clinical features of the patient, the

82
presence of elevated basal serum GH concentrations, and
the failure of serum GH concentrations to suppress
following a glucose load.
c. ACIlI - results in Cushing's disease [see Adrenal
section]

III. HYPOPITUITARISM - hypothalamic disorders can give rise to isolated


anterior lobe hormone deficits but hypopituitarism generally results from
extensive destruction of anterior lobe tissue and leads to panhypo-
pituitarism (Simmond's disease).
A. temporally, symptoms are sequentially related to
CLINICAL FEATURES -
deficiencies of gonadotropins, GH, ACTH, TSH, and PRL.
1. GONADOTROPIN DEFICIENCY - may cause delayed puberty in children;
amenorrhea, loss of secondary sex characteristics, and atrophy
of ovaries and external genitalia in adult females; and
aspermia, loss of secondary sex characteristics, and testic-
ular atrophy in adult males.
2. GH DEFIC~ - before puberty, it can cause pituitary dwarfism
(short stature but with normal proportions) and delayed sexual
maturity. After puberty, it is generally asymptomatic.
3. AC'I'H DEFICIENCY - produces symptoms of hypoadrenalism.
4. TSII DEFICIENCY - produces symptoms of hypothyroidism.

B. .QYB!

1. ~~UAR - rarely suprasellar tumors will impinge upon and


destroy anterior and/or posterior pituitary tissue to produce
hypopituitarism and/or diabetes insipidus. Craniopharyngioma
is a tumor (usually benign) of children and young adults
derived from remnants of Rathke's pouch. The tumor histologic-
ally resembles embryonic tooth enamel formation (ameloblast-
oma) with squamous and columnar- epithelium set in loose
fibrous tissue. Calcification frequently occurs and can be
seen on X-rays. CerminollJas arise in the pineal gland from germ
cells that were embryologic "leftovers' .
2. INTRASELLAR

a. Non-functioning adenoma - is the most frequent cause of


hypopituitarism. As the neoplasm enlarges in size, there
is compression of the remainder of the gland resulting
in a gradual loss of function. For this reason, the
tumors tend to be relatively large when discovered.
Histologically, the cells generally are sparsely
granulated (chromophobe adenoma) and may be derived from
undifferentiated precursor cells. Others, however, may
contain cells with a clear cytoplasm (null cell adenoma)
or cells packed with mitochondria (oncocytoma). In some
instances, a functioning adenoma may undergo secretory
exhaustion, ischemic necrosis, hemorrhage, or infarction
and also lead to hypopituitarism.
b. Sheehan's syndrome - results from infarct of anterior
lobe secondary to hypotension or shock. Although there
may be multiple etiologies, classically it is due to
obstetric hemorrhage and hypotension (postpartum
pituitary necrosis). Inability to lactate may be first
sign but often is identified only when there is failure
to resume menses.
c. Empty sella s~rome - refers to absence of pituitary
gland in an ot erwise enlarged or normal sella. This may
be the end result of a number of preexisting intrinsic
pituitary problems, previous surgical removal, or due to
a defect in diaphragma sella which allows herniation of
arachnoid membrane into sella causing pituitary atrophy
under conditions of chronic increase in CSF pressure.

83
IV. POSlERIOR pnlJITARY SYNDROMES

A. oxnOCIN - apparently neither excessive nor deficient production of


oxytocin has associated clinical effects.
B. ADH - ADH is released in response to changes in the osmolality of
body water. Together with thirst, which promotes water ingestion,
ADH regulates water balance to maintain steady-state solute
concentrations within tissues.
1. EXCESS SECRETION (SIADH) - results in renal retention of water
with hypervolemia, hemodilution, hyponatremia, and an inabil-
ity to dilute urine. The usual cause is ectopic production of
ADH by various neoplasms (classically oat cell of lung), but
it also may be seen with inflammation, etc. The features of
SIADH are hyponatremia with appropriately low plasma osmol-
ality; urine osmolality greater than plasma osmolality; exces-
sive renal sodium excretion; and normal renal and adrenal
function.
2. DEFICIENT SECRETION (DIABETES INSIPmus) results in clinical
symptoms similar to diabetes mellitus (polyuria, polydipsia,
etc.) and is generally due to posterior pituitary tissue
destruction, disruption of hypothalamic-hypophyseal axis, or
idiopathic causes. Patients with severe diabetes insipidus can
be identified by low urine osmolality, high plasma osmolality,
and high urine concentration following administration of
synthetic ADH. In some cases, there may be sufficient ADH
secretion but an end-organ unresponsiveness (nephrogenic
diabetes insipidus) to the hormone.

THYROID
I. REVIEW OF NORMAL

A. EtlSRYOLOGY - the adult thyroid, consisting of two lateral lobes


connected by an isthmus, begins as a tubular evagination from the
foramen cecum at root of tongue and grows caudally anterior to the
trachea. The distal portion proliferates to form thyroid while prox-
imal portion degenerates. Aberrant locations (substernal, lingual)
may result from improper descent. Failure of the proximal portion to
degenerate may result in thyroglossal duct cysts which may not
become apparent until adulthood. They lie in midline (as opposed to
branchial cleft cysts), contain mucinous fluid, and are lined by
squamous or cuboidal epithelium with striking underlying lymphocytic
infiltrate. Abscesses or draining sinus tracts may develop.
B. STRUClVRE - the thyroid is composed of variably sized follicles set
in a fibrovascular stroma containing the parafollicular or "e" cells
that secrete calcitonin. Normally, follicles are lined by cuboidal
epithelium with apical microvilli that enclose an eosinophilic
thyroglobulin-containing colloid. With increased activity, the
epithelium becomes more columnar as the colloid is absorbed while
with inactivity colloid accumulates and there is flattening of the
epithelium.
C. REGULATION - thyroid hormones are very sensitive to metabolic demands
and thyroid activity is constantly changing. Hypothalamic TSHRF
stimulates pituitary TSH which stimulates production of thyroid
hormones. Thyroxine (T4) is the major secretory product; however, Ts
is the most potent thyroid hormone. During periods of activity,
thyroglobulin is taken up from the colloid by the follicular cells
and undergoes proteolysis to release Ts and T~ which are secreted
into perifollicular capillaries. T, T, and TSH can be quantita-
tively measured by RIA procedures. The t~yroid hormones circulate in
blood bound to plasma proteins, primarily thyroxine-binding globulin
(TBG). The binding is so strong that only a very small amount of the
circulating hormones is unbound or "free" and biologically active.
Since the assays for T4 and Ts typically measure the total hormone,
changes in TBG concentration may result in changes in the measured

84
total hormone concentration but a "normal" biologically active level
is maintained. Pregnancy, estrogen therapy, and a genetically
determined increase in TBG can lead to an elevated total T4 and T3 •
Alternatively. total T4 and T~ levels are decreased when plasma TBG
is decreased, as may occur 1n liver disease, nephrotic syndrome,
androgen treatment, or a genetic defect in TBG synthesis. Although
the total hormone concentration is increased or decreased as appro-
priate in order to keep the TBG about one-third saturated, the free
hormone concentration is maintained as "normal" to provide euthyroid
status. The rise in serum T3 and T4 , in turn, have negative feedback
on the hypothalamic and pituitary secretion of TSHRF and TSH.
II. CLINICAL PRESENTA1l0N - is usually related to:

A. LOCAL EFFECTS - an enlarging gland may encroach on adjacent structures


and produce dysphagia, inspiratory stridor, hoarseness, dyspnea, etc
B. HYPER11IYROIDISM (thyrotoxicosis) - is a hypermetabolic state produced
by increased T4 and/or Ts and measurement of T3 by RIA appears to be
a sensitive test for hyperthyroidism. The disorder is clinically
manifested by hyperkinesia, emotional lability, increased appetite
(but decreased weight), muscle weakness and fine tremors, heat
intolerance, warm moist skin, sweating, tachycardia, cardiac
arrhythmias, fine oily hair, ocular changes, and diarrhea. Sudden
release of excessive amounts of thyroid hormone (thyroid storm) is
an emergency condition manifested by fever, tachycardia, arrhyth-
mias, obtundation, and coma. Major causes of hyperthyroidism include
Grave's disease, toxic multinodular goiter, and toxic adenoma.
c. HYPOTHYROIDISM - results from a deficiency of thyroid hormones and may
result from intrinsic thyroid disease (primary hypothyroidism) or
from a deficiency of TSH (secondary hypothyroidism) as a consequence
of pituitary or hypothalamic disease. Rarely is hypothyroidism due
to general end-organ resistance to thyroid hormone. The measurement
of serum TSH is regarded as the most sensitive indicator of primary
hypothyroidism. With thyroid failure, the serum TSH level is
increased. In the patient with clinical features compatible with
hypothyroidism, a TSH level that is not elevated suggests secondary
hypothyroidism.
1. LATE CHILDHOOO OR ADULT HYPOTHYROIDISM (myxedema.) is a
hypometabolic state manifested by insidious onset of tired-
ness, apathy, and lethargy. Mental faculties become dulled and
speech becomes slowed. The skin becomes rough and the hair
becomes coarse and dry. Mucopolysaccharide is deposited in
connective tissue throughout the body giving the subcutaneous
tissue a "doughy" consistency. The periorbital soft tissue
becomes puffy, the tongue becomes enlarged, and the heart
becomes flabby and dilated. Increased peripheral vascular
resistance and decreased cardiac output leads to narrowed
pulse pressure, and decreased perfusion of skin results in
cold intolerance. Major causes include surgical, chemical, or
radiation ablation; primary idiopathic myxedema; and
Hashimoto's thyroiditis.
2. GESTATIONAL OR EARLY INFANCY HYPOTHYROIDISM (cretinism) - may arise
from nutritional lack of iodine or defects in thyroid hormone
synthesis. A deficiency early in fetal life will retard brain
development causing severe mental retardation, spasticity, and
deaf-mutism (neurologic cretinism). A deficiency late in
pregnancy or early infancy usually becomes manifest by feeding
difficulty, somnolence, hoarse low pitched cry, and failure to
thrive. If untreated, there is impaired skeletal growth, dry
skin, and protuberant abdomen. Later manifestations include
wide set eyes,· broad flat nose, enlarged tongue, and
periorbital puffiness.

85
II 1. THYROIDITIS
A. AcurE INFECTIVE THYROmITIS - results from bacterial or fungal seeding of
the gland which may cause painful swelling but is usually self--
limiting with no sequelae except focal scarring.
B. SUBAcurE GRAHULCIIATOUS (DEQYERVAIN'S) THYROmITIS - is probably viral in
origin, and typically presents in young adult females as an acute
febrile "flu-like" systemic illness. There may be sudden, painful,
and diffuse (but irregular) enlargement of thyroid and/or transient
thyrotoxicosis which gradually subsides over several months. Micro-
abscesses may form early in the disease but eventually, with
follicular destruction, there is a granulomatous "foreign body"
inflammatory reaction to pools of colloid. Focal scarring may be the
end result but generally is not clinically significant.
C. SUBACUTE OR CHRONIC LYMPHQCYfIC THYROmITIS - of unknown etiology, this
presents as painless mild-moderate enlargement of the thyroid and/or
transient hyperthyroidism which is usually self-limiting. There is
a lymphocytic stromal infiltrate without plasma cells or germinal
centers and there may be mild fibrosis.
D. RIEDEL'S THYROmITIS - also of unknown etiology, this is a rare
condition more commonly seen in middle aged females and character-
ized by aggressive fibrosis of the thyroid parenchyma and adjacent
structures. Clinically it presents asa hard, non-tender, fixed
nodule of irregular outline which may cause stridor, dysphagia,
hoarseness and dyspnea, and which may be easily confused with
cancer. Up to 50% of patients may be clinically hypothyroid.
E. HASHII1OTO'S THYROmITIS - characteristically presents in middle aged
women as a gradual thyroid enlargement, usually painless, associated
with insidious onset of hypothyroidism.
1. PATHOGENESIS - the disease is autoimmune in origin. Because of
its association with HLA-DR5, some feel that there is a
genetic predisposition to formation of autoantibodies to
follicular cell antigens as the result of a deficiency in
antigen-specific suppressor T cells. Without these, cytotoxic
(killer) T cells are free to attack follicular cell antigens
while helper T cells assist in the B cell production of
autoantibodies (microsomal, thyroglobulin, and TSH-receptor).
TWo forms of TSH-receptor autoantibodies may be produced: TGI
(tbyroid growtb immunoglobulin) and TSI (tbyroid stimulating
immunoglobulin)._In addition, blocking antibodies for each of
these may be formed. In Hashimoto's thyroiditis, the
TSH-receptor autoantibodies may consist mostly of TGI (or TGI
and TSI with TSI blocking antibody). At any rate, there is
growth of the gland but generally no hyperfunctioning.
2. ANATOIIC FEATURES - gross ly, the gland is usually symmetrically
enlarged, firm, and rubbery. Microscopically, there is
destruction of follicles (perhaps mediated by cytotoxic T
cells) and extensive infiltration of the stroma by lympho-
cytes, immunoblasts, plasma cells, and macrophages. Germinal
centers are often prominent and the cytoplasm of residual non-
functional follicular cells are packed with mitochondria
giving an eosinophilic granular appearance (Rurtble cells or
oncocytes). Some degree of fibrosis is usually present.

IV. GOITERS
A. DIFFUSETOXIC GOITER (GRAVE'S DISEASE) - is a symptom complex seen
predominantly in young, adult women which is characterized by
hyperthyroidism, moderate thyroid enlargement, ocular changes, and
localized thickening of the skin over dorsum of feet and legs.
1. although genetically associated with HLA-DR3, the
PATHOGENESIS -
patho~enesis is basically same as Hashimoto's thyroiditis. In
Grave s disease, both TGI (which stimulates gland growth) and
TSI (which stimulates hormone production) are produced as a

86
result of an antigen-specific T suppressor cell deficiency and
results in an enlarged, hyperfunctioning gland. Additionally,
autoantibodies that are specifically directed against, or at
least cross react with, eye-muscle antigens are apparently
produced. Subsequent lymphocytic infiltration of, and
mucopolysaccharide deposition in, the extraocular eye muscles
and retro-orbital fatty tissue account for the proptosis and
ocular changes. Similar changes in the dermis and subcutaneous
tissues create the pretibial "myxedema" associated with the
disorder. Although the eye problems usually regress spontan-
eously, they may progress to produce blindness despite
successful treatment of the thyrotoxicosis.
2. ANATa1IC FEATURES grossly, there is moderate diffuse
enlargement of the thyroid with the parenchyma having a soft
meaty appearance. Histologically, the classic appearance is
increased numbers of tall columnar follicular epithelial cells
creating papillary infoldings into the follicle lumen which is
almost devoid of colloid material. There is an increase in the
stromal vascularity and a heavy stromal lymphocytic infiltrate
which may form lymphoid follicles. Extrathyroid tissue which
may show morphologic changes include the heart, skeletal
muscle, liver, and lymphoid organs.
B. (simple goiter, colloid goiter) - refers to a
DIFFUSE NONTOKIC GOITER
gradual painless enlargement of the thyroid due to compensatory
hyperplasia secondary to a defect in production or an inability to
secret functional thyroid hormone. At the time of diagnosis,
however, most patients are euthyroid.
1. ETIOUOOY - endemic goiters are generally confined to specific
geographic locations and are the result of deficient dietary
iodine intake or, in some cases, regional environmental
goitrogens. Sporadic goiters are less common and typically
occur in adolescent or young adult women. They are presumed to
be due to TSH stimulation of the gland. Some probably arise in
a marginally euthyroid patient when there is an increased
demand for thyroid hormone or when environmental goitrogens
are introduced. Other causes include the uncommon hereditary
defects in hormone synthesis or transport, excess production
of TBG, or other unknown (autoimmune?) mechanisms.
2. DEVELOPMENT initially, TSH stimulation causes increased
follicular cell activity reflected by increased cell mass and
tall columnar epithelial cells lining colloid-depleted
follicles (hyperplastic stage). This produces moderate diffuse
enlargement of the gland which has a meaty appearance on cut
surface. As a euthyroid state is reached, colloid (generally
thyroglobulin deficient) accumulates unevenly in follicles
flattening the epithelium and enlarging the gland even further
(colloid involution). As the colloid accumulates, the cut
surface develops a gelatinous appearance.
C. GOITER (adenomatous goiter) - is the end result of a
MULTINCIlULAR
long-standing diffuse nontoxic goiter and can cause tremendous
thyromegaly. Since they are derived from simple goiters, they are
more frequently seen in females but in an older age population.
Although the goiter may be functional (toxic multinodular goiter),
more frequently patients are euthyroid, and although the entire
gland is usually multinodular, the nodularity may be asymmetric and
more prominent within one lobe. Irregular scarring creates nodules
of varying size which, in turn, contain follicles of varying size
and colloid content. Focal hemorrhage, calcification, and microcysts
may form. Due to the size and irregularity, these nodules can easily
be clinically confused with cancer. Clinical symptoms are generally
related to local effects of increasing size (inspiratory stridor,
dysphagia, superior vena cava syndrome) and/or hyperthyroidism (not
as severe as Grave's disease and not associated with ocular or skin
problems) .

87
V. SOLITARY THYROID NODULES - although thyroid nodules are common and
thyroid carcinoma is relatively rare, there is no reliable method to
distinguish non-neoplastic from neoplastic or benign from maligP.ant short
of histologic study. Slightly over 50% of clinically palpable "solitary"
thyroid nodules prove to be nodules of multinodular goiters and, of the
remainder, about 3/4 are solitary benign adenomas.
A. BENIGN LESIONS - although lipomas, hemangiomas, etc do occur, they are
unusual and of little significance. The vast majority of benign
lesions are adenomas which become more frequent with increasing age
and usually present as small to moderate sized (less than 4 cm)
discrete nodules. They are rarely functional but they may painlessly
enlarge slowly over time, enlarge suddenly and painfully due to
hemorrhage, or remain relatively static and asymptomatic. Occasion-
ally, they (as well as nodules of multinodular goiters) may undergo
cystic degeneration. They are considered to have little, if any,
malignant potential. Derived from follicular epithelial cells,
almost all are follicular adenomas. Criteria for classification as
an adenoma include: (1) complete fibrous encapsulation of the
nodule; (2) compression of adjacent thyroid parenchyma; (3)
different histologic appearance of the neoplastic and non-neoplastic
tissue; and (4) lack of multinodularity in rest of gland.
B. MALIGNANT LESIONS - although a solitary nodule in a male is more likely
to be malignant, the incidence of malignancy is higher in females.
Nevertheless, thyroid carcinomas are relatively infrequent occur-
rences. Previous radiation exposure increases the risk of malignancy
but other forms of thyroid disease do not appear to increase the
risk.
1. PAPILLARY ADENOCARCINc:JtA (65%) - can arise at any age but tend to
present in young adult females as a non-tender neck mass which
is very s low growing and may have been present for years.
While some papillary carcinomas consist solely of papillary
structures, many show varying proportions of follicular
elements and a few will be devoid of any papillary structure
but possess characteristic nuclear changes (ground glass or
optically clear nuclei) that indicate that the tumor will
behave as a papillary carcinoma. Another useful histologic
feature is the presence of psammoma bodies, small concentric
calcifications felt to represent degenerated papillary tips,
which are present in about 50% of the cases. All lesions of
the thyroid which have any element of papillary architecture
should be considered malignant. Some may be multifocal within
the thyroid. They spread via lymphatics to regional lymph
nodes and up to 20% of patients have cervical lymph node
metastases at presentation. Nevertheless, the overall prog-
nosis is excellent, but the prognosis can be negatively
influenced by tumor extension through the thyroid capsule,
increased age of patient, the degree of anaplasia, and the
presence of distant metastases.
2. FOLLICULAR CARCINc:JtA (25%) - tends to present in middle-aged
females as a firm, irregular, nodular mass which also grows
relatively slowly. It may appear grossly benign and encapsu-
lated or aggressive and infiltrative. Both forms tend to show
central fibrosis with necrosis, hemorrhage, and cyst forma-
tion. The architecture ranges from a well differentiated
glandular pattern to sheets of anaplastic cells. The tumor
tends to invade vessels and spread hematogenously to distant
organs (lung, bone, liver, etc). Although the small, well
differentiated encapsulated forms have survival rates
comparable to papillary carcinomas, the less differentiated
forms tend to be more aggressive so that there is a poorer
overall prognosis.
3. UNDIFFERENTIATED CARCINc:JtA (10-15%) - occurs in an older population
and is an aggressive rapidly growing anaplastic tumor with
progressive local invasion and widespread metastases. The
average survival from time of diagnosis less than 6 months.

88
4. MEDULLARYCARCDOIA (5-10%) - arises from the parafollicular
("C") cells of neural crest origin and occur sporadically or
as part of genetically inherited syndromes. Most of the tumors
secrete calcitonin but they have the capability of secreting
a wide variety of polypeptides. The cells generally will
contain neurosecretory granules and are arranged in nests
surrounded by a fibrovascular stroma which characteristically
demonstrates deposition of amyloid in the tumor stroma.
Although the clinical presentation may vary widely, most of
the patients have some degree of diarrhea due to calcitonin or
prostaglandin secretion.
a. Sporadic form - is the most frequent form and tends to
present in middle aged adults as a painless, relatively
large solitary mass, 50% of which have metastasized at
the time of diagnosis to regional lymph nodes, lung,
liver, or bone. The overall survival, therefore is about
40% over a 10 year span.
b. Genetic form - 10-15% are identified in adolescents as
part of the autosomal dominant multiple endocrine neo-
plasia syndromes and occur in association with pheo-
chromocytoma and other endocrine tumors. These tend to
be multicentric, bilateral, and smaller. Because of
their genetic nature and association with other
endocrine tumors, they are usually identified at an
early stage so that the 10 year survival runs 60-70%.

PARATHYROID
I. REVIEW OF NORMAL - derived from the pharyngeal pouches, the parathyroid
glands tend to lie posterior to the thyroid gland but may be found in
aberrant locations. The major secretory cell is the chief cell which is
also the precursor of the clear (wasserhelle) cells, which have abundant
cytoplasmic glycogen, and the oxyphils, which have increased numbers of
cytoplasmic mitochondria. Fat cells accumulate during childhood and
adolescence to comprise about one third of the volume of each gland. After
secretion, PTH (parathormone) is cleaved into an active N-terminal
polypeptide chain with a short half-life and a non-functional (but more
easily measured) C-terminal polypeptide with a longer half-life. PTH
maintains serum calcium levels by regulating renal vitamin D metabolism,
increasing renal calcium reabsorption and phosphate secretion, and
increasing reabsorption from the bone stores. Increasing serum calcium
levels have a negative feedback on further PTH secretion.
I I. HYPERP ARATHYROII>ISM

A. is reflected by elevated PTH levels even in


PRDfARY HYPERPARA11fYROmISM -
the face of hypophosphatemia, hypercalcemia, and increased urine
calcium excretion. Serum alkaline phosphatase levels are also
increased. Over time, if not recognized and treated, this will lead
to the "stones" of nephrocalcinosis and renal stones; "bones" of
bone demineralization (osteomalacia), osteitis fibrosa cystica ~von
Recklinghausen's disease of bone), and "brown tumors"; "moans' of
psychiatric disturbances, neurologic abnormalities, and muscle
weakness; and "groans" of peptic ulcers, pancreatitis, and abdominal
pains. Hypertension, chondrocalcinosis, and metastatic calcification
of soft tissues may also occur.
1. ADENOMA - is the most frequent cause and represents a neoplas-
tic monoclonal proliferation of a parathyroid cell line
(generally chief cells) which is almost always functional.
Although multiple adenomas may rarely occur, the vast majority
are solitary lesions which are slightly more common in females
and tend to involve the inferior glands. The expanding
neoplastic cells displace the normal fat cells, and the non-
neoplastic parathyroid tissue becomes compressed at the
periphery of the expanding nodule.

89
2. PRIMARY HYPERPLASIA - occurs with perhaps more frequency than was
previously recognized but the etiology has not been clearly
elucidated. Most hyperplastic glands show chief cell
proliferation with occasional clusters of oxyphil cells that
tend to crowd out the native fat cells. All four glands are
affected but not necessarily equally; the superior glands
tending to be affected more prominently. Occasionally, only
one gland is grossly enlarged and may be exceedingly difficult
to distinguish from an adenoma, but the differentiation
between adenoma and hyperplasia has therapeutic implications
in that surgical excision of an adenoma should be curative
while with surgical excision of a single hyperplastic gland,
hypercalcemia will persist due to the activity of the
remaining glands.
3. CARCINCItA - is a rarity. The marked variability in cell
morphology that is occasionally encountered in a benign
adenoma may simulate malignant change, so a diagnosis of
carcinoma should be made with caution. Generally, carcinomas
are slow growing and eventually regionally invasive. They show
cytologic atypia with increased mitoses and may demonstrate
c~psular or vascular invasion. Like many endocrine tumors,
however, the best criterion of malignancy is the demonstration
of metastases but distant metastases are unusual.
B. SECONDARY HYPERPARADlYROmISII - is due to secondary (compensatory)
hyperplasia of the glands (and elevation of serum PTH) in response
to chronic hypocalcemia (usually due to chronic renal failure) or
unresponsiveness of peripheral tissue to the effects of PTH. The
glands are usually smaller than what is seen with primary hyper-
plasia and if the underlying cause is corrected within a reasonable
period of time, the glandular hyperplasia and associated hormonal
effects are reversible. On the other hand, if the underlying cause
persists, the changes may become irreversible (tertiary hyperpara-
thyroidism) .
III. HYPOPARAlHYROIDISM - is generally cliaracterized by low PTH, hypocal-
cemia, and hyperphosphatemia. On occasion, if the PTH is non-functional,
measured serum PTH levels may be normal or elevated. Hypoparathyroidism is
most often the result of inadvertent removal of the parathyroids during
thyroidectomy or radical neck dissection. Radiation and primary hypo-
functioning (probably autoimmune process) of the glands are less common
causes. Clinical symptoms include weakness, paresthesias, muscle cramps or
tetany, cataracts, headache, and abdominal pain. Pseudohypoparathyroidism
is a familial syndrome resulting from an end-organ resistance to PTH.
These patients have hypocalcemia and increased PTH. They also develop
skeletal and developmental defects.

ADRENAL CORTEX
I. REVIEW OF NORMAL

A. HISTOLOGY AND EUNC'fION

1. ZONA RETICULAR IS - the inner layer of the cortex contains


irregular clusters of darkly staining "compact" cells which
produce glucocorticoids (cortisol) and androgenic steroids
(testosterone). Glucocorticoids act to increase appetite and
promote fat deposition in face, neck, and trunk; increase
protein breakdown and nitrogen excretion; increase
gluconeogenesis in liver but inhibit glucose uptake by muscle;
impair both the inflammatory and immunologic defense
mechanisms; and interfere with water transport into cells and
thereby increasing ECF volume. The androgenic steroids act to
promote masculinization and inhibit feminization.

90
2. ZONA FASCICULATA - the middle layer of the cortex consists of
columns of cells with finely vacuolated clear cytoplasm
containing steroid precursors (mainly cholesterol). The zona
fasciculata and zona reticularis work in tandem. Cells of the
reticularis synthesize glucocorticoids and androgens from the
precursor substances that are stored in the fascicu1ata. In
times of increased production, the zona fascicu1ata decreases
in width due to cytoplasmic lipid depletion while the zona
reticularis increases due to increased production demands.
3. ZONA GLOtIERULOSA - the outer layer of the cortex consists of
clusters of cuboidal cells with scanty cytoplasm containing
few lipid droplets. These cells produce mineralocorticoids
(aldosterone) which act to regulate extracellular fluid volume
and potassium metabolism. Aldosterone acts on the kidney to
secrete potassium and retain sodium and water.
B. REGULATION

1. ZONA RETICULARIS AND FASCICULATA - hypothalamic CRF stimulates


pituitary corticotrophs to release ACTH which causes the zona
fasciculata and reticularis to secrete glucocorticoids and
androgens. Cleaving ACTH from a larger precursor polypeptide
chain also liberates melanotropins; therefore increased ACTH
production from pituitary is accompanied by increased melano-
tropins, hyperpigmentation of the skin, and hyperplasia of the
adrenal cortex. Cortisol has a negative feedback on the
pituitary and hypothalamus.
2. ZONA GLmERULOSA - mineralocorticoid secretion is regulated by
serum potassium and the renin-angiotensin system. Although
ACTH may need to be present, it does not respond directly to
increased ACTH to any significant extent.
II. MAJOR ANATOMIC LESIONS
A. CORTICAL HYPERPLASIA - is generally the result of pituitary hyper-
secretion of ACTH, but may also occur in cases of ectopic production
of ACTH. Hyperplasia is a bilateral process and the adrenals often
show mUltiple hyperplastic cortical nodules.
B. ADENaq§ - are relatively common but most are nonfunctional. Grossly,
they appear as expanding, yellow-orange nodules of varying size. The
nodules contain lipid-laden cells resembling the cells of the zona
fascicu1ata admixed with a few cells resembling the zona reticular-
is. Larger adenomas may show hemorrhage, calcification, or cystic
degeneration and may be difficult to distinguish from carcinomas.
Smaller well-differentiated lesions, on the other hand, may be
difficult to distinguish from a simple focus of nodular hyperplasia.
C. CARCIN«I'IAS - although rare, these tumors are generally functional,
large, invasive, and highly malignant. Even by gross and microscopic
appearance, they may be difficult to distinguish from large aden-
omas. In general, however, they tend to be larger and are more
likely to have foci of necrosis. Adrenal carcinomas also have a pro-
pensity to invade veins and lymphatics and can disseminate widely.
I I I. HYPERADRENALISM

A. EXCESS GLUCOCORTIcoms cause Cushing's syndrome. This typically


presents in middle age females and, in its fully developed form, is
characterized by moon facies, buffalo hump, truncal obesity, abdom-
inal stria, hirsutism, menstrual disorders, emotional lability, easy
bruisability, susceptibility to infection, impaired glucose toler-
ance, muscle weakness, osteoporosis, etc. With increased serum
cortisol levels, the pituitary basoI1hils, for unknown reasons,
develop a hya1inized cytoplasm (Crooke s hyaline degeneration). The
di(:l.gnosis of Cushing's syndrome requires the demonstration of excess
cortisol and the lack of suppression of cortisol secretion by
dexamethasone, a potent glucocorticoid which suppresses ACTH.

91
1. PRDlARY HYPERCORTISOLISH - is associated with a low serum ACTH and
is due to either a cortisol-secreting cortical neoplasm (in
which case, the contralateral adrenal gland will usually be
atrophic) or, occasionally, bilateral nodular hyperplasia
(apparently the result of factors other than ACTH stimulation)
2. SECONDARY HYPERCORTISOLISH - is associated with high serum ACTH and
is due to an ACTH-producing pituitary adenoma (Cushing's
disease which is the most common etiology of hypercortisolism)
or ectopic production of ACTH by non-endocrine tumors
(especially bronchogenic carcinoma). This generally leads to
bilateral diffuse or nodular adrenal cortical hyperplasia in
which the zona reticularis (and to a lesser extent the zona
fasciculata) is expanded.
3. IATROGENIC - chronic exogenous steroid use may produce the
clinical manifestations of Cushing's syndrome. Exogenous
administration of steroids, however, is the most common cause
of adrenal cortical atrophy (mostly of the zona fasciculata
and zona reticularis) due to suppression of pituitary ACTH.
B• EXCESS ANDROGENS

1. INHERITED (A1TrOSOMAL RECESSIVE) DISORDERS - affect the activity of


various enzymes in the steroidogenesis pathway resulting in
congenital adrenal hyperplasia. The metabolic defects all
block the production of cortisol thereby eliminating the
negative feedback on the pituitary, increasing the serum ACTH,
and leading to bilateral diffuse or nodular hyperplasia.
Clinical features of these adrenogenital syndromes include:
a. Simple virilizing - partial 21 hydroxylase deficiency
shunts the steroid pathway away from cortisol production
and toward androgen production. In females, this
presents as virilization of infants or, if less severe,
as infertility problems after puberty. In males, this
produces precocious pu~erty.
b. Virilizing and salt losing - 21 hydroxylase deficiency
shunts the steroid pathway away from cortisol production
and toward androgen production, but it also interferes
with aldosterone production leading to hyponatremia and
hyperkalemia.
c. Virilizing and hypertensive - 11 hydroxylase deficiency
shunts pathway away from cortisol and toward androgens
as well as causing buildup of an aldosterone precursor
which is a potent mineralocorticoid that causes
hypertension, hypernatremia, and hypokalemia.
2. in this case, carcinomas are the more
ANDROGEN SECRETING NEOPLASM -
likely cause with adenomas less likely.
C. EXCESS HINERALOCORTIcomS will produce metabolic derangements
characterized by hypernatremia and hypertension, hypokalemia and
hypokalemic alkalosis, and increased urine potassium.
1. PRIMARY HYPERA[J)OSTERONISH(Conn's syndrome) - most often appears
in middle aged females and presents as moderate hypertension
(due to the hypernatremia) which may be associated with
chronic fatigue and weakness (and rarely paralysis) and
cardiac arrhythmias due to hypokalemia. Due to elevated aldos-
terone levels, the serum renin levels are depressed. The most
common cause is an aldosterone-secreting cortical adenoma; but
occasionally idiopathic bilateral adrenal hyperplasia (a con-
dition more frequently seen in children) may be responsible.
2. SECONDARY HYPERA[J)OSTERONISH resul ts from the persistent
stimulation of renin secretion (as the byproduct of renal
ischemia, hypovolemia, or renin producing tumors) and is
therefore associated with increased levels of renin. Clinical
signs and symptoms are similar to Conn's syndrome.

92
IV. HYPOADREN~M

A. ADDISON'S DWASE (primary chronic adrenocortical insufficiency) -


results from any chronic destructive process that destroys more than
90% of cortical tissue. Clinically, there is an insidious onset of
weakness and fatigue, anorexia, weight loss, nausea and vomiting,
hypotension, and hyperpigmentation of exposed and non-exposed skin.
Eventually, serum Na+, Cl-' HCOs-' and glucose levels fall while serum
Ir increases. Serum cortisol and urine cortisol (17-hydroxycortico-
steroids) are decreased in primary adrenocortical failure but the
lack of adrenal response to ACTH stimulation must be demonstrated
before the diagnosis can be made.
1. mIOPATHIC ATROPHY (probably autoimmune in origin) - may occur as
an isolated disease but not infrequently occurs in association
with other autoimmune diseases as part of a spontaneous or
inherited clinical syndrome. A majority of patients have
autoantibodies to adrenal antigens. Grossly the glands are
small with a thin, atrophic cortex draped around a relatively
normal appearing medulla.
2. D~~ OR nwILTRAT~ P~S include tuberculosis,
metastatic tumors, infection, trauma, hemorrhage, amyloid,
etc. The glands are usually grossly enlarged but microscopic-
ally show extensive cortical destruction.
B. ACUTE ADREJW. INSUFfICIENCY

1. ADDISONIAN CRISIS - may develop in patients with Addison's


disease who experience sudden stress (trauma, infection,
surgery, etc.). This may lead to coma, hypotension, shock, and
vascular collapse.
2. IATROSENIC - may result from abrupt withdrawal of exogenous
steroids in a patient whose cortex has been suppressed by
these drugs or inadequate steroid administration to a patient
with non-functioning adrenals. -
3. IIEHORRIIAGE - usually is the result of trauma (especially in
newborns); anticoagulant therapy; or bacteremia (Wsterhouse-
Frfderfchsen syndrome). In the later, patients develop fever
and extensive total body hemorrhage manifested as petechia and
purpura on skin and mucus membranes. This is followed by circ-
ulatory collapse and death unless appropriate intervention is
available.
C. ADREJg:ORTICAL INSUFFICIENCY - is the result of decreased ACTH
SECOtIJARY
secretion from the pituitary from whatever cause. Since ACTH levels
are low, these patients lack the cutaneous hyperpigmentation and the
other clinical features are usually less striking than those in
Addison's disease. Also, the zona glomerulosa is not affected so
there will be little or no signs of hypoaldosteronism. For the
diagnosis to be conclusively established, the capacity of the
adrenals to respond to exogenous ACTH must be demonstrated.

ENDOCRINE PANCREAS
I. DIABETES MELLITUS - is the most common endocrine disease and affects
approximately 2% of the general population in the United States. The
disorder results' from a deficiency in production of insulin and/or
impaired end organ response to insulin. Insulin is an anabolic hormone
involved not only in carbohydrate metabolism but protein and lipid
metabolism as well. Its primary role is to increase the rate of glucose
transport into fibroblasts, adipose cells, and striated muscle but it also
functions to promote hepatic and skeletal muscle glycogen formation,
convert glucose to triglycerides, and promote nucleic acid and protein
synthesis (stimulating cell growth and differentiation). Although other
compounds can stimulate insulin release, hyperglycemia is the primary
stimulus for release and synthesis of insulin. A common feature of all

93
types of diabetes mellitus is hyperglycemia, which over a period of time
can lead to non-enzymatic glycosylation of various proteins (including
collagen) leading, in some cases, to irreversible alterations of basement
membranes, etc.
A. GESTATIONAL DIABETES (GOM) - refers to glucose intolerance that appears
for the first time during pregnancy, and occurs in 3-5% of all
pregnancies. The major impact of GDM is the increased morbidity and
mortality for the offspring. These risks can be reduced or
eliminated by control of maternal hyperglycemia, which can usually
be accomplished with diet alone. Screening for GDM at 24 to 28 weeks
gestation is recommended for all pregnant women. The recommended
screening test for GDM is measurement of venous plasma glucose one
hour after 50 g oral glucose challenge. Women with a positive
screening test (plasma glucose > 140 mg/dL) should have an oral
glucose tolerance test (OGTT) to confirm the diagnosis of GDM.
B. TYPE I (.JUVENILE ONSET, INSULIN DEPENDENT) - is seen in 15% of diabetics
and is the result of inadequate production of insulin. The mass and
number of 6 cells in the pancreatic islets are reduced and there may
be severe atrophy of islets with virtual absence of 6 cells. These
patients are felt to have a genetic susceptibility to developing
autoimmune reactions to their 6 cells which becomes expressed when
triggered by some environmental factor.
C. TYPE II (ADULT ONSET,· NON-INSULIN DEPENDENT) - is, by far, the more common
form of diabetes mellitus. In these patients, although there may be
a deficiency or delay in insulin secretion from the beta cells,
there is also an end-organ resistance to the effect of insulin.
There is a decrease in the number of cellular insulin receptor sites
as well as functional abnormalities in the receptor activities. This
type of diabetes also has a genetic component but it is not well
understood. Often, these patients are obese (which hampers insulin
action even in non-diabetics) but do not need insulin injections.
Microscopically, the pancreatic islets are normal in number but show
variable degrees of hyalinization (this may also occur in
non-diabetic patients and is not diagnostic). Fibrosis of the islets
also may occur. -
D. ctltPLICATIONS - in general, complications appear earlier and are more
severe in the Type I diabetes mellitus.
1. KETOAcmOSIS - is most likely to occur in Type I diabetes and is
basically a defect in lipid metabolism. With a deficiency of
insulin, there is mobilization of adipose tissue resulting in
increased free fatty acids. Glucagon (which is in excess)
increases hepatic oxidation of the fatty acids to produce
ketone bodies. Amino acid uptake by muscle is inhibited and
protein catabolism occurs giving a negative nitrogen balance.
Increased ketogenic amino acids in the serum compounds the
problem. Since cellular uptake of glucose is inhibited, as
glucose levels rise water and electrolytes are drawn out of
the cell and excreted in the urine causing dehydration and
contributing to the developing metabolic acidosis. The
patients may lapse into unconsciousness and have a "fruity"
odor to the breath and labored Kussmaul breathing.
2. VASCULOPATHY - is not specific for diabetics but tends to be
more severe and more widespread. The vascular disease involves
large vessels (atherosclerosis) contributing to cardiovascular
disease, strokes, and gangrene of the lower extremities;
medium sized vessels (hyaline arteriolosclerosis) contributing
to hypertension and diabetic nephropathy; and small vessels
(microangiopathy) contributing to retinopathy, neuropathy,
nephropathy, etc.
3. NEPHROPATHY (most frequent cause of death in Type I) - is
usually heralded clinically by proteinuria and followed by
azotemia leading to renal failure within five years. Major
histologic changes include arteriolosclerosis of afferent And
efferent arterioles; glomerulosclerosis (although not the most

94
frequent pattern, nodular glomerulosclerosis, or Kimmelstiel-
Wilson disease, is virtually diagnostic of diabetes);
pyelonephritis; and necrotizing papillitis.
4. OPmBAUIDPATHY - diabetic retinopathy is one of the more common
causes of blindness in the United States. Degeneration of
vascular pericytes contribute to microaneurysms which form and
may rupture or leak serum to produce the retinal exudates and
hemorrhage seen clinically. Basement membrane thickening and
increased vascular permeability reduces blood flow and
stimulates proliferation of new, but poorly formed, vessels
between the vitreous and the inner tissue layer of the retina
(retinitis proliferans). Leakage of serum and hemorrhages from
these poorly formed vessels ultimately result in fibrosis
which tends to pull the retina from the underlying anchoring
choroid causing retinal detachment. Cataracts and glaucoma are
also more frequently seen in diabetics.
5. NEUROPATHY - damage to Schwann cells and axons by unknown
mechanisms (vascular ischemia or metabolic abnormalities may
play a role) may produce motor and sensory deficits of the
lower extremities. A common manifestation is the neuropathic
ulcer occurring along the pressure points of the sole of the
foot.
6. INFECTION - increased susceptibility to infection probably
relates to poor vascular status and dysfunction of leukocytes.
Similar processes contribute to the poor wound healing seen in
diabetics.
II. PANCREATIC ISLET CELIJ TUMORS - normal islets consist of ex cells
(secrete glucagon), B cells (secrete insulin), 6 cells (secrete somato-
statin), and PP (pancreatic polypeptide) cells (secrete polypeptides whose
functions are not well defined). Other non-hormone producing cells have
been identified including enterochromaffin cells from which pancreatic
carcinoid tumors arise and DI cells which secrete vasoactive intestinal
peptide (VIP). Islet cell tumors may be solitary or multiple, functional
or non-functional, benign or malignant.
A. INSULINOO (B cell tumor) - is the most common. The majority are
solitary adenomas that vary in size but generally are small. 10% are
malignant tumors but, in general, islet cell tumors should not be
considered malignant until there is evidence of metastases. Although
most insu1inomas secrete insulin, only about 20% ever become
clinically apparent. Symptoms, when they occur, are related to
hypoglycemia.
B. GASTRINOO - although a gastrin producing cell has not been identified
in the normal pancreas, certain tumors of pancreatic islet-cell
origin have been shown to secrete gastrin and appear similar to the
gastrin-secreting cells of the intestine and stomach which can also,
but less frequently, form gastrin-secreting tumors (predominantly
duodenal). These tumors are associated with gastric acid hyper-
secretion from hyperplastic gastric parietal cells responding to the
increased gastrin stimulation. The hypersecretion of gastric acid
results in intractable peptic ulceration of duodenum, stomach, and
jejunum (Zollinger-Ellison syndrome). In addition, these patients
develop fluid and electrolyte disturbance secondary to diarrhea and
malabsorption. Slightly over 50% of the gastrinomas are malignant
with most showing metastases at the time of diagnosis. The remainder
are benign adenomas which may be multiple and may be associated with
other endocrine tumors.

95
DIFFUSE ENDOCRINE SYSTEM
"Neuroendocr~ne" or nUD (Amine frecursor !lptake and Decarboxylation) cells,
which have many physical similarities by E.M. (i.e. neurosecretory granules),
have been identified scattered diffusely throughout the body. Originally felt to
be derived from cells that migrated from the neural crest to endocrine glands
during embryogenesis, it is now felt that some cells are somehow secondarily
programmed by the neural crest and/or that non-endocrine tissues, undergoing
neoplastic transformation, may develop the ca~ability to e~ress polypeptide
hormones either through gene rearrangement or 'derepression" of a preexisting
gene. These often come to clinical attention as the result of hyperplastic or
neoplastic functional activity, but also may present simply as unexplained mass
lesions.
1. PARAFOLLICULAR "c" CELLS - of the thyroid give rise to the thyroid
medullary carcinomas.
II. GANGLION CELLS and PHEOCHROMOCYTES - are the differentiated neural
crest cells of the adrenal medulla but also occur in other locations.
Normally, the cells of the adrenal medulla produce and store cate-
cholamines (epinephrine and norepinephrine). Metabolites of the catechol-
amines may be found in urine and include met anephrine, normetanephrine,
vanillylmandelic acid (VMA), and homovanillic acid (HVA). These cells may
give rise to:
A. - derived from ganglion cell line, this is one of the
NEURml.ASTCltA
more common tumors of childhood and, in general, is confined to the
pediatric age range. Although they can occur in a variety of
locations, most arise in the adrenal medulla (50-80%) or the
posterior mediastinum. Usually, the tumors are bulky and soft with
areas of hemorrhage, necrosis, and may have areas of calcification
(which may be a radiographic clue). Histologically, these are one of
the "blue tumors". of childhood consisting of small round cells with
scant cytoplasm and hyperchromatic nuclei. The classic histologic
feature is the presence of Homer-Wright rosettes although these are
not always present. At the time of-diagnosis, the tumors of infancy
are generally localized tumors but those occurring after the age of
two have often metastasized widely with predilection for bone marrow
and liver. In infants, the tumor sometimes mature into a ganglio-
neuroblastoma or a ganglioneuroma. Generally, however, the prognosis
for patients older than 2 years is poor. Clinically, presenting
symptoms are usually due to rapid tumor growth or the effects of
metastases (weight loss, abdominal mass, malaise). Although patients
generally are not hypertensive, most secrete norepinephrine and the
urinary metabolites will be elevated.
B. PHEOCH~ derived from pheochromocytes, these are more
frequently seen in middle aged adults. Known as the "10% tumor", 10%
are extra-adrenal and located in the periaortic ganglia or organ of
Zuckerkandl, 10% are bilateral, 10% are malignant, and 10% are
inherited as an isolated autosomal dominant disease or in
association with other endocrine neoplasms (multiple endocrine
neoplasia syndromes). Grossly, the tumors vary greatly in size but
are generally round and well demarcated from the surrounding adrenal
tissue. As they grow larger, they tend to develop areas of necrosis
and hemorrhage. Clinically, these patients may present with
sustained hypertension, sustained hypertension with paroxysms of
severe hypertension, or intermittent hypertension. Sudden release of
catecholamines (primarily norepinephrine) into the circulation may
result from manipulation of the tumor, stress, postural changes,
etc., and result in paroxysms of hypertension, diaphoresis, anxiety,
nausea and vomiting, abdominal pain, tremors, or more ominously MI
or CVA. Urinary catecholamines, metanephrine, and VMA are elevated.
III. PARAGANGLION CELLS - are similar to the pheochromocytes but are
scattered throughout the body associated with the autonomic nervous
system. Paragangliomas are rare tumors but, like pheochromocytomas, have
the capacity to secrete catecholamines. Those derived from the
parasympathetic chemoreceptor system (carotid body tumors, glomus jugulare

96
tumors, aortic body tumors, etc.) tend to be nonfunctional tumors. Those
derived from the abdominal aorticosympathetic paraganglia (retroperitoneal
paragangliomas or extraadrenal pheochromocytomas) are more likely to be
functional tumors. Generally, they are benign but some may recur locally
and, rarely, a few may metastasize widely.
IV. MERKEl.. CELLS - are present in the basal layer of the epidermis and
produce rare dermal carcinomas predominantly of the head and upper
extremities in elderly patients. Electron micrographs demonstrate
neurosecretory granules but elaboration of endocrine substances is not
generally seen. These tumors may disseminate widely and prove fatal in 15-
20% of cases.
V. ENTEROCHROMAFFIN CELLS - give rise to carcinoid tumors (argentaffin-
omas). Occurring most frequently in the GI tract (appendix, small bowel,
rectum), these potentially malignant tumors may be found anywhere "neuro-
endocrine" (APUD) cells are encountered. They tend to be small mucosal
lesions that occasionally are multiple. They may be asymptomatic or
produce symptoms related to local invasion, obstruction (from desmoplastic
tissue response to the neoplasm), or elaboration of secretory products.
The metastatic potential seems to be related to the site of origin, the
depth of penetration of the tumor, and its size. A majority of extra-
appendiceal carcinoids are associated with metastases to regional lymph
nodes or to the liver at the time of diagnosis. In patients who have
metastatic liver involvement from intestinal carcinoids or who have extra-
intestinal carcinoids, a symptom complex known as carcinoid syndrome may
occasionally occur. This is characterized by vasomotor disturbances
(transient flushing and cyanosis of the skin), intestinal hypermobility
(diarrhea, cramps, nausea, vomiting), bronchoconstriction (cough, dyspnea,
wheezing), cardiac involvement (endocardial fibrosis and valvular
deformity of the right side of heart) and hepatomegaly (from hepatic
metastases). These symptoms are probably mediated by tumor secretion of
serotonin (5-hydroxytryptamine), histamines, and kinins. Increased levels
of 5-hydroxyindolacetic acid (5-HIAA), a metabolic product of serotonin,
may also be found in the serum and urine. 5-year survival in appendiceal
carcinoids approaches 100%, rectal carcinoids 85% and gastric, small
bowel, and colonic carcinoids 50%. In patients who have the carcinoid
syndrome, however, the disease tends to be progressive with an ultimately
fatal outcome.

MULTIPLE ENDOCRINE NEOPLASIA SYNDROMES


I. MEN I (WERMER SYNDROME) - consists of hyperplasia or neoplasia of the
pituitary, parathyroid, and pancreatic islet cells and is frequently
associated with the Zollinger-Ellison syndrome. Less often, the thyroid
and adrenal cortex may also be involved.
II. MEN II
A. ~ IIa (Sipple syndrome) - consists of pheochromocytoma, thyroid
medullary carcinoma, and occasionally parathyroid adenoma or
hyperplasia.
B. MEN ub (sometimes classified as MEN III) - same as MEN IIa but
without parathyroid disease and with the addition of multiple
mucocutaneous neuromas. It tends to run a more aggressive course.
III. CLINICAL PRESENTATION is variable but may include hypertension
(pheochromocytoma), ulcers (gastrinoma), Cushing's syndrome (adrenal
cortical hyperplasia/adenoma), hypocalcemia (parathyroid
hyperplasia/adenoma), hypoglycemia (insulinoma) , or mass effects
(medullary carcinoma).

97
BREAST

I. REVIEW OF NORMAL

A. ANAnl'IIC STRUCTURE -at birth, rudimentary branching ducts are present


beneath the nipple. In response to increasing sex hormone levels
during puberty, there is rapid ductal growth and branching, prolif-
eration of fibrous interductal stroma, and development of rudiment-
ary gland buds emanating from small terminal ductules. The nipple
and outer portion of lactiferous ducts are lined by stratified
squamous epithelium which, as ducts get smaller, turns into pseudo-
stratified columnar epithelium (segmental ducts), then double
layered cuboidal epithelium (subsegmental ducts), then flattened
cuboidal epithelium with myoepithelial cells (terminal ducts and
ductules). The terminal ductules and the gland buds (terminal duct-
lobular unit) are embedded in a loose myxomatous stroma (periductal
and intralobular connective tissue) and individual lobules are
separated by a more dense fibrous (interlobular) stroma.
B. PHYSIOLOGIC CHANGES - under estrogen influence, the epithelium of the
terminal ductules and gland buds proliferate and increasing proges-
terone levels during later part of the menstrual cycle stimulate
intralobular stromal growth and edema. Under the influence of
placental hormones, numerous secretory glands branch out from the
gland buds so that with increased gestational age, the breast
becomes packed with secretory glands which are lined by a single
layer of cuboidal cells and which obscure the stromal component of
the breast parenchyma. After lactation the glands regress and
atrophy. At menopause, there is further ductular and glandular
atrophy which may result in the appearance of small cysts (cystic
atrophy) and replacement of the fibrous stroma by adipose tissue.
II. DEVELOPMENTAL DISORIlERS

A. POLYnfELIAfPOLYHASTIA at about six weeks gestation, bilateral


thickenings of the epidermis forms ridges (milk lines) running
between the upper and lower limb buds along ventral surface of body.
These ridges progressively atrophy except for the superior portions
which give rise to the nipples. Failure of the milk line to regress
may lead to supernumerary nipples (polythelia) or breasts
(polymastia).
B. CONGENITAL INVERSION OF THE NIPPLE - may simulate the nipple retraction
sometimes seen in association with breast malignancies.
III. HYPERTROPHY

A. NEONATAL - during gestation, maternal hormones stimulate proliferation


of ductal epithelium and periductal connective tissue and,
occasionally, abortive secretory activity. These changes quickly
regress after birth due to withdrawal of hormonal support.
B. PREPUBERTAL - usually bilateral, this suggests excess sex hormone
levels (ovarian, adrenal, pituitary tumors, etc). These changes
usually regress with lowering of hormone levels.
C. VIRGINAL - unilateral or bilateral enlargement of obscure etiology.
It may be due to increased sensitivity of breast tissue to hormonal
stimulation. Generally this does not regress and may require
reduction mammoplasty.
D. GYNE~IA - refers to hypertrophy of the male breast and is usually
the result of excess estrogen influence (most often due to hepatic
cirrhosis, lung carcinoma, estrogen therapy, testicular tumors,
aging, etc) or high prolactin levels (pituitary tumors).

98
IV. INFLAMMATORY DISK<\SE
A. ~ KMrrITIS - is most commonly seen in lactating women where cracks
or fissures in the nipple provides portal of entry for bacteria.
Clinically the breast is red, swollen, and painful. Tissue necrosis
with resultant scar formation may lead to dimpling of skin or nipple
retraction simulating clinical signs of malignancy.
B. PLASl'IA CELL ItASTITIS (mammary duct ectasia) - is uncommon and of obscure
etiology, but may be due to chronic obstruction of lactiferous ducts
by inspissated lipids with ultimate rupture. Clinically it is
typified by an ill-defined area of pain, tenderness, and induration,
and histologically there is dilatation of ducts containing cellular
debris and lipid-laden macrophages, erosion of the duct epithelium,
and marked periductal and stromal granulomatous inflammation with
large numbers of plasma cells. .
C. FAT NECROSIS - is a localized, often granulomatous, inflammatory
reaction to necrosis of fatty tissue. With resolution, a dense
fibrous scar develops which may contain dystrophic calcifications or
cause skin dimpling or nipple retraction simulating malignancy.
V. FIBROCYSTIC CHANGE - is the most common "disorder" of breast and is the
result of exaggeration and distortion of normal cyclic alterations that
occur during the menstrual cycle and most frequently becomes symptomatic
in mid to late reproductive life. Most likely related to relative
hyperestrinism (reflected by overstimulation of terminal ductal epithelium
and stromal growth) and/or an abnormal end-organ response to hormones,
there is a wide variability in clinical manifestations, gross appearance,
and histology.
A. FlB~IS - is a common denominator in most patients with fibrocystic
disease but when occurring as an isolated lesion (an infrequent
occurrence), often presents as a unilateral, upper outer quadrant,
poorly defined, rubbery, mobile mass which may be tender to
palpation. Proliferation of fibrous stroma compresses terminal ducts
and glands with obliteration of periductal and intralobular loose
connective tissue. There is no predisposition to cancer.
B. CYSTIC CHANGE - stromal hypertrophy and progressive hyperplasia and
dilatation of terminal ducts without complete regression after
menses results in a diffuse ill-defined increase in tissue
consistency punctuated by discrete nodules. Usually multifocal and
often bilateral, these macrocysts may produce pain and tenderness
(especially prior to menses). Solitary cysts, however, may be
clinically confused with cancer especially if microcalcifications
develop in the wall. Grossly, cysts may be brown or blue (blue dome
cysts) depending on fluid contained within. Cysts are usually lined
by flattened epithelium but apocrine metaplasia is not uncommon and
is almost always an indicator of benignity. The presence of cysts
does not imply any increased risk for the development of cancer.
C. ADENOSIS - intralobular fibrosis with proliferation of terminal ducts
and gland buds results in a firm, relatively well defined, often
tender, mass lesion. Usually, but not always, it is associated with
other cystic changes. Florid ductular proliferation may result in
"back-to-back glands" (adenosis) and fibrosis may compress ducts
into nests and ribbons of epithelial cells (sclerosing adenosis)
easily confused with cancer. It does not, however, carry an
increased risk for subsequent development of cancer.
D. EPI11IELIAL HYPERPLASIA - hyperplasia of terminal duct and ductular
epithelium may partially occlude lumina or produce papillary
projections into lumina (ductal papillomatosis). In general, these
changes do not cause discrete mass lesions but are often associated
with other fibrocystic changes that do. Microcalcifications may be
present. The association between fibrocystic disease and breast
cancer is proportional to the extent of epithelial hyperplasia and
the degree of epithelial atypia.

99
VI. BENIGN NEOPLASMS

A. FIBROADElDIA - is the most common benign breast tumor and possibly is


the result of focal hypersensitivity to estrogen. These tend to
occur in women under the age of 35 and typically present as a
solitary, well defined, discrete, rubbery, mobile 2-4 cm mass
arising in the upper outer quadrant. Histologically there may be a
prominent glandular component (pericanalicular), a prominent stromal
pattern (inrracanalicular), or a mixture of the two. Large (10-15
cm), bulky, lobulated, gianr fibroadenomas may distort breast and
cause pressure necrosis of overlying skin.
B. PHYLLmES TUMOR (cystosarcoma phyllodes) - is similar to the fibro-
adenoma but it tends to have a more cellular myxoid stroma which, in
a small number of cases, may become malignant resulting in a sudden
and rapid increase in size. In general, phyllodes tumors behave
favorably if the tumor is < 5 cm and the patient < 20 years old.
C. INTRADUCTAL PAPILLOItA is a solitary papillary neoplasm usually
occurring in the lactiferous duct of middle aged to elderly females
that produces a watery or bloody nipple discharge and small
subareolar mass. A minority of cases may show malignant change
(inrraducral papillary carcinoma), but even so, does not tend to
metastasize and carries a favorable prognosis.
D. NIPPLE APENOO is a benign sweat gland tumor (hidradenoma
papil1 iferum) that occurs most often in elderly women. Located
immediately beneath the nipple, it may lead to nipple ulceration.
VII. MALIGNANT NEOPIJASMS - second only to lung cancer as the leading cause
of cancer deaths in women, 5-8% of all women will develop breast cancer.
Occurrence is rare before 25 years of age, but the incidence increases
with age to peak levels during the perimenopausal years. Formerly, breast
cancer carried a poor prognosis but with improved diagnostic techniques
and earlier diagnosis, currently the overall 5-year survival rate is
approximately 75%.
A. PREDISPOSING FACTORS

1. GENETIC FACTORS - a family history of breast cancer (especially


premenopausal cancer in mother or sister) greatly increases
risk over normal population. Family history of other types of
cancer (except for ovarian and endometrial carcinomas) does
not appear to increase risk. .
2. HORtlONAL FACTORS - breast cancer is more common in those patients
whose breasts have not achieved full anatomic maturity and
physiologic function (1. e. prolonged lactation). This includes
nulliparous women, women who have not had children until late
in life (after 35), and women who have had children but were
unable or unwilling to nurse. This suggests a role of estrogen
activity unopposed by progesterone over a long reproductive
life span as an etiologic factor. This is supported by an
increased incidence of breast carcinoma in women with early
menarche and/or delayed menopause; in post-menopausal women
with increased estrogen levels (due to tumor, ovarian cortical
hyperplasia, exogenous administration, obesity, etc); in those
with a previous history of breast, ovarian, or endometrial
cancer; and even in men receiving estrogen therapy for
prostatic cancer.
3. ENVIRONMENTAL FACTORS - there has been some statistical evidence
to implicate radiation exposure, geographic location, high
intake of dairy fats, etc.
B. SIGNS AND SYMPTOMS a firm, non-tender, non-mobile "lump" with
ill-defined b01:ders may be fixed to skin or deep muscle. Fibrous
desmoplastic response may cause dimpling of skin or deviation or
retracrion of nipple. LymRhatic involvement of skin may result in
edema and "peau d' orange I appearance. With routine mammography,
however, more and more subclinical lesions are being identified.

100
C. LOCATION - most common location is upper outer quadrant (45%) followed
by the nipple or subareolar area (20%), upper inner quadrant (15%),
lower outer quadrant (10%), and lower inner quadrant (10%).
n. C~IFICATION - almost all malignant tumors arise from the terminal
duct-lobular unit and are therefore adenocarcinomas although they do
not always have a glandular architecture. 90% of these are derived
from the terminal ductal epithelium with an additional 7-8% arising
from lobular (glandular) epithelium.
1. CARCINOMAS ARISING FROM DUCTAL EPITHELIUM

a. Intraductal carcinoma-in-situ (nCIS) - generally occurs


after menopause and most are discovered on biopsy
because of suspicious areas identified on mammography.
Arising from the terminal ducts, malignant epithelial
cells proliferate, with differing histologic patterns,
within the confines of the ductal basement membrane. The
most common pattern is for tumor cells to completely
fill and distend the duct with central necrosis and
dystrophic calcification (comedocarcinoma). In other
instances, tumor cells mar. brid~e the lumen of the ducts
to create evenly rounded I holes (cribriform fattern) or
they may have a papillary appearance, a so id appear-
ance, etc. nCIS is generally slow growing and if no
infiltration is identified (comedocarcinoma is the most
likely to show microinvasion and is associated with a
higher risk of recurrence), the prognosis is generally
good. Without treatment approximately 30% of these
patients will develop invasive carcinoma, and a small
proportion may develop invasive carcinoma of the
contralateral breast.
b. Infiltrating ductal. NOS (scirrhous) - by far the most
conwon form of breast cancer, this has a peak incidence
around menopause and presents as a hard, gritty,
gray-white mass with irregular or indefinite borders.
Although there is microscopic variability, there is
usually a dense hyalinized fibrous tissue stroma
(desmoplasia) infiltrated by nests and strands of
malignant cells. A majority have axillary lymph node
metastases at the time of diagnosis and the five year
survival is only a little better than 50%.
c. Medullary carcinoma - these uncommon tumors usually
present as a deeply situated, relatively large circum-
scribed mass. The tumor appears soft and may show areas
of necrosis or hemorrhage. Microscopically, it is char-
acterized by sheets of large malignant cells with scanty
stroma and accompanied by a prominent lymphocytic infil-
trate. The tumor tends to have "pushing" rather than
infiltrative margin and are generally slow growing and
slow to metastasize. Ten year survival is around 85%.
d. Mucinous (colloid) carcinoma - occurring in older women,
these tumors are relatively rare but are large and bulky
with a mucoid gelatinous appearance. Microscopically,
malignant cells may be floating in pools of mucin, form
well-defined mucin producing glands, or sheets of
"signet ring" cells. These tend to be slow growing, are
slow to metastasize, and have a relatively good
prognosis.
e. Paget's disease of the breast - usually occurs in older
women and is characterized by long history of burning,
itching, or soreness of the nipple which may produce
fissuring, weeping, crust formation and, ultimately,
ulceration. These nipple changes are associated with an
underlying carcinoma (usually of a major excretory duct)
which mayor may not be clinically evident. Microscopic-
ally the nipple changes are characterized by large,

101
pale, vacuolated cells (Paget cells) in the epidermis
which stain positively with PAS and mucin stains and
represent intraepithelial metastases from the underlying
carcinoma. Prognosis depends on the extent of the
underlying tumor.
2. CARCIlDIAS ARISING FlO! UEULAR EPITHELIUti

a. Lobular carcinoma-in-situ (LeIS) - arises from the


terminal ramifications of the intralobular terminal
duct. It does not cause mass lesions and usually is
discovered in tissue biopsied for other reasons. The
terminal ducts and acini become distended by prolif-
eration of malignant cells which have a monotonous,
deceptively benign appearance. I t may be multifocal
within the involved breast and is bilateral in about
25%. There is some question as to whether this lesion
itself progresses to invasive carcinoma, but it does
imply a high risk of developing subsequent carcinoma.
b. Infiltrating lobular carcinoma - although much less
common, these tumors have similar gross, microscopic,
and prognostic characteristics as infiltrating ductal
carcinoma. The cells are smaller and tend to be arranged
in an "indian file" pattern or oriented in concentric
rings around normal ducts (targetoid pattern). At one
time these were felt to have a greater incidence of
bilaterality than ductal carcinoma but this may not be
true. Several studies have shown approximately equal
incidence of bilaterality and evidence suggests that
ductal carcinoma is also associated with an increased
risk of contralateral malignancy. Lobular carcinoma
does, however, show a somewhat greater incidence of
axillary metastases than infiltrating ductal carcinoma.
3. is not a specific type of cancer but
"INFLAHl'lATORY" CARCINotIA -
rather is a clinical term used to describe any breast
carcinoma which has widely infiltrated and obstructed the
subepidermal lymphatics and blood vessels to cause the sudden
onset of heat, redness, and swelling that mimics an acute
inflammation. These tend to occur in the large pendulous
breasts of young women, especially during lactation when the
breast contains wide-open blood and lymphatic channels. As
might be surmised, the prognosis is poor.
E. SPREAD - the principal route of metastasis is via lymphatics,
especially the axillary lymph nodes since the majority of lesions
are found in the upper outer quadrant. However, there may also be
involvement of the internal mammary chain and supraclavicular nodes.
Distant metastases are found most frequently in the lung, bone
(osteolytic), liver, adrenal, brain, and ovaries.
F. STAGING

1. STAGE I - tumor less than 2 cm., no palpable lymph nodes, no


evidence of distant spread (85% survival).
2. STAGE II - axillary lymph node metastasis QL tumor between 2
and 5 cm with no palpable lymph nodes and no evidence of
distant spread (66% survival).
3. STAGE III - tumor greater than 5 cm, clinically palpable lymph
nodes with fixation, no evidence of distant spread (41%
survival) .
4. STAGE IV - distant metastases (10% survival).

G. THERAPY

1. LOCALIZED CARCINmA (Stage I or II) - local excision with clear


margins, axillary node removal and subsequent radiation;
modified radical mastectomy with or without radiation;
possible adjuvant hormonal or chemotherapy.

102
2. may be treated by hormonal manipulation (if
HETASTATIC DISEASE -
tumor is estrogen receptor positive (ER+) about 50% will
respond to hormonal therapy. If the tumor is progesterone
receptor positive (PgR+), about 75% will respond. I f the tumor
is estrogen receptor negative (ER-), there is only about 15%
that will respond); cytotoxic drugs; and/or endocrine
ablation.
H. include fewer than 3 axillary lymph nodes
FAVORABLE PROGNOSTIC FACTORS -
showing metastasis; tumor less than 2 cm in size (prognosis improves
with decreasing tumor size); progesterone/estrogen receptor-
positivity; low nuclear grade; mucinous or papillary histologic
pattern; and increasing number of cells in the S-phase fraction.

103
FEMALE GENITAL TRACT

1. REVIEW OF NORMAL - in both sexes, the primitive gonad forms along a


genital ridge from specialized mesenchyme that is covered by the coelomic
(germinal) epithelium. At about six weeks gestation, an invagination of
this coelomic epithelium forms the paramesonephric (muellerian) duct which
lies adjacent to the previously existing mesonephric (wolffian) duct. In
the female, the paramesonephric duct develops to ultimately form the
fallopian tubes and, by fusing together in the midline, the uterus,
cervix, upper vagina. The lower 2/3 of the vagina develop from the uro-
genital sinus which grows cephalad to fuse with the upper portion. At the
same time, the mesonephric duct degenerates. Aberrations in this
developmental scheme can lead to such unusual conditions as agenesis of
structures, septate structures, duplicated structures, etc.
II. SEXUALLY TRANSMfITED DISEASES

A. GONOCOCCUS (N. GONORRHOEA) - the initial site of infection in women is


usually Bartholin is or Skene's glands. Acute inflammation of
Bartholin's gland may lead to occlusion of the duct and abscess
formation. With resolution of suppurative exudate, Bartholin's duct
cyst may result. The organism spreads upward over the mucosal
surface secondarily infecting the endocervix and creating a purulent
exudate. Traversing the endometrial cavity (usually during or right
after menses), gonococcus is one of the leading causes of purulent
salpingitis. If untreated, the fimbria agglutinate, pus accumulates
and a pyosalpinx is formed. With time, pus is reabsorbed and
replaced by clear fluid to form hydrosalpinx. In males, the organism
infects periurethral glands and can spread to prostate, seminal
vesicles, etc. and generally presents with dysuria and a milky
discharge from the urethra.
B. CHLAlfiDIA (c. TRACHCl'lATIS) - is a common organism that infects the same
tissues as gonococcus but produces few clinical symptoms. In
females, it probably is responsible for a majority of the cases of
salpingitis resulting in infertility. In males, it may produce a
non-specific urethritis with mild dysuria and a mucoid discharge.
C. HERPES SDiPLEK. II - this causes painful red papules which develop into
crops of vesicles which then ulcerate. Dysuria is frequent and
systemic symptoms of fever and malaise may be present. Ulcers heal
in 10-20 days but may recur over a lifetime. Histologically, there
are intraepidermal vesicles formed by acantholysis due to balloon
degeneration of infected epidermal cells. Characteristic features on
cytologic scrapings of the lesions include large multinucleated
giant cells and intranuclear eosinophilic inclusions.
D. CONDYUI'IA ACUHINATA (IMIAN PAPILUI'IA VIRUS) - appear as either flat or
verrucous alterations of squamous epithelium (venereal warts). They
are most frequently seen, but not limited to, the perineal/peri-
anal/penile skin but may also affect the vagina and cervix. The
verrucous condylomas show hyperkeratosis, parakeratosis, acanthosis
and koilocytosis of the epidermis. Flat condylomas may result from
infection by different viral strain and do not have the verucoid
appearance but do tend to have more cellular atypia of epidermis.
These also occur in the vagina and cervix and may be associated with
subsequent dysplasia/neoplasia.
E. SYPHILIS (T. PALLmw) - the spirochete penetrates small abrasions in
the skin or mucous membranes and after an incubation of 3 weeks
causes a painless papule at the site of infection which soon
ulcerates (chancre) but will eventually heal spontaneously. Weeks to
months later, during the stage of secondary syphilis, systemic
symptoms (headache, low-grade fever, lymphadenopathy, etc) develop
and syphilitic warts (condylom8 l8t8) appear. These, too, heal
spontaneously to possibly be followed by the cardiovascular and
neurologic effElcts of tertiary syphilis.

104
F. CHANCROm (M. DUCREYI) - causes genital ulcers ("soft chancre") and
regional lymphadenitis. The latter, if untreated, may progress to
form an inguinal abscess (bubo). Histologically, the most notable
feature below the ulcer is lumenal occlusion and thrombosis of blood
vessels due to rapid endothelial cell proliferation. The organism
can only rarely be shown in tissue with special stains but smears of
the lesion often demonstrate the short, gram-negative rods.
G. GRANUUI!A INGUINALE (CALYI1tlATOBACTERIUtI GRAH1IUI'IATIS) - occurs in perianal or
genital area as a solitary lesion or a small group of lesions that
appear as ulcers filled with granulation tissue. Lesions spread by
peripheral extension and can become quite large. The dermis contains
a dense infiltrate of macrophages and plasma cells and occasional
neutrophilic abscesses. The most conspicuous histologic finding is
the presence of intracytoplasmic inclusion bodies (Donovan bodies)
within macrophages. Their demonstration is requisite for the
diagnosis which may be accomplished by special stains of tissue or
smears of biopsy material.
H. CAHl)mA (c. ALBICANS) - this is a common disorder in females especially
in diabetics and in conditions of high progesterone states (preg-
nancy, BCP, etc.). Vaginal infections are characterized by itching
and a white, curdy discharge. Diabetic vulvitis results from repeat-
ed candida infections and causes a thickened, red, pruritic vulva.
I. TRICIKItONAS (T. VAGINALIS) - in females who are symptomatic (itching,
dysuria, dxspareunia), the vaginal mucosa has a bright red
"strawberry' appearance with thick, frothy, grayish discharge. Wet
mount may identify the flagellated pear shaped protozoan. In males,
it is generally asymptomatic.

VULVA/VAGINA
I. DEVELOPMENTAL DISORDERS

A. may not be recognized until puberty when there is


ItIPERFORATE HYtlEN -
failure to menstruate. Physical exam may reveal vagina, uterus, and
fallopian tubes distended with blood (hematocolpos, hematometria,
and hematosalpinx).
B. GARTNER'S DUCT CYST (remnants of mesonephric/wolffian duct) - may
appear in anterolateral vaginal wall submucosa and measure 1 - 2 cm
in size.
II. INFLAMMATORY DISORDERS - the vulva and vagina are prone to infection
because of the warm, damp environment and the effects of estrogen.
Relative hypoestrinism in the post-menopausal periods tends to decrease
vascularity and result in epithelial atrophy with decreased cornification
of the epithelium. This makes the epithelium more prone to trauma and
infection. In elderly patients, this is called senile atrophic vulvitis
or vaginitis.
III. VULVAR DYSTROPHY - generally occurs in middle aged to elderly post-
menopausal women as white plaque-like lesions of skin (leukoplakia) which
are often mUltiple. The primary symptom is itching.
A. ATROPHIC DYSTROPHY (1 ichen sclerosis) - is characterized by atrophic
labia, narrowed introitus, and smooth vulvar skin with yellow-blue
papules that coalesce into thin gray parchment-like areas that are
susceptible to trauma and infection. Histologically, characterized
by epithelial atrophy, dermal fibrosis, and nonspecific chronic
inflammation. It does not progress to cancer.
B. HYPERTROPHIC DYSTROPHY (with or without atypia) - clinically similar to
atrophic dystrophy, microscopically there is hyperkeratosis and
acanthosis with or without cellular atypia. Those lesions which do
have atypia have a low « 10%) malignant potential. This form may
also be seen in prepubertal females but usually regresses at the
time of puberty. .

105
IV. NEOPLASIA

A. VULVA

1. HmRADEJDIA PAPILLIFERUII - is benign tumor derived from apocrine


sweat glands which presents as a nodular mass usually on or
between labia. Histologically there is a complicated papillary
architecture which may be mistaken for adenocarcinoma.
2. BmEN'S D~ (squamous cell carcinoma-in-situ) - appears to
be increasing in frequency and tends to affect women under 40.
It may present as raised, red, velvety lesions often involving
the labia but also has a tendency to be multicentric (espec-
ially with involvement of the periclitoral and 'perianal skin).
The histologic severity varies but condylomatous features may
be present. HPV can be identified in 80% of these lesions.
Only about 10-20% of patients will progress to invasive
squamous carcinoma of the vulva (usually elderly or immunosup-
pressed) but 25% are associated with carcinoma-in-situ or
invasive carcinoma of the cervix and/or vagina.
3. SQUAtIOUS CELL CARCINCIIA - usually occurs in post-menopausal women
but has shown an increasing frequency in younger populations.
Clinically looks like vulvar dystrophy in early stages with
itching and local discomfort the predominant symptoms.
However, with time, they will become firm and indurated with
possible central ulceration. They may be multicentric - both
geographically and temporally. Although most squamous
carcinomas are histologically well differentiated, at time of
diagnosis 65% have metastasized to regional lymph nodes
(inguinal, femoral, pelvic) and will later metastasize to
viscera. Lesions > 2 cm and with lymph node metastases have
poor 5 year survival (25%). Smaller lesions treated with
vulvectomy and pelvic lymphadenectomy have 60-80% 5 year
survival. Verrucous carcinoma (which may also occur on the
penis) is a variant of squamous cell carcinoma in which the
cells are extremely well differentiated although the gross
appearance is that of a large fungating tumor. Typically this
cancer will invade locally but will not metastasize.
4. EXTRAIIAtItIARY PAGET'S DISEASE - presents as a red, crusted, well
demarcated lesion usually on labia majora in women who give a
history of chronic pruritus and irritation. Vacuolated tumor
cells are present singly and in clusters within the epithelium
and probably arise from adnexal epithelium. They are PAS and
mucicarmine positive and are usuallf confined to the epidermis
and skin appendages. Unlike Paget s disease of the breast,
underlying adenocarcinoma is uncommon. This intraepithelial
neoplasia may persist for years with lateral spread but with-
out invasion. Once invasion occurs, however, the prognosis is
poor.
B. !&WI6 - the most frequently identified malignancy involving the
vagina is metastatic and primary malignancies are less common.
1. most are well-differentiated and arise
SQUAI'IOUS CELL CARCDOtA -
in the posterior fornix and invading cervix and perivaginal
structures by direct extension. The upper 1/3 of vagina drains
to iliac nodes while lower 2/3 drain to femoral, inguinal, and
pelvic nodes. It may come to clinical attention as the result
of vaginal discharge or spotting and is primarily seen in
older women. Prognosis depends on the clinical stage.
2. ADENOCARCD«ItA - arising on the anterior wall (upper 1/3), clear
cell carcinomas may occur rarely (.001%) in young (15 - 27 yo)
daughters of women treated with DES during pregnancy. The
carcinoma is preceded by the more common adenosis (which
occurs in 30-50% of DES exposed patients and represents a
persistence of fetal histology where there is a delayed
transformation of glandular epithelium to squamous

106
epithelium). Adenosis appears as red areas contrasted against
the normal pink mucosal background to· give a "cobblestone"
appearance but often disappears by the 4th decade.
3. SARCmA BOTRYOmES - is a rare form of rhabdomyosarcoma that
occurs in young children, has a polypoid grape-like
appearance, invades locally, and metastasizes widely. Poor
prognosis in general.

CERVIX

I. REVIEW OF NORMAl" - the junction between the squamous epithelium of the


exocervix and the columnar mucin secreting epithelium of the endocervix
(transition zone) migrates anatomically over time. At birth, columnar
epithelium is present on the exocervix but progressively recedes due to
replacement by squamous metaplasia. Although occasionally gradual, the
squamo-columnar junction is usually abrupt. In perimenopausal and
postmenopausal women, the junction may be high in the endocervical canal
and may interfere with the accuracy of routine PAP screening. The mucus
secreting endocervical "glands" are, in reality, clefts. Cervical mucus
changes with hormone levels and can be used as a diagnostic tool. If
cervical mucus is air-dried on a glass slide, a crystalline or "ferning"
pattern indicates estrogen predominance while an amorphous appearance
indicates progesterone predominance. This can be used to differentiate
between the two major causes of amenorrhea - pregnancy (increased
progesterone) and anovulation (increased estrogen).
II. CERVICITIS

A. ACmE - gonococcus is probably the most commonly identified agent.


Although the cervix is frequently a secondary site of infection, a
thick purulent discharge is usually present. Chlamydia may be a more
common etiologic agent but is less symptomatic and more difficult to
diagnose. The organism is hard to culture but may be identified by
monoclonal antibodies. Other organisms may cause cervicitis
particularly post-abortion, post-partum, or post-trauma. Unlike
gonococcus, other organisms tend to spread via lymphatics (rather
than over the mucosal surface) and therefore cause less purulent
exudation and discharge but may lead to bacteremia, peritonitis,
bowel obstruction from adhesions, etc.
B. CHRONIC - nonspecific lymphocytic infiltrates are present to some
extent in virtually all adult women. Severe chronic inflammations
may develop lymphoid follicles (follicular cervicitis). Stenosis and
obstruction of the endocervical "glands" may result in mucus reten-
tion and cystic dilatation (Nabothian cysts). This imparts a pebbly
appearance to the endocervical canal but has little clinical
significance.
III. POI"YPS
A. INF~TORY - occur in - 5% of women and are usually solitary. They
arise from the endocervical canal and produce soft sessile or
pedunculated lesions of varying size composed of loose fibromyxoma-
tous stroma containing cystically dilated glands, thick-walled blood
vessels, and varying degrees of inflammatory infiltrate. With
trauma, the surface mucinous epithelium erodes and easily bleeds so
that they come to clinical attention due to irregular vaginal
spotting.
B. HYPERPLASTIC (microglandular hyperplasia, "pill" polyp) - are usually
seen in hyperprogesterone conditions (pregnancy, BCP) and consists
of tightly packed hyperplastic endocervical glands. Clinically looks
similar to inflammatory polyps but potentially may be confused with
adenocarcinoma by microscopic appearance.

107
IV. CERVICAl, CARCINOMA - although adenocarcinomas arising from endocervical
glandular epithelium do occur (10% of cervical malignancies), the vast
majority of cervical malignancies are squamous cell carcinomas. In the
U.S., the frequency of cervical dysplasia/neoplasia appears to be
increasing but the frequency of invasive carcinoma and mortality due to
cervical carcinoma has decreased.
A. ETIOLOGY- a recent study suggests that women who have used oral
contraceptives have a significantly greater risk of developing
cervical carcinoma and that the risk increases in proportion to the
duration of usage. The bottom line risk factors for developing
cervical carcinoma, however, appear to be early onset of sexual
activity, increasing numbers of sexual partners, and the promiscuity
of sexual partner. With the exception of cigarette smoking, all
other risk factors (lower socioeconomic groups, race, number of
children, frequency of sex, circumcision of partner, etc) are
probably related to the first three and suggests that a sexually
transmissible agent may be involved. Suspected agents include:
1. HPV (strains 16, 18, 31) - the koilocytotic changes charac-
teristic of cells infected with the human papilloma virus
(HPV) are seen in dysplastic epithelium and HPV DNA has been
found in both dysplastic and neoplastic epithelial cells. HPV
is a transforming virus and strains 16, 18, and 31 are felt to
represent high risk strains in terms of carcinogenic potential
while strains 6 and 11 (those most typically associated with
condyloma acuminata) are felt to be lower risk strains.
2. HSV I I - statistically, women with genital herpes have a higher
incidence of dysplasia and carcinoma than the general
population. Antibodies to HSV II antigens (specifically AG-4)
are higher in women with dysplasia/neoplasia and the AG-4
antigen can be demonstrated in neoplastic cells in 90% of
cervical cancer biopsies compared to 10% of non-cancer
cervical biopsies. Herpes virus is primarily a cytopathic
virus but it may be that HSV II hastens the transforming
properties of HPV.
B. PATHOOEHESIS - invasive carcinoma follows a long history of progres-
sively worsening dysplastic changes. Dysplasia, therefore, must be
considered a premalignant lesion. Atypical intraepithelial cells are
characterized by increased N/C ratio, greater nuclear pleomorphism,
increased mitoses, and loss of polarity. These first appear in basal
layer and, with increasing severity of dysplasia, progressively
involves a greater percentage of the epithelial thickness and dis-
rupts the normal maturation sequence. Dysplasia/neoplasia (cervical
intraepithelial neoplasia) virtually always begins at squamocolumnar
junction and can be graded as mild dysplasia (CIN I), moderate dys-
plasia (CIN II), severe dysplasia (CIN III), or carcinoma-in-situ
(also CIN III). The newer Bethesda classification utilizes two
categories (low grade and high grade intraepithelial neoplasia).
Each level of dysplasia/CIN may persist or progress to a more severe
level. The more severe, the shorter the time interval to the
development of CIS. Whether or not dysplasia can regress has been
the subject of controversy but at least it can be easily eradicated.
C. EVALUATION - although dysplasia/neoplasia begins at squamo-columnar
junction, there are no consistently recognizable abnormalities to
the naked eye. Diagnostic evaluation involves:
1. PAP SMEARS - 95% reliable (5% false negatives) assuming proper
specimen collection and competent evaluation.
2. COLPOSCOPY AND BIOPSY - the colposcope is a lighted instrument
which magnifies cervical mucosa 20x. Areas of abnormality
(thickened white mucosal plaques, abnormal vasculature, etc)
are biopsied and examined histologically.

108
D. - grossly, invasive carcinomas may appear infiltrative,
tlQRPHOLDGY
ulcerative, or exophytic (most common appearance with mass lesion
protruding above surrounding mucosa). There are three microscopic
patterns of squamous cell carcinoma, large cell nonkeratinizing
(most frequent), large cell keratinizing, and small cell. (Adenocar-
cinomas derived from endocervical epithelium tend to occur at a
somewhat older age and present at a more advanced stage with a
correspondingly poorer prognosis. Adenosquamous carcinoma combines
elements of both, arises from the reserve cells of the endocervical
epithelium, and also has a less favorable prognosis.) Growth by
local extension may involve urinary bladder, ureter, rectum and
vagina. Metastases occur primarily via lymphatics to regional and
periaortic lymph nodes, then to liver, lung, bone, etc.
E. P~IS - is generally related to the clinical stage of the tumor
with overall 5 year survival rates approximately 60%. Chemotherapy
is notoriously ineffective. Death often results from uremia due to
ureteral obstruction.

UTERUS
I. DYSFUNCTIONAL UTERINE BLEEDING - abnormal uterine bleeding in absence
of organic lesion of endometrium or uterus.
A. ANOVULATORYCYCLE failure of ovulation results in prolonged
estrogenic stimulation without progesterone-induced secretory
change. This may result in mild hyperplasia.
B. OVULATORY CYCLE

1. INADEQUATE LUTEAL PHASE - results from low progesterone output


from corpus luteum and is clinically manifested by infertility
and either amenorrhea or increased bleeding.
2. IRREGULAR SHEDDING - possibly due to delayed involution of corpus
luteum with prolonged progesterone stimulation. Secretory
endometrium may be admixed with proliferative endometrium.
Clinically manifested by profuse, regular menstrual bleeding
lasting 1-2 weeks.
II. INFLAMMATORY DISEASE

A. ACUTE ENDmlETRITIS - although the normal endometrium is generally


resistant to acute infection, when it occurs it is most commonly
seen in post-abortion or post-partum states when there has been
retained fetal or placental parts. Microabscesses or neutrophilic
destruction of endometrial glands will be present. Accumulations of
pus may develop within the endometrial cavity (pyometra) especially
if there is some obstruction of the endocervical canal.
B. CHRONIC ENlHltETRITIS - may be seen in post-abortion/post-partum states
or associated with IUDs, but in 15% of cases are without definable
underlying cause. Clinically, patients may present with pelvic pain,
abnormal bleeding, and/or infertility. Histologically, it is
characterized by presence of plasma cells in endometrial stroma.
III. ADENOMYOSIS - refers to the presence of endometrial tissue (glands and
stroma) buried within myometrium and not in continuity with the rest of
the endometrium. It is felt to arise from abnormal downgrowth of basal
endometrium into the myometrium. Rarely are these foci responsive to
cyclic hormonal change but they may cause symmetric uterine enlargement
and clinically may present as menorrhagia, menstrual cramps, or
dyspareunia.
IV. ENDOMETRIOSIS - estimated to occur in 20% of adult females (usually 3rd
and 4th decade) and a significant cause of infertility, endometriosis is
characte~ized by presence of benign, potentially functional endometrial
tissue outside of the uterine confines. Although it may involve any
tissue, the most common sites of involvement are the ovaries followed by
uterine ligaments, rectovaginal septum, and pelvic peritoneum.

109
A. PATIIOGENESIS -is unknown, but theories include possible metaplasia of
coelomic epithelium to muellerian duct tissue, regurgitation of
endometrial tissue through the fallopian tubes during menses, and/or
lymphatic/hematogenous dissemination.
B. MORPHOUOOY - grossly, the lesions appear as red-blue to yellow-brown
nodules on or beneath serosal surfaces. Functional bleeding induces
fibrous adhesions with consequent problems of pelvic visceral
distortion. Large "chocolate" cysts may develop in ovaries. Histo-
logic diagnosis rests on the identification of ectopic endometrial
glands and stroma or hemosiderin associated with ectopic placement
of either glands or stroma.
C. CLINICAL PRESENTATION - is generally due to pain and/or infertility, but
symptoms may depend on the site of involvement and functional
activity of the tissue. Most often, the tissue is functional and
bleeds cyclically. Patients may complain of dysmenorrhea and pelvic
pain from periuterine adhesions; pain on defecation due to rectal
involvement; dysuria from bladder involvement, etc.
V. ENDOMETRIAL POLYPS - may be solitary or multiple, are often pedunc-
ulated and of varying size, and tend to occur postmenopausally. They may
be composed of functional endometrium or, more commonly, hyperplastic
endometrium with cystically dilated glands and cellular stroma. Many are
asymptomatic but they may cause intermittent bleeding and a small
proportion may harbor adenocarcinoma.
VI. ENDOMETRIAL HYPERPLASIA - is an increased proliferation of both epi-
thelial and stromal elements with a concomitant increase in endometrial
volume. Hyperplasia is seen primarily in the post-menarcheal or peri-meno-
pausal age groups and is associated with prolonged or excessive estrogen
stimulation. Estrogen secreting tumors, increased adrenocortical function,
Stein-Leventhal syndrome, and exogenous estrogen administration, there-
fore, may be associated with endometrial hyperplasia. Clinically, hyper-
plasia causes abnormal or excessive bleeding (Le. spotting or menor-
rhagia) but the majority of cases are self-limiting and spontaneously
regress. If they do not, however, over a period of years there is, related
to the degree of hyperplasia, a definite risk of progression to endo-
metrial adenocarcinoma.
A. CYSTIC (MILD) HYPERPLASIA - cystically dilated glands of varying sizes
are lined by mitotically active columnar epithelium. There is
increased stroma but scant stromal mitoses. Minimal risk of subse-
quent carcinoma.
B. ADENc:x1ATOUS (MOOERATE) HYPERPLASIA - irregularly thickened grayish mucosa
shows increased number of irregularly shaped glands. There is a
moderately increased risk for development of subsequent carcinoma.
C. ADENOMATOUS HYPERPLASIA KITH ATYPIA (ATYPICAL HYPERPLASIA) - increased mitoses
and glandular crowding with cellular atypia ranging from mild to
severe. High rl.sk of subsequent carcinoma.
VII. ENDOMETRIAL GLANDULAR TUMORS - are the most common female genital
cancer and appears to be increasing in frequency with 90% occurring after
menopause. The development of carcinoma is related to prolonged or
excessive estrogen stimulation of the endometrium. Risk factors include
obesity, diabetes, hypertension, and infertility (many have a history of
anovulatory cycles). Most common symptoms are irregular vaginal bleeding
and leukorrhea.
A. MORPHOUOOY- these may be focal or diffuse but eventually the
endometrial cavity becomes filled with nodular and partially
necrotic tumor. 60%-75% are adenocarcinomas varying from well-
differentiated (Grade I) to poorly differentiated (Grade III). 20%-
30% contain foci of squamous differentiation. If the squamous
component is benign, the tumor is an adenoacanthoma and behaves like
a Grade II adenocarcinoma; if malignant, it is an adenosquamous
carcinoma and the prognosis is poor.

110
B. tumor may extend into the myometrium and through the
NATURAL HISTORY -
serosal surface to involve adjacent structures. Lymphatic spread is
to regional and periaortic lymph nodes and hematogenous spread is to
lung, liver, bone, etc. Prognosis is dependent on depth of myome-
trial invasion, degree of cellular differentiation, and type of
tumor. Tumors in older women tend to be less well-differentiated and
more invasive than those in younger women.
VIII. ENDOMETRIAL STROMAL TUMORS - are characterized by the presence of
endometrial stroma of varying cytologic atypia present in myometrium.
A. BENIGN SfRQI'IAL NODULES - appear as expanding nodules of endometr ial
stroma buried within the myometrium.
B. ENDOLYMPHATIC SfRCl1AL MYOSIS - represents a low-grade sarcoma in which
endometrial stromal tissue in the myometrium tends to invade
lymphatics and blood vessels. 50% recur and 15% show distant
metastases.
C. ENDCI1ETRIAL SfRCl1AL SARCnIA - usually arises high in fundus filling the
endometrial cavity and growing into the myometrium with extensive
vascular invasion. Variably differentiated cytology but mitoses >
10/hpf. 50% 5 yr survival.
IX. MALIGNANT MUELLERIAN TUMORS - tend to occur in elderly postmeno-
pausal patients and present with bleeding. Derived from muellerian
mesoderm, the tumor consists of malignant glandular and stromal
components. Stromal component may differentiate into mesodermal structures
such as cartilage, muscle, bone. There is only a 25% 5 yr survival.
A. CARCINOSARCClI'IA - homologous sarcoma and carcinoma.
B. HESODERHAL MIXED TUMOR - heterologous sarcoma and carcinoma.
X. SMOOTH MUSCLE TUMORS

A. LEIotrlOMA - is probably the most common neoplasm in women. They


usually are first noticed to arise in the 3rd to 4th decade and tend
to decrease in size after menopause. Grossly these are well
circumscribed, grey-white, rubbery spheroids which have a whorled
cut surface. They may be pedunculated, submucosal, subserosal, .or
intramural and are often multiple and of varying size. Some may
undergo cystic degeneration, hyalinization, or calcification.
Histologically, they are composed of interlacing bundles of benign
smooth muscle cells. Malignant transformation is distinctly unusual.
When symptoms occur, they are usually related to the size of the
tumors (pressure on bladder or rectum, sensation of heaviness) to
abnormal bleeding patterns (particularly submucosal tumors), or to
pain from degeneration, infarction, and hemorrhage.
B. LEIotrlOSARCOMA - is uncommon. They may present as fleshy masses
invading into uterine wall or as polypoid mass growing into endo-
metrial cavity. They are felt to arise de novo rather than from
preexisting leiomyoma. They vary in histologic appearance from low-
grade (well-differentiated) to high-grade (poorly differentiated).
Crit~ria of malignancy may simply rest on number of mitoses/HPF and
cytologic atypia. In the uterus, smooth muscle tumors which have a
mitotic rate of > 5 mitoses per 10 HPF should be considered
malignant. Overall 5 yr survival is 40-50%.

FALLOPIAN TUBES
Transport of ova and sperm is an active process aided by the ciliated epithelium
of the tubal mucosa. Anything that interferes with the tubal epithelium (minor
inflammatory changes) or the tubal mobility (peritubal adhesions, etc.) may also
interfere with fertility.

111
1. INFLAMMATORY DISEASE (most common disorder of tubes)

A. SALPINGITIS -is usually an extension of a preexisting cervicitis or


endometritis. Gonorrhea and chlamydia are the most frequent causes
of salpingitis and, unless recognized and treated, may result in
infertility.
B. ~-OVARUW ~ - usually results from microabscess formation in
cortex of ovary that secondarily involves the fimbria of the
fallopian tube in the inflammatory process.
II. SAIJPINGITIS ISTHMICA NODOSA - refers to nodular lesions at the isthmus
of tube characterized by increased thickness of the muscular wall which
contains gland-like spaces lined by tubal epithelium. Etiology not known
but process may be similar to adenomyosis.
III. CYSTS - embryologic remnants of mesonephros, muellerian and wolffian ducts
are present in mesosalpinx and may become cystic (paratubal cysts,
hydatids of Morgagni).
IV. NEOPLASIA - is unusual but benign adenomatoid tumors (mesotheliomas) and
primary adenocarcinomas can occur. Malignant involvement, however, is much
more likely to represent metastases.
V. ECTOPIC PREGNANCY - occurs when the fertilized ovum implants in an area
other than the endometrium. It is increasing in frequency and most occur
in the tube but on rare occasions may occur in the ovary or peritoneal
cavity. Predisposing factors include those that inhibit tubal transport of
the ovum (chronic salpingitis, peritubal adhesions, cysts, tumors, etc),
but in about half of the cases, no underlying pathology can be identified.
Since the tubal mucosa has a limited ability to undergo decidual change,
the developing placental tissue is poorly anchored and hemorrhage occurs
at the implantation site creating a hematosalpinx. Ectopic pregnancy is
the most common cause of hematosalpinx. As the placenta grows, it burrows
through the thin wall of the tube and may rupture the tube causing life-
threatening intraperitoneal hemorrhage. Clinically, at the time of
rupture, there is abrupt onset of severe abdominal pain and the patient
may go into shock.
OVARY

I. REVIEW OF NORMAL - in both sexes, the primitive gonad forms along a


genital ridge from specialized mesenchyme that is covered by the coelomic
(germinal) epithelium. The migration of germ cells from the yolk sac via
the root of the mesentery to the primitive gonad induces the formation of
sex cords from the mesenchymal stroma (some feel that the sex cords are
derived from the germinal epithelium). The granulosa cells are eventually
derived from these sex cords and surround the oocyte to fOrm the primord-
ial follicle. The stromal cells adjacent to the granulosa cells differen-
tiate into the concentric perifollicular theca interna and externa. The
granulosa cells of the ovary and the Leydig cells of the testes have the
same embryologic origin and both are capable of producing both estrogenic
and androgenic steroids. Although ovarian. stromal ce11s will preferential-
ly produce androgens, they may also produce estrogens and progesterone.
The prp-dominance of one over the other is due to presence or absence of
appropriate enzymes. Several follicles develop during each cycle but only
one ruptures and becomes a corpus luteum secreting progesterone compounds
to support the luteal (secretory) phase of the menstrual cycle. The others
undergo atresia. The theca cells initially undergo hyperplasia and
luteinization and ultimately hyalinization to form a corpus albicans.
II. NON-NEOPIJASTIC OVARIAN ENIJARGEMENT - is genera11y caused by develop-
ment of cysts within the ovary. These may be asymptomatic or present with
abdominal pain and/or abnormal menstrual cycles.
A. are commonly found in ovaries (particularly
"GEmINAL" INCLUSION CYSTS -
in premenopausal period), may be mUltiple and of varying size, and
result from downgrowth of the surface epithelium into the ovarian
cortex.

112
B. PHYSIOLOGIC OR FVNCTIONAL CYSTS - result from exaggeration of normal
physiologic cyclic changes.
1. FOLLICLE CYSTS - are extremely common and consist of one or more
cysts developing from follicles that are undergoing atresia.
Ranging up t02 cmin diameter, they are lined by granulosa
cells or flattened atrophic cells and contain clear fluid.
Usually, they are of no clinical significance although rarely
they may continue to secrete estrogens with resulting endo-
metrial hyperplasia.
2. CORPUS LUI'EUIt CYSTS - results from cystic enlargement of corpus
1uteum which usually has central hemorrhage. Usually of no
significance, but if it ruptures through capsule, it may mimic
ectopic pregnancy and persistent secretion of progesterone may
cause menstrual irregularities.
3. THECA LUTEIN CYSTS - may develop with improper atresia of
unruptured follicles so that there is hyperplasia and
persistence of luteinized theca cells stimulated by conditions
in which there are high circulating levels of gonadotropins
(pregnancy, hydatidiform moles, etc.). Often this is a
bilateral condition and, on occasion, these cysts may rupture
to cause hemorrhage.
C. POLYCYSTIC OVARIES - one of the more common causes of infertility, the
ovaries are bilaterally enlarged, with multiple cysts underlying a
thick white collagenous capsule. The cysts are lined by granulosa-
theca cells which may be luteinized (and androgen secreting). Cor-
pora lutea are absent. Clinically, there are a variety of symptoms
ranging from abnormal bleeding and hypermenorrhea to symptoms of
virilization or the Stein-Leventhal syndrome (2° amenorrhea, obes-
ity, hirsutism, infertility). The etiology is not known but probably
involves a dysfunction of the hypothalamic-pituitary-ovarian axis so
that abnormal secretion of gonadotropin releasing factor from the
hypothalamus results in continuous ovarian stimulation by FSH 'and LH
without the monthly cycling. For unknown reasons, wedge resection of
the ovary will restore normal cycling in a majority of patients.
D. SIIQIAL HYPERPLASIA - occasionally, particularly in postmenopausal
women, proliferation of ovarian stromal cells will result in nodular
masses (primarily in the ovarian medulla) of both ovaries that
clinically present as ovarian enlargement. Many of the stromal cells
are luteinized (hyperthecosis) and produce androgens which may lead
to virilization. Peripheral conversion of androgens to estrone,
however, may lead to hyperestrogenic symptoms such as endometrial
hyperplasia and carcinoma.
III. NEOPLASTIC OVARIAN ENlJARGEMENT - most ovarian neoplasms are benign
(80%) but ovarian cancer is the third most frequent genital tract
malignancy. Although less frequent than endometrial and cervical cancer,
ovarian cancer results in greater mortality (50%) than the other two
combined. This is primarily due to late presentation. Unless endocrin-
ologically active, they tend to be asymptomatic until the size of the
tumor causes symptoms of abdominal pain and distension, GI or urinary
tract compression or invasion, or abdominal or vaginal bleeding. Ovarian
neoplasms tend to be most prominent during reproductive years with the
malignant forms tending to occur pre- or perimenopausally. The risk of
ovarian neoplasia appears to be increased in women with a family history
of ovarian tumors and women who have not borne children. Unlike breast or
endometrial cancer, estrogen does not appear to playa role.
A. are the most common of
TUMORS DERIVED FRmI SURFACE (GERMINAL) EPITHELIUM -
the ovarian neoplasms making up about 60-70%. The paramesonephric
(muellerian) duct is embryologically derived from coelomic
epithelium and, in the female ultimately supplies the epithelial
lining of the fallopian tube, uterus, and endocervix. The surface
epithelium of the ovary (also derived from coelomic epithelium)
therefore has the potential to differentiate into tubal epithelium
(serous secreting, ciliated columnar), endometrial epithelium (non-
ciliated columnar) or endocervical epithelium (mucus-secreting,
113
nonciliated columnar) and ovarian neoplasms arising from this
surface epithelium may mimic any of these cell types. Because of
their location, the malignant forms are often not recognized until
they have spread, many by seeding diffusely throughout the
peritoneum (which may cause massive ascites). Even benign tumors,
however, can torse, infarct, and behave similar to an acute abdomen.
1. - comprise 25-30% of all ovarian tumors and are
SEROUS TUMORS
most common in the 20-50 year age group.
a. Serous cystadenoma - generally unilocular cystic struc-
ture filled with clear fluid which can reach large size.
Capsule is generally smooth and glistening but may have
few papillary projections on internal or external sur-
face of the cyst wall. The cyst is lined by tall cili-
ated and nonciliated columnar epithelium (similar to
that seen in the fallopian tube) without cellular
atypia. 20-30% are bilateral and as size increases they
may become multilocular.
b. Serous cystadenocarcinoma - makes up about half of all
malignant ovarian tumors. Cyst wall shows papillary
projections on both sides and there are usually solid,
nodular thickenings of the capSUle. Histology shows
piling up and stratification of the epithelium with
penetration of cyst wall and invasion of the capsule or
ovarian stroma. 65-70% are bilateral. Although psammoma
bodies are commonly seen in, and are characteristic of,
serous tumors, they do not necessarily imply benignity
or malignancy. Depending on cellular atypia the tumors
are graded I (well-differentiated) to III (poorly
differentiated), however the clinical stage appears to
be of greater prognostic significance than the grade.
c. Serous borderline tumor - shows the cyst lining cells
beginning to stratify. Although cellular atypia and
mitoses are present, the atypia is not as severe nor are
the mitoses as frequent as is seen in carcinoma. A
cellular stroma separates the neoplastic glands and
there is no evidence of capsular or ovarian stromal
invasion although there may be peritoneal seeding. The
10 year survival is approximately 75%.
2. - comprise approximately 20% of ovarian tumors
MUCINOUS TUMORS
and are most frequently seen in the 30-60 year age range.
a. Mucinous cystadenoma - are about equal in incidence to
the benign serous tumors. Gross ly, they also appear
similar to serous tumor but have a greater tendency to
be unilateral, multiloculated, and larger. They contain
a mucinous, gelatinous fluid. Histologically, the cyst
epithelium is a non-ciliated mucous secreting epithelium
similar to that seen in the endocervix.
b. Mucinous cystadenocarcinoma - are much less frequent
than the serous counterpart but basically the same gross
and histologic criteria apply for diagnosis (piling up
of the epitheliums, complex papillary and glandular
formations creating a cribriform pattern or solid
cellular proliferations, and invasion of the ovarian
stroma). The malignant cells have decreased amounts of
cytoplasmic mucin but it may cause pseudomyxoma peri-
toneii if peritoneal seeding occurs.
c. Mucinous borderline tumor - shows piling up and strati-
fication of the epithelium but to a lesser degree. Mild
to moderate cellular atypia and occasional mitoses may
be present. The 10 year survival 68% (96% i f confined to
ovary) .

114
3. ENDOfETRIom CARCINOMAS - comprise 20-25% of ovarian tumors and
are malignant from inception. 30-40% are bilateral. Histolog-
ically, these are more of a glandular neoplasm than papillary
neoplasm and closely resemble endometrial carcinoma: Although
up to 30% of patients will have a coexistent endometrial
cancer, this tumor is felt to arise de novo rather than as a
metastasis from the endometrium or a malignant transformation
of a focus of endometriosis. Foci of squamous differentiation
within the tumor is a helpful diagnostic sign. The overall 5
year survival is 40-50%. Rarely, other endometrial-like tumors
(mixed mesodermal tumors, endometrial stromal sarcomas, etc)
can also arise in the ovary.
4. BRENNER TUHORS - comprise 2% of ovarian tumors and can be seen
at any age but peak incidence is between 40-70. They tend to
be unilateral, small, solid, and benign. Although the propor-
tions vary, generally there is a fibrous stroma punctuated by
nests of epithelial cells resembling either squamous
epithelium or transitional cell epithelium. Occasion.ally the
nests may have small cystic spaces in the center and resemble
mucinous glandular structures. The stromal component may also
contain lipid and appear luteinized and has been reported in
association with endothelial hyperplasia and adenocarcinoma.
Borderline Brenner tumors resemble low-grade papillary
transitional cell carcinomas and-malignant Brenner tumors
resemble higher-grade transitional cell carcinomas.
B. make up 10-20% of ovarian
TUKORS DERIVED FROtI SEX CORD AHD/OR STROfA -
tumors and are the most likely to be hormonally active.
1. - comprise 5% of ovarian tumors and
GRANULOSA-THECA CELL TUtlORS
have peak incidence in the post-menopausal years. Usually
unilateral, they vary in size and may be solid and/or cystic.
Although pure granulosa cell and pure theca cell tumors occur,
generally there is a variable mixture of the two rendering
histologic variability, but this has little prognostic value.
Granulosa cells may form microfollicles (Call-Exner bodies)
and theca cells are sometimes difficult to tell from fibrous
stroma. Although they generally follow a benign course, they
should be considered potentially malignant. Those with greater
proportion of granulosa component are more frequently malig-
nant while those with greater froportion of theca component
are more frequently functiona . The functional tumors may
cause precocious puberty or, in adults, they may be associated
with estrogen associated lesions (endometrial hyperplasia and
carcinoma, breast carcinoma, etc). Since the steroid synthetic
pathway in these tumors differs from normal granulosa cells in
that they can produce androstenedione as well as estrogens,
occasionally they may be virilizing.
2. FIB~ - comprise 5% of ovarian tumors. Most are unilateral,
solid, round masses 5-10 cm in size. A thecal component may be
present and secrete estrogen (fibrothecoma). For unknown
reasons, when the tumor > 6 cm in size, 40% of patients will
develop ascites and right-sided pleural effusion (Heig's
syndrome).
3. (arrhenoblastoma, androblastoma) - shows
SERTOLI-LEYDIG CELL TUtlORS
peak incidence in the 2nd-3rd decade. Usually unilateral solid
tumors with focal hemorrhage and necrosis, they contain
sertoli cells, Leydig cells, and a primitive gonadal stroma.
The histology varies considerably but tends to recapitulate
testicular development. With the less well differentiated
tumors, the stroma becomes increasingly prominent and heterol-
ogous elements (mucinous intestinal epithelium, cartilage,
skeletal muscle, etc) may appear. On occasion these may have
estrogenic activity, but the majority, if functionally active,
cause defeminization (amenorrhea, hair loss, breast atrophy)
or virilization (hirsutism, male hair distribution, lowering
of voice, clitoral hypertrophy, muscular build, etc.) due to

115
elaboration of androgenic hormones. Most. follow a benign
course and early surgical excision may reverse some of the
endocrine effects.
4. HILUS (HlLAR) CELL TUMORS - appear in post-menopausal women as
benign, small, unilateral tumors composed of large lipid laden
cells felt to arise from ovarian hilus cells (the counterpart
of the testicular Leydig cells) and may be virilizing. Intra-
cytoplasmic Reinke crystalloids confirm the diagnosis.
5. LIPID CELL TUMORS - these cells may resemble Leydig cells and/or
adrenal cortical cells. Although some feel that they may arise
from adrenal rests, others feel that they simply represent
Leydig cell tumors devoid of Reinke crystalloids. Usua11y
sma11 , benign, and may be hormona11y active (estrogen and
androgen).
6. SERTOLI CELL TUMORS - are rare. They tend to appear in young
adults most frequently and the majority are functional with
45% secreting estrogens and 20% secreting androgens. The cells
resemble the Sertoli cells that are present in the Sertoli-
Leydig cell tumors.
C. comprise 10-20% of ovarian tumors. Most
TUMORS DERIVED FRDtI GERH CELLS -
are benign but the malignant forms constitute the most common form
of ovarian malignancy in children.
1. TERA~ - 46XX karyotype implies parthenogenetic origin from
single postmeiotic haploid germ cell.
a. Mature cystic teratoma (dermoid cyst) - make up 95% of
germ cell tumors. They most often arise in young adults,
are unilateral, and usually < 10 cm. These tumors are
prone to torsion and infarction and clinica11y, patients
may present with abdominal pain, pelvic mass, irregular
periods, etc. Grossly they appear as a unilocular or
multilocular cyst lined by squamous epithelium and con-
taining che~sy sebaceous debris, matted hair, cartilage,
teeth, etc. Microscopic elements are derived from all
germ layers and may consist of mucus glands, cartilage,
respiratory or GI epithelium, skin and skin adnexa,
brain, ,thyroid, etc. Struma ovarii refers to a teratoma
consisting predominantly of thyroid tissue (10% of these,
patients are hyperthyroid and thyroid carcinomas may
originate in this tissue).
b. Immature (malignant) teratoma - is a rare lesion and
tends to arise in a slightly younger age group (teens)
than the mature teratomas. Almost always unilateral,
these are predominantly solid tumors showing necrosis
and hemorrhage and which grow rapidly with local
extension and metastases. Histology shows immature
elements (particularly immature neuroepithelium) in
addition to mature elements and, unless the tumor is
sampled carefully, these elements may be missed.
2. DY~RH~ - comprise only 2% of all malignant ovarian tumors
but over 50% of malignant germ ce11 tumors. The majority arise
in 2nd-3rd decade, and are usually unilateral, yellow-white to
grey-pink fleshy masses which may be of variable size but are
usually relatively large when first discovered. The malignant
cell is derived from primordial germ cells and resembles the
undifferentiated embryologic gonad. Sheets and cords of medium
sized cells with vesicular nuclei and clear or granular
glycogen rich cytoplasm are separated by scant fibrous stroma
infiltrated by mature lymphocytes and occasional granulomas.
Although all are malignant, cellular atypia is variable and
only 30% are aggressive. They are radiosensitive and the 5
year survival runs 70-90%.

116
3. ENDmERtlAL SINUS (YOLK SAC) TUMOR arises in children and
adolescents and is derived from malignant germ cells showing
extra-embryonic yolk sac differentiation with histologic
resemblance to the endodermal sinus. Schiller-Duval bodies are
the characteristic histologic feature. The cells are rich in
alpha fetoprotein and alpha I-antitrypsin. Rapid aggressive
growth and poor response to therapy originally characterized
these tumors, however current survival rates are greater than
50% even with advanced disease.
4. EMBRYONAL CARCINa1A - is a rare tumor occurring in children and
adolescents. They are unilateral and are usually large when
first discovered. They secrete HCG and alpha fetoprotein.
5. CHORIOCARCINa1A - is usually seen in combination with other germ
cell tumors. Pure choriocarcinomas, however, are almost always
seen in younger patients and produce a unilateral hemorrhagic
mass composed of malignant syncytiotrophoblasts and cyto-
trophoblasts. It produces high quantities of HCG. They
disseminate widely, are poorly responsive to therapy, and
generally fatal.
D. METASTATICTUMORS - are relatively common with breast, GI tract
(Krukenberg tumor) and other pelvic organs as the primary sites.

GESTATIONAL TROPHOBLASTIC NEOPLASMS


I. HYDATIDIFORM MOLE - occurs in approximately 1: 2000 pregnancies and
tends to have a higher incidence in older women (40-50 yrs). It is felt to
arise from loss or inactivation of maternal chromosomes in the fertilized
egg. The embryo dies at an early stage but the placenta continues with
abnormal growth. Clinically these often present in the 4th to 5th month of
pregnancy with the passage of vesicular tissue from the vagina, and there
may be a history of uterine bleeding since early pregnancy. The uterus is
usually larger than would be expected for the gestational age but no fetus
is detectable. HCG levels are elevated and there may be bilateral ovarian
enlargements due to theca-lutein cysts. Grossly, a delicate bulky tumor
consisting of grape-like cystic structures fills the endometrial cavity.
Microscopically, there is hydropic swelling of avascular villi with
variable degrees of trophoblastic hyperplasia and anaplasia. The major
complication is the potential for the development of choriocarcinoma.
II. INCOMPLETE (PARTIAl,) MOLE - refers to a mixture of normal and hydropic
villi without appreciable trophoblastic proliferation. A fetus with a
triploid phenotype is generally present. There is no implication of
subsequent development of choriocarcinoma.
III. INVASIVE MOI-E (chorioadenoma destruens) - invasion into and/or through
the myometrium by molar villi and trophoblastic tissue can result in
uterine perforation. This is clinically characterized by bleeding
associated with irregular uterine enlargement. The neoplasm may embolize
to distant sites (1. e. lungs) but the embolic lesions will usually
spontaneously regress when the uterine lesion is removed. The response to
chemotherapy is usually good and can be monitored by HCG levels. .
IV. CHORIOCARCINOMA - is a trophoblastic malignancy which may arise from
either a preexisting normal pregnancy (20%) or an abnormal pregnancy (50%
of which are hydatidiform moles). Clinically, it is characterized by
irregular bleeding during an otherwise apparently normal pregnancy or
continued bleeding after miscarriage. The tumor is usually soft and
friable with areas of hemorrhage and necrosis. It consists of neoplastic
trophoblastic tissue containing both cytotrophoblasts and syncytiotropho-
blasts without the presence of chorionic villi. The tumor spreads early
and widely by lymphatic and hematogenous dissemination. Metastases are
usually present at the time of diagnosis and most commonly involve lungs,
vagina, brain, liver, and kidney. As long as viable syncytiotrophoblasts
are present, there are markedly elevated levels of HCG which can be used
to monitor the course of the disease. Chemotherapy with methotrexate and
actinomycin has been very effective with cures being possible even in the
face of widespread disease. Unfortunately choriocarcinomas arising outside
of the gestational setting do not show the same response or prognosis.

117
MALE GENITAL TRACT
PENIS
I. DEVELOPMENTAL DISORDERS

A. in males, abnormal development of the


HYPOsPADIAS/EPISPADIAS -
urethral canal may result in the urethral opening lying along the
ventral (hypospadias) or the dorsal (epispadias) surface of the
penile shaft. This is often associated with other genitourinary
malformations (cryptorchism, bladder exstrophy, etc.). Either may
produce partial urinary obstruction predisposing to urinary tract
infection.
B. PHnmsIS - may be developmental or acquired and refers to the
inability to retract the foreskin over the glans because of an
abnormally small preputial opening. This prevents adequate
cleansing of the glans and predisposes to infection. Inflammation
of the glans and prepuce (balanoposthitis) may also result in
adhesions to the foreskin and an acquired phimosis. Paraphimosis
refers to the inability to replace the foreskin after retraction
usually due to constriction and swelling of the glans.
II. PREMALIGNANT AND MALIGNANT CONDITIONS

A. ERYDIBOPLASIA OF QUEXRAT - is essentia11y the same disorder as Bowen's


disease of the vulva and appears on the penis grossly as red,
raised, plaque usually on the glans and microscopically shows
severe epithelial dysplasia/carcinoma-in-situ, decreased keratin,
and chronic inflammation. A small number may progress to invasive
squamous cell carcinoma.
B. BOHENOm PAPULOSIS - also represents a squamous ce11 carcinoma-in-
situ that clinically presents as multiple (rather that solitary)
lesions of the penis and tends to occur in a younger age group.
e. SQUAMOUS CELL CARCIN(»IA - reaches a peak incidence in the fifth to
seventh decade and occurs more frequently in uncircumcised males.
It usually begins near coronal sulcus as a white plaque-like
lesion which eventually ulcerates (or less commonly grows in a
fungating fashion). The tumors are usually well-differentiated and
slow growing with only 30% showing metastases to superficial
inguinal nodes at the time of diagnosis. Prognosis depends on
depth and extent of invasion and nodal involvement.
D. VE~ CARCI~ (giant condyloma) - is an uncommon condition
associated with HPV infections and represents a well differen-
tiated papillary squamous cell carcinoma that spreads horizon-
tally. It is recurrent and locally invasive but does not
metastasize until late in its course.
TESTIS
I. CRYPTORCHIDISM - refers to the failure of the testis to descend into
the scrotum from its embryologic position in the coelomic cavity and is
usually unilateral. Although there is early arrest of germ cell produc-
tion, gross atrophy usually does not become apparent until puberty.
These patients have an increased risk of subsequent testicular cancer in
both the cryptorchid testis (regardless of surgical repositioning) and
the contralateral "normal" testes.
II. NON-NEOPLASTIC ENLARGEMENT OF SCROTAL CONTENTS

A. HYDmxELE - refers to an accumulation of serous fluid within the


tunica vaginaU.s secondary to trauma, infection, systemic edema,
etc.
B. ~DXiLE - refers to an accumulation of blood within the tunica
vaginal is secondary to trauma or hemorrhagic diathesis.
e.· CHYUXELE - refers to an accumulation of lymphatic fluid within the
tunica vaginalis due to lymphatic obstruction by tumor, parasites,
etc.

118
D. SPE~TIXELE - refers to a cystic dilatation of epididymal ducts
containing semen.
E. GRANULCltIATOUS ORCHITIS - is probably an autoimmune disease that
usually presents as a painful mass in the testis. Histologically,
there is granulomatous inflammation which must be differentiated
from infectious granulomas.
F. TORSION - refers to a twisting of the spermatic cord (usually due
to trauma) which may induce infarction, hemorrhage, and testicular
enlargement.
III. NEOPLAS~S - like their ovarian counterpart, testicular neoplasms can
arise from the surface epithelium (tunica vaginalis), sex cord/stromal
tissue, or germ cells. pnlike the ovaries, however, the vast majority of
testicular neoplasms take origin from the germ cell. Testicular
neoplasms are increasing in frequency, have a peak incidence in young
adults (15-30 years), and generally cause painless enlargement of the
testis.
A. GERM CELL TUMORS (95% of testicular neoplasms)
Germ Cell
I
I
SEMINOIfA
I
EMBRYONAL CARCINOIfA
I
I
Extraembryonal
I I
E b
m 'Yomr.1 tissue
I
Trophoblast
I
Yolk sac Ectoderm Mesoderm
1
Endoderm

I I I
CHORIOCARCINOIfA YOLK SAC TUMOR TERATOIfA

1. SEMI~ - is the most common testicular tumor and is the


male counterpart of the ovarian dysgerminoma. It is most
prevalent in the 4th or 5th decade. They are bulky, firm,
lobulated tumors without necrosis or hemorrhage. They first
spread to common iliac and para-aortic lymph nodes but are
very radiosensitive and, in general, have an excellent
prognosis.
a. Classic seminoma - consists of monotonous sheets of
cells with round nuclei, clear cytoplasm, and distinct
cytoplasmic borders. There is variable amount of
fibrous stroma with a prominent lymphocytic infil-
trate, and occasional granuloma formation.
b. Anaplastic seminoma - shows greater cellular pleo-
morphism and increased numbers of mitoses.
c. Spermatocytic seminoma - typically affects elderly
males and shows cellular pleomorphism with giant cells
but is a slow growing tumor that does not metastasize.
2. has a peak incidence in the third decade
EMBRYONAL CARCINOIfA -
and is often a small infiltrative mass lesion consisting of
large pleomorphic cells with a variable architectural
pattern and often exhibiting necrosis and hemorrhage. Up to
90% may contain HCG and AFP. It does not respond well to
radiation but if found early, prognosis is relatiyely good.
3.. ENDODE~L SINUS (YOLK SAC) TUMOR - although not a common tumor,
it is the most frequently seen testicular tumor in babies
and young children and consists of undifferentiated cells
with a variable architectural pattern. Like its ovarian
119
counterpart, the tumor is rich in alpha fetoprotein and a-1-
antitrypsin. If found early, and with current methods of
treatment, the prognosis is generally good.
4. CHORIOCARCINotIA - is uncommon in its pure form. It is usually
small, necrotic, and hemorrhagic consisting of both
cytotrophoblasts and syncytiotrophoblasts. Like the ovarian
choriocarcinomas, HCG is present and the tumor spreads
quickly and widely via the bloodstream. Initial clinical
complaints may be due to metastases. Prognosis is generally
poor.
5. TERA~ (5%) - are tumors which contain a mixture of tissue
derived from the three embryologic germ cell layers. The
mature teratoma tends to occur in younger individuals and
contains mature benign tissue. The immature teratomas tend
to behave clinically in a malignant fashion but may show
only immature tissue rather than frankly malignant tissue.
Malignant teratomas contain cytologically malignant tissue.
6. MIXED TUMORS (40%) - contain a mixture of two or more "pure"
germ cell tumors, most commonly a mixture of teratoma and
embryonal carcinoma (teratocarcinoma). Most will contain HCG
or AFP. The prognosis is dependent on the more aggressive
element.
B. in general, these are benign tumors but, as
SEX CORDISTR(lfAL TUMORS -
in the ovary, they may be endocrinologically active.
1. LEYDIG CELL TUMOR (2%) - can occur at any age and may secrete
androgens or estrogens creating feminization or precocious
masculinization in boys. The only clinical effect in adult
males would be gynecomastia in estrogen secreting tumors.
They are bilateral in up to 10% of cases and microscopically
may show Reinke crystalloids.
2. SERTOLI CELL TUMORS - are rare tumors which may also secrete
androgens or estrogens and may contain cells resembling
ovarian granulosa cells. Most are benign.
C. unlike the ovary, these tumors are rare in
SURFACE EPITHELIAL TUMORS -
the testis and consist primarily of the testicular mesothelioma
(adenomatoid tumors). .

PROSTATE
I. INFLAMMATORY DISEASE - may be asymptomatic or associated with low
back pain or symptoms of urinary tract infection.
A. AcnE P~ATITIS - is often an extension from other genitourinary
bacterial infections or complications of trauma (catheterization,
transurethral resection) leading to a swollen acutely tender
gland.
B. CHRONIC PROSTATITIS - has a more insidious onset. When bacterial in
origin, it is frequently a source of recurrent urinary tract
infections. More commonly, however, no bacteria can be cultured.
C. G~LotIA~ PROSTATITIS - may be a reaction to prostatic secretions
that gain access to the stroma. Infectious etiologies (TB, etc),
however, must be ruled out when granulomas are encountered.
II. BENIGN PROSTATIC HYPERTROPHY (BPH) - is actually a misnomer since
the lesion primarily represents a nodular hyperplasia of the gland. It
is a common disorder which usually begins in the fifth decade and which
increases in frequency with increasing age (95% of men over age 70).
Only 5-10% of patients, however, will require surgical treatment.
Although the etiology is unclear, there is an apparent relative
imbalance between androgens and estrogens with the increasing estrogen
levels seen with advancing age somehow sensitizing the prostatic tissue
to the effects of testosterone since BPH will only occur in the presence
of intact testes. The periurethral tissue is the most susceptible and

120
leads to a spongy nodular enlargement of the median and lateral lobes
which may cause symptoms related to partial urinary obstruction or
retention of urine and predispose to urinary tract infections. There
does not appear to be any causal relationship to the subsequent
development of prostatic cancer. Microscopically, there is epithelial
hyperplasia causing papillary budding and infolding of epithelium into
the glandular lumina, inspissated prostatic secretions, focal squamous
metaplasia, and fibromuscular hypertrophy.
III. PROSTATIC CARCINOMA - is probably the most common malignancy in
males, but ranks behind lung and colon cancer in terms of mortality.
Unusual before the age of 50, it increases in frequency with advancing
age so that prostatic cancer occurs in up to 70% of men by the age of
80. Many of these cancers, however, are small, biologically indolent,
and identified incidentally on microscopic examination of the prostate
for other reasons (Stage A carcinoma). Again, the etiology is unclear
but it appears that genetic, environmental, and hormonal factors are
involved. The tumor originates in the "peripheral" zone of the gland. It
frequently involves the posterior lobe and may be palpated as a hard
irregular nodularity on rectal examination (Stage B carcinoma). Micro-
scopically almost all are adenocarcinomas and most of these are well-
differentiated with few mitoses and l~ttle pleomorphism. "Back-to-back"
glandular crowding without intervening stroma, a single layer epithel-
ium, and capsular, vascular, or perineural "lymphatic" invasion are all
indicators of malignancy. The major symptom is urinary obstruction, but
this usually does not occur until late in the course of the disease so
that the majority of patients present with extension of the tumor
through the prostatic capsule (Stage C) or metastases (Stage D).
Lymphatic spread is to regional lymph nodes and hematogenous spread is
generally to bone (where the tumor produces osteoblastic lesions). Death
is usually due to disseminated disease or obstructive nephropathy. In
patients at risk, serum PSA (prostatic specific antigen) may be of value
in screening for the disease.

121
URJ:NARY TRACT

KIDNEY
I. REVIEW OF NORMAL
A. BBRYOLOGX - the kidney arises from a succession of developmental
changes that occur along the nephrogenic cord which resides in the
urogenital ridge on the dorsal wall of the coelomic cavity. Running
lengthwise through the nephrogenic mesenchyme is a tubular structure
(pronephric duct) connected at the caudal end to the cloaca. The
development of the cephalad pronephros occurs early and degenerates
as the mesonephros is developing. The pronephric duct at this point
becomes the mesonephric (Jr!olffian) duct. As the mesonephros is
degenerating, the true kidney is being developed from the most
caudal portion of the nephrogenic cord, the metanephros. A ureteric
bud develops from the caudal end of the mesonephric duct and
penetrates into the metanephric mesenchyme. The stalk of the
ureteric bud becomes the ureter while its expanded cranial end
becomes the renal pelvis. The pelvis divides successively to form
the major calyces and minor calyces. Blind ended collecting tubules
penetrate the nephrogenic mesenchyme and give off approximately
twelve generations of dichotomously branching arching ducts. The
leading portions of these ducts induce the development of adjacent
metanephric vesicles which elongate rapidly to form the distal
convoluted tubule, the loops of Henle, the proximal convoluted
tubule, and Bowman's space. The distal convoluted tubule and the
proximal portion of the collecting duct eventually fuse. The
nephron, therefore, develops from the metanephric mesenchyme while
the collecting system develops from the ureteric bud. As the fetus
matures, the kidneys ascend cranially from their initial pelvic
location and derive their vascular supply from a succession of more
cranially located renal arteries.
B. ANATOHX - the cortex consists of glomeruli, proximal and distal con-
voluted tubules and collecting ducts. It normally measures approx-
imately 1.5 cm in thickness and includes the columns of Bertin which
extend between the medullary pyramids. The medulla consists of the
renal pyramids whose tips (papillae) protrude into the minor caly-
ces. The pyramids contain the loops of Henle and the collecting
ducts. A glomerulus consists of endothelium; visceral and parietal
epithelial cells (which define Bowman's space); and the "supporting"
mesangial cells and their matrix. In addition, a glomerular basement
membrane is interposed between the endothelium and the foot pro-
cesses of the visceral epithelial cells (podocytes). The glomerular
filter consists of fenestrated endothelial cells, the basement
membrane, and the foot processes. Substances penetrate or are
excluded by this filter on the basis of size and charge.
II. CONGENITAL/DEVEI..OPMENTAL DISORDERS
A. RaUL ~IS - bilateral agenesis is incompatible with life and is
associated with pulmonary hypoplasia, oligohydramnios, and Potter's
facies (wide-set eyes, beaked nose, micrognathia, and low-set ears).
Unilateral agenesis is more common and seen most frequently in
males. The remaining kidney may become hypertrophied and there may
be association with esophageal atresia or congenital heart disease.
B. PELVIC KmNEY - results from the failure of kidney to ascend to its
proper location. May be identified as a pelvic mass but generally
asymptomatic although somewhat more prone to infection and ureteral
obstruction.
C. HORSESHOE KJPNEY - majority are fused at the lower pole. Again, usually
asymptomatic but prone to infection and ureteral obstruction.

122
D. CYSTIC DISEASE

1. CHIlJ)HOOO POLYCYSTIC KIDNEY (POTTER'S TYPE I)- autosomal recessive


disease that is almost always bilateral and usually eventuates
in death soon after birth. Characterized by dilated collecting
ducts giving a uniform spongy appearance to the kidneys.
Although the cortical nephrons are decreased in number, they
are relatively normal in appearance. It is associated with
cystic changes in intrahepatic bile ducts, and those who
survive childhood may later present with jaundice and signs of
liver failure.
2. DYSPLASTIC KIDNEYS (POTTER'S TYPE II) - results from failure of the
ureteric bud to induce nephron formation from the metanephric
mesenchyme. Both the collecting ducts and abnormal nephrons
are decreased in number and the collecting ducts are dilated.
Bilateral involvement is probably more common but leads to
death soon after birth. It may, however, be unilateral and
these are the patients who come to medical attention because
of abdominal mass. The kidney may be large and polycystic or
small. Microscopically, there are abnormal collecting ducts
surrounded by collars of undifferentiated mesenchyme. The
majority will show islands of cartilage formation.
3. ADULT POLYCYSTIC KIDNEY (PO'lTER' S TYPE III) - autosomal dominant
inheritance with high penetrance. Affects up to 1:500
individuals and is almost always bilateral. This results from
interference with nephron induction as in Type II but at a
later stage of development. There are, therefore, a variable
number of normally formed and functioning nephrons. These
patients usually do not come to medical attention until the
3rd or 4th decade of life. They may present with hematuria
resulting from hemorrhage into the cysts, but will eventually
develop chronic renal failure. This may also be associated
with benign hepatic cysts or saccular "berry" aneurysms of the
cerebral vasculature.
4. MEDULLARY SPONGE KIDNEY - is a disease of unknown etiology which
is often discovered incidentally during workup for some other
problem. It consists of benign dilatation of the collecting
ducts in the medulla of the kidney which may predispose to
infection or stone formation.
5. UREMIC-MEDULLARY CYSTIC DISEASE - possible autosomal recessive
disease which causes renal failure in teenagers and young
adults. Small medullary cysts (especially at the cortico-
medullary junction) are associated with the development of
cortical tubular atrophy, interstitial fibrosis, chronic
inflammation, and decreased numbers of glomeruli. The initial
symptoms may be related to loss of tubular function with loss
of concentrating ability (polyuria nocturia), sodium wasting,
and tubular acidosis.
III. RENAl, CALCULI - calculi may arise when there is an excessive amount of
stone constituents (calcium, oxa1ates, uric acid, cystine, etc) present in
the urine and a favorable environment for the precipitation of those
constituents. Acid urine pH will facilitate precipitation of uric acids
and cystine. Alkaline urine pH will precipitate phosphates which may
combine with magnesium and ammonium (converted from urea by Proteus and
other urea-splitting bacteria) to form large staghorn calculi that may
fill the calyces and pelvis. The presence of bacteria and the stasis of
urine are also predisposing factors. The majority of stones contain
calcium which may be mixed with oxalate, phosphate, or hydroxyapatite.
Large stones tend to be asymptomatic and but predispose to infection and
may produce hematuria due to trauma to the urothe1ia1 surfaces. Smaller
stones are more likely to access and obstruct the ureter and create
clinical symptoms of colicky abdominal pain radiating from the flank to
the groin.

123
IV. URINARY OBSTRUCTION - may result from stones, BPH, congenital defects,
tumors, functional disorders, pregnancy, etc. and predisposes to infection
and stone formation. Hydronephrosis refers to the dilatation of the renal
pelvis and calyces due to the obstruction to the outflow of urine and is
associated with progressive atrophy of the kidney. Depending on the site
of ·obstruction, it may be unilateral or bilateral. The renal changes
become irreversible after about three weeks of complete obstruction or
several months of partial obstruction. If the obstruction is acute, there
may be pain related to acute dilatation of collecting ducts or stretching
of the renal capsule. Gradual unilateral obstruction, either partial or
complete, may be asymptomatic and found only during evaluation of other
problems. Bilateral partial obstruction may first become manifest by loss
of concentrating ability with resulting polyuria and nocturia. Hyperten-
sion may ensue. Bilateral complete obstruction will obviously lead to
anuria.
V. GLOMERULAR DISEASE - injury to the glomerulus is often reflected by the
presence of blood (hematuria) or protein (proteinuria) in the urine
implying that the filter which normally excludes these substances is
damaged.
A. HECHAHISItS OF GLatERULAR DAMAGE

1. LOSS OF GLOMERULAR BASEMENT MEMBRANE (GBI1) POLYAHIONS allows


increased filtration of anionic compounds such as albumin.
2. HYPERFILTRATION - a decreased number of functioning nephrons
places an increased demand on those that are functional. To
maintain normal filtration, this demands an increased glomer-
ular filtration rate (GFR) resulting in increased glomerular
blood flow and increased capillary pressures. This leads to
increased permeability to plasma proteins which accumulate in
urine or in the mesangium, the latter leading to mesangial
proliferation and sclerosis of the glomeruli. This reduces the
number of functioning glomeruli and a vicious cycle is
initiated.
3. IHHUNOLOGIC

a. Natiye "fixed" antigens - on the GBM or associated with


the podocytes may induce antibody formation. The former
would give a diffuse, linear staining on immunofluores-
cence while the later would give a granular, subepithel-
ial staining on immunofluorescence.
b. Non-glomerular antigens - may become attached to, and
incorporated within, the glomerular structure (i.e. DNA,
lectins, cationic proteins bound to anionic GBM, etc.)
and be secondarily attacked by antibody. This would lead
to a granular appearance on immunofluorescence.
c. Circulating antigen-antibody complexes - may become
filtered and trapped in the glomeruli, activate
complement, and attract neutrophils and monocytes which
liberate digestive enzymes. When there is antibody
excess, however, the Ag-Ab complexes are usually large
and are picked up by the RES. With small production of
Ab, the Ag-Ab complexes usually are soluble and do not
become trapped, but with moderate Ab production, the
insoluble complexes may be filtered out. High cationic
compounds tend to traverse the GBM and become trapped in
the subepithelial region, anionic compounds tend to be
trapped in the subendothelial region, and neutral
compounds tend to get trapped in the mesangium.
B. GLOMERULAR RESPONSE TO INJURY

1. CELLULAR PROLIFERATION - may involve endothelial, epithelial, or


mesangial cells.
2. INFLAMMATORY (EXUDATIVE) RESPONSE - primarily by poly and monocyte-
macrophage.

124
3. either true thickening or
GLmfERULAR BASEItEKf IlElBRANE THICltENIN8 -
by the derosition of electron dense deposits (most commonly
subepithe ial).
4. HYALINIZATION AIm SCLEROSIS - amalgam of plasma protein, basement
membrane material, and mesangial matrix destroys glomerular
architecture.
C. NEPHRITICSYNDRCIE - is characterized by hematuria, red blood cell
casts, azotemia, hypertension, and oliguria.
1. ACUTE PROLIFERATIVE (DIFFUSE PROLIFERATIVE, POST-INFECTIOUS)
GLDlERULONEPIIRITIS - most frequently results from trapping of
immune complexes involving exogenous antigens, but less
frequently may be due to endogenous antigens. Although this
does affect adults, it is seen most frequently in children 1-3
weeks after Group A a-hemolytic streptococcal infection. In
children, there is malaise, fever, oliguria, hematuria,
nausea, periorbital edema, and mild-moderate hypertension. Red
blood cell casts and mild protein « 1 gm) are present in the
urine. In adults, there may be abrupt onset of hypertension or
edema. In the acute phase, there is often elevated ASO titers,
decreased C3, positive cryoglobulins, and elevated ESR. In
acute stages, the kidneys may be swollen and "flea-bitten".
Microscopically, the flomeruli are hypercellular
(proliferation of mesangia, endothelial, and to a lesser
extent, epithelial cells),. have a diffuse neutrophilic and
monocytic infiltrate, and· are bloodless. On
immunofluorescence, there is granular deposition of IgG and C3
in the subepithelial region. In children, 95% recover
clinically within two months of onset and morphologically
within three years while a few may progress to chronic
glomerulonephritis or rapidly progressive glomerulonephritis.
The prognosis in adults is controversial but probably a little
poorer.
2. IlEtlBRANOPROLIFERATIVE GLCIIERULONEPHRITIS (MPGN) - is a disease which
affects children and young adults. Light microscopy shows
hypercellular glomeruli due to endothelial and mesangial
p,roliferation creating a centrilobular accentuation and a
'tram-track" appearance to the basement membrane due to
mesangial interposition (mesangial matrix and cellular
cytoplasm being forced between the endothelial cells and GDM.
On EM, electron dense deposits are found in subendothelial
(Type I), intramembranous (Type II, Dense Deposit Disease), or
subendothelial and subepithelial (Type III) locations. Most
cases slowly progress to chronic renal failure but some
patients may develop RPGN. There is a high incidence of
recurrence in transplants.
3. CRESCEJaIC OR RAPmLY PROGRESSIVE GLCIIERULONEPIIRITIS (RPGN) - is a clin-
icopathologic syndrome in which there is a rapid pro%ression
to renal failure in patients with glomerular "crescent forma-
tion (proliferating parietal epithelial cells, monocyte-
macrophages, and fibrin). This may have an abrupt onset of
oliguria and hematuria with lesser degrees of hypertension,
edema, and proteinuria. Crescent formation is always an
indicator of severe underlying glomerular disease and may be
associated with:
a. Imrune Complex Disease (Post-infectious GN) - a few
patients (more commonly adult patients) with post-
infectious GN develop persistent oliguria leading to
anuria.
b. Goodpasture's Disease (Anti-GDK) - is seen most
frequently in young adult males who develop an anti-GDM
antibody which may also cross reacts with pulmonary
alveolar DM. Pulmonary involvement may occur first and
is usually manifested by recurrent hemoptysis or
pulmonary hemorrhage. In the kidney, there is early
focal and segmental then diffuse fibrinoid necrosis in

125
the capillary tufts, degeneration of the epithelial and
endothelial cells with disruption of the GBM and fibrin
deposition. On immunofluorescence, there is linear IgG
and C3 deposition along the GBM.
c. Idiopathic Crescentic GN (no immune deposit disease) -
is generally a diagnosis of exclusion. Immunofluores-
cence patterns are inconsistent.
IV. IgA NEPHROPATHY (Berger's Disease) is a form of focal
glomerulonephritis characterized by recurrent self-limiting
episodes of hematuria in children or young adult males often
following an upper respiratory tract infection. Less
frequently it may present as the nephrotic syndrome. The
histology varies from focal GN with segmental mesangial
proliferation to diffuse mesangioproliferative GN to RPGN but
on immunofluorescence, deposits of IgA, properdin, and C3 are
identified diffusely throughout the mesangium. It may be a
genetic abnormality with overproduction of IgA or excessive
exposure to antigens eliciting an IgA response. It is slowly
progressive with 20% progressing to chronic renal failure.
D. NEPHROTIC SYNDROME - is characterized by proteinuria (> 3.5 gmjday),
hypoalbuminemia (reversed albumin:globulin ratio), hyperlipidemia
(increased LDL andj or VLDL) , lipiduria (free fat and oval fat bodies
in urine), and edema (pitting edema most marked in dependent and
periorbital soft tissue due to hypoalbuminemia and salt and water
retention). Complications include infections (due to loss of
immunoglobulins and complement) and thrombosis (due to loss of
anticoagulant factors).
1. MINIMAL CHANGE DISEASE (lipoid nephrosis) - represents the most
common cause of nephrotic syndrome in children but also
accounts for 15-20% of adult cases. The pathogenesis is
uncertain but there is a selective proteinuria to low
molecular weight proteins. Light microscopy is normal and
immunofluorescence shows no consistent evidence of Ig, C', or
electron dense deposits in the glomeruli. Electron microscopy,
however, reveals visceral epithelial distortion with
cytoplasmic vacuolization, swelling and retraction ("fusion")
of foot processes, and flattening of the epithelial cells
against the capillary basement membrane. Response to
corticosteroid therapy is dramatic but a significant
proportion of patients will have periodic relapses after
steroid withdrawal. The disease does not appear to progress to
chronic renal disease.
2. FOCAL SEGMENTAL GLOMERULOSCLEROSIS (FOCAL SCLEROSIS) - although this
may represent non-specific glomerular change, there is a group
of patients that develop an insidious onset of nephrotic
syndrome but also have micro-hematuria, hypertension, non-
selective proteinuria, a poor response to steroids, deposits
of IgM and C3 in sclerotic mesangium, and a high (50%)
progression to chronic renal failure. The lesions begin in the
juxtamed1illary glomeruli and initially consisting of focal and
segmental collapse of capillary structure with adhesions to
Bowman's capsule. On EM, in addition to the changes of lipoid
nephrosis, there is focal loss of epithelial cells and
thickening of the capillary basement membranes in the affected
areas. Presumably, protein is able to enter the mesangium
where it initiates a mesangial reaction with sclerosis and the
accumulation of a PAS positive material (hyalinosis). With
time, there is progression to global sclerosis. There is a
high incidence of recurrence (25-50%) in transplants.
3. MEMBRANOUS GLOtIERULONEPHROPATHY - is the most common cause of
nephrotic syndrome in adults. Grossly, the kidneys are large,
swollen, and pale. With light microscopy, there is normal
glomerular cellularity but uniform diffuse thickening of the
capillary walls which, as demonstrated by EM, is due to
irregular subepithelial deposits of electron dense material

126
(Stage I). Granular deposits of IgG and C' may be seen on
immunofluorescence. GBM material subsequently accumulates
between deposits forming "spikes" (Stage II) which eventually
surround the deposits (Stage III) and incorporate them into
the GBM (Stage IV). Additionally, there is a loss of
epithelial foot processes. Although up to 15% may be
associated with known antigens, the remainder are idiopathic
and presumably the result of chronic Ag-Ab reactions. Al though
some spontaneous remissions occur, most, over a variable time
span progress to renal failure.
VI. PYELONEPHRITIS (TUBULOINTERSTmAL NEPHRITIS) is most commonly
infectious in origin with dominant organism being gram negative bacteria
(usually E. coli) from patient's intestinal tract. The normal kidney is
resistent to blood borne infection, therefore the most common route of
infection.is bacterial ascension from the urinary bladder.
A. AanE PYELONEPHRITIS - is often associated with urinary obstruction,
instrumentation of the urinary tract, vesicoureteral reflux,
pregnancy, prior renal disease, and diabetes. Symptoms include acute
onset of fever and malaise, costovertebral angle pain, dysuria,
frequency, and urgency. Histology shows interstitial suppurative
inflammation and tubular necrosis. Abscesses, which are often
multiple are most readily apparent in cortex. The acute inflammatory
cells rupture into tubules and travel to collecting ducts.
Laboratory studies reveal pyuria, WBC casts, and positive culture
with greater than 105 organisms. Complications may include
necrotizing papillitis, (usually bilateral and may be multiple),
pyonephrosis, and perinephric abscess. Healing leads to scarring
with cortical depression and calyceal deformity. Appropriate
antibiotic therapy is needed to eradicate the causative organisms.
The incidence of recurrence, however, is as high as 33%.
B. CHRONIC PYELONEPHRITIS - is a major cause of chronic renal failure and
may be the result of chronic obstruction leading to recurrent
infections superimposed on obstructive damage or, more commonly, the
result of reflux nephropathy. Clinically, it may be characterized by
repeated bouts of acute pyelonephritis or may be insidious
presenting as renal insufficiency and hypertension. Tubular damage
leads to polyuria and nocturia. Grossly, these are small kidneys
with broad irregular cortical scars and deformed blunted calyces.
Microscopically, there is tubular atrophy and/or dilatation with
protein casts (thyroidization), interstitial fibrosis, and chronic
inflammation. Periglomerular fibrosis and, in the late stages,
glomerular sclerosis may develop. On culture, bacteria mayor may
not be present.
C. DRUGS AND TOXINS - may produce an interstitial nephritis either through
immunologic hypersensitivity to a wide variety of drugs or direct
toxic damage to the tubules by a wide variety of toxic agents.
Depending on the drug or toxin, this may induce sudden damage with
acute renal failure or cumulative damage over a longer period of
time leading to chronic renal failure.
VII. . HYPERTENSION blood pressure is determined by cardiac output
(influenced by blood volume) and peripheral resistance of arterioles.
Peripheral resistance is regulated by vasoconstrictors (angiotensin II,
leukotrienes, thromboxane, catecholamines) and vasodilators (kinins and
prostaglandins).
A. RENAL CONTROL

1. - decreased pressure in afferent


SECRETION OF PRESSOR SUBSTANCES
arterioles, decreased sodium or chloride reaching macula
densa, or direct sympathetic (a-adrenergic) stimulation of
juxtaglomerular (JG) cells cause release of renin. Renin acts
on a circulating liver derived IX globulin (angiotensinogen) to
produce AG I which is cleaved to AG II which increases
peripheral resistance. Renin also stimulates aldosterone

127
secretion from adrenal cortex (zona glomerulosa) to retain
sodium and water which increases blood volume. There is a
negative feedback on renin secretion by decreased stimulation
of stretch receptors of afferent arterioles due to increased
blood pressure; increased blood volume which leads to
increased GFR leading to decreased sodium reabsorption in
proximal tubule leading to increased sodium reaching macula
densa; increased blood pressure decreases the sympathetic
stimulation of JG cells; direct suppression of JG cells by AG
II. Any condition resulting in increased renin therefore
produces increased blood pressure.
2. MAINTENANCE OF FLUm All) ELECTROLYTE BALANCE - sodium is controlled
by the GFR (decreased GFR causes less filtered Na+ and
increased proximal tubular reabsorption), aldosterone
(increases Na+ reabsorption in distal tubule), and natriuretic
factor (increased volume causes Na+ loss, by an unknown
mechanism). Therefore mechanism that leads to increased Na+
and ~O retention promotes increased blood pressure.
3. RENAL ANTIHYPERTENSIVES - include prostaglandins, kallikrein-
kinin, and neutral lipid factor. Therefore, mechanisms which
decrease secretion of antihypertensives promotes increased
blood pressure.
B. RENAL ItANIFESTATIONS

1. BENIGN (ESSEHrIAL) HYPERTENSION - shows bilateral, fine cortical


granularity. Thickening and hyalinization of walls of
arterioles and small arteries (arteriolonephrosclerosis) is
not the cause of increased blood pressure but once formed may
perpetuate or accelerate increased blood pressure. These
vascular changes cause ischemic damage leading to glomerular
damage, tubular atrophy, and interstitial fibrosis. Collagen
is deposited inside Bowman's capsule. The larger arcuate and
interlobular arteries show reduplication of the elastic lamina
and fibrosis of media.
2. MALIGNANT HYPERTENSION - results from accelerated progression of
previously "benign" hypertension. Fibrinoid necrosis of
arterioles and "onion skinning" of interlobular arteries lead
to ischemia and infarction. Increased renin, AG II, and
aldosterone are present in the serum. A vicious cycle is
established where increased blood pressure causes endothelial
damage, leading to fibrin deposition in vessels, leading to
renal ischemia, leading to increased renin, AG II, and
aldosterone, leading to increased blood pressure, leading to
further endothelial damage. Clinically, there may be abrupt
onset of cardiovascular or CNS symptoms (headache, N+V, visual
scotomas, etc). Unless treated, death by cardiovascular
disease, stroke, or uremia generally follows within a year.
VIII. ACUTE RENAL FAII_URE may be due to diffuse vascular disease
(polyarteritis, malignant hypertension, etc), severe glomerular disease
(RPGN), acute interstitial disease (hypersensitivity to drugs), massive
pyelonephritis, cortical necrosis (DIC), urinary obstruction, or acute
tubular necrosis.
A. ACUTE TUBULAR NECROSIS (ATH)

1. ISCHEMIA (hypotension, shock, decreased RBF) causes patchy


epithelial necrosis throughout the tubule (more prominent in
proximal tubule) with tubulorrhexis (rupture of BM) and
hyaline or granular cast formation in distal tubule and
collecting duct.
2. TOXINS (heavy metals, solvents, antibiotics, mushrooms,
antifreeze etc.) generally cause proximal tubular epithelial
necrosis without rupture of the basement membrane and sparing
of the distal tubule.

128
3. the clinical onset generally begins 24-36 hours
CLINICAL COURSE -
after the initiating insult. Oliguria and decreased GFR leads
to fluid overload, uremia and electrolyte retention. This may
last up to 3 weeks. Oliguria arises from tubular fluid leaking
into the interstitium and tubular obstruction from
interstitial edema or cast formation. Decreased glomerular
filtration is the result of decreased perfusion or prolonged
vasoconstriction, decreased glomerular capillary permeability,
and tubular obstruction. Recovery is heralded by diuresis
which may cause electrolyte loss. As tubular damage is
repaired the diuresis abates. In general, toxic ATN has better
progress because unless other organs are damaged by toxin,
there is no severe underlying disease.
IX. CHRONIC RENAL FAII,URE - when GFR is 20-30% of normal, azotemia
(increased BUN and creatinine) will develop and is often associated with
hypertension. When GFR is 15-20% of normal azotemia is complicated by the
symptoms of the uremic syndrome.
A. FLum. ELECTROLYTE. AND Acm/BASE MANIFESTATIONS - tubular damage impairs
ability to concentrate urine leading to polyuria and nocturia.
Decreased glomerular filtration will increase Na+ and water
retention leading to edema. Retention of W produces metabolic
acidosis and resultant Kussmaul breathing. Increased P04- and
decreased Ca-- leads to increased PTH and parathyroid hyperplasia.
Decreased serum Ca-- from inability of kidney to synthesize 1,25 OH2
Vito D. leads to renal osteodystrophy.
B. CARDIOPULMONARY MANIFESTATIONS - volume overload and salt retention may
lead to congestive heart failure. Hypervolemia may cause
hypertension, and uremia may produce pericarditis, pleuritis, and
pneumonitis.
C. HEMATOLOGIC MANIFESTATIONS - decreased production of erythropoietin,
uremic hemolysis, and GI bleeding contribute to the development of
normochromic, normocytic anemia. Abnormal platelet functioning leads
to bleeding diatheses.
D. GASTROINTESTINAL MANIFESTATIONS - nausea and vomiting, GI bleeding.
E. DERtlATOLOGIC MANIFESTATIONS - itching, accumulation of urochrome pigment,
uremic frost.
F. NEUROLOGIC MANIFESTATIONS - myopathy, neuropathy, encephalomyopathy
leading to coma.
X. NEOPLASMS

A. RENAL CORTICAL "ADEN(IIA5" - are fairly common asymptomatic benign lesions


< 2 cm in size and yellow in color. Histologic architecture may be
papillary, tubular, crstic, or solid.
B. RENAL CELL CARCINCl'IA (RCC - the most frequent form of renal cancer in
adults, RCC tends to occur in mid to late life and has a male
predominance. Patients may be asymptomatic until the tumor reaches
a size to cause space-occupying problems or it may present as
hematuria with or without a palpable mass. It may have endocrine-
like effects and produce polycythemia (erythropoietin), hypercal-
cemia (PTH), hypertension (renin), Cushing's (glucocorticoids), etc.
Grossly, the tumor is usually solitary, bulky, unilateral, yellow in
color with foci of necrosis and hemorrhage, and has a tendency to
invade the renal vein. Microscopically, the architecture may be
papillary, tubular, solid, or trabecular. The characteristic cell is
a clear cell with abundant clear cytoplasm containing glycogen and
lipid. Granular cells and spindle cells may also be present. The
clinical course is very unpredictable; the tumor can metastasize
widely with the most common locations being lung and bone.
C. HI~' TUMOR - is one of the more common tumors of childhood. It may
present with abdominal pain, GI obstruction, hypertension, hemat-
uria, or pulmonary metastases. Grossly, they tend to be large
un1.lateral (90%) masses and may extend across midline. The cut
surface has a variable appearance. Histologically, since the tumor
is derived from nephrogenic mesoderm, it may contain muscle, bone,

129
cartilage, and fat with epithelial cells forming abortive tubules
and primitive glomeruli embedded in a spindle cell stroma. Striated
muscle cells are the most commonly identified mesenchymal
components.
D. TBANSmotW. CELL CABCDQIA. (5-10% of adult renal cancer) - tends to
produce early hematuria and are therefore often identified at an
earlier stage. They may cause urinary obstruction and hydroneph-
rosis. They may be associated with concurrent transitional cell
carcinomas elsewhere within the urinary tract and the presence of
multicentric lesions suggests this may be the effect of a car~inogen
in urine. Prognosis varies depending of the histologic grade of the
tumor.
XI. RENAL MANIFESTATIONS OF SYSTEMIC DISEASE

A. D~ MELLDruS - the renal involvement is generally more severe in


insulin dependent diabetes.
1. include hyaline arteriolosclerosis of lm:th the
VASCULAR CHANGES -
afferent ADd efferent arterioles.
2. GLClERULAR CHANGES

a. Diffuse glomerulosclerosis - is the most common lesion.


There is diffuse glomerular basement membrane thickening
and mesangial cell proliferation. Expansion of the
mesangial matrix eventually crowds out the cells of the
glomerulus leaving a hyalinized, scarred, non-functional
glomerulus.
b. Nodular gloaerulosclerosis (Ki-elstiel·IUIson)
although less common, is 'more suggestive of a diabetic
etiology. "Hyalinized" mesangial nodules are formed in
the peripheral areas of the mesangium.
c. Capsular drop - refers to plasma protein coagulum which
becomes adherent to the parietal epithelium and is
characteristic of diabetes.
d. Fibrin cap - refers to plasma protein coagulum between
the glomerular basement membrane and the epithelial
cells. This, however, is a non-specific change also
associated with other glomerular diseases.
3. TUBULODfl'ERSTITIAL
CHANGES include acute or chronic
pyelonephritis, papillary necrosis (acute necrosis and
sloughing of variable numbers of the renal papillae results in
colicky pain, hematuria, and acute renal failure. It may also
be seen in non-diabetic patients with renal infections,
urinary obstruction, or analgesic abuse), thickening of
tubular basement membranes, and glycogen deposits in the
tubular epithelial cells (Armanni-Ebstein lesions).
B. SYSTEIIC Lupus ERYTHEMATOSUS - the renal lesions of SLE are primarily
confined to glomerular changes resulting from the deposition of DNA-
antiDNA complexes.
1. DIFFUSE PROLIFERATIVE GN (50%) - presents as a nephritic syndrome
with hematuria and hypertension. Microscopically, there is
diffuse glomerular hypercellularity, focal necrosis, deposi-
tion of immunoglobulins and complement in capillary loops and
mesangium creating a "wire loop" appearance. By E.M., the
deposits are characteristically in a subendothelial location.
2. FOCAL PROLIFERATIVE GN (30%) - clinically presents as hematuria
and proteinuria. There is focal, segmental mesangial hyper-
cellularity and thickening with focal necrosis.
3. HESANGIAL NEPHRITIS ( 10%) - shows granular IgG and C3 in
mesangium and mild increase in mesangial cells and matrix.
Presents as microscopic hematuria and proteinuria.
4. DIFFUSE HEI'IBRANOUS (10%) - presents as nephrotic syndrome or
recurrent proteinuria.

130
URINARY BLADDER
I. ACUTE CYSTITIS - is almost always initially due to enteric organisms (;E...
coli, etc) and are more frequent in females due to a shorter urethra and
tendency for bacteria to colonize in the vagina and distal urethra. Major
predisposing factors include residual urine ( congenital or acquired
diverticulae, benign prostatic hypertrophy), stasis of urine (low urine
output, urinary retention), and mucosal trauma (instrumentation, catheter,
foreign bodies, etc). Grossly, the mucosa may appear hyperemic with
exudates, ulcerations, and/or hemorrhage. Microscopically, simply shows
hyperemia with an inflammatory infiltrate. Clinically manifest by
frequency, dysuria, lower abdominal or pelvic pain.
II. CHRONIC CYSTITIS
A. ItALAKOPLAKIA - is characterized by yellow mucosal plaques comprised of
foamy macrophages with granular cytoplasm, giant cells, and lympho-
cytes. Laminated concretions (Michaelis-Gutmann bodies) are present
both within and without macrophages. This is due to chronic E. coli
infection and the granules represent phagosomes stuffed with
bacterial debris.
B. alRONIC IHfER5fITIAL CYSI'ITIS (Hunner's ulcer) - is a cystitis of unknown
etiology presenting with suprapubic pain, frequency and urgency,
typically in middle aged women. The mucosa is edematous with
petechial hemorrhages and focal mucosal ulcerations. Pronounced
chronic inflammation and fibrosis of the muscular wall of the
bladder is present.
c. CXSI'ITIS CYSI'ICA - mucosal epithelium becomes trapped beneath surface
due to chronic inflammation and forms cystic spaces. The epithelium
may show mucous metaplasia (cystitis glandularis).
III. CALCULI - generally result from precipitation of urinary salts, especial-
ly phosphates in association with magnesium and calcium. This is promoted
by urea-splitting organisms which result in alkaline pH.
IV. NEOPLASMS - over 90% arise from urothelium in a sequence of hyperplasia
(increased epithelial thickness ~ 7 layer) ~ dysplasia (increased nuclear
atypia, mitoses, cell immaturity) ~ carcinoma-in-situ ~ carcinoma. They
tend to be multiple, both geographically and temporally. They often
present as painless hematuria.
A. PAPILLm'IA - true benign papillomas are rare. They tend to arise on
lateral walls or trigone and usually single and polypoid in
appearance with delicate papillary fronds.
B. TRANSITIONAL CELL CARCINCIU - more frequently seen in males, they tend to
appear in middle adulthood. Excretion and concentration of carcino-
gens in urine and/or chronic mucosal irritation have been implicated
as an etiology. Up to 70% are papillary, noninvasive low grade
tumors. Papillary lesions look alike grossly but histologically may
range from "papilloma" (Grade I) to Grade III. They are usually
small (less than 0.5 em), red, and possibly multicentric. As the
histologic grade worsens, the tumors tend to be flatter and more
likely to become invasive. As tumors recur, there tends to be
greater cellular anaplasia.
c. SQUAMOUS CELL CARCINCIU - is less common and usually is an invasive,
fungating, or ulcerative lesion that arises from preexisting
squamous metaplasia.
D. ADENOCARCINCIU - is relatively rare and arises from urachal remnants
in the dome of the bladder, metaplastic mucus secreting epithelium,
or cystitis cystica.

131
GASTROINTESTINAL TRACT

ESOPHAGUS

I. DEVELOPMENTAL/STRUCTURAL/FUNCTIONAL DISORDERS

A. A~IA may be associated with other gastrointestinal or


genitourinary abnormalities, vertebral defects, or cardiovascular
malformations. The atretic portion is usually located at or near the
tracheal bifurcation. This usually results in an blind upper segment
which may have a hypertrophied wall and a lower segment which often
(80-90%) is connected to the trachea or a bronchus by a fistulous
tract (tracheo-esophageal fistula). Less commonly, there is a fis-
tulous tract to the upper segment. It is clinically manifested by
excessive salivation, regurgitation, and coughing from reflux of
gastric acid. If there is fistulous communication between the upper
segment and trachea. there may be paroxysmal suffocation from food
in the tracheobronchial tree and aspiration pneumonia.
B. STENOSIS (narrowing of lumen) - may be limited or extensive and either
congenital (presents as feeding difficulties from birth) or acquired
due to post infectious or posttraumatic scarring. neoplasia. sclero-
derma, extrinsic compression. etc.
C. HEBS/RINGS - refer to circumferential mucosal or mucosal/muscular
folds constricting the esophageal lumen. Clinically this usually
manifests as painless dysphagia. They occur in the upper esophagus
of middle aged or elderly women where they may be associated with
severe iron deficiency anemia and atrophic glossitis (Plummer-Vinson
syndrome) or they occur in the lower esophagus at squamocolumnar
junction (Schatzki's rings) occasionally associated with hiatal
hernias.
D. HIATAL HERNIA - refers to a protrusion of a portion of the stomach
above the diaphragm. Most are asymptomatic but a minoritx «10%) of
patients will develop reflux esophagitis and "heartburn .
1. SLmING HERNIA (80-90%) - occurs when traction from a scarred or
congenitally shortened esophagus pulls the cardia of the
stomach through the esophageal hiatus. The hernia may be
accentuated by swallowing.
2. ROLLING (PARAESOPHAGEAL) HERNIA - occurs when a portion of stomach
protrudes through a defect in the esophageal hiatus to lie
alongside the esophagus and form an intrathoracic sac which,
although infrequent, may become strangled and infarcted. This
hernia may be accentuated by increased intraabdominal
pressure.
E. ACHALASIA - is a disorder of esophageal motility and characterized by
the failure of the physiologic gastroesophageal sphincter to relax.
Loss of normal rerista1sis in the distal 2/3 of the esophagus leads
to a functiona obstruction and increased intraluminal pressures.
Although the etiology is unknown. there is a lack or reduction in
the number of ganglion cells of the myenteric Auerbach's plexus in
the body of the esophagus. There may also be a hypersensitivity of
the gastroesophageal sphincter to humoral constricting mediators.
Proximal to the gastroesophageal sphincter. the esophagus is dilated
and the wall may become thinned from progressive dilatation or
thickened from compensatory muscular hypertrophy. Clinically. there
will be progressive dysphagia, eventual regurgitation of ingested
food, and occasionally, aspiration pneumonia. There is a slightly
increased incidence of developing esophageal carcinoma in the
dilated segment due to chronic irritation of the mucosal surfaces.
F. DIVERTICULA - refer to developmental or acquired outpouching of the
mucosa and submucosa with or without its underlying muscularis.
Symptoms are usually related to regurgitation and/or aspiration of
food.

132
1. PULSION DIVERTICULA - are the most common and are related to
increased intraluminal pressures. They occur immediately
proximal to the upper esophageal sphincter on the posterior
wall, in the mid-esophagus (at or near the tracheal bifurca-
tion); or immediately proximal to the gastroesophageal
sphincter. Zenker's diverticulum is a congenital defect of the
upper esophagus in which all layers of the esophagus are
involved in the diverticulum formation.
2. TRACTION DIVERTICULA are due to healing of inflammatory
processes external to the esophagus with fibrosis creating
esophageal distortion. This usually occurs at or below the
tracheal bifurcation.
II. VASCULAR DISEASE

A. ESOPHAGEAL VARICES - portal hypertension, from any cause, diverts the


normal portal venous flow into the azygous system via the coronary
(left gastric) veins and esophageal veins. This produces tortuous
distended veins beneath the esophageal mucosa or in the periesoph-
ageal tissue which run parallel to the long axis. They are generally
asymptomatic until rupture occurs causing massive hematemesis.
Rupture may be idiopathic or the result of trauma (vomiting).
Patients who have had previous variceal bleeding are at risk for
recurrent attacks and there is a 40% mortality with each episode.
B. MALLORY-HEISS SYNDIDIE - linear, longitudinal lacerations of mucosa and
submucosa occurring along the lesser curvature of the stomach or
distal esophagus are most commonly seen after bouts of severe or
prolonged vomiting, but are occasionally seen after other "trauma"
(coughing, resuscitation, instrumentation, etc.). Most frequently
seen in, but not limited to chronic alcoholics, it may lead to fatal
hematemesis but, in general, has a much better prognosis than
ruptured varices and there is less of a tendency for recurrence.
III. INFLAMMATORY DISEASE

A. REFLUX ESOPHAGITIS - mayor may not be associated with hiatal hernia.


The histologic alterations depend on severity and chronicity of
reflux; acid levels; and presence of bile salts, bile acids and
pancreatic enzymes. Superficial epithelial necrosis, peptic ulcera-
tion, and submucosal inflammation are present to varying degrees in
the distal third of the esophagus. With time, this may lead to
fibrosis and strictures. The distal esophagus may also undergo
metaplastic change to a columnar secretory epithelium resembling
both gastric and intestinal mucosa (Barrett's esophagus), and
approximately 10% of these patients will develop adenocarcinoma of
the esophagus. Clinically reflux may be asymptomatic or present with
dysphagia, retrosternal pain, and occasional hematemesis or melena.
B. INFECTIOUS ESOPHAGITIS - herpes virus and candida infections are more
frequently seen than in the past due to increased numbers of
patients in immunocompromised states. Generally, these cause small
ulcerative lesions of the mucosal surface.
C. CHRONIC ESOPHAGITIS - may evolve from predisposing factors such as
reflux, uremia, alcohol abuse, and smoking but many cases are idio-
pathic. The mucosa may become atrophic or dysplastic and submucosal
fibrosis creates clinically significant strictures. The incidence of
squamous cell carcinoma is also increased.
IV. ~OPLASIA - although increasing in frequency, esophageal carcinoma is a
relatively uncommon malignancy in this country which tends to appear in
the elderly. Clinically, there is gradual onset of dysphagia and obstruc-
tion leading to weight loss and malnutrition. This may be complicated by
bleeding, infections with ensuing sepsis, erosion into aorta causing
massive hematemesis, or erosion into trachea allowing aspiration of food.
The inliidious onset leads to late diagnosis and direct extension to
contiguous structures often makes surgical resection impossible. In
general, there is a 5-10% five year survival but with cytologic screening
of at-risk patients and early diagnosis, survival may reach 90%.

133
A. SQUAtIOUS CELLCABCDDIA - comprises about 75-85% of esophageal cancers,
most frequently arises in the mid-esophagus, and is more common in
Blacks and males. Predisposing factors include chronic alcohol and
tobacco abuse and/or pre-existing structural or functional abnor-
malities of the esophagus contributing to a chronic esophagitis. The
development of carcinoma is probably preceded by asymptomatic,
slowly progressive mucosal dysplasia which over the course of months
to years becomes malignant, encircles the mucosa as it penetrates
into the submucosa, and presents as either a polypoid fungating mass
protruding into lumen (60%), a necrotic ulcerative lesion that may
erode through esophageal wall and into surrounding structures (25%),
or a diffuse infiltration into the wall causing rigidity and lumenal
narrowing (15%). Initially, the tumors tend to spread by direct
extension to involve periesophageal soft tissue and adjacent
mediastinal structures.
B. AOENOCARCDgIA - comprises 15-25% of esophageal malignancies and arises
from the submucosal mucous glands or preexisting Barrett's esophagus
and therefore is most frequently seen in the distal esophagus.
Adenocarcinoma is more common in whites and has a high male: female
ratio.
STOMACH
I. DEVELOPMENTAL/STRUCTURAL/FUNCTIONAL DISORDERS

A. PYLORIC STENOSIS - familial forms are relatively frequent (1: 300 -


1:900) with a 4:1 male predominance (frequently the first born
male). Hypertrophy and hyperplasia of the circular muscle of the
pylorus (perhaps secondary to muscular hypersensitivity and spasm)
may be palpated as a mass in the newborn. Edema and inflammation
aggravate the stenosis leading to obstruction and projectile
vomiting characteristically beginning around the third or fourth
week of life. On rare occasions, symptoms may not develop until
adulthood. Acquired forms of pyloric stenosis (postinflammatory
scarring or neoplastic infiltration of the pylorus) may also lead to
symptomatology of pyloric obstruction.
B. GASTRIC DILATATION - may result from pyloric obstruction or ileus and
can cause tremendous dilatation and thinning of the gastric wall.
c. ~ - although uncommon, this is a disastrous occurrence usually
due to trauma to a distended stomach and is rapidly followed by
shock and death.
II. INFLAMMATORY DISEASE - the normal gastric mucosa secretes mucus and
bicarbonate to form a protective barrier against the back diffusion of H+
into mucosa and leakage of Na+ from mucosa. Additionally, prostaglandin
synthesis and secretion may exert a protective effect on the surface epi-
thelial cells.
A. ACUTE (EROSIVE) GASTRITIS - is associated with a wide variety of
predisposing factors. The protective mucosal barrier may be altered
by aspirin, toxins, disturbance of mucosal blood perfusion, etc.
Also, the opening of A-V shunts in the gastric mucosa (by neurogenic
or chemical means) may shunt sufficient blood away from the mucosal
surface to produce ischemic damage allowing back diffusion of acid.
Sudden closure of the shunts and return of blood flow to the injured
mucosa may then result in congestion and hemorrhage. Histologic
changes vary from localized hyperemia, edema, and mild mixed
inflammatory infiltrates of the lamina propria (acute superficial
gastritis) to focal hemorrhage and superficial sloughing of mucosa
(acute hemorrhagic gastritis) to diffuse hemorrhage and erosion of
the mucosa (acute hemorrhagic erosive gastritis). With increasing
severity, there is more evidence of bleeding, a greater inflammatory
infiltration, and the initially asymptomatic lesions now produce
symptoms of epigastric pain, nausea and vomiting, and with severe
cases, massive hematemesis.

134
B. also comprises a continuum of morphologic
CHRONIC (NONEROSIVE) GASTRITIS -
and histologic changes which is often associated with concomitant
Helicobacter (formerly Campylobacter) pylori infections. Histologic
changes in chronic superficial gastritis consist of mild localized
or diffuse superficial lymphocytic and plasmacytic infiltrates of
lamina propria while chronic atrophic gastritis is characterized by
heavier and deeper inflammatory infiltrates with thinning of the
mucosa and atrophy of glands, partial loss of parietal cells, and
surface epithelial atypia. Gastric atrofhY is characterized by
flattening or loss of normal rugal folds; ymphocytic, plasmacytic,
and occasionally eosinophilic infiltrates; marked glandular atrophy
with almost total loss of parietal cells; and replacement of surface
or pit epithelium by goblet cells (intestinal metaplasia) which
frequently show atypical or dysplastic changes. Clinically, chronic
gastritis is often asymptomatic but may present with vague abdominal
pain, nausea, or vomiting. The major clinical significance, however,
is the association of other more serious disorders in patients with
chronic gastritis. These include:
1. PERNICIOUS ANEMIA - is associated with a fundal gastric atrophy
(type A gastritis) and loss of parietal cells resulting hypo-
or achlorhydria. These patients produce antibodies directed
against intrinsic factor, intrinsic factor-Vitamin B12
complexes, and gastric parietal cells. In addition to B12
deficiency anemia, these patients are more prone to developing
gastric cancers.
2. GASTRIC PEPTIC ULCERS - are often accompanied by chronic gastritis
in the antral portion of the stomach (type B gastritis) but
the relationship between the two is unclear.
3. GASTRIC CARCINOHA - is associated with concomitant chronic
gastritis.
III. ACUTE PEPTIC ULCERATION ("stress ulcer") - is basically an extension
of acute erosive gastritis where the mucosal erosion penetrates the
muscularis mucosa. They tend to arise initially in the proximal stomach
but, depending on the severity, multiple ulcers may involve the entire
stomach. Usually they are small, circular ulcers with a reddish-brown base
rarely extending deeper than the submucosa. They initially appear within
24 hours after severe trauma i.e., extensive burns (Curling s ulcers),
acute brain damage (Cushing's ulcers), severe medical illnesses, surgery,
or shock. They may also be associated with steroid therapy, aspirin abuse,
and smoking. The pathogenesis is uncertain but it may involve disturbances
in vascular perfusion of the mucosa, damage to mucosal acid barrier, and
back diffusion of gastric acid. Acid must be present for the ulcers to
occur but usually acid concentration is not increased above normal except
in the case of Cushing's ulcers where brain damage causes increased vagal
activity with resultant acid hypersecretion. Generally, stress ulcers are
asymptomatic but may cause mild to moderate, and occasionall~ massive,
bleeding. Usually they heal without sequelae once the "stress' has been
removed. They do not progress to chronic peptic ulcers.
IV. CHRONIC PEPTIC ULCER DISEASE - chronic peptic ulcers affect approx-
imately 5-10% of general population. The peak incidence is in mid-adult
life and is uncommon before the age of 20 or in premenopausal women.
Resulting from acid-pepsin digestion of mucosa, these are usually solitary
lesions occurring in the duodenum (most frequent) or stomach. Approx-
imately 10% of patients have both duodenal and gastric ulcers. They can,
however, develop in Barrett's esophagus, Meckel's diverticulum, and
surgical gastroenteric anastomoses. The incidence of duodenal ulcers is
slowly declining while the incidence of gastric ulcers has remained
steady.
A. duodenal ulcers are increased in frequency in patients
DUODENAL ULCER -
with a genetic predisposition (type 0 blood, MEN I, etc.), those who
smoke and/or drink, and those who have concurrent medical disease
(C·OPD, hepatic cirrhosis, rheumatoid arthritis, etc). The mechanism
of ulceration probably relates more to the delivery of increased
acid-pepsin secretion to the duodenum rather than altered mucosal

135
resistance although both mechanisms may playa role and pathogenetic
mechanisms may vary from individual to individual. Generally, in
duodenal ulcer patients the mean acid secretion is higher than that
found in normal patients or those with gastric ulcers. The total
parietal cell mass of the gastric mucosa may be increased and
roughly correlates with degree of acid secretion. Parietal cells may
be more sensitive to gastrin stimulation or gastrin secretion may be
abnormally prolonged due to impaired feedback inhibition by
increased acid or secretin levels. Excessive stimulation of parietal
cells by the vagal nerve may occur, and rapid emptying of gastric
contents into duodenum or impaired duodenal secretion of protective
prostaglandins (PGE 2 ) or bicarbonate may occur.
B. GASTRIC UUER - the mechanism of ulceration probably relates more to
a function of altered mucosal resistance than to increased acid
secretion. Most gastric ulcer patients have preexisting or
concurrent chronic gastritis and are normo- or hyposecretors of
acid-pepsin. The mechanisms may be related to antroduodenal reflux
of bile acid, pancreatic secretions, and lysolecithinj deranged
mucosal blood flowj or other impairment of the mucosal barrier.
There is an increased incidence in patients who smoke and/or drink,
abuse aspirin, or have COPD. There is less evidence of a genetic
predisposition than with duodenal ulcer patients.
C. UUER MORPHOLOGY - most peptic ulcers are solitary. If duodenal and
gastric ulcers are both present (10% of patients), duodenal is the
first to appear. Duodenal ulcers most often occur on the anterior
wall of the first portion of duodenum while gastric ulcers are
usually found in the antrum with slightly greater predilection for
the lesser curvature. Ulcers are usually under 2.0 cm diameter but
may be greater than 4.0 cm diameter with the gastric ulcers tending
to be the larger. The crater appears punched-out with a straight
wall perpendicular to the base of the ulcer which usually lies
within the submucosa or muscularis propria. The crater base is
usually free of exudate "and, in gastric ulcers over time, scarring
creates mucosal rugal folds that radiate outward from the central
crater. Mucosal surfaces along the margin of gastric ulcers are
generally not indurated but do show underlying chronic gastritis and
may undergo intestinal metaplasia.
D. CLINICAL PRESENTATION - patients may be asymptomatic. Alternatively,
duodenal ulcers may present with intermittent burning epigastric or
substernal pain that classically begins within a few hours after
eating and is relieved by antacids, milk, or food which tends to
neutralize the excess acid. In patients with gastric ulcers,
however, food may be irritating and induce vomiting which is more
likely to lead to anorexia and weight loss. Referred pain may be
present in thorax, back, or upper abdomen. Complications include
bleeding (30% of patients, 25% of deaths), perforation (5% of
patients, 65% of deaths), or pyloric obstruction due to scarring.
Malignant transformation is rare in gastric ulcers and virtually
nonexistent in duodenal ulcers.
V. NEOPLASIA

A. BENIGN LESIONS - include leiomyomas and benign polyps but are generally
asymptomatic.
B. occurring in mid to late adulthood, these are most
GASTRIC CARCINOHA -
often found in the distal antral and pyloric regions, but there has
been an increasing frequency of proximal tumors. Predisposing
conditions inc.lude benign neoplastic polyps, chronic atrophic
gastritis, and gastric atrophy. Patients with previous partial
gastrectomies also have an increased incidence. Environmental and
dietary factors (nitrites) probably also playa significant role in
the pathogenesis. All gastric carcinomas are adenocarcinomas and
consist either of cells reminiscent of intestinal columnar epi-
thelium and/or gastric mucus producing cells. The former tend to
produce expanding masses which consist of nests and sheets of
malignant cells tending to show glandular differentiation while the
latter tend to be less cohesive and diffusely infiltrating eliciting
a striking desmoplastic stromal response or sometimes producing
136
abundant mucin ("signet-ring" carcinoma). These tumors may grow into
the gastric lumen as fungating or polypoid masses, extend diffusely
into the gastric wall creating ill-defined mural thickening (linitis
plastica), or create ulcerative lesions with indurated, heaped up
margins and a shaggy, necrotic base. If the tumors are identified
early while still confined to the mucosa and submucosa (early
gastric cancer), gastric resection will markedly improve the
prognosis. If not identified early, they will eventually invade and
penetrate the muscularis propria (advanced gastric carcinoma), seed
the peritoneum, and metastasize. Commonly, clinical symptoms are
non-specific and include weight loss, abdominal pain, anorexia, and
vomiting. Occult blood loss may be demonstrated by 'coffee-ground"
vomitus or guaiac positive stools. The non-specific nature of the
presentation often delays the diagnosis resulting in a poor (10%-
20%) overall five-year survival rate. Poor prognostic indicators
include a proximal location and advanced stage.

SMALL BOWEL/COLON/RECTUM
I. DEVELOPMENTAL/STRUCTURAL/FUNCTIONAL DISORDERS
A. SltALL BOHEL

1. ATRESIA - single or multiple segments of bowel represented only


by a solid fibrous cord may be due to intrauterine vascular
insult. The majority occur in the ileum and are manifested by
persistent vomiting due to intestinal obstruction.
2. DIVERTICULA (pulsion) - are uncommon but when present may be
solitary or multiple. They tend to occur along mesenteric
border (where mesenteric vessels and nerves enter the bowel
wall). Rarely, this may result in bacterial overgrowth and B12
deficiency, bleeding, or perforation.
3. MECKEL'S DIVERTICULUM "" arises from persistence of the proximal
portion of the embryonic omphalomesenteric (vitelline) duct.
This occurs in 1-4% of the general population with equal sex
ratio although males are more likely to be symptomatic.
Variable in size, they are located along the antimesenteric
border of the ileum usually within twelve inches of the
ileocecal valve. Microscopically, about 50% contain hetero-
topic gastric mucosa and, therefore, may develop peptic
ulceration and bleeding (occasionally mimicking appendicitis).
Rarely, they may perforate.
4. PANCREATIC REST - refers to submucosal (usually duodenal) foci
of ectopic but normal pancreatic tissue which may protrude
into the lumen. Only clinical significance is to differentiate
it from a neoplasm.
5. HERNIA - refers to a serosal lined outpouching of peritoneum
which occur in areas of weakness or defect in the abdominal
wall (femoral or inguinal canals, umbilicus, previous surgical
sites, etc.). Segments of bowel may enter these sacs and are
particularly prone to do so during periods of increased intra-
abdominal pressure (defecation, lifting objects, etc). If the
venous drainage of the bowel is compromised, ensuing venous
stasis and edema traps the bowel within the hernia sac (incar-
ceration) and may result in ischemic necrosis (strangulation)
and possible perforation.
6. FIBROUS ADHESIONS - peritonitis from any cause (bacterial,
surgical, etc) may result in the development of fibrous
adhesions between loops of bowel or between the bowel and
abdominal viscera or peritoneal wall. Since the small bowel is
relatively mobile, portions may get trapped between adhesions
resulting in symptoms of obstruction.
7. INTUSSUSCEPTION - occurs when a portion of bowel (intussusceptum)
becomes telescoped into a more distal portion (intussuscip-
iens). This usually occurs in previously healthy children and
involves the telescoping of the terminal ileum into the cecum.

137
In adults, it is often associated with polyps or tumors which
get caught up in peristaltic action. As the proximal portion
is being propelled further distally by peristalsis, its
attached mesentery is being pulled along also and may lead to
vascular compromise and infarction. Clinically, it is
characterized by acute obstructive symptoms.
8. VOLWLUS - refers to a twisting of bowel on its mesenteric
attachment resulting in obstruction, vascular compromise, and
infarction. This occurs most often in the small bowel due to
its mobility but may also occur at the sigmoid colon or cecum.
B. COLON

1. tlALRarATION - occurs when the cecum fails to descend to its


normal right lower quadrant position. Usually of little
significance but may confuse diagnosis of acute appendicitis
since the appendix does not lie in its characteristic
location.
2. IMPERFORATE ANUS - may simply be the failure of anal membranous
septum to perforate but, on the other hand, it may be as-
sociated with agenesis, atresia, or stenosis of the anal
canal. There may also be associated fistulas to the female
genital tract or the male urinary tract.
3. HIRSCHPRUNS'S DISEASE (congenital megacolon) - is an uncommon
congenital disease with a strong male predominance (9:1). It
is characterized by the failure of Meissner's (submucosal) and
Auerbach's (myenteric) plexuses to develop. This is initially
manifested at the anorectal junction and extends proximally to
a variable extent but is usually limited to the rectum and
sigmoid. A lack of peristalsis creates a functional obstruc-
tion and results in dilatation of the colon proximal to the
aganglionic segment. Clinically, infants tend to be consti-
pated from birth although there may be occasional passage of
diarrheic liquid stool. As the -colon dilates, there is
abdominal distension and vomiting.
4. DIVERTICULAR DISEASE (diverticulosis/diverticulitis) - refers to
single or, more commonly, multiple outpouchings of colonic
mucosa and submucosa through the muscularis propria. The
etiology is not clear but it may be related to weakness of the
bowel connective tissue either through disease processes or by
normal aging; increased intraluminal pressures created by
alteration of quantity and character of stool (low residue-low
fiber diets); or a primary intrinsic defect of colonic
muscular coat. In the majority of patients, there is hyper-
trophy of the circular and longitudinal muscular layers
(prediverticular disease) with subsequent herniation of the
mucosa and submucosa through the colon wall at points of
weakness between the mesenteric and antimesenteric taenia
where the vessels penetrate the circular muscle (divertic-
ulosis). Most diverticula (95%) occur in the sigmoid colon,
but they may involve the entire length of the colon. Secondary
inflammation (diverticulitis) may follow fecal impaction
within a diverticulum and infrequently result in pericolic
abscesses, peritonitis, or sinus/fistula tracts. Fever and
leukocytosis are clinical indicators of diverticulitis. Most
patients are asymptomatic, however, but symptoms may occur
even in the absence of diverticulitis. The onset is insidious
with intermittent or continuous lower abdominal discomfort,
constipation, and distension. Progression results in worsening
constipation, abdominal pain, and rectal bleeding. A mass may
be palpable in the left lower quadrant. Chronic diverticulitis
may lead to fibrous thickening of the wall and decreased
luminal diameter. .
5. MELANOSIS COLI - is an asymptomatic discoloration of mucosal
surface of the colon due to accumulation of brown-black
pigment within macrophages of the lamina propria. Associated
with chronic anthracene laxative abuse.

138
II. VASCULAR DISEASE
A. ISCHEtIIC BQNEL DISEASE - may affect small bowel and/or the colon.
1. TRANSIIURAL INFARCTION - is generally the result of mesenteric
vascular occlusion. Although the overall incidence of trans-
mural infarction is relatively low, it has an increasing
frequency with age, and is a surgical emergency. Numerous
anastomoses between branches of the celiac, superior mesen-
teric, and inferior mesenteric arteries reduces the incidence
of arterial infarction. The colon is additionally protected by
virtue of some vascular supply from the posterior abdominal
wall (colon infarcts are most commonly seen in watershed areas
of arterial supply i.e. the splenic flexure and mid-rectum).
The outcome of any vascular occlusion, therefore, depends on
the degree of occlusion, the rate at which it develops, the
physical location in the vascular bed, the patency of other
collateral vessels, and the p02 of the blood. Arterial
occlusion (60%) can arise from thrombus formation on an
atherosclerotic plaque (most common), emboli, or intrinsic
vascular disease while venous occlusions (40%) are usually due
to thrombus formation secondary to trauma, intraperitoneal
infection, or external compression. Within 18 hours, vascular
occlusion, either arterial or venous, will result in patchy or
confluent hemorrhagic infarction of the bowel which is first
manifest by vascular congestion and mucosal, submucosal, and
subserosal hemorrhage. The borders of arterial infarcts are
usually distinct while those of venous infarcts tend to be
blurred. Edema and thickening of the bowel wall, sloughing of
the mucosal surface, and inflammatory infiltrates follow.
Fibrinous or fibrinopurulent exudate appears on the serosal
surfaces within 24 hours and without surgical intervention,
secondary bacterial contamination, peritonitis, and perfor-
ation can occur. Clinically, patients tend to be older males
with thrombus formation in atherosclerotically compromised
mesenteric vessels who develop abrupt onset of severe abdom-
inal pain with nausea, vomiting, possible bloody diarrhea, and
ensuing shock. Bowel sounds are decreased and, with the onset
of peritonitis, a rigid abdomen develops. Without interven-
tion, sepsis, shock, blood loss, or perforation will culminate
in death within 48 hours.
2. AcurE HEMORRHAGIC ENTEROPA11IY - multifocal areas of ischemic damage
involving the mucosa and submucosa (mucosal infarction) or the
mucosa, submucosa, and muscularis propria with sparing of the
serosa (mural infarction) is generally due to tissue hypoper-
fusion secondary to shock, cardiac failure, infections,
vasoconstrictive drugs, etc. Histologically, this appears
similar to transmural infarctions except for the depth of
involvement and less of an acute inflammatory infiltrate.
Clinically, it presents with abdominal pain, cramping, and
bloody diarrhea and should be suspected in patients having
predisposing factors to hypotension. Prognosis is generally
better than with transmural infarcts and can often be managed
medically.
B. ANGICDYSPLASIA - refers to abnormal dilatation (telangiectasia) of thin
walled vessels in the lamina propria of the cecum and/or ascending
colon (often immediately below the surface epithelium) which
predisposes to intestinal bleeding. Less frequently, it may also
involve the stomach and duodenum. The etiology is unclear but may be
related to entrapment of penetrating vessels in the muscular wall of
the bowel with development of incompetency of the precapillary
sphincters in the mucosal vessels. Clinically asymptomatic until
bl~eding occurs in mid to late adulthood.
C. maroRRHOlDS - are extremely common but except for pregnant women are
unusual under the age of 30. Persistently elevated venous pressure
in hemorrhoidal plexus causes variceal dilatation of veins beneath
rectal (internal hemorrhoids) and/or anal (external hemorrhoids)
139
mucosa. When symptomatic, they produce pain, itching, and rectal
bleeding characterized by streaks of blood on the external surface
of the stool. Predisposing factors include chronic constipation with
straining, pregnancy, and portal hypertension. Thrombosis, super-
ficial ulceration, fissure formation, and hemorrhagic infarction are
possible complications.
III. MALABSORPTIVE DISEASES - most absorption of foodstuffs occurs in the
duodenum and jejunum except for vitamin B12 and bile salts which are
absorbed in the ileum. Malabsorption is characterized by the variable
absorption of ingested protein, carbohydrates, fats, vitamins, minerals,
or water and may result from lack of adequate nutritional factors in the
diet; defects in gastric, hepatobiliary, or pancreatic function which
interferes with the breakdown of protein, carbohydrates, or fats into
molecules of absorbable size; alteration of the absorptive surface area of
the bowel; defects in transport of absorbable molecules across the mucosal
barrier of the bowel; or defects in transport of absorbed foods into
portal circulation. Clinically, malabsorption may be characterized by
anemia, muscle wasting and weight loss, abdominal distension, borborygmi,
and abnormal stools (steatorrhea, diarrhea, etc).
A. CELIAC DISEASE (gluten-sensitive enteropathy, non-tropical sprue) -
results from an immunologic hypersensitivity to gliadin (a glyco-
protein constituent of the protein gluten found 'in wheat, oats,
barley, and rye products) which induces, usually beginning in child-
hood, increased levels of 19A and IgM antibodies in the intestinal
mucosa. Whether the antibodies are cytotoxic or whether there is
cell mediated destruction of the surface epithelial cells of the
jejunum is not certain. There is, however, accelerated sloughing of
the jejunal epithelium (most severe in the upper jejunum and
decreasing distally) leading to severe atrophy of the jejunal villi
(thereby decreasing the absorptive surface area), increased depth of
the intervillous crypts due to epithelial hyperplasia (in an attempt
to repopulate the surface epithelium), and increased chronic inflam-
matory cell infiltration of the lamina propria. Residual surface
epithelial cells show cytologic alterations including irregular
nuclear positions and poor staining. Diarrhea is usually the
presenting complaint, but it may present simply as an unexplained
anemia. Elimination of wheat, oats, barley, and rye products from
the diet will restore normal physiologic function and, in most
cases, will restore normal histology of the jejunum although there
is an increased incidence of subsequent small bowel lymphomas and,
to a lesser extent, adenocarcinomas.
B. TROPICAL SPRUE - is most likely the sequelae of an as yet unclassified
bacterial enteritis. Histologic changes vary from normal to findings
similar to celiac disease but, unlike celiac disease, the histologic
changes tend to involve the small bowel uniformly. It is most
frequently seen in adolescents and young adults and is clinically
characterized by steatorrhea and folate deficiency anemia. Most
patients can be cured by broad spectrum antibiotic therapy with
folic acid supplementation.
C. HHIPPLE'S DISEASE - is a systemic disease with a marked male
predominance. The etiology is most likely infectious. Although the
disorder is often cured by antibiotic therapy and although EM
reveals rod-shaped bacilli within lysosomes of macrophages and along
the brush border of the epithelial cells, no organisms have been
consistently isolated. The characteristic feature is the presence of
macrophages filled with PAS positive, diastase resistant, granules
(the bacilli-bearing lysosomes) which are found most commonly in
lamina propria of small bowel. Villi become distended and blunted by
the macrophages giving a shaggy appearance to mucosal surface.
Lymphatic obstruction results in dilatation of lacteals and, if
these rupture, lipogranulomas may form. Occasionally fine lipid
vacuoles can be seen within surface epithelial cells. The disorder
usually presents clinically as severe malabsorption (steatorrhea and
emaciation), but it may present with migratory polyarthralgia, CNS
symptoms, or lymphadenopathy.

140
D. DISACCHARmASE DEFICIENCY - may be acquired in association with other
forms of bowel disease or result from a congenital inborn error of
metabolism in which disaccharides cannot be broken down into
absorbable monosaccharides. When manifested by adults, any of the
enzymes may be deficient but when first manifested in infancy, there
is usually a lactase deficiency. At first feeding, infants develop
abdominal distension and explosive, watery, frothy stools. There is
little, if any, histologic changes of the mucosa and treatment is by
dietary elimination of milk and milk byproducts.
E. ABEIALIPOPRDTEINEI1IA - is an autosomal recessive inborn error of
metabolism manifested in infancy by failure to thrive, diarrhea, and
steatorrhea. These infants are unable to synthesize the apoprotein
Butilized in the lipoprotein coat of chylomicrons, VLDL, and LDL.
Triglycerides accumulate in mucosal cells and there is severe hypo-
lipidemia with decreased chylomicrons, VLDL and LDL.
F. REGIONAL ENTERITIS, CHRONIC PANCREATITIS, and VIRAL ENTERITIS - may also
result in malabsorption.
IV. INFLAMMATORY DISEASE

A. ACUTE APPENDICITIS - is probably initiated by obstruction of mucus


drainage from the lumen (fecaliths, lymphoid hyperplasia, fibrosis,
tumor, etc) which may lead to distension, vascular compromise, and
secondary bacterial invasion. As polys emigrate from vessels, a
fibrinopurulent exudate appears on the serosal surface. Suppurative
necrosis of the mucosa and muscularis further compromises the
vascular supply resulting in gangrenous necrosis and predisposing to
rupture. Appendicitis is clinically characterized by diffuse
abdominal discomfort eventually localizing to the right lower
quadrant (MCBurney's point) accompanied by nausea, vomiting,
anorexia, and diarrhea/constipation. As the inflammation involves
the peritoneal wall, abdominal guarding with rebound tengerness,
fever, and leukocytosis develop. Without surgical intervention,
rupture, peritonitis, sepsis, abscess formation, or portal
thrombosis may lead to death.
B. CROHN'S DISEASE (REGIONAL ENTERITIS) - is a systemic inflammatory disease
primarily characterized by segmental, transmural, and granulomatous
involvement of the small bowel and/or colon, but which less commonly
may involve any portion of the G.I. tract from the oral cavity to
the anus. Skin, skeletal muscle, joints, bone, etc. may also be
involved and it may be associated with other diseases related to
autoimmunity such as ankylosing spondylitis, erythema nodosum,
autoimmune hemolytic anemia, and sclerosing cholangitis.
1. INcmENCE - appears to be increasing in frequency with slightly
greater predilection for females. Peak incidence occurs in the
adolescent-young adult years. The etiology is unknown but
various theories include genetic predisposition, infection,
autoimmune reactions, and cellular immune deficiencies.
2. MORPHOLOGY - the gross appearance is characterized by segmental
involvement ("skip" lesions) of the bowel. Affected areas are
sharply demarcated from intervening normal bowel. Most often
small bowel and colon are both involved (50-60%), however, the
small bowel alone (30-35%) or the colon alone (granulomatous
colitis - 20%) may be affected. Classically, in involved areas
there is transmural thickening and fibrosis of the bowel wall
with narrowing of the bowel lumen, linear serpentine ulcera-
tion of the mucosal surface running along the long axis of the
bowel (cobblestone mucosa), and encirclement of the bowel by
thickened, edematous, and fibrotic mesenteric fat (creeping
fat). Characteristic microscopic findings include non-specific
chronic inflammation affecting all layers of the bowel wall
with non-caseating granuloma formation, and lymphoid aggre-
gates. Approximately 40% of cases, however, fail to reveal
granulomatous inflammation. There is marked thickening,
chronic inflammation, and fibrosis of the submucosa and
subserosal areas with relative preservation of the muscularis

141
propria. The mucosa shows varying degrees of necrosis and
ulceration, goblet cell hyperplasia, and over time, dysplastic
changes of the surface epithelium.
3. CLINICAL COURSE - the disease begins as intermittent bouts of
fever, abdominal pain, and diarrhea. As the disease progres-
ses, the intervals between attacks become shorter and the
symptoms become more pronounced leading to nausea, vomiting,
anorexia, weight loss, slow blood loss, and electrolyte
imbalance. Complications include bowel obstruction secondary
to fibrous strictures (especially of terminal ileum),
adhesions, fistula formation, peritoneal abscess, hemorrhage,
and malabsorption of protein, vitamin B12 , folic acid, and
iron. Patients with this disease have an increased risk of
developing adenocarcinoma of the colon or, less likely, small
bowel. Recurrence rate after surgical resection is high.
C. is an acute and chronic inflammatory disease of
ULCERATIVE COLITIS -
unknown etiology causing extensive ulceration of the mucosal sur-
faces of the colon and, infrequently, the terminal ileum. With
somewhat greater frequency, patients with ulcerative colitis develop
extraintestinal manifestations similar to those found in regional
enteritis. Theories on etiology are similar to regional enteritis
and include the possibility of an as yet unidentified infectious
agent and/or an immunologic mechanism. Some authors feel that
ulcerative colitis and regional enteritis are simply different
manifestations of the same disease process and should be lumped
under the term "inflammatory bowel disease".
1. MORPHOLOGY - ulcerative colitis always begins in the rectum and
spreads proximally without "skip lesions" and, although
infrequent, may spread into terminal ileum ("backwash
ileitis"). Early lesions begin as small mucosal hemorrhages
and crypt abscesses. The abscesses may undermine surrounding
mucosa and produce mucosal sloughing and ulceration. With
progression, the ulcers may extend to the muscularis propria.
On the rare occasion that the process extends through the
muscularis, pericolic abscesses may form. Residual islands of
edematous, inflamed mucosa stand out in contrast to the
surrounding ulceration and are called inflammatory pseudo-
polyps. Moderate fibrosis of the bowel wall with non-specific
chronic inflammatory infiltrates may occur but rarely to the
extent seen in regional enteritis. Although fistula formation
can occur, it is less frequent than in regional enteritis and
usually is confined to the perianal-perirectal regions. The
epithelium of the ulcer margins may undergo metaplastic and
dysplastic changes, and this may account for the high inci-
dence of malignant transformation.
2. CLINICAL COURSE - ulcerative colitis is a recurrent disease
initially manifested most often in young adulthood as abdom-
inal pain, cramps, and bloody diarrhea often subsequent to
emotional stress. Symptom free intervals may vary from months
to years. With recurrent attacks, weight loss, fever, and
electrolyte disturbances may occur. During acute attacks,
patients may develop marked colonic distension (toxic mega-
colon) requiring immediate surgical intervention. Malignant
transformation is about five times more common than in
regional enteritis and the risk is partially dependent on the
duration of the disease (the longer the disease is present,
the greater the risk), the age of onset (the younger the age
of onset, the greater the risk), the chronicity (the shorter
the symptom free intervals, the greater the risk), and the
extent (the greater the involvement, the greater the risk).
D. - refers to the presence of a membrane-like
PSEUDOl1EI1BRANOUS COLITIS
inflammatory exudate consisting of mucin, neutrophils, and fibrin,
which is patchily distributed over the mucosal surface of the colon
or, to a lesser extent, small bowel. Underlying the membrane are
inflammatory infiltrates and varying degrees of mucosal necrosis. It

142
Occurs with various conditions that allow the proliferation of
Clostridium difficile. This may occur after the use of various
antibiotics (especially clindamycin and lincomycin) or subsequent to
severe trauma or medical/surgical illness. The cytopathic effects of
the bacterial toxin (suppurative inflammation and focal mucosal
necrosis) initially develops within crypts and spews an inflammatory
coagulum onto the surface of the bowel which may coalesce into large
patches. It is clinically characterized by either profuse diarrhea
developing during antibiotic therapy which generally will resolve
within two weeks after discontinuation of the drug or, alter-
natively, copious bloody diarrhea which may develop many days after
antibiotic therapy has been completed and may rapidly lead to
electrolyte imbalance and possible death particularly in those
patients who are already seriously ill.
v. GASTROINTESTINAL POLYPS - may occur anywhere along the gastrointestinal
tract but are most commonly found within the colon.
A• HAl'lAR'lJIIATOUS POLYPS

1. .JUVENILE RETENTION POLYP most occur in childhood but


approximately 10% are found in adults. The rectum is most
frequent site and although typically solitary, they may be
multiple. They are felt to arise from developmentally
malformed glands that gave rise to mucous retention cysts.
Grossly they appear as a smooth, unfissured, pedunculated
masses containing mucus or pus filled cysts. Microscopically,
they consist of dilated, cystic glandular spaces lined by
normal colonic epithelium within an expanded lamina propria
containing a mild acute and chronic infiltrate.
2. PEUTZ-.JEGHERS POLYP - Peutz-Jeghers syndrome is an autosomal
dominant disease first manifest in adolescence and young
adulthood and characterized by polyposis of entire G.T. tract
(with predilection for jejunum, ileum, and stomach) in
association with melanin pigmentation of the buccal mucosa,
lips, and digits. The polyps are hamartomas consisting of
highly convoluted normal mucosa overlying a branching core of
smooth muscle. Atypia and mitoses are rare but infrequently
may be associated with G.1. malignancies as well as other
visceral neoplasms.
B. HYPERPLASTIC POLYPS -are the most common type of polyp in adults. Loss
of growth restraint on the normal proliferative zone at the base of
the colonic crypts results in lengthening of the crypts creating an
epithelial polyp. Grossly they appear as small, pink, sessile
mucosal nodules, often multiple, usually less than 0.5 cm in
diameter. Microscopically, there is cellular crowding, hyperchrom-
atism and increased mitotic activity in the proliferative zone at
the base of the crypts but the nuclei retain their basal position
within the cell. As the cells mature and migrate toward the surface,
the crypt lumen acquires a serrated or "saw-tooth" appearance. Due
to a similar proliferative stimulus acting on fibroblasts, a
thickened collagen layer also develops beneath the surface epithel-
ium. Clinically they are asymptomatic and malignant transformation
is virtually nil.
C. NEOPLASTIC POLYPS - incidence increases with increasing age.

1. (adenomatous polyp) - is the most common neoplasm


TUBULAR ADENotIA
of large bowel but may also occur in small bowel or stomach.
They may occur singly or mUltiply but rarely appear before 40
years of age unless associated with one of the polyposis
syndromes. They typically appear as pedunculated polyps with
a tan, spherical, irregularly fissured head measuring up to 1
cm in diameter, occasionally larger. Within the head, gland-
ular spaces are lined by pseudostratified epithelium which has
lost its normal differentiation and shows varying degrees of
cellular atypia and mitotic activity. As the size of the polyp
increases (usually reflecting an increase in the villous
143
component), the cellular atypia increases and the elongated
tortuous glands become crowded "back-to-back" to form a
cribriform histologic pattern with little or no intervening
stroma. This generally signifies malignant transformation,
however since the bowel mucosa does not contain lymphatics,
intraepithelial or intramucosal malignancy (carcinoma-in-situ)
is of little clinical significance until the malignant cells
invade the muscularis mucosa where they may be borne off to
distant sites by the submucosal lymphatics. Malignant
transformation is most likely to occur in areas showing a
villous architecture. Clinically, these are usually
asymptomatic but they may bleed secondary to trauma or become
infarcted secondary to torsion of the stalk.
2. VILLOUS ADENotIA - is usually large (> 2 cm), solitary, firm,
grey-tan, sessile lesion that can occur throughout the colon
but most frequently occurs in the rectosigmoid area. Micro-
scopically, more than 50% of the lesion reveals long, slender,
villous or papillary projections having a fibrovascular core
and lined by pseudostratified epithelium with varying degrees
of cellular atypia. Approximately 30-50% have undergone
malignant transformation and, of these, about half will be
invasive having penetrated the muscularis mucosa. The larger
the tumor, the greater the chance of invasive carcinoma. These
may be asymptomatic but the larger lesions may produce rectal
bleeding and/or copious mucoid diarrhea with resultant loss of
protein, fluid, and electrolytes (particularly potassium).
3. TUBULOVILLOUS ADEtDiA - microscopically, there are components of
both tubular and villous adenomas with the villous component
ranging between 25-50%. As with tubular adenomas, the
malignant potential is proportional to degree of villous
component, therefore, the incidence of malignant transforma-
tion lies between that of the tubular adenoma (which may
contain up to 25% villous component) and the villous adenoma
(which contains more than 50% villous component).
D• POLYPOSIS SYJI)ROtIES

1. FAtlILIAL POLYPOSIS COLI is an autosomal dominant disease


characterized by appearance during teenage years of thousands
of tubular adenomas of the colon (rarely involving small bowel
and stomach). Unless resected, carcinoma develops after
approximately 10-20 years and invariably by the age of 40. A
colectomy with ileorectal anastomosis is required to eliminate
the risk of malignant transformation.
2. GARDNER'S SYHD1DtE - autosomal dominant disease characterized by
adenomatous polyposis involving colon, small bowel, and
stomach associated with benign lesions of skin (epidermoid
cysts), soft tissue (fibroma, lipoma), and/or bone (osteomas -
particularly membranous bones of the head). There is
inevitable development of colon carcinoma.
3. TURCOT'S SYNDIDIE - autosomal recess i ve disease characterized by
colonic adenomatous polyposis associated with malignancies of
the central nervous system (medulloblastoma, glioblastoma,
ependymoma) and a high risk of developing colon cancer.
VI. NEOPLASIA

A. BENIGN LESIONS (leiomyoma, fibroma, lipoma, etc.) can occur anywhere


throughout the G.1. tract. Occasionally, they may become large
enough to induce symptoms, but in general, are small and
asymptomatic. The possibility of malignancy must be ruled out,
however, whenever a mass lesion is identified.
B. SHALL BOHEL CARCINotIA - is uncommon and almost all are adenocarcinomas
which usually arise in the ileum and produce a constricting mass but
rarely cause symptoms until late in the course of the disease. They
spread to involve regional lymph nodes, liver, lungs, and other
viscera. Carcinomas that involve the ampulla of Vater, however, may
present early with signs and symptoms of obstructive jaundice.
144
C. - approximately 5% of the U. s. population will
COLOREcrAL CARCINtJIA
develop colorectal carcinoma. The vast majority are adenocarcinomas
and are found (in decreasing order of frequency) in the sigmoid
colon, rectum, right colon, transverse colon, and descending colon
although there is some suggestion of a recent rightward shift. There
is an increasing incidence with age - the majority appearing after
the age of 50.
1. PATHOGENESIS - is not known but both genetic and dietary factors
have been implicated. A family history of colorectal carcinoma
increases the risk. Ingestion of animal fats increases bile
acid secretion and concentration within the bowel. This
becomes converted to secondary bile acids and fatty acids by
the action of normal anaerobic bacterial flora. These may be
directly toxic to the mucosa or potentiate the action of
various carcinogens. Additionally, decreased fiber content
prolongs transit time through bowel and may therefore increase
exposure time of colonic mucosa to carcinogens.
2. LEFT-smED LESIONS - begin as small plaques which tend to remain
superficial and over the course of 1-2 years completely
encircles the lumen. The tumor then begins to invade deeper
into the wall of the colon and as it compromises the blood
supply, ulceration of the central portions occur. It may
spread by direct extension into surrounding fat or metastasize
to regional lymph nodes and other viscera (especially liver).
They are often heralded by rectal bleeding, occult blood,
diarrhea, constipation, and decreased caliber of stools.
3. RIGHT-SmED LESIONS - begin as mucosal indurations but tend to
develop into a fungating mass lesions that protrude into the
lumen. They eventually may spread to mesentery by direct
extension .or metastasize to regional lymph nodes and other
viscera. They often present as weakness, malaise, weight loss,
melena, and anemia. .
4. HOOIFIED OUJ(ES I CLASSIFICATION - the major prognostic indicators of
a poor prognosis are depth of invasion and metastasis to more
than 4 lymph nodes.
5-YEAR
SURV I YAL
A Limited to mucosa or submucosa 90%
Extension into but not through 75%
muscularis propria. No lymph
node involvement.
Extension through wall. No 55%
lymph node involvement.
Limited to wall. Regional 50%
lymph node involvement.
Extension through wall with 30%
regional lymph node involvement.
D Distant metastases 25%

D• APPENDICEAL MUCINOUS TUI'IORS

1. NON-NEOPLASTIC "MUCOCELE" - local obstruction of the lumenal


drainage of the appendix or hyperplasia of the mucosa with
excessive mucus production may lead to a distended appendix
due to retention of mucoid secretions.
2.< APPENDICEAL MUCINOUS CYSTADENDCARCINtJIA - is a malignant neoplasm
cht:l,racterized by the excessive production of mucin with
resultant cystic dilatation of the appendix. Histologically,
the tumor is often well differentiated and mimics the benign

145
mucocele, but clinically it is aggressive and will seed the
peritoneal cavity and fill it with abundant mucoid secretions
(pseudomyxoma peritonei).
3. APPENDICEAL nucINOUS CYSTADEID'IA - is the most frequent cause of a
mucoid filled cystically dilated appendix. Generally
asymptomatic, it may rupture and spill mucoid material into
the peritoneal cavity. It does not, however, seed the cavity
nor produce pseudomyxoma peritonei.

146
HEPATOBILIARY SYSTEM
a.:n.d
EXOCRINE PANCREAS

LIVER
I. REVIEW OF NORMAL - the liver weighs about 1400 to 1600 grams in the
normal adult. It receives blood from two sources, the hepatic artery and
the portal vein and is drained by the hepatic veins into the inferior vena
cava. Histologically, the classic hexagonal hepatic lobule is centered
around the central vein with plates of hepatocytes radiating outward to
the portal tracts. The hepatocytes surrounding the portal tracts are the
limiting plate. The functional hepatic acinus, however, is centered around
the portal tract and delimited by the central veins. Blood flows from the
portal area to the central vein. Therefore, the least well perfused area
of the liver is the centrilobular region (zone 3 of the acinus), the best
perfused area is the periportal region (zone 1). Bile, on the other hand,
flows in the opposite direction i.e. from the centrilobular region to the
portal tracts througlt the small bile canaliculi, the intermediate canals
of Hering, and the interlobular bile ducts.
II. BILIRUBIN METABOUSM - the majority of bilirubin is derived from the
breakdown of red blood cells in the reticuloendothelial system. The heme
pigment is converted to biliverdin, then to bilirubin. Soluble in lipid
but insoluble in blood most, but not all, bilirubin is tightly bound to
albumin for transport to the liver and, in this form, cannot be excreted
in the urine or pass the blood brain barrier. If the amount of bilirubin
exceeds the capacity of albumin to bind it, however, the unconjugated
bilirubin may cross the blood brain barrier where it tends to localize in
the cerebral cortex and basal ganglia, causing kernicterus. Kernicterus
will usually result in bt:ain damage and, if severe enough, can cause
death. Bilirubin dissociates from album:i.n at the liver cell membrane and
enters the hepatocyte where it is made water soluble by conjugation to
glucuronic acid through the action of glucuronyl transferase. This form,
when present in the blood, is only loosely bound to albumin and may be
excreted in the urine. The hepatocyte excretes the conjugated bilirubin
into the bile canaliculi. Lecithin, bile acids, bile salts, cholesterol,
calcium, and other electrolytes are added to the bile by active transport
as it passes through the biliary system to the gallbladder where it is
concentrated. Upon appropriate stimulus, such as the ingestion of a fatty
meal, the ampulla of Vater relaxes and the gallbladder contracts to
discharge bile into the duodenum! The diglucuronide is split in the bowel
and the bilirubin is converted by small bowel bacteria into urobilinogen.
Some of the urobilinogen is reabsorbed into the portal circulation and
passed back to the liver, comprising the enterohepatic circulation, and
some passes into the general circulation to be excreted by the kidneys.
III. JAUNDICE (ICTERUS) •. refers to a yellow-green discoloration of the skin
and the sclera produced by accumulation of bilirubin in the tissues and
body fluids. Depending upon the discriminatory ability of the examiner,
jaundice may become detectable when serum levels reach between 1. 8 and 3.0
mg/dl. The most frequent causes of jaundice in the United States are viral
hepatitis, cirrhosis, extrahepatic bilia.ry obstruction, and drug induced
cholestasis. Mechanisms of bilirubin accumulation include:
A. f?U55S PRmUCTION OF BILIRUBIN

1. HEKOLYTIC ANEl'IIA- results in increased levels of unconjugated


bilirubin. Serum bilirubin levels rarely exceed 5 mgldl
because the liver is capable of handling most of this
overload. If there is hepatic hypoxia or intercurrent hepatic
disease however, more severe jaundice may result. Tightly
bound to albumin, unconjugated bilirubin cannot pass into the
urine or pass the blood brain barrier.
2. MASSIVE HEMORRHAGE OR INFARCI'ION - particularly of the GI tract or
lungs may increase production of bilirubin as the destroyed
red cells are resorbed.

147
B. the precise method for uptake of bilirubin
REDUCED HEPATIC CELL UPTAKE -
into the liver cell is poorly understood, although it is known that
the unconjugated bilirubin separates from albumin prior to being
taken· up. Inside the cell, bilirubin is picked up by acceptor
proteins and ultimately transported to the endoplasmic reticulum to
be conjugated.
1. DRUGS -may cause reactions which reduce uptake of bilirubin.
2. thought to be transmitted as an autosomal
GIlBERT'S SYNDROtIE -
dominant disease, is characterized by a mild, chronic
elevation of unconjugated (indirect) bilirubin, which is
usually detected only as an incidental lab finding or when
some unrelated event transiently elevates bilirubin levels
further to produce jaundice. Al though there is decreased
glucuronyl transferase activity, the major defect appears to
be in hepatic uptake of bilirubin.
C. IKPAIRED CONJUGATION

1. OF THE NEHBORN - is thought to be due to


PHYSIOLOGIC JAUNDICE
immaturity of the hepatic conjugating system. This occurs
around three to five days of life in full-term infants and may
be more pronounced and more prolonged in premature babies.
Jaundice in this case results from unconjugated bilirubin.
2. CRIGLER-NAJJAR SYNDROtIE

a. Type I is the more severe form with levels of


unconjugated bilirubin ranging from 15-48 mgjdl. Due to
a defect in glucuronyl transferase activity, conjugation
of bilirubin does not occur and kernicterus invariably
develops with affected individuals ultimately dying of
neurologic problems. It is probably transmitted as an
autosomal recessive disorder.
b. Ine II - is transmitted as an autosomal dominant
disorder, gives bilirubin values of 6-25 mgjdl with
normal conjugated (direct) bilirubin concentrations, and
is usually not fatal.
D. II1PAIRED EXCRETION OF CONJUGATED BILIRUBIN - excretion of conjugated
bilirubin may be blocked at any level within the hepatobiliary tract
from the liver cell to the ampulla of Vater and cause cholestasis.
Cholestasis, whether as the result of intrahepatic or extrahepatic
obstruction, tends to have similar clinical signs and symptoms and,
to a large extent, similar morphologic alterations in the liver as
well. Because extrahepatic jaundice can be treated by surgical means
and intrahepatic jaundice generally responds to medical treatment,
it is clinically of importance to know whether jaundice results from
intrahepatic or extrahepatic causes.
1. INTRAHEPATICCAUSES - in general, all forms of intrahepatic
jaundice are associated with increased levels of the trans-
aminases and LDH, and also may result in decreased prothrombin
levels with consequent prolonged prothrombin times. Bile plugs
are present in distended canaliculi and bile pigment can be
seen in hepatocyte cytoplasm. Alterations in the cytoplasmic
organelles and the accumulation of bile acids produce a
feathery degeneration of the hepatocytes. Ultimately there
will be proliferation of interlobular bile ducts.
a. Hereditary disorders
(1) Dubin-Johnson syndrome - an autosomal recessive
disorder of bile secretion resulting in a
generally mild conjugated bilirubinemia, chronic
or recurrent jaundice, and accumulation of a
peculiar dark pigment in hepatocytes.

148
(2) Rotor syndrome - is similar to the Dubin-Johnson
syndrome but is less common and lacks the hepatic
pigmentation.
b. Estrogens, contraceptive agents, anabolic steroids -
may hamper the transfer of bilirubin and other organic
anions across the liver cell membrane to produce
intrahepatic blockage of bilirubin excretion.
c. Disruption of liver architecture (cirrhosis, hepatitis,
etc) - causes intrahepatic cholestasis.
2. EXTRAHEPATIC CAUSES - result from narrowing or obstruction of the
biliary tract or the ampulla of Vater. Gallstones lodged in
the common duct or carcinoma affecting any duct structures or
the head of the pancreas are the most common causes of
posthepatic obstructive jaundice.
3. HORPHO~Y - in both intrahepatic obstruction and extrahepatic
obstruction, the liver may assume a green discoloration.
Microscopically, bile plugs may be present in distended canal-
iculi and cytoplasmic bile pigment can be seen in the Kupffer
cells and hepatocytes. Alterations in the cytoplasmic organ-
elles and the accumulation of bile acids produce a feathery
degeneration of the hepatocytes that are surrounded by mono-
nuclear cells. With extrahepatic obstruction or obstruction of
the large intrahepatic bile ducts, bile backs up into the
intrahepatic ducts and induces proliferation of the portal
tract bile ducts which also become twisted and tortuous.
Ultimately canaliculi may rupture to produce "bile lakes"
containing dead or dying bile stained hepatocytes.
4. CLINICAL FEATURES - in obstructive jaundice, there is an increase
of conjugated bilirubin in the serum, and the stools become
clay-colored (acholic stools) due to the inaccessibility of
bile to the GI tract. Lack of bile in the gut also causes
decreased absorption of fats and fat soluble vitamins causing
a temporary malabsorption syndrome to develop. Bilirubinuria
develops but urine urobilinogen levels decline. The accumu-
lation of bile acids in the blood causes pruritus. With
chronic cholestasis, plasma cholesterol levels increase due to
both increased hepatic synthesis and decreased excretion.
5. LABORATORY FItmlNGS - serum alkaline phosphatase levels, which
tend to represent damage to biliary epi'thelium, are markedly
increased. Obstruction of bile outflow also results in
hepatocellular damage with elevation of the parenchymal
enzymes (LDH and transaminases) as well, although not to the
extent seen with primary liver cell damage.
IV. DRUG INDUCED DISEASE - most of the drug induced diseases are associated
with hepatic necrosis and/or partial or complete intrahepatic obstruction
of bile flow.
A. CLASSIFICATION BY DRUG

1. DlRECI' HEPATaroXINS are those causing injury in most


individuals to whom the agents are given.
2. ItmIRECI' HEPATOTOXINS are those causing injury in only
occasional individuals.
3. HEPATIC ALLERGENS - are agents which appear to have little direct
toxicity but cause injury by hypersensitivity reaction.
B• CLASSIFICATION BY EFFECI'

1. drugs that produce cholestasis (anabolic steroids,


CHOLESTASIS -
chlorpromazine, etc.) interfere with excretion of bilirubin
and other organic anions. Microscopically, the only findings
are bile pigment within swollen liver cells and Kupffer cells,
and small bile plugs within canaliculi. Upon removal of the
drug, the changes promptly disappear.

149
2. NECROSIS - certain drugs (halothane, isoniazid, PAS, etc.)
produce either zonal or massive necrosis of the liver. This
may result from a direct cytotoxic effect of the particular
drug or the development of a hypersensitivity reaction.
Histologically, changes from focal necrosis to confluent areas
of necrosis or even massive destruction of the entire liver
may be found.
3. CHOLESTASIS AND NECROSIS - are produced by agents such as the
sulfa drugs, tetracyclines, thiouracil, etc. Most patients
show no particular untoward effects when taking these drugs,
but some individuals become jaundiced and develop focal
necrosis within the liver. Histologic findings include
cholestasis and sometimes focal swelling or coagulation
necrosis of the cells with only a scant mononuclear infiltrate
in areas of necrosis. Upon eliminating the drug involved, such
signs usually clear quickly.
C. LABORATORY FINDINGS - in individuals with only cholestasis, alkaline
phosphatase may be elevated slightly without elevation of transam-
inases. With necrosis, the transaminase levels increase. In cases of
massive hepatic necrosis, extremely high transaminase levels may be
followed by a precipitous drop to undetectable amounts.
V. CIRCULATORY DISORDERS
A. CONGESTION - because hepatocytes are highly sensitive to hypoxia,
vascular stasis is an important cause of hepatocellular injury. When
acute, the liver is swollen and engorged with blood. The liver
fraction of LDH generally becomes elevated almost immediately and
the transaminases may become slightly elevated. When chronic, the
sinusoidal dilation fans out irregularly toward the periphery,
sometimes appearing to fuse with dilated sinusoids in adjacent
lobules, producing congestive bridges. The congested central
portions of the lobule stand out against the paler liver tissue and
is termed "nutmeg liver". On microscopic section, sinusoids may be
distended and packed with red cells. The long-standing pressure may
cause atrophy or necrosis of the adjacent liver cells. Severe, rapid
congestion of the liver may cause the sinusoids to rupture and
produce central hemorrhagic necrosis. Subsequent fibrosis of the
necrotic areas may lead to cardiac sclerosis since congestive heart
failure is frequently the cause.
B. HEPATIC VEIN THRCl1BOSIS (BUDD-CHIARI SYNDIPIE) - is usually the result of
thrombosis due to a hypercoagulable state or a primary carcinomas of
the liver that grows into the hepatic veins. Such blockage of out-
flow of blood results in sinusoidal congestion, and if the blockage
is sudden, acute painful enlargement of the liver with intractable
severe ascites develops. If the obstruction develops insidiously,
liver enlargement is gradual and less painful, but ascites develops
nonetheless. As a consequence of the central congestion, central
veins become thickened, sometimes with atrophy of the adjacent
hepatocytes and delicate fibrosis in the central regions of the
liver lobules much like the changes seen in cardiac sclerosis. Veno-
occlusive disease is a similar process that may occur with
chemotherapy but involves the small to medium sized branches of the
hepatic veins.
C. PORTAL VEIN THIOIBOSIS - extrahepatic obstruction is usually related to
intravascular thrombosis, extrinsic compression by tumor, or direct
invasion by tumor. Within the liver, primary or metastatic cancer or
a strategically located abscess may cause portal obstruction. The
liver is not enlarged or tender but portal hypertension will occur
and result in congestive splenomegaly, possible ascites, and
circulatory diversion through systemic venous anastomoses. Zahn's
infarct is an occlusion of a portal vein branch that does not lead
to a true infarct· but rather to a well demarcated purplish discolor-
ation of the parenchyma resulting from congestion of the sinusoids.

150
VI. INFLAMMATORY DISEASES
A. BACTERIAL

1. CHOLANGITIS - is characterized by purulent bacterial infection


in the intrahepatic bile ducts. The process generally begins
in the extrahepatic biliary system as a complication of
obstruction of the common bile duct by gallstones or, less
commonly, by carcinoma of the head of the pancreas or some
portion of the biliary tract. The infection, usually due to
enteric organisms, then spreads into the intrahepatic biliary
tree (ascending cholangitis). Pus can be expelled from the
bile ducts and there is inflammatory infiltrate in surrounding
portal connective tissue. Clinically, cholangitis causes a
high fever and a large tender liver. Signs and symptoms are
similar to those of acute cholecystitis. Severe cholangitis
may lead to formation of abscesses in the liver.
2. LIVER ABSCESS - pyogenic bacteria may seed the liver through the
bile ducts, portal vein, hepatic artery, lymphatics, or by
direct spread of adjacent infections. Less commonly, perfor-
ations of the gastrointestinal tract or other intraperitoneal
infections, sometimes 'fo11owing surgery, may give rise to
subdiaphragmatic or subhepatic abscesses, which then involve
the liver. Large abscesses such as these must be drained for
proper treatment.
B. PAMSIIIC - these disorders include amoebic abscesses, echinococcus
(hydatid cysts), schistosomiasis, liver flukes, and leishmaniasis
(kala azar). The are uncommon in the U.S. but may be seen in the
immigrating patients.
C. VIRAL - may be caused by any number of viral agents under appropriate
conditions. In general usage, however, the term applies to a group
of viruses that are specifically hepatotropic. It may occur as an
inapparent infection without symptoms or signs of disease, as a mild
anicteric disorder of short duration and relatively mild severity,
or as an icteric disorder that varies from mild and brief to pro-
longed and severe.
1. ETIOLOGY

a. Hepatitis A (HAV) - is a RNA virus which causes an acute


self-limited form of hepatitis usually acquired by
ingestion of feca11y contaminated food or water. Often
associated with childhood or rural epidemics, outbreaks
also occur when conditions are crowded or sanitary con-
ditions are poor. The incubation period for type A hepa-
titis is two to six weeks and this is the period of peak
infectivity since the virus is shed in the feces during
this time. By the time clinical symptoms appear, how-
ever, the virus is no longer shed. A short period of
viremia occurs just before the onset of clinical symp-
toms but transfer of the virus via blood is rare. Unlike
the other types of hepatotropic viruses, HAV is not
known to have a carrier state or to progress to chronic
hepatitis. Serum antibodies to HAV are initially of the
IgM type followed by the longer lasting IgG type.
b. Hepatitis B (HBV) - consists of a double shelled 42 nm
particle (Dane particle) containing an inner 28 nm core
particle which envelops DNA polymerase and double
stranded circular DNA. The virus produces a disease
which may be clinically asymptomatic or exceedingly
fulminant. The incubation period for hepatitis B is six
weeks to six months. The Dane particle is probably the
infectious virion and is transmitted primarily by the
parenteral route - usually through blood products or
contamination of needles by infected blood. Hepatitis B

151
virus has also been identified in stool, urine, saliva,
sweat, semen, and menstrual blood and may be transmitted
by aerosols or by sexual contact. Serum markers include:
(1) HBsAg (hepatitis B surface antigen, also called
Australia antigen) - is an early marker of acute
infection. In self-limited cases, it will
disappear after several months. Persistence for
over 6 months implies chronic disease.
(2) Anti-HBs antibody - is not detectable until HBsAg
disappears. The time lag between disappearance of
HBsAg and ap~earance of anti-HBs antibody is
ca11ed the 'window" period. Rising titers
indicate clinical recovery and confers lifetime
immunity. Not detectable in carriers.
(3) Anti-HBc antibody - antibody to the core antigen
(HBc-Ag) is usually detectable approximately 30
days after appearance of surface antigen, often
at onset of symptoms, and is an important marker
during the window period. A sensitive indicator
of acute infection, levels decline after months
to a few years. The titer remains elevated in
chronic cases including carriers. It does not
confer immunitx.
(4) HBeAg - the "e' antigen is an early indicator of
infection appearing shortly after surface antigen
and when the disease is most transmissible. It
peaks early and disappears before surface antigen
disappears. Persistence indicates probable
chronic active state.
(5) Anti-HBe antibody rises as the antigen
disappears. It is also a marker for the window
period, indicating recent acute disease. Titers
fa11 slowly over 1 to 2 years, but remains
elevated in chronic cases. Does not confer
immunity.
c. Hepatitis C (Non-A, Non-B, NANBV) - a single stranded
RNA virus, most cases arise as the result of transfusion
of contaminated blood and tend to have a greater predi-
lection to produce chronic disease than HBV. Other
cases, however, may be spread by the fecal-oral route
and tend to produce a more fulminant acute disease than
HAV. Recently developed screening kits to test for
antibody against the protein C-IOO-3 have allowed test-
ing of blood supplies and hopefu11y the incidence of
hepatitis C will begin to decline.
d. Hepatitis Delta (HDV) - is an RNA virus which is depen-
dent on the HBV outer coat to replicate and therefore
requires coexistent HBV infection, either in an active
or carrier form. The virus is probably transmitted much
in the same manner as HBV.
2. ACUTE HEPATITIS - the clinical and morphologic changes are
similar with all of the hepatotropic viruses. Although the
incubation periods differ, biochemical evidence of hepatic
damage (elevated ALT, AST, LDH) and peak infective periods
precede -the development of clinical symptoms. If symptoms
develop, there is a prodromal period in which the patients
develop a flu-like syndrome with malaise, weakness, loss of
appetite and nausea. Clinical symptoms may abate (esp.
hepatitis A) without the development of increased bilirubin
levels (anicteric hepatitis) or they may worsen with fever,
chi11s, headache, myalgias, vomiting, and diarrhea fo11owed by
the appearance of jaundice. As the bilirubin (primarily
conjugated) levels rise, however, the prodromal symptoms begin
to subside although the urine may darken (bilirubinuria) due
to the conjugated bilirubin. If the bile canaliculi become

152
obstructed by hepatic edema or inflammation, the stools will
become lighter in color and an intense pruritus may develop
due to retention of bile salts. On physical examination the
liver is slightly enlarged and usually tender, frequently with
a blunted edge. Most cases subside with only 5-10% of patients
(those with hepatitis C or hepatitis B and none of those with
hepatitis A) entering into a "carrier" state. An even lesser
number progress to fulminant hepatic necrosis and death. The
morphologic changes in acute hepatitis are generally indis-
tinguishable regardless of the causative agent. Characteristic
features include:
a. Liver cell injury injured cells swell and the
cytoplasm appears watery and vacuolated (ballooning
degeneration). Such changes are most evident in the
centrilobular regions.
b. Hepatic necrosis individual necrotic cells may
disappear leaving no trace while others undergo a
peculiar coagulation and appear as round eosinophilic
bodies (Councilman bodies). Occasionally, hepatocellular
necrosis is more extensive reaching from one lobule to
another and forming so-called bridging necrosis. In
extremely severe cases, massive necrosis of the liver
may occur.
c. Kupffer cell hypertrophy/hyperplasia
d. Portal infiltration - of macrophages and lymphocytes.
Within areas of focal liver cell necrosis, nests of
lymphocytes and macrophages may also be present.
e. Regeneration of liver cells - during recovery, evidence
of regeneration of hepatocytes (binucleate cells,
increased mitoses) is present.
f. Intrahepatic cholestasis - is variably present with
small droplets of bile pigment seen in hepatocytes and
Kupffer cells and inspissated bile within the
canaliculi .
g. Lobular disarray - all of the preceding changes alter
the normal architecture of the hepatic lobule which may
later interfere with normal hepatic function.
3. CHRONICHEPATITIS - may be defined as any hepatitis which
persists for more than six months. Although often the result
of hepatotropic viruses (excepting HAV), there are a variety
of other causes.
a. Chronic persistent hepatitis - develops in less than 10%
of cases of acute viral hepatitis. Vague recurrent symp-
toms of malaise, weakness, loss of appetite, right upper
quadrant discomfort and occasionally mild jaundice are
present in about 50% of the cases. -The others remain
asymptomatic. Transaminase levels may be slightly in-
creased to about 2-3 times normal, and bilirubin levels
usually are less than 5 mgjdl. On liver biopsy, there is
nonspecific portal mononuclear cell inflammation with
preservation of the limiting plates. Intralobular nec-
rosis and inflammation are mild or absent. Most patients
recover but a few convert to chronic active hepatitis.
b. Chronic active hepatitis - is characterized by progres-
sive liver destruction. About 90% of these cases are
either viral or cryptogenic (now felt to mostly repre-
sent previous hepatitis C infection) in origin, but the
incidence of drug induced chronic active hepatitis
appears to be increasing. Strongly associated withHLA-
B8 and DRw3, non-specific autoantibodies are present in
such patients, who may respond to immunosuppressive
drugs. Some patients develop LE cells, thus the designa-
tion "lupoid hepatitis". Transaminase levels are elev-
ated, gamma globulins are increased, and PT is
increased. Clinically, these patients -do not appear

153
particularly ill when they first present. They have
poorly defined complaints of weakness and fatigue with
loss of appetite as seen in acute hepatitis. Jaundice
usually is not present until late in the course when the
disease is fairly severe. Itching is common. Liver
histology reveals mononuclear inflammation spilling out
of the portal areas with piecemeal necrosis (destruction
of the limiting plates) and bridging necrosis. Kupffer
cells are prominent and there may be periportal fibrosis
and abnormal bile duct proliferation.
D. TOXIC- is most frequently due to ethanol. In the early stages of
alcoholic toxicity, the liver is enlarged, smooth, yellow, soft and
greasy. There is considerable fatty change (steatosis) in the liver
on microscopic examination. With abstention, steatosis will subside,
but with progression, hepatic cells may be swollen with some bile
stasis apparent in the bile canaliculi and in hepatocytes. Pink
hyaline material, known as alcoholic hyaline (Nallory bodies), may
be present within the cytoplasm of scattered liver cells some of
which are necrotic and surrounded by neutrophils. Exaggeration of
this process leads to the clinical condition of alcoholic hepatitis.
Again, with abstention, these changes will usually regress but may
lead to eventual hepatic cirrhosis.
VII. CERRIIOSIS - is characterized by necrosis, regeneration of liver cells,
fibrosis, and disruption of normal liver architecture.
A. is the most common type in the U. S. and
LAENNEC I S (ALCOHOLIC) CIRRHOSIS -
about 90% of the individuals give a history of chronic alcoholism,
although only 10-15% of chronic alcoholics develop cirrhosis. In
these patients, with continued alcohol damage to hepatocytes, the
liver decreases in size and demonstrates a fine nodularity (micro-
nodules). Hepatocytes often still contain fat and alcoholic hyaline
may still be visible. Bile stasis may also be present. Progressive
fibrosis begins around portal areas with larger nodules probably
representing regeneration. As the disease advances, the liver may
weigh less than 1200 gms with a fibrotic consistency. As the liver
shrinks, fat tends to disappear and the liver becomes progressively
darker brown. Cirrhosis may develop gradually over a period of
several decades. Eventually non-specific symptoms of weight loss,
anorexia, nausea, vomiting and vague gastrointestinal complaints
occur. Sometimes the first evidence of liver disease is simply the
development of jaundice. The patient may develop ascites, spleno-
megaly, and gastrointestinal bleeding from esophageal varices or
gastritis. Death usually occurs as a result of hepatic failure,
infection, gastrointestinal hemorrhage, or rarely, the development
of hepatic carcinoma.
B. POSTNECROTIC CIRRHOSIS - comprises between 10 and 30% of the total cases
of cirrhosis and may follow a single episode of viral hepatitis, or
drug or chemical toxicity which results in a marked degree of acute
hepatocellular necrosis. More often, however, it develops insidious-
ly, possibly following chronic hepatitis. The liver may be misshapen
and small. Large, irregular and variable sized regenerative nodules
(macronodules) are interspersed among depressed scars of varied
size. Sometimes the nodularity is diffuse and more uniform, up to 3
to 4 cm in diameter. Microscopically, there is irregular coarse
scarring with only scattered bile ducts and blood vessels in the
scars. Occasional areas of persistent normal liver parenchyma may be
present between scars, but most areas consist of regeneration of
liver cells in large disorganized masses with prominent bile stasis.
The necrosis usually has been so severe that portal triads may be
adjacent to each other. When symptoms occur, they are usually the
result of liver failure or portal hypertension. Some patients with
this form of cirrhosis develop hepatocellular carcinoma; it is this
type of cirrhosis with which liver carcinoma is most frequently
associated.

154
C. BILIARY CIRRHOSIS

1. PRIHARY BILIARY CIRRHOSIS - is associated with immunologic distur-


bances, high serum cholesterol levels, and skin xanthomas.
Antimitochondrial antibodies are found in a majority of
patients.
a. Morphology - grossly, the liver is large and green,
initially smooth but later developing a coarse nodular-
ity. Microscopically, in the earlier stages, septal and
larger interlobular bile ducts are damaged and surround-
ed by a dense infiltrate of lymphocytes, histiocytes,
plasma cells and occasionally eosinophils. Granulomas
may form. As the disease progresses, ductular prolifer-
ation is observed and granulomas are less common. Later
the inflammation subsides and relatively acellular
fibrous septa extend from portal tracts into and around
lobules. Periportal cholestasis may become severe.
Finally, regeneration of nodules is seen accompanied by
end-stage liver disease.
b. Clinical - 90% of the patients who develop biliary
cirrhosis are middle-aged females. The disease starts
most frequently as pruritus but jaundice will eventually
develop. Even then, patients usually feel well and have
a good appetite but the course is steadily down hill
with deep jaundice reflecting a late stage when patients
often complain of backache and pain over the ribs. Duo-
denal ulcers and hemorrhage are common. Portal hyper-
tension, with bleeding esophageal varices, is frequent
but usually a late development. Laboratory findings
include marked elevation of the alkaline phosphatase,
and serum IgM may be very high.
2. may result from mechanical obstruc-
SECONDARY BILIARY CIRRHOSIS -
tion of the biliary outflow tract, ampulla of Vater, or extra-
hepatic bile ducts. The liver is intensely green due to the
obstruction of bile outflow. Where bile canaliculi have rup-
tured, la.rge accumulations of bile pigment called "b:ile lakes"
may be found associated with necrosis of surrounding hepato-
cytes. Clinical findings include jaundice and itching. Because
of the failure to excrete bile into the intestinal tract,
lipid absorption is poor and may be associated with
hemorrhagic problems related to inadequate absorption of
vitamin K.
D. (hemochromatos:is) - individuals who are unable to
PIGI1ENTARY CIRRHOSIS
handle iron in proper metabolic pathways and those who have exces-
sive iron intake store iron in the liver and other parenchymal
tissues of the body. In the liver, this induces an appearance
similar to a moderately advanced stage of Laennec cirrhosis.
Microscopically, brownish pigment granules are found within the
cytoplasm of liver cells and in bile duct epithelium. The pigment
also is present within fibrous tissue. Most patients with pigmentary
cirrhosis develop hepatic failure or portal hypertension. Indivi-
duals with pigmentary cirrhosis also have a much higher incidence
than the normal population of primary hepatocarcinoma.
E. WIUIDN'S DISE~ (hepatolenticular degeneration) - is an inborn error
of copper metabolism which is inherited as an autosomal recessive
trait and which usually appears in the second or third decade of
life. There is inadequate synthesis of the ~lasma copper binding
protein ceruloplasmin. In patients with Wilson s disease, inadequate
levels of ceruloplasmin permit copper to be transported loosely com-
plexed with albumin from which it is readily dissociated to be
deposited in the liver, basal ganglia of the brain, the cerebral
cortex, kidney, and the cornea. Wherever copper is deposited, damage
to tissues occurs. Copper deposits in the eye are found in Desce-
met's membrane at the limbus of the cornea, producing the
characteristic Kayser-Fleischer rings which are pathognomonic of

155
Wilson's disease. In the liver, hepatocellular necrosis is
accompanied by formation of hyaline acidophilic inclusions similar
to those present in alcohol abuse. Scarring follows this stage of
necrosis, producing delicate trabeculae or broad massive areas of
collapse suggesting postnecrotic cirrhosis. Laboratory tests show
low serum copper levels, low serum ceruloplasmin levels and
increased excretion of copper in the urine.
VIII. PORTAL HYPERTENSION - is most frequently seen as a complication of
cirrhosis. Obstruction of the hepatic veins results in a backup of blood
causing increased pressure in the portal vein but arterial shunting of
blood into the portal vein may be a major contributor to the increased
portal pressure. Increased portal pressure will lead to:
A. ~nES - is the intraperitoneal accumulation of watery fluid con-
taining 1 to 2 gms/dl of protein (primarily albumin). Glucose,
sodium, and potassium are present in essentially the same concentra-
tion as in the blood. Drainage of such fluid produces considerable
loss of proteins and electrolytes and may disrupt fluid and electro-
lyte metabolism. The rate of exchange of the peritoneal fluid with
plasma is estimated as high as 80% per hour. Removal of the large
quantities of ascitic fluid frequently is followed by prompt
reaccumulation.
B. COLLATERAL VENOUS CHANNELS - are particularly significant in the lower
esophageal plexus producing esophageal varices. The hemorrhoidal
plexus connects the superior mesenteric vein of the portal system
with the inferior mesenteric system so that up to half of the
patients with cirrhosis develop severe hemorrhoids. In patients in
whom the fetal umbilical vein has not become totally obliterated,
the typical caput medusae may develop around the umbilicus.
C. SPLENCl'IEGALY bec()mes prominent with weights up to 1000 gms being
common. Hypersplenism may develop, producing anemia, leukopenia, and
thrombocytopenia. If the latter occurs in combination with abnormal-
ities of clotting factors, the patient may develop a hemorrhagic
diathesis.
IX. NEOPLASIA

A. BENIGN

1. HBUNGImu - cavernous hemangiomas are the most common of the


benign tumors which may be found in the liver. Usually they
are small but they may grow to a size of 3 cm and appear as
nodules imbedded in liver substance. Histologically. there are
large endothelial lined vascular spaces which may become
sclerotic with fibrous replacement (sclerosing hemangioma).
Generally, they are of no clinical importance.
2. HEPATIC ADENOMAS - are generally small and are not sharply
demarcated from surrounding liver tissue. Microscopically,
they are composed of well differentiated liver cells with
sinusoids and plates, but without normal lobular structure .. If
the adenomas do not establish connections with the biliary
system, bile stasis will cause them to turn green. Although
not common, hepatic adenomas are seen most frequently in women
of reproductive age and, in some studies, have been associated
with use of birth control pills.
3. BILE DUCT ADENOMAS - are usually small encapsulated, spherical,
yellow white nodules. Microscopically, multiple small acini
lined by epithelium similar to that present in small bile
ducts is seen. These channels are surrounded by fibrous
stroma. They usually are located in the subcapsular area and
are usually seen in adult males. .
4. HEMANGIOENDOTHELIOMA - is a well encapsulated tumor that usually
is found in infant females. It sometimes is associated with
hemangiomas elsewhere. Although considered benign, the tumor
has malignant potential. Jaundice and enlarged liver are

156
common with abdominal distension and vomiting and diarrhea
frequent complaints. Congestive heart failure may occur
because of arterio-venous shunts that form the tumor.
B• tlALIGNART

1. HEPA~~ - accounts for 75 to 85% of malignant tumors of


the liver and tends to occur most frequently in older men.
About 75% of hepatocellular carcinomas arise in livers
affected by cirrhosis and about 2% of all cases of cirrhosis
develop into hepatocellular carcinoma. Additionally, a number
of chemicals (vinyl chloride, thorotrast, aflatoxin, etc) have
also been implicated. The clinical presentation includes
hepatomegaly, abdominal pain, and abdominal mass. About 15% of
the patients are jaundiced. Detection of elevated alpha feto-
protein levels in patients with long-standing liver disease
may indicate that hepatocellular carcinoma has developed.
Grossly, hepatocellular carcinoma may appear as a solitary
massive tumor, as multiple nodules scattered throughout the
liver with an appearance similar to that of metastatic
disease, or as a diffuse infiltrative lesion difficult to
distinguish from the underlying cirrhosis. Bile pigmentation
is frequent in hepatocellular carcinoma because there is no
functional drainage through bile ducts. These tumors have a
propensity to invade hepatic veins and may cause Budd-Chiari
syndrome. Microscopically, the cells resemble hepatocytes. In
well differentiated tumors, cells tend to form cords sometimes
separated by sinusoids and may elaborate bile. Such tumors may
appear almost like normal liver. Others, however, are composed
of anaplastic giant cells with abundant cytoplasm and multiple
nuclei, frequently in mitosis.
2. CHOLUmI~~ - occurs in adults and is not associated with
cirrhosis. They arise from bile duct epithelium and fo-rm acini
or papillary structures lined by mucous secreting cells. No
bile is found within the tumor cells, although bile may be
detected in the tumor mass when it has been produced by liver
cells which cannot drain adequately into the external biliary
tract creating signs of obstructive jaundice. Patients with
thorotrast exposure, hemochromatosis, or polycystic renal
disease are at increased risk.
3. ~I~ - consists of pleomorphic tumor cells with large
hyperchromatic nuclei, giant cells in frequent mitosis and
irregular vascular channels. The cells may appear spindle
shaped, and cirrhosis is present in 20 to 40% of the cases.
These have been linked to vinyl chloride and thorotrast
exposure.
4. HEPATOB~ - is almost always seen before age two and con-
sists of epithelial and/or mesenchymal components. Fetal and
embryonal type hepatocytes may be seen with primitive spindle
cells, osteoid tissue, cartilaginous tissue and muscle cell
elements.
5. IlETASTATIC - most neoplasms in the liver are of metastatic
origin, many of which arise in the gastro-intestinal tract,
particularly in the colon and spread through the portal veins
into the liver. Generally, scattered nodules of tumor tend to
be relatively uniform in size. Pancreas, lung, breast, and
kidney tumors may also frequently metastasize to the liver.
X. REYE'S SYNDROME - is a disorder of undetermined etiology consisting of
fatty liver with associated encephalopathy. It may affect children up to
age 15 years, but is most common between ages 1 and 3. In the typical
case, the child appears to be recovering from an upper respiratory
infection when vomiting develops and, within 24 hours, irritability,
restlessness, lethargy, convulsions, and coma intervene. The course is
rapid and the fatality rate is high. About half the time, mild to moderate
enlargement of the liver is detected but jaundice is often absent or mild.
Serum bilirubin is usually normal, transaminases are increased, prothrom-
bin time is prolonged, and blood ammonia levels normally are increased
157
although the ammonia levels do not cor,relate well with the status of the
patient. Spinal fluid glutamine levels have the best direct correlation
with the course of this disease. Liver biopsy reveals diffuse small lipid
deposits in all hepatocytes (microvesicular steatosis) without significant
necrosis or inflammation.
XI. HEPATIC FAILURE - may occur as a result of a single episode of massive
destruction or as an acute decompensation in chronic liver disease. Both
viral hepatitis and cirrhosis are frequent causes of hepatic failure.
Jaundice by itself is not indicative of liver failure but is almost
invariably present when failure occurs. Patients with liver failure
develop personality alterations, confusion, and mental obtundation ranging
from mild lethargy to coma (hepatic encephalopathy). A characteristic
flapping tremor of outstretched hands referred to as "liver flap"
(asteriKis) also occurs. Patients are unable to metabolize estrogens
which, in the male results in hypogonadism, loss of libido, and gyneco-
mastia. Palmar erythema and spider angiomas of the skin have also been
attributed to increased estrogens. Patients sometimes develop a pungent
sweet-sour odor known as fetor hepaticus and the urine is particularly
pungent. Associated renal insufficiency or failure is present in the
hepatorenal syndrome although there often is no evidence of morphological
changes in the kidneys. Patients in hepatic failure do not synthesize
albumin, globulins, or prothrombin properly, with a consequent increase in
amino acid levels in the blood as well as the expected problems associated
with deficiencies of these proteins. Anmonia accumulates because of an
inability to form urea.

GALLBLADDER AND BILIARY TRACT


1. CONGENITAL - anomalies of the gallbladder range from complete agenesis
to total reduplication to aberrant locations. Angulation or kinking of the
gallbladder fundus can cause an expanded bulbous tip known as a phrygian
cap. Generally, these anomalies are rare and, for the most part, not
clinically important. Anomalies of the bile ducts, however, may be life
threatening and include localized or diffuse biliary agenesis or atresia.
II. CHOLELITHIASIS - gallstones generally form in the gallbladder from three
primary constituents: cholesterol (the most common constituent), calcium
bilirubinate, and calcium carbonate. The prevalence of stones can be re-
lated to excess amounts of unconjugated bilirubin or cholesterol secreted
into the bile by the liver, or disturbances in bile acid and bile salt
metabolism. Although genetic predisposition probably plays a role,
gallstones are more frequently seen in women, increase in incidence with
age, and are related to obesity and high caloric diets.
A. PADIOGENESIS- cholesterol, insoluble in water, is maintained in
solution in the form of a micelle containing a central core of
cholesterol, surrounded by hydrophilic bile salts and lecithin. When
the amount of cholesterol exceeds the capacity of the bile salts and
lecithin to maintain it in solution and when factors are present
that will promote precipitation of cholesterol crystals, gallstones
begin to grow by accretion.
1. CHOLESTEROL STONES - in a "pure" form, these comprise approx-
imately 10% of all gallstones. They occur singly or multiply,
vary from small to 5 or 6 cm. in size, and are radiolucent.
Smaller stones are usually spherical and translucent with an
obvious crystalline appearance. As they enlarge, they develop
a smooth egg-shell surface, which reveals a radial crystalline
structure on cut section.
2. CALCIUM BILIRUBINATE STONES - are less common than cholesterol
stones and are encountered in diseases in which excessive pro-
duction of bilirubin is a principle feature. These usually are
radiolucent, jet black stones measuring up to I cm in size.
These almost never occur singly.
3. CALCIUM CARBONATE STONES - are rare, but are radio-opaque.

158
4. account for about 80% of all biliary
HlXED AND ctI1BIHED STONES -
calculi in surgically removed gallbladders. Mixed stones are
those having varying proportions of all three of the stone
forming constituents of bile. Combined stones refer to those
in which either the central core or external layers are pure
and the remainder is a mixture of constituents. Mixed stones
are usually multiple whereas combined stones are sometimes
solitary.
B. ~LlCATlmm - although most gallstones are asymptomatic, about 25%
of patients will develop one or more of the following complications.
1. usually due to smaller
msTRUCTIOH OF CYSfIC OR CCI'IHOH BILE DUCT -
stones which become impacted within a duct to cause partial or
total obstruction. Nausea, vomiting, and severe pain (b1l1ary
co11c) results from spastic episodic contraction of the gall-
bladder musculature in an effort to expel the obstructive
stone. If the stone remains in the cystic duct, mucoid disten-
sion (hydrops) of the gallbladder may result but the patient
will not be jaundiced. If the stone lodges in the common bile
duct or the ampulla of Vater, however, jaundice usually
results.
2. CHOLECYsrITIS [see below]
3. GALLSTONE ILEUS - rarely, gallstones may erode the wall of the
gallbladder and adherent intestinal loops to create chole-
cystointestinal fistulas. Such stones may pass unnoticed
through the stools, but very large stones may obstruct the GI
tract, particularly at the ileocecal region.
4. C~R [see below]

III. CHOLECYSTITIS

A. usually starts as acute right upper quadrant pain


Ag1I'E CHOLECYsrITIS -
referred to the right shoulder. Fever, nausea, vomiting, white count
elevation, and rigid abdominal wall are usually present with severe
cramping pain. A mild degree of jaundice is present about 25% of the
time.
1. PATHOGENESIS - obstruction of biliary outflow from the gall-
bladder, whether partial or complete, creates conditions for
progressive concentration of bile and chamicalirritation of
the gallbladder wall. Secondary bacterial infection is often
present. The inflammatory response to the bacteria and chem-
ical injury may further compromise the blood supply creating
additional insult to the mucosa (gangrenous cholecyst1tis). In
a large majority of cases, stones can be found impacted at the
neck of the gallbladder or within the cystic duct. Occasion-
ally, however, no physical obstruction is identified.
2. MORPHOLOGY - the gallbladder is usually enlarged and tense with
a variegated color ranging from red to green-black. The serosa
may be covered by a fibrinous exudate, and the lumen is filled
with blood, pus, bile, and often additional stones. In areas
of mucosal necrosis, ulcers may penetrate the wall, creating
abscesses or causing generalized chemical peritonitis. Other
complications include ascending cholangitis and hepatic
abscesses. Histologically, edema, leukocytic infiltration,
vascular congestion, and sometimes abscess or gangrenous
necrosis is seen. A prominence of eosinophils indicates a
subacute cholecyst1tis, and calcification of the wall may
ultimately lead to a porcela1n gallbladder.
B. is often characterized by intolerance to fatty
CHRONIC CUOLECYSfITIS -
foods, belching, and postprandial epigastric distress, sometimes
including nausea and vomiting.

159
1. PATHOSaESIS - the exact evolution of chronic cholecystitis is
obscure, as it is rarely preceded by a well-defined episode of
acute cholecystitis. Almost all, however, are associated with
cholelithiasis and are therefore most commonly seen in obese
females.
2. MORPHOLOGY - the most frequent finding is fibrous thickening and
chronic inflammation of the wall of the gallbladder althoufh
both may be minimal. If obstruction is present, the gal -
bladder may be distended, but the mucosa is generally intact.
Only about 30% of cases have concomitant bacterial super-
infection. Outpouching of the mucosa through the wall
(Rokitansky-Aschoff sinuses) are found in about 90% of
chronically inflamed gallbladders.
IV. CHOLESTEROLOSIS - is also commonly referred to as strawberry gall-
bladder, because lipid-laden macrophages accumulate in the tips of the
mucosal folds and resemble the pale seeds that punctuate the surface of a
strawberry. It may be found in up to 10% of the general autopsy popula-
tion, but rarely causes clinical symptoms. It is not thought to predispose
to cholecystitis and is unrelated to blood levels of cholesterol.
V. NEOPLASIA
A. BENIGN LESIONS - papillomas and adenomas of the gallbladder are both
derived from the mucosal epithelium and usually are relatively
small. The papilloma grows as a pedunculated branching structure and
the adenoma is a flat sessile thickening. Neither are clinically
significant except to differentiate them from malignant growths.
B. CARCINmIA OF GALLBLADDER - arises from the mucosal epithelium and,
although relatively rare, is the most common malignancy of the
biliary system. Predominantly a disease of adulthood, it is most
frequently seen in the 6th and 7th decades of life and females are
involved four times as often as males. Most are adenocarcinomas
although a small number are squamous cell carcinomas. In 65% to 95%
of the cases of carcinoma of the gallbladder, cholelithiasis and
chronic cholecystitis are present. Although some carcinomas are
fungating lesions that expand into and fill the lumen while
simultaneously burrowing into the wall, the more common form is an
infiltrating lesion that simply produces an indurated mucosal plaque
while infiltrating into the wall and surrounding tissue and elicit-
ing an exuberant desmoplastic response. Ulcerations and fistulas may
develop and the tumor may grow into adjacent viscera, including
liver, loops of bowel,or stomach. Metastatic sites also include
perihilar lymph nodes and lungs. Although most of these neoplasms
develop within the fundus and neck of the gallbladder, they are
generally asymptomatic until they spread to the adjacent bile ducts.
Symptoms take the form of loss of appetite, nausea and vomiting, and
intolerance to fatty foods with frequent belching. Jaundice occurs
when the tumor has infiltrated major bile ducts or extended into the
liver bed to obstruct an intrahepatic bile duct, or when metastasis
to porta hepatis nodes puts extrinsic pressure on the bile duct.
About half the time a palpable mass is present in the right upper
quadrant and with about the same frequency, right upper quadrant
pain or colic is present. Weakness and weight loss progress as the
tumor enlarges. The prognosis is generally poor due to late
diagnosis.
c. CARCINOO OF THE BILE Ducr5 AND AMPULLA OF VATER - although less frequent,
like carcinomas of the gallbladder these malignancies occur during
mid to late adulthood, but show a slightly greater incidence in men.
Again, most are adenocarcinoma and grow either as nodular, fungat-
ing, or infiltrative lesions. The most frequent location is the
ampulla of Vater and the lower end of the common duct. Stones are
identified in only about one third of these patients, but chronic
inflammation or irritation may playa role in pathogenesis. Because
of their location, these tumors tend to cause obstructive lesions
earlier in the course of the disease. Jaundice is therefore a
prominent feature of these tumors, but may remit if a part of the
tumor becomes necrotic, breaks off, and relieves the obstruction. In

160
spite of the early manifestations, a majority of these tumors have
spread at least locally by the time of diagnosis. The overall
outlook of tumors of the biliary ducts is poor. Tumors of the
ampulla, however, have a somewhat better prognosis than tumors
involving other areas of the biliary ducts.

EXOCRINE PANCREAS
The exocrine function of the pancreas resides in the ductal and acinar tissue
whose function is to manufacture and secrete digestive enzymes that are used to
breakdown food into small components capable of being absorbed by the small
bowel. The digestive enzymes are synthesized in an inactive form, stored in the
zymogen granules of the acinar cells, and, when stimulated by the duodenal
hormone cholecystokinin, released into the acinar lumen where trypsin becomes
activated and, in turn, activates the other enzymes.
I. DEVELOPMENTAL DISORDERS - the pancreas originates as dorsal and ven-
tral diverticular buds off of the caudal foregut (duodenum). As the
ventral pancreas (which ultimately forms much of the head of the pancreas)
rotates to fuse with the dorsal pancreas (which forms most of the body and
tail), the ventral duct fuses with the distal portion of the dorsal duct
to form the main pancreatic ductal system. If the ventral bud fails to
rotate, an annular pancreas develops which may predispose to duodenal
obstruction, especially in children.
II. ACUfE PANCREATITIS - results from diffuse enzymatic destruction of
pancreatic tissue and, in its severe form (acute hemorrhagic pancreatitis,
acute pancreatic necrosis), constitutes a medical emergency. In a large
number of patients, a specific etiology is never uncovered; however, it is
particularly common in alcoholics, and gallstones are present in 40-70% of
the patients. The conditions leading to liberation of activated digestive
enzymes within the pancreatic tissue probably revolve around abnormal or
inappropriate intracellular activation of the enzymes, injury to the
acinar cell, and/or pancreatic duct obstruction. Whatever the mechanism,
activated proteases, lipases, and amylases are liberated into the
pancreatic tissue resulting in enzymatic fat necrosis, hemorrhage, and
autodigestion of the pancreas.
A. MORPHOLOGY -grossly the pancreas appears swollen and hemorrhagic with
chalky white precipitates (enzymatic fat necrosis) throughout the
abdomen. Histologically, there is proteolytic destruction of pancre-
atic tissue, necrosis of blood vessels with associated hemorrhage,
enzymatic fat necrosis, and acute inflammation.
B. CLINICAL - most cases present in middle age with the sudden onset of
severe unrelenting pain which may radiate sharply into the back.
Electrolyte disturbances, vascular collapse, and shock may quickly
intervene. Serum amylase levels rise within the first 24 hours and
serum lipase follows suit 2-3 days later. Both enzymes can be found
in the urine. Deposition of calcium in the necrotic tissue contri-
butes to a hypocalcemia which, if persistent, is a poor prognostic
sign. 20%-50% of these patients will die in the first week of their
disease, and if they survive, may develop pancreatic abscesses,
pseudocysts, or duodenal obstruction.
I II. CHRONIC (RELAPSING) PANCREATITIS - probab ly results from recurrent
episodes of mild or subclinical acute pancreatitis that eventually leads
to fibrosis of a large portion of the exocrine parenchyma. Although the
etiology is multifactorial, it is most frequently seen in middle-aged
alcoholic males. With alcoholics, the pancreas is firm in consistency with
both calculi and multiple foci of smaller calcifications. Acini are
atrophic, fibrous tissue is markedly increased, and chronic inflammation
surrounds dilated ducts containing inspissated protein secretions. For the
most part, however, the islets survive intact until late in the course of
the disease. Repeated attacks of mild to moderate abdominal pain is
typical but some patients may be totally asymptomatic until sufficient
pancreatic tissue is destroyed to produce symptoms of pancreatic
insufficiency. During the initial attacks, there may be mild fever and

161
jaundice with slight elevations of serum amylase and alkaline phosphatase,
but as the disease progresses weight loss, hypoalbuminemia, steatorrhea,
and diabetes ensue.
IV. PSEUDOCYST - as a sequela of acute/chronic pancreatitis or abdominal
trauma, pseudocysts present as a solitary, unilocular collection of fluid
and necrotic debris that produces abdominal pain and predisposes to
intraperitoneal hemorrhage or peritonitis. The absence of an epithelial
lining distinguishes these from the less common true pancreatic cysts.
v. NEOPLASMS

A. CYSTADENOMA - is an uncommon neoplasm arJ.s1ng in older patients from


ductal epithelium and is usually solitary and multilocular. Its
malignant counterpart, cystadenocarcinoma, has a similar gross
appearance but is histologically invasive.
B. ADENOCARCINOMA - typically a tumor of mid to late life, they arise
primarily from exocrine ductal epithelium and are increasing in
frequency. Both environmental and genetic factors have been
implicated in the development of these tumors. Although some are
diffusely spread throughout the pancreas, the majority arise in the
head of the pancreas. Tumors of the body and tail of the pancreas
tend to be hard, irregular masses that have infiltrated widely
throughout the pancreas and contiguous structures and have distant
metastases. Because of a long interval of silent growth, anorexia,
weight loss, abdominal and/or back pain, or symptoms of metastatic
disease are often the presenting symptoms. This late onset of
symptoms delays diagnosis and portends a poor prognosis. Tumors of
the head of the pancreas are more often small with infiltrative
margins that invade the duodenum or biliary structures. Patients
often present with painless jaundice due to impingement upon biliary
tract structures. Early symptoms aid in early diagnosis and
prognosis is correspondingly improved, but the overall 5-year
survival for pancreatic carcinoma is less than 5%.

162
SK:::tN

ACUTE INFLAMMATORY DISEASES


I. URTICARIA - may be acute or chronic. It is an erupt ion of hives (wheals),
which are erythematous, edematous, sharply circumscribed pruritic plaques.
Acute urticaria manifests wheals that last less than 24 hours but
typically recur episodically. They are often allergic in etiology (Type I
IgE mediated hypersensitivity) and result from the action of locally
released histamine and other vasoactive substances on the post-capillary
venules. Others may be due to trauma, drugs, etc. Chronic urticaria
manifests wheals that last longer than 24 hours, and tend to appear
continually, often for months. They are usually not allergic in etiology
and their pathogenesis is unknown. Urticaria seldom requires a biopsy for
diagnosis and the histologic findings (edema withperivenular accumulation
of lymphocytes and neutrophils followed by interstitial infiltration of
eosinophils) are not diagnostic.
II. ECZE~ - is an inflammatory reaction pattern of skin to a wide variety
of inciting agents including internal allergens, external allergens
(contact dermatitis), irritants, trauma of many kinds and certain
microorganisms. Some patients with hay fever and asthma also display
chronic eczema associated with their genetically-determined atopic state
(atopic eczema). Epidermal changes (spongiosis and spongiotic vesicles)
are superimposed on a background of dermal inflammation and edema.
Clinically, the initial lesion is erythema and edema with gross or
microscopic epidermal vesiculation (depending on anatomic site and
etiology). The lesions are pruritic and, unless interrupted by treatment,
the picture evolves to one of crust and scale (from dried vesicles), then
lichenification and hyperpigmentation (reflecting reactive epidermal
acanthosis and hyperkeratosis).
III. ERYfHE~ MULTIFORME - is a common cutaneous vascular react ion- pattern
to any of several possible sensitizing substances, most often medications
and infections (especially Herpes Simplex virus). I t is thought to be
immune-complex mediated, with a reaction cascade starting in a superficial
dermal vessel, and then expanding peripherally. The cutaneous reaction
goes through sequential phases of erythema, edema, hemorrhage, and
epidermal necrosis, resulting in a concentric pattern ("target" or ' iris"
lesion). Microscopically, there is a heavy lymphohistocytic perivascular
infiltrate, and endothelial swelling. Peripherally, the lymphocytes are
seen invading the epidermis causing vesiculation and keratinocyte
necrosis. Centrally, the process is more advanced with exocytosis of
lymphocytes into the epidermis around areas of keratinocyte necrosis
(zonal necrosis). With elimination of the cause, this process resolves in
3-4 weeks and mild cases require no treatment. However, severe cases, with
extensive mucosal and cutaneous lesions, may cause profound illness
requiring systemic corticosteroid therapy.
IV. ERYfHEMA NODOSUM - is a deep dermal and subcutaneous vascular inflam-
matory reaction pattern, manifesting clinically as tender red nodules,
usually on the legs, and more frequently seen in women. It may be
idiopathic, or induced by certain drugs (oral contraceptives, etc) or
infections (TB, coccidioidomycosis, leprosy, etc), and it may be recurrent
and episodic. The histology reveals septal panniculitis and perivascular
inflammation, without vascular necrosis.
V. ACNE - acne vulgaris affects essentially all adolescents and may, in
severe cases, lead to disfiguring scars. The onset is generally earlier in
females but the severity greater in males. Acne begins with hyperplasia of
sebaceous follicles at puberty due to rising levels of testosterone. Sebum
production increases, and the normal sebaceous microbial flora (especially
Propionibacterium acnes) proliferate releasing fatty acids from the sebum.
The second major event is hyperkeratinization and occlusion of the neck of
the sebaceous follicles that are not associated with a hair. This produces
the primary lesion of acne - the comedo. Dependin,g on the level of
obstruction of the follicle, a comedo may be "o.p,en' (high obstruction
holds the follicle orifice open and produces the blackhead) or "closed"
.(lower obstruction allows the orifice to close and results in the

163
"whitehead"). The closed comedo is more significant in that it may
progress to form a pustule, red papule, nodule, or cyst. These last
lesions commonly coalesce ("acne conglobata") and leave scars. The
severity of acne is in part familial and in part dependent on how rapidly
androgen level rise. Other acneiform lesions that clinically resemble acne
in some respects include:
A. ~~A - occurs in an older age group and may resembles acne except
that comedos do not develop. The central face is prominently
involved, and exhibits patulous sebaceous orifices, pustules,
nodules, erythema and telangiectasia. In men, it may eventuate into
rhinophyma (the "w.e. Field nose") due to reactive connective tissue
formation.
B. PERIORAL PAPULAR DERMATITIS - is seen mainly in women, and usually
follows the prolonged use of potent topical corticosteroids or oil-
containing cosmetics (cold creams, etc) on areas of the face. The
lesions are erythematous papules.

INFECTIOUS DISEASES
I. I~PEllGO - is caused by Group A B-hemolytic streptococci or in the case
of neonates and infants, coagulase-positive staphylococci. The initial
lesions are subcorneal bullae which rupture to leave a heavy, yellowish
(honey-colored) crust of dried serum. The disease is contagious with
exposed skin (face and hands) typically being involved. Histologically,
the subcorneal blisters are filled with neutrophils. Occasional
acantholytic cells may be seen in base of bullae and the underlying
epidermis shows spongiosis.
II. ERYSIPELAS - is a superficial cellulitis caused by streptococci. The
lesion appears dusky red with indurated border and microscopically there
is diffuse neutrophilic infiltrate and edema of dermis. Some patients
experience periodic recurrences of erysipelas in same area.
III. DE~TOPHYTOSES - consists of a group of diseases produced by a
variety of dermatophytic fungi (trichophyton sp., microsporum sp., etc)
that are confined to the stratum corneum. These include tinea barbae,
tinea capitis, tinea corporis, tinea cruris, tinea pedis, tinea versi-
color, etc. The diseases differ in respect to location on the body, age of
involvement, etc., but generally produce irregular annular lesions with
irregular interior clearing and peripheral scale. The diagnosis of
dermatophyte infection can be made by microscopic examination of a KOH
preparation of the peripheral scales.
IV. VARICEI,LA-ZOSTER - both varicella and zoster are caused by the same
herpesvirus, varicella-zoster virus. The histopathologic changes in both
are similar to those of herpes simplex lesions.
A. VARICELLA (CHICKEN POX) - caused by exposure of a non immune person to
the virus. Almost all are children. A generalized cutaneous eruption
follows exposure by about 2 weeks. Lesions begin as small papules
which become vesicular and then crust. These tend to develop over
several days so that lesions of variable age are often present
together at the height of involvement. Occasionally some lesions may
become pustular and lead to scarring.
B. ZOSTER (SHINGLES) - occurs in partially immune patients that have been
previously exposed to the virus and is caused by reactivation of
latent virus harbored in cranial and/or spinal sensory ganglia that
then spreads along sensory nerves to the skin. Most cases occur in
adults and is usually self-limiting. The cutaneous eruption
typically consists of exquisitely painful groups of vesicles
distributed within a sensory nerve (or nerves) dermatome that stop
abruptly at the midline.

164
V. mJMAN PAPILLOMA VIRUS (HPV) - produces various kinds of "warts"
(verruca vulgaris, plantar warts, condyloma acuminata, etc) depending on
the strain of the virus. Approximately 56 strains have been identified to
date. These warts have varying degrees of epithelial atypia (koilocytosis,
etc), acanthosis, parakeratosis, and hyperkeratosis with downward growth
of the lateral rete ridges. HPV has also been implicated in the
development of various neoplasias (Bowen's disease, verrucous carcinoma,
laryngeal carcinoma, cervical carcinoma, etc).
VI. MOLLUSCUM CONTAGIOSUM - is caused by a virus of the poxvirus group.
Variable numbers of small, flesh-colored, dome-shaped, umbilicated,
pruritic papules erupt on the skin and spontaneously disappear with time.
Histologically, there is acanthosis and numerous epidermal cells (with the
exception of basal cells) are filled with large, basophilic intracyto-
plasmic inclusion bodies known as molluscum bodies. Little inflammatory
infiltrate is usually present in the dermis except around lesions that are
undergoing spontaneous involution.

PAPULOSQUAMOUS DISEASES
These disorders seem to be reactions to various external "triggers" and are
characterized by well circumscribed "scaly" lesions. They should be differen-
tiated from dermatophytic and eczematous lesions which also may appear "scaly"
(but which are often not well circumscribed).
I. PSORIASIS - is a genetically predetermined cutaneous response to various
stimuli that involves accelerated epithelial turnover and is characterized
by dry well demarcated, occasionally annular, salmon-colored plaques that
are covered by silvery micaceous scales, which when scraped away, reveal
pinpoint areas of bleeding (Auspitz phenomenon). The lesions typically
involve the scalp, elbows, lower back and buttocks, and knees. One form
(sebopsoriasis) tends to involve oily skin areas (scalp, face, and central
chest. Localized psoriasis may occur in areas of trauma (Koebner's phen-
omenon). Involvement of the nails produces a characteristic orange dis-
coloration immediately proximal to the detachment of the nail and the
presence of "thimble-like" pitting of the nails is virtually pathogno-
monic. Histologic features include regular elongation of the rete ridges
(acanthosis) with blunting of the lowermost portion and concomitant
elongation of papillae with papillary edema; thinning of the stratum
malpighii overlying the elongated papillae (which may contain a small
spongiform pustule); thinning or absence of the stratum granulosum; and
marked parakeratosis (which may contain [rfunro microabscesses). On occasion
the pustules become grossly apparent and lead to a pustular psoriasis.
II. PITYRIASIS ROSEA - is a self-limiting, generally asymptomatic disease of
young adults and is probably due to a retroviral infection. Some patients
have a prodrome of upper respiratory infection. The initial lesion is
Renerally a single, almost perfectly oval ring with peripherally attached
cigarette paper" like scales and is often misdiagnosed as "ringworm".
This "herald lesion" is followed by the appearance over the next few days
of similar but smaller lesions over the trunk and extremities. Extensive
eruptions may be mistaken clinically for secondary syphilis, but they
spare the palms. Histologically, there is an "interface dermatitis" with
a mononuclear cell infiltrate in the upper dermis, spongiosis and
occasional spongiotic vesicles, mild acanthosis and focal parakeratosis.
III. LICHEN PLANUS - is a cutaneous reaction pattern to a variety of stimuli.
It presents with multiple small, flat, violaceous, pruritic papules which
tend to coalesce into larger angular plaques which may have fine white
linear markings (Wickham's stria). The lesions tend to show symmetrical
involvement of the wrists, forearms, and elbows and, like psoriasis, may
appear in areas of trauma (Koebner's phenomenon). The oral mucosa and the
glans penis may also be involved. The histologic appearance includes
hyperkeratosis; degeneration and necrosis of basal cells (which may
produce colloid (Civatte) bodies in the papillary dermis; irregular "saw-
tooth" acanthosis; and a dense band-like dermal infiltrate that extends to
the dermal-epidermal junction and is sharply demarcated at the lower
margin.

165
VESICULOBULLOUS DISORDERS
Bullous (blistering) diseases are classified in several ways. The initial
approach is by histologic localization of the level of the blisters (subepi-
dermal, intraepidermal, etc). Additional studies (immunofluorescence, electron
microscopy, etc) are often also needed.
I. EPIDERMOLYSIS BUl.LOSA (ED) - comprises a group of at least 17 bullous
diseases which are also known as "mechanobullous diseases" since the
lesions are elicited by mechanical trauma. Most are inherited, either
autosomal dominant or autosomal recessive, with the autosomal recessive
forms being more severe and often fatal in infancy. The pathogenesis is
related to inherited structural weaknesses in various components of the
epidermal basal cell layer or dermal-epidermal junction area. The
diagnosis is. determined primarily by identifying the level of the
structural defect by electron microscopy.
II. DERMATITIS HERPETIFORMIS (DH) - is an idiopathic disease that appears
in late childhood through early to mid-adulthood. It is associated with a
mild or asymptomatic gluten sensitive enteropathy and the formation of IgA
antibodies that react with anchoring fibrils in the uppermost dermis. TIle
lesions which are located on the extensor surfaces of the extremities,
shoulders and gluteal region tend to be intensely pruritic and arise in a
grouped or clustered ("herpetiform") configuration. The disease is not
related to herpes viruses. Most patients have HLA-B8 histocompatibility
antigens. The histologic characteristics are subepidermal blister form-
ation, infiltration of eosinophils beneath the blister, and papillary
dermal neutrophilic abscesses lateral to the blister site (lateral papil-
litis). Lesions suitable for biopsy, i.e. unexcoriated papules, may be
hard to find. Fortunately, the diagnosis can be confirmed by direct
immunofluorescence demonstrating a granular pattern of IgA autoantibodies
in normal skin which are accentuated at the tips of the dermal papillae.
The disease is incurable but controllable with drugs (dapsone,
sulfapyridine) .
III. LINEAR IgA DISEASE - is a rare bullous disease which is similar to
dermatitis herpetiformis microscopically but which does not res~ond well
to drugs. A childhood form ("chronic bullous disease of childhood', and an
adult form exist. Clinically, the lesions are distributed somewhat
differently than in DH and the patients do not have the associated gluten-
sensitive enteropathy and HLA-B8 antigen. The direct immunofluorescence,
however, is the best way of distinguishing from DH since the IgA
autoantibody pattern is linear rather than granular.
IV. PEMPHIGUS - is a bullous disease which occurs in two basic forms:
pemphigus vulgaris/vegetans and pemphigus foliaceous/erythematosus. Young
to middle age adults are affected. It is difficult to control even with
high doses of steroids, and it is sometimes fatal. Autoantibodies to
certain components of the epidermal intercellular spaces cause
acantholysis at characteristic levels of the epidermis, resulting in
blister formation. The blisters are fragile and rupture easily, therefore
appearing as crusted erosions. The mouth is usually affected first,
followed by skin lesions that vary in severity. The histologic hallmark of
the pemphigus group is acantholysis: high epidermal in p. foliaceous/-
erythematosus, and low epidermal (suprabasilar) in p. vulgaris/vegetans.
IgG autoantibodies and C3 can be seen in the intercellular spaces by
direct immunofluorescence and this causes the loss of cohesion between
cells. By indirect immunofluorescence, the autoantibodies can also be
demonstrated in the serum, and the levels in the serum tend to parallel
the severity of the disease.
V. BULLOUS PEMPHIGOII) (DP) - is the commonest of the bullous diseases.
Although clinically somewhat similar to pemphigus, it occurs mainly in the
elderly. The bullae are generally large, tense, and widely distributed
including the axillae, inguina, and flexor surfaces of the forearms. They
may cause considerable debility in the elderly patient. The mucous mem-
brafies are usually, but not always, spared. The histologic characteristics
are subepidermal bulla formation, eosinophil infiltration in and below the
bulla, and spontaneous re-epithelization of the bulla floor. These

166
findings, however, are non-diagnostic, and direct immunofluorescence is
usually necessary. A linear subepidermal pattern (in the lamina lucida) of
IgG and C3 deposition is characteristic. Steroids may be required to
control the disorder.
VI. HERPES GESTATIONIS - is a rare bullous/inflammatory disease of preg-
nancy. Although the cause is unknown, it is nQt related to herpes virus
infection and histologically resembles bullous pemphigoid. Linear deposits
of C3 and, in most cases, IgG autoantibodies can be demonstrated in the
epidermal basement zone of unaffected skin. In the third trimester of
pregnancy, pruritic lesions appear, typically on the trunk, and tend to
persist until after delivery. The disease often recurs in subsequent
pregnancies, and whether or not there is any effect on the fetus is
controversial.
VII. PORPHYRIA CUTANEA TARDA (PCT) - is the commonest of the porphyrias. I t
is related to hepatic deficiency of uroporphyrinogen decarboxylase, which
may be inherited. The clinical manifestations are usually precipitated by
exposure to hepatotoxic substances, such as ethanol, estrogens, iron, and
certain chlorinated hydrocarbons. There is a broad range of cutaneous
manifestations, most notably photosensitivity, with blisters and
fragility. Chronic changes include hyperp igment at ion , hypertrichosis, and
sclerodermoid lesions of sun-exposed skin. The mechanisms of these changes
are poorly understood. Histologically, there is subepidermal blister
formation with scant inflammatory infiltrate, homogenization of collagen
around capillaries, and prominent dermal papillae. Direct
immunofluorescence is of little additional value. The important diagnostic
test is the demonstration of increased urinary excretion of uroporphyrins
and coproporphyrins.

CONNECTIVE TISSUE (AUTOIMMUNE) DISORDERS


I. LUPUS ERYTHEMATOSUS (IJE) - there are two major forms of lupus
erythematosus: nonsystemic and systemic. In the former only the skin is
involved, while in the latter, there is systemic involvement with
significant patient morbidity and mortality.
A. CHRONIC CUTANEOUS (pISCQm) I.E - large, discrete, reddish "discoid"
patches with surface scaling and follicular plugging appear
typically on the face and scalp. The lesions of so-called
disseminated discoid LE occur predominately on the upper trunk and
arms and facial lesions may be present or absent. Microscopically,
the lesions show epithelial atrophy with hydropic degeneration of
cells in the basal layer. There is hyperkeratosis and follicular
plugging, dermal edema, and patchy lymphocytic infiltrate localized
about cutaneous appendages.
B. ~IC LE (ILE) - although this represents a systemic disease, the
cutaneous lesions may be the first to appear and the severity often
parallels the severity of visceral involvement. The initial cuta-
neous lesions in SLE are often nonspecific but in well developed
lesions, the changes are similar to discoid LE with epithelial
atrophy and degeneration of the basal layer. Laboratory tests are
used to confirm or rule-out a diagnosis of SLE. Besides the findings
associated with involvement of other organs, tests such as the
fluorescent anti-nuclear antibody (ANA) are positive in up to 90% of
untreated SLE patients. More specific is the anti-dsDNA antibody
test; its demonstration carrying a grave prognosis because of the
high association with renal disease. Another test of substantial
value is the so-called lupus band test which may demonstrate, by
direct immunofluorescence, granular deposition of immunoglobulins
(1gA, 19M or 19G) and/or complement (C3) at the dermal-epidermal
junction of involved and uninvolved sun-exposed skin.

167
II. SCLERODERMA - two forms of scleroderma also exist: circumscribed and
systemic.
A. CIRCUtlSCRIBEDSCLERClJERtIA (noRPHEA) - is usually limited to skin or
subcutaneous tissue. In the most common form, there are round to
oval indurated but smooth-surfaced plaques that are ivory in color
and have a violaceous border ("lilac ring"). Early histologic
changes include thickening of the reticular dermis and a moderately
severe dermal lymphocytic infiltrate. Older lesions are more
sclerotic and inflammatory cells are largely absent. The epidermis
appears atrophic and the reticular dermis is thickened and dense.
Eccrine sweat glands are often atrophic in involved areas.
B. SYSTEHIC SCLERClJERtIA (PROGRESSIVE SYSTEHIC SCLEROSIS. pss) - invo 1ves the
fibroblast production of excessive amounts of collagen. The
cutaneous lesions tend to be poo~ly circumscribed and usually begin
peripherally on the hands and feet and extend centrally. Initially
edematous, the skin becomes progressively sclerotic leading to
deformity of the digits. Diffuse hyperpigmentation may also occur.
In addition to the cutaneous manifestations, there is variable, and
sometimes fatal, involvement of internal organs (particularly of the
esophagus, GI tract, heart, lungs, and kidneys).
III. DERMATOMYOSITIS - as the name implies, both skin and skeletal muscle
are involved. In children, dusky lilac suffusion of the upper eyelids
"heliotrope rash" is characteristic but the rash may affect face, trunk
and upper extremities. Although muscular weakness may be present,
abdominal pain and gastrointestinal hemorrhage often dominate the clinical
picture. Soft tissue calcifications may appear late in the disease. Adult
dermatomyositis, .on the other hand, is different than the childhood form
in that it lacks the associated abdominal pain, GI bleeding, and muscular
calcifications seen in childhood disease. In addition there is an
association with malignant neoplasia, often occult and preceded by the
skin lesions from months to years. Besides the heliotrope rash on the
eyelids commonly associated with childhood dermatomyositis, other areas of
cutaneous involvement include the chest, knees, elbows, knuckles and
distal phalanges. These lesions often resemble those of SLE both
clinically and histopathologically.
IV. MIXED CONNECTIVE TISSUE DISEASE (MCTD) - has clinical features of SLE,
PSS, and polymyositis but no anti-dsDNA antibody. There is little renal
involvement, and the disorder responds fairly well to steroid therapy.

EPIDERMAL TUMORS AND TUMOR-LIKE CONDITIONS


Most tumors of the epidermis are related to sun damage and, therefore, tend to
appear on sun-exposed portions of the skin. There appear to be two forms of sun
damage related to the subsequent development of epithelial tumors: those related
to chronic sun exposure which increa.se the risk of squamous cell and basal cell
carcinomas; and those related to severe blistering sunburns in childhood which
increase the incidence of nevi and malignant melanomas.
I. EPITHEUAL CYSTS - are relatively common lesions presenting as slow-
growing subcutaneous nodules. Usually unimportant although ruptured cysts
will induce a florid foreign body granulomatous response and may be
painful.
A. EPmERHOm (EPITHELIAL INCLUSION) CYSTS - are derived from the squamous
epithelium and are lined by normal squamous epithelium which
desquamates keratin into the cystic cavity. They occur principally
on the head, neck, and trunk.
B. PILAR (TRIClilLEMHAL) CYSTS - are derived from the epithelium of the pilar
units and therefore do not demonstrate the normal layers of the
superficial epithelium. Most appear on the scalp and some may
undergo partial calcification. Clinically, the contents of these
cysts tend to have a yellowish hue while that of the epidermoid
cysts tends to be more white.

168
II. PSEUDOCARCINOMATOUS (PSEUDOEPITHEUOMATOUS) HYPERPLASIA - is a
reactive epithelial hyperplasia associated with certain infectious agents
that cause chronic inflammation, chronic inflammation of noninfectious
etiology, and the margins of chronic ulcers. Additionally, granular cell
tumors located in the dermis (or tongue) may induce pseudocarcinomatous
hyperplasia in the overlying epidermis. Downward proliferation of the
hyperplastic epidermis into the dermis along with cytologic atypia
suggests a squamous cell carcinoma both clinically and histologically. The
differentiation from true cancer may at times be difficult.
III. ADNEXAL TUMORS - are derived from apocrine glands, eccrine glands,
sebaceous glands, and pilar units. These are genetically predetermined,
most are benign, and they have similar clinical appearances. Although a
solitary lesion could clinically be mistaken for a basal cell carcinoma,
these are generally multiple tumors occurring principally around the
central face, forehead, and base of nose in younger patients who have a
strong family history of similar lesions.
IV. SEBORRHEIC KERATOSIS - is the most frequent tumor of the skin and in
many instances is a sun-related lesion appearing in areas that were
sunburned (face, trunk, and occasionally extremities) during childhood.
The lesions, however, only appear after the age of 45. Clinically, they
frequently are multiple, have a "stuck on" appearance, and vary in color
from tan to black. There are several variant forms (verrucous, acanthotic,
reticular, stucco, etc), but important clinical features in common are
that they are well demarcated from surrounding skin and are benign. Histo-
logic features are hyperplasia of the basal epithelium (basaloidacantb-
osis) and hyperkeratosis (forming born pseudocysts). They usually don't
come to clinical attention unless they are cosmetically displeasing or
until one becomes irritated or "activated". When this occurs, the lesion
becomes pruritic, loses its color, and grows rapidly. Histologically,
there is marked spongiosis of the epithelium with inflammatory infiltrate
of the dermis.
V. KERATOACANTHOMA - is a rapidly evolving but benign, firm, nodular
lesion with a central keratin-filled crater that may simulate squamous
cell carcinomas. They tend to occur on the sun-exposed areas (digits and
upper lip). Most will regress spontaneously within a few weeks but a few
may reach large size. Histologically, it is an invaginated acanthotic
lesion with central hyperkeratosis and a ground glass appearance of the
epithelial cytoplasm. The microscopic appearance may be identical to a
squamous cell carcinoma, and the diagnosis rests on clinical history.
There is a suggestion of viral etiology since approximately 10% have been
associated with HPV.
VI. ACTINIC (SOI_AR) KERATOSIS - is related to chronic sun damage and usually
presents as multiple keratoses occurring on sun-exposed skin, most notably
the face and hands. Histologic changes include solar degeneration of the
upper dermis, dysplasia of the epidermis (notably in the lower 1/3), and
hyperkeratosis (which may on occasion form cutaneous borns). Occasionally,
there may be epithelial atrophy. These lesions may progress to invasive
squamous cell carcinoma, particularly when they involve the ear, lip, or
dorsum of the hand.
VII. BOWEN'S DISEASE (SQlIAMOUS CEU_ CARCINOMA-IN-SITU) - can occur on both
exposed (sun related damage) and non-exposed (HPV or arsenic related
damage) areas of skin. The lesion initially appears as a solitary erythem-
atous patch with an irregular but well-defined border that often clinic-
ally appears more inflammatory than neoplastic, which may delay the
diagnosis. The patch, however, progressively increases in size. Micro-
scopic exam shows a total loss of maturational sequence in the epithelium
and trans epidermal nuclear atypism with dyskeratosis. Conversion to
invasive squamous cell carcinoma is unusual, but once it occurs, there is
a likelihood of regional or distant metastasis.
VIII. SQUAMOUSCEIJL CARCINOMA - usually arises from areas of actinic (solar)
keratosis, especially on the lip and ear, and is therefore related
primarily to sun exposure. They begin as indurated keratoses that with
time become nodular and tend to ulcerate. Histologically, the tumor
consists of irregular masses, cords, and nests of neoplastic squamous
cells that invade the underlying dermis. Keratin "pearl" formation is seen
in tumors with higher degrees of differentiation. In general, cutaneous

169
squamous cell carcinomas have a much lower metastatic potential than those
arising from mucous membranes (lip, cervix, bronchus, etc) except for
those that arise from burn scars (Marjolin ulcer) and chronic leg ulcers.
IX. BASAL CELL CARCINOMA (BCC) - is the most common malignant tumor of the
skin, is also related to chronic sun radiation damage, and occurs on sun-
exposed areas of the skin. There also appears to be a genetic component to
the development of these lesions. They appear most commonly at the root of
the nose and along the scalp line where they usually present as usually a
solitary slightly translucent or "pearly" nodule with a depressed center
and superficial telangiectases. Histologically, they consist of is lands of
basaloid cells with peripheral palisading invading into, but tending to
show slight separation or "clefting" from the underlying dermis. The
neoplastic cells also tend to form their own stroma which is different
from collagen. There are many clinical variants (pigmented, superficial,
morphea-like, etc) which may be confused with other benign and malignant
dermatologic conditions. Although BCC rarely metastasizes, its infiltra-
tive nature makes definition of the surgical boundaries difficult and
recurrences are common.

DERMAL TUMORS AND INFILTRATES


Structural components of the dermis (fibroblasts, blood vessels, peripheral
nerve, smooth muscle) and cellular migrants through the dermis (mast cells,
histiocytes, lymphocytes, etc) may all give rise to dermal tumors, some of which
are peculiar to the skin and others which are similar to tumors of soft tissue
elsewhere in the body.
I. DERMATOFIBROMA (benign fibrous histiocytoma) - is a common tumor that
can arise at any age but mostly in young to middle aged adults. They occur
most frequently on the lower extremities of females as a firm, slow-
growing nodule which, because of variable coloration, may occasionally
mimic melanoma clinically. Histologically, the tumor is relatively well
demarcated remaining separated from the overlying epidermis (which
frequently shows a basaloid hyperplasia) but occasionally extending into
the underlying subcutaneous tissue. There also tends to be trapping of
collagen at the periphery of the lesion. The lesion may contain fibro-
blasts, capillaries and histiocytesin varying proportions. Histologic
variants include those with a storiform pattern of spindle cells, bands
and bundles of round-oval cells that dissect through the collagen, those
with prominent vascular proliferation, and those with large "monster"
cells.
II. DERMATOFIBROSARCOMA PROTUBERANS - is a locally aggressive, slow
growing form of malignant fibrous histiocytoma which tends to occur on the
trunk and proximal extremities of young to middle aged adults (more
frequently males). It usually begins as an indurated plaque that develops
mUltiple nodules. Histologically, the tumor is composed of crowded spindle
cells arranged in a storiform pattern which has a broad expanding border.
There is often atrophy and ulceration of the overlying epidermis and the
subcutaneous tissue is frequently involved. There is a high rate of
recurrence after resection.
III. NEVUS FLAMMEUS (portwine stain) - not a true neoplasia, this usually
appears after birth as the result of weakening and dilatation of the
dermal blood vessels that gradually increases with age. They may be
associated with other clinical syndromes (Sturge-Weber, etc).
IV. CAPILLARY HEMANGIOMA (strawberry mark) - usually appear shortly after
birth, grow relatively rapidly, and may appear cosmetically disfiguring
but will usually spontaneously resolve by mid-childhood. It consists of
endothelial hyperplasia and proliferation of capillaries with small
lumens.
V. PYOGENIC GRANULOMA - is basically a reactive granulation tissue
process with proliferation capillaries in an edematous stroma with mixed
inflammatory infiltrates and an epidermal "collarette" around the base of
the lesion. The overlying epidermis is often ulcerated.

170
VI. MASTOCYfOSIS - refers to a group of diseases characterized by mast ce11
proliferation. The cutaneous form is urticaria pigmentosa and generally
occurs in children as multiple brown papules or plaques which when rubbed
develop edema and erythema (Darier's sign). A similar reaction will occur
when non-Iesional skin is stroked (dermatographism). This results from the
degranulation of the accumulated mast cells. Systemic symptoms may involve
palpitations, flushing, diarrhea, pruritus, etc.
VII. XANTHOMAS - are cutaneous accumulations of lipid-laden macrophages
often admixed with multinucleated giant cell and a variable degree of
inflammatory cells. They are frequently related to familial hyperlipid-
emias and/or lymphoproliferative disorders. Eruptive xanthomas appear on
the buttocks and posterior thighs and fluctuate with serum triglyceride
levels; tuberous xanthomas occur on the elbows, knees, fingers and
buttocks; tendinous xanthomas appear over the achilles tendon and extensor
tendons of the fingers; plane xanthomas occur in the palmar creases; and
xanthelasma appears on and around the eyelids.
VIn. LYMPHOMA - primary cutaneous lymphoma is known as mycosis fungoides and
is of T-cell origin. The clinical presentation varies but early lesions
may present as eczematoid patches that do not respond to topical
treatment. They may progress to slightly indurated plaques and ultimately
fungating nodules. Histologic exam shows an upper dermal infiltrate of
mononuclear cells that invade the epidermis singly and lie in epidermal
"lacunae" (Pautrier's microabscess). These cells have hyperchromatic,
pleomorphic, convoluted (cerebriform) nuclei and when they invade the
circulation produce Sezary's syndrome.

MELANOCYTIC TUMORS
There appear to be three major factors in the development of pigmented lesions
(sun burn, genetic predisposition, and hormonal status) only one of which is
under behavioral control. Acute sunburn produces individual degenerating and
necrotic keratinocytes ("sunburn cells") at all levels of the epidermis. Chronic
sun exposure causes basophilic degeneration of the upper dermis (solar
degeneration). Atrophic, wrinkled skin is more a reflection of chronic sun damage
than it is of age. Protection of basal cells and young keratinocytes from
ultraviolet radiation by capping them with melanin is the function of the
epidermal melanocytes.
I. PIGMENTED NEVI - at birth, 99% of all individuals are devoid of
pigmented nevi. By the age of 10, approximately 66% of individuals have
pigmented nevi, and they increase in number in relation to increased sun
exposure and severity of prior sunburn. By the age of 80, however, most
pigmented nevi have spontaneously disappeared. Nevi consist of nevus cells
which are similar to melanocytes except that they occur in clusters and
they do not have the dendritic processes of normal melanocytes. The life
cycle of a nevus begins with nevus cell proliferation at the dermal-
epidermal junction (junctional nevus). These usually appear in childhood
and in ~regnant women (melanocytes have estrogen and progesterone
receptors), may be irregular in color, and have an irregular border. These
nevus cells then grow into the underlying dermis to produce a more
homogeneously colored, soft, dome-shaped compound nevus. With time,
junctional activity disappears leaving behind an intradermal nevus which
eventually disappears. All growth occurs from cellular activity at the
dermal-epidermal junction.
A. - can be defined as those nevi present at
CONGENITAL PIGHEHI'ED NEVUS
birth or developing within two weeks of birth or all nevi greater
than 5 cm. in diameter. They may occur on non-sun exposed skin.
Although they vary in size, the risk of malignant melanoma is
increased only in those larger that 20 cm in size. The histology may
vary, but nevus cells tend to be located in the mid to deep dermis
(with a nevus free zone in the upper dermis) and in the subcutaneous
fat suggesting that these lesions may arise from arrested migration
of melanocytes to the epidermis.

171
B. BLUE NEWS - is a well circumscribed, firm nodule of uniform blue-grey
color due to the presence of pigmented nevus cells in the deep
dermis.
C. DYSPLASTIC NEVUS - clinically, these nevi tend to be large (0.5-1.5 cm)
with an irregular border and irregular pigmentation. Histologically,
the nests of nevus cells are not distinct, merge from one rete ridge
to the next, and are irregular in size and shape. There is melano-
cytic atypia and an eosinophilic alteration of the collagen at the
dermal-epidermal junction. Typically, these nevi appear in large
numbers on both exposed and non-exposed skin as part of the familial
dysplastic nevus syndrome, and these patients are at high risk for
developing malignant melanoma.
II. MALIGNANT MELANOMA - all melanomas originate at the dermal-epidermal
junction and are comprised of loose nests of large abnormal melanocytes
which have irregular nuclear outlines often enclosing a large eosinophilic
nucleolus. The tumor may then grow laterally along the dermal-epidermal
junction (radial growth) and/or invade into the deeper dermis (vertical
growth). Worrisome clinical features include asymmetry with irregular
borders, variegated coloration (tan, black, red, blue, brown), large
diameter, elevation from surrounding skin, and a family history. The
prognosis for all melanomas is related to the depth of invasion (the
deeper the invasion, the poorer the prognosis). Those that penetrate less
than 1.5 mm into the dermis have an excellent prognosis. It is therefore
extremely important to recognize these lesions early in their evolution.
A. LENTIGO KALIGNA tlELANotIA - arise from a preexisting lentigo maligna
(Hutchinson's freckle) which is a slowly enlarging unevenly
pigmented macular lesion with irregular borders that usually occurs
on the face of elderly persons. About 30% undergo malignant
transformation. These melanomas spread laterally through the lower
epidermis and upper dermis and only late in its course develops
vertical growth into the deeper dermis.
B. ACRAL LENTIGIHOUS tlELANQHA - arise on the hands and feet around the nails
or on the palms and soles. These also have a long radial growth
phase but ultimately develop downward growth somewhat earlier than
the lentigo maligna melanomas.
C. SUPERFICIAL SPREADING tlELANotIA - is the more common and clinically appears
similar to a dysplastic nevus. It has a long radial growth phase but
also grows upward to give the epidermis a "buckshot" appearance
histologically. After six months to two years, it too will develop
a downward growth.
D. NOOULAR tlELANotIA - has, from its inception, downward growth as well as
radial growth. The histologic appearance of the malignant cells is
variable (round cells, spindle cells, etc) but loss of cohesion is
a common feature. Clinically, this produces a nodular lesion which
may ulcerate. Widespread metastases occur fairly early, and the
prognosis is correspondingly grim.

172
SKELETAL SYSTEM

BONE
I. CONGENITAL/DEVELOPMENTAL DISORDERS
A. includes a number of
OSTEOGEJIESIS lJIPERFEC'lA ("brittle bone disease ll )
-
hereditary disorders which have in common abnormalities in the
synthesis of Type I collagen which result in defects of the bone,
eyes, ears, joints and teeth. Severe forms may be present at birth.
Extremely fragile bones lead to fractures before or at birth and
these children rarely survive until their first birthday. Less
severe forms usually present as recurrent fractures beginning in
early childhood. These patients often have blue sclerae and
abnormally formed teeth. They may develop hearing impairment or
deafness due to fracture of middle ear ossicles, and lax joints
result in dislocations and kyphoscoliosis. Morphologically, these
patients have a reduced bone mass with very thin cortical bone.
B. ADUIIJBOPLASIA (dwarfism) - is an autosomal dominant disease which
often arises by spontaneous mutation and causes impaired
proliferation of cartilage in the ePiihyses resulting in decreased
growth of the long bones. Clinica ly, these patients have a
disproportionately large head and body with shortened arms and legs.
II. PYOGENIC OSTEOMYELmS - oc.curs more frequently in children and is
usually secondary to transient bacteremia. Although a number of organisms
can be involved, the principal organism is Staphylococcus aureus. A
suppurative inflammatory response accompanied by intense edema results in
ischemic necrosis of bone (the sequestrum). If the lesion does not heal,
a chronic osteomyelitis may develop with continued inflammation and sinus
tract formation.
III. METABOLIC BONE DISEASES
A. ~ - refers to an absolute decrease in bone mass below the
level necessary to maintain sufficient mechanical support. Probably
the most common of the metabolic diseases, the incidence increases
with age. It is typically a disorder of post-menopausal females
although males are equally susceptible. The exact etiology is
unknown although malnutrition, lack of exercise, and endocrine
disturbances may a role. The bony trabeculae and the bone cortices
are thinned resulting bone fractures, usually of the femoral neck.
B. ~IA/R~ - are both due to vitamin D deficiency (caused by
dietary inadequacy, lack of exposure to sunshine, intestinal malab-
sorption, chronic renal disease, etc), with rickets occurring in
children and osteomalacia in adults. Characterized primarily by in-
adequate mineralization of bone matrix, there is no initial decrease
in mass although density is decreased; osteoid is correspondingl1t
increased. Rickets is characterized clinically by a II pigeon-breast'
deformity of the thoracic cage, a beading ("rachitiC rosary") along
the costochondral junctions, lumbar lordosiS, and bowed legs. In
osteomalacia, changes similar to those seen in rickets may occur but
common manifestations include frontal bossing, deformities of
weight-bearing bones, and incomplete pathologic fractures.
C. OSTEITIS FDROSA cxsrICA (von Recklinghausen' s disease of bone) - is
associated with advanced primary hyperparathyroidism. The basic
lesion is osteoclastic resorption of bone with fibrous replacement.
Cysts may form within the fibrous tissue. The mandible and maxilla
are initially affected but the entire skeletal system may eventually
develop lesions. So-called IIbroWll tumors" containing giant cells may
also be seen in advanced cases.
D. OSTEITIS DEFORI1ANS (PAGET'S DISEASE OF BONE) - of obscure etiology
(defective connective tissue metabolism, viral infection?), this
disorder occurs after age 40 and may be present in up to 3% of
elderly persons. It may be monostotic or polyostotic and can involve
any bone. Bizarre bone remodeling results in a tile-like or mosaic

173
pattern that is diagnostic of Paget's disease. Patients may present
with pathologic fractures and X-rays reveal enlarged, radiolucent
bones. A small number of patients subsequently develop an aggressive
form of osteogenic sarcoma.
IV. NEOPLASIA

A. BOHE-FORl'IING (OSTEOBLASTIC) TUHORS

1. OSTE~ - is a rare benign tumor, usually found in the skull,


and comprised of dense normal bone.
2. OSTEom OSTE~ is a benign, but painful, lytic lesion
occurring in the diaphyses of long bones (femur and tibia most
frequently) in patients under age 30.
3. OSTEOGENIC SARCOtIA (OSTEOSARCOtIA) - is the most common primary bone
tumor and usually affects young people ages 10 to 25 except in
those cases arising from Paget's disease which usually occur
in patients over age 50. In young age group the femur, tibia
and humerus are common sites and about 75% occur in the
vicinity of the knee. Pain is frequently an early feature and
many have metastasized (particularly to lung) at the time of
diagnosis. They usually begin in the medullary cavity and
penetrate outward through the cortical bone. The histologic
appearance may vary but all produce "malignant" osteoid as the
characteristic feature.
B. CARTILAGE-FORKING (CHONDROGENIC) TUHORS

1. EX~IS - is a benign, knobby neoplasm which protrudes from


metaphyseal surface of long bones - usually the lower femur or
upper tibia. As an isolated defect it has little significance,
but an inherited condition known as "hereditary multiple car-
tilaginous exostosis" is associated with numerous exostoses
that possess a definite potential for malignant
transformation.
2. ENCHONDIDIA - is a benign neoplasm that typically occurs in the
small bones of the hands and feet of young adults. It may be
erosive and cause pain, swelling, or pathologic fractures.
Multiple enchondromas are characteristic of Oilier's disease
and an inherited form associated with hemangiomas is Haffuci's
syndrome.
3. CHONDROGENIC SARCOtIA (CHONDROSARCOtIA) - is the malignant tumor of
cartilage forming cells (chondroblasts). Most occur in an
older age group than osteosarcoma, usually after age 35, and
usually involve pelvic bones, vertebrae and ribs. Growth is
much slower and prognosis much better than osteosarcoma.
C. GIANT CELL TUMOR - occurs in patients over age 20 with frequent sites
being the ends of long bones. There is bone or cartilage formation
but numerous giant cells are seen within a background of neoplastic
plump or spindled fibroblast-like cells. They may be benign or
malignant depending on presence or absence of anaplastic features of
the fibroblast-like cells. Other "bone tumors" may contain giant
cells but this does not make them a giant cell tumor.
D. EHING' 5 SARCqIA - is a rare but extremely malignant neoplasm that
occurs in older children and adolescents. Usually arising within the
marrow cavity, the tumor cons ists of sheets of small, round
hyperchromatic cells which may be confused with metastasis from
neuroblastoma. Early, widespread metastasis is common and prognosis
is generally poor although recent advances in treatment have been
encouraging.
E. I1ETASTATIC CANCER - bone is a common site of disseminated cancer, most
of which are carcinomas. Common primary cancers which tend to
involve bone in metastatic spread include breast, lung, prostate, and
kidney.

174
JOINTS AND RELATED STRUCTURES
I. INFECTIVE ARTHRmS

A. PYOGENIC ARTHRITIS -usually represents hematogenous dissemination from


a distant focus of infection. In children, the most common organism
is Hemophilus influenzae while, in adults, the organisms are more
likely to include staphylococci, streptococci, pneumococci and
gonococci. The disorder is usually monoarticular and involves one of
the larger joints, i.e. hip, knee, ankle, shoulder, elbow, wrist.
Clinical manifestations include redness, swelling, pain and
tenderness of involved joint. If it is not effectively treated, it
may become chronic and destructive.
B. TUBERCULOUS ARTHRITIS - may occur with tuberculous osteomyelitis of
spine (Pott's disease) which extends into intervertebral discs. It
may also occur as a monoarticular form within large joints. Early
diagnosis and treatment is necessary to prevent the destructive
consequences of this usually chronic disorder.
II. OSTEOARTHRmS (DEGENERATIVE JOINT DISEASE) - is the most common
disease of joints. It is a degenerative rather than an inflammatory
disorder that is associated with aging but may occur after trauma or
infection of the joint. The joints affected are commonly the weight-
bearing (vertebral, femoral) joints and the distal interphalangeal joints.
The common pathologic change is degeneration of articular cartilage. Clin-
ically, there is slowly progressive joint stiffness. Spur formation in
distal interphalan~eal joints of fingers produces nodular swellings called
"ileberden's nodes'. Involvement of the spine may cause compression of
nerve roots with radicular pain, spasms and atrophy of skeletal muscle.
Bony spurs may project from the margins of the joint space, limiting
motion and causing pain. Bone spurs or fragments of articular cartilage
may break off to create loose foreign bodies known more commonly as "joint
,
m~ce ".
III. RHEUMATOID ARTHRITIS - is an idiopathic, chronic, systemic inflammatory
(probably autoimmune) disease which may affect skin, skeletal muscle,
bone, eyes, heart, blood vessels, lungs and other organs. The classic
feature is a progressively deforming arthritis which usually affects
multiple joints, most often small joints of the hands and feet. Classic-
ally, the onset is usually in the third and fourth decades with a pro-
drome of fatigue, malaise and low-grade fever. Prodromal symptoms are
followed by the appearance of joint stiffness, most apparent in the
morning upon waking. Any joint may be involved but the classic presenta-
tion is bilateral symmetric involvement of proximal interphalangeal and
metacarpophalangeal joints which may be enlarged, tender, painful, warm,
and reddened. As the disease progresses, there is progressive joint
induration and stiffness to the point of permanent and disabling
ankylosis. Atrophy of related skeletal muscle follows because of disuse.
The hands may assume a "clawlike" appearance with characteristic ulnar
deviation of ankylosed fingers; the overlying skin is shiny, erythematous
and atrophic. An anti-IgG antibody (rheumatoid factor) is found in nearly
all patients with classic disease. Early lesions show an acute nonspecific
synovitis with edema and a mixed inflammatory infiltrate of the synovia.
As the disease progresses, there is a diffuse proliferative synovitis with
replacement of the synovial lining by a highly vascularized, polypoid mass
of inflammatory tissue infiltrated by chronic inflammatory cells - so
called pannus formation. Advanced lesions show erosion of the entire
articular surface with obliteration of the joint space followed by fibrous
adhesions or bony ankylosis between opposing joint surfaces.
IV. GOUTY ARTHRITIS - is characterized by transient but recurrent attacks
of acute arthritis due to precipitation of monosodium urate crystals in
the synovial spaces and may develop in anyone with elevated uric acid
blood levels. Most cases of gout are due to a genetic disorder of purine
metabolism and predominately affects adult males, but other cases may
arise from excessive cellular turnover (leukemias, etc) and/or renal

175
disease. Other manifestations of gout include deposition of urate crystals
in other tissues such as joint capsules, perichondrial tissue, bursae,
heart valves and kidney creating inflammatory foci known as tophi.
Recurrent attacks of acute gouty arthritis over a period of years will
eventually create a chronic disabling disorder.
V. TUMORS AND TUMOR-LIKE CONDmONS
A. G~ION - is a common, small, cystic, non-neoplastic lesion found
within the connective tissue of a joint capsule or tendon sheath. It
consists of a central area of myxoid degeneration surrounded by
collagenous capsule. Excision is usually curative.
B. SYNOVIAL SARCOIA - is an uncommon lesion that tends to arise in the
deep soft tissues of lower and upper extremities from synovial
cells, usually of bursae. They tend to be highly aggressive and
frequently metastasize to lungs and pleura.

176
NEUROMUSCULAR SYSTEM

CENTRAL NERVOUS SYSTEM


I. CELLULAR COMPONENTS AND REACTION TO INJURY

A. NE~ - is the functional unit of nervous system.


1. CDlPONEHl' PARTS

a. Cell body - varies -in shape depending on location and


contains Nissl bodies, neurofibrils (microtubules and
neurofilaments), and occasionally pigment (melanin or
lipofuscin). Integrity of Nissl bodies reflect the
health of the cell. Nissl bodies tend to disperse in
reaction to injury and therefore reflect the health of
the neuron.
b. Dendrites afferent processes which show various
patterns depending on location.
c. ~ - efferent processes which vary in diameter and
length and are encased by myelin sheath.
d. Hyelin sheaths - surround axons and act as electrical
insulation. In PNS, myelin sheaths formed by Schwann
cells while in CNS, myelin is formed from
oligodendroglia.
2. NEURONAL REACTION TO INJURY

a. Central chromato~sis (reversible) - results from axonal


damage. Cell bo y becomes swollen and rounded with
dissolution of the central Nissl bodies and peripheral
displacement of the nucleus. The closer the axonal
damage to the cell body, the more rapid and severe the
central chromatolysis.
b. Simple chromatolysis (reversible) - dissolution of all
Nissl substance. Nucleus retains central location but
ghost cells or shadow cells result.
c. Acute ischemic necrosis (non-reversible) - results from
hypoxic damage. Cell body becomes swollen with eosino-
philic alteration of cytoplasm (red neuron). Nucleolus
is lost and nucleus becomes pyknotic. Neuron eventually
dissolves and drops out. First appears 5-6 hours after
insult. Hippocampal, Purkinje, and cortical neurons are
most susceptible.
d. Axgnal degeneration (dying-back or retrograde degener-
ation) - affects entire neuron. Axonal damage results in
progressive proximal degeneration of axon from point of
injury. The axon swells, fragments, and disappears.
Chromatolysis occurs in the cell body.
e. Wallerian degeneration - disruption of axonal integrity
results in axonal disintegration distal to point of
injury. Axons in PNS may regenerate, particularly if
Schwann cells remain intact, but axons of CNS will not
regenerate to any significant extent.
f. Demyelination - disintegration of myelin sheaths may be
primary event or secondary to axonal damage. Integrity
of myelin is dependent on integrity of axon but not
necessarily vice versa. PNS may remyelinate but CNS will
not.

177
B. ~ (neuroectodermal origin)
1. protoplasmic astrocytes are found primarily in the gray
FORtIS -
matter and have numerous thick branching processes. Fibrillary
astrocytes have fewer, thinner, processes show less branching
and contain glial fibrils that are not present in protoplasmic
astrocytes. All astrocytes have processes which abut on blood
vessels (vascular footplate) to form a perivascular glial
membrane. This vaso-astral network acts as an interstitial
framework for the CNS.
2. REACfIONS TO INJURY

a. Astrocytosis (cellular gliosis) - 4-5 days after injury,


astrocytes undergo hyperplasia, hypertrophy, and develop
an eosinophilic cytoplasm with eccentric nuclei
(gemistocyte). Protoplasmic astrocytes develop glial
fibers to become fibrillary astrocytes. The astrocytes
act as scar tissue by replacing damaged tissue with
fibrillary processes. Eventually the cells are crowded
out leaving a fibrillary gliosis or glial scar.
b. Clasmatodendrosis - fragmentation and loss of cell
processes with disintegration of cell body occurs with
severe injury and represents death of astrocytes.
c. Alzheimer Type II astrocytes - reaction of protoplasmic
astrocytes seen in liver disease. Enlarged, pale nuclei
in otherwise normal cytoplasm.
d. CODorS. 8IDylacea (brain sand) - are basophilic laminated
structures which form in terminal processes of astro-
cytes seen with increasing afe and found predominately
around vessels and in subpia and subependymal areas.
e. Rosenthal fibers - elongated, dense, tapering bodies
associated with long standing gliOSis.
C. OL~ROGLIA (neuroectodermal origin) - are small, dark cells with
relatively few, short, thin processes. They are most numerous in the
white matter where they line up in an interfascicular arrangement
and form myelin sheaths around nerve fibers early in fetal life.
They are necessary to maintain the integrity of existing myelin but
will not replace destroyed myelin. They are also present in gray
matter where they serve (along with microglia) as satellite cells to
neurons. Their reaction to injury basically involves cellular
swelling with the formation of a perinuclear halo.
D. E~ (neuroectodermal origin) - are specialized ciliated glial
cells which line the ventricular cavities and central canal of
spinal cord. Cilia arise from intracytoplasmic structures called
blepharoplasts. Irritation of the ventricular cavity may result in
local proliferation of the subependymal glial tissue which may
protrude into the lumen (ependymal granulation) to cause granular
ependymitis. Usually not significant unless it obstructs the CSF
pathway (aqueductal or foraminal gliosis).
E. MICROGLIA (mesodermal origin) - are small elongated cells with
numerous processes which function as CNS reticuloendothelial cells.
In the gray matter they also serve, along with oligodendroglia.as
satellite cells to neurons. Within a few days after CNS injury,
microglia increase in number. The cell body becomes bulkier as the
cytoplasmic processes are drawn back into the cell. They become
mobile with phagocytic properties and become known as compound
granule or gitter cells. They act as macrophages and phagocytize
myelin and tissue debris, migrate to and accumulate around blood
vessels, and are eventually borne off in the circulation. Injury to
cortical neurons may evoke a focal increase in satellite cells
(satellitosis). I f the neuron dies, microglial cells phagocytize the
necrotic neuron in process of neuronophagia.

178
II. NONSPECIFIC CNS REACTION TO INJURY
A. INCREASED ItrrRACRANIAL PRESSURE (CSF pressure> 200 nun H20) - occurs when
CNS volume exceeds physical capacity for expansion. May be result of
local or generalized mass effect resulting from tumor, abscess,
edema, etc. Clinically manifested as intermittent headache, mental
slowness, confusion, and papilledema. Complications include:
1. LATERAL HERNIATION - reflected as either hippocampal (uncal)
herniation (hippocampal gyrus forced under tentorium
cerebelli), contralateral cerebral peduncle laceration caused
by edge of tentorium (Kernohan's notch), and cingulate
(subfalcial) herniation (cingulate gyrus forced under falx
cerebri) .
2. OUTHARD HERNIATION - transcalvarial herniation is brain herniates
through calvarial defect - usually traumatically induced.
3. DOHNHARD HERNIATION - tonsillar and medullary herniation through
foramen magnum.
4. INFARCTION - resulting from herniations which compress and
occlude vessels. Most frequently involves compression of
posterior cerebral against tentorium cerebelli and anterior
cerebral against falx cerebri.
5. SECONDARY BRAIN STEH HEtlORRHAGE - may be associated with rapidly
expending supratentorial lesions.
B. CEREBRAL EDEMA - may be local or generalized and cause increased
intracranial pressure and resultant complications. Edema may be
cytotoxic (fluid accumulation within cells usually secondary to
metabolic disturbance), vasogenic (extracellular fluid accumulation
due to damaged capillary endothelial cells), or interstitial (fluid
accumulation in periventricular white matter secondary to increased
passage of CSF across ventricular walls). _
C. HYDROCEPHAWS - refers to increased CSF volume causing ventricular
distention. CSF is produced by choroid plexus and reabsorbed
primarily by the arachnoid villi granulations. Increased CSF volume
may be result of overproduction of CSF, inability of arachnoid villi
to transfer CSF to venous drainage, or blockage of CSF pathways. A
non-communicating (obstructive) hydrocephalus results if blockage
occurs within the brain and a communicating hydrocephalus if
blockage occurs in subarachnoid space. If hydrocephalus occurs
before cranial sutures close, there is resultant enlargement of
head. After cranial sutures are closed, acute hydrocephalus will
cause symptoms related to increased intracranial pressure and is
often associated with trauma, infections, subarachnoid hemorrhage or
tumors. Slowly progressive hydrocephalus, however, may not show
elevated CSF pressures (normal pressure hydrocephalus) and may be
clinically manifested by progressive dementia, gait disturbances,
and incontinence. As ventricles dilate, the ependyma becomes
flattened and disrupted and interstitial edema may ensue with injury
to surrounding white matter. Hydrocephalus ex vacuo refers to
compensatory ventricular dilation secondary to brain atrophy.
III. TRAUMATIC INJURY - with even a minor blow to the head, pressure waves
are transmitted through the brain parenchyma and may cause submicroscopic
splits in individual axons. If the head is put into motion by the force of
the impact, the skull accelerates and decelerates faster than brain
resulting in impact lesions on the parenchyma and stretching and injury to
the vessels which traverse the space between the brain and skull on both
the ipsilateral and contralateral surfaces. The ipsilateral lesion
resulting at the primary impact site is the "coup" lesion while the
contralateral lesion, which is often larger, is the "contrecoup" lesion.
A. CON~ION - is a clinical diagnosis characterized by transient loss
o( consciousness with retrograde and anterograde amnesia. There is
little, if any, damage to the CNS and no residual sequelae.

179
B. EPmURAL HEI1ATCI1A - refers to bleeding between the skull and dura
mater. It appears as dark red gelatinous blood clot which adheres to
the dura. Although atypically it may be of venous origin, most
frequently epidural hematomas result from tearing of the middle
meningeal artery and are located over the temporal areas. Clinical
course is characterized by transient loss of consciousness with a
subsequent lucid interval followed, within 24 hours, by coma and
signs of increased intracranial pressure. Unless surgically drained,
they tend to expand rapidly in volume with ensuing brain herniation
and death.
C. SUBDURAL HEI1ATOtIA - refers to collection of blood in the potential
space between the dura and leptomeninges. These usually result from
rupture of bridging veins that cross this space and are most often
located over the convexities (15% bilateral). Usually associated
with blunt trauma without skull fracture.
1. Acne - usually associated with underlying parenchymal damage
and become clinically manifested by fluctuating levels of
consciousness within a few days after the trauma. The CSF may
be clear, bloody or xanthochromic depending on associated
injuries.
2. CHRONIC - become clinically apparent weeks or months after
often otherwise insignificant head trauma. Manifested by
slowly developing confusion and inattention which may lead to
coma. As organization occurs around the periphery, both a
visceral and parietal (usually thicker) membrane develop.
Unless drained, the hematoma may slowly expand resulting in
herniation and/or brain stem hemorrhage. The CSF is usually
xanthochromic with increased protein content.
D. omnrusION -usually superficial areas of hemorrhagic necrosis of the
cortex resulting from crushing of CNS tissue by blunt force. Most
often seen on inferior surface of frontal lobes, anterior tip of
temporal lobes, and occipital poles. Initial foci of hemorrhagic
necrosis are replaced by glial scar to form irregular yel10wish-
brown depressed areas. These may act as foci of seizure activity
(particularly temporal lobes).
E. SUBARACHNOm HEI'IORRHAGE - trauma is most common cause of subarachnoid
hemorrhage and results from rupture of corticomeningeal vessels.
Usually accompanied by underlying contusion.
IV. VASCULAR DISEASE

A. HYPOXIC ENCEPHALOPATHY -may be produced by any process that reduces


effective oxygenation of the brain. To ensure adequate oxygenation,
the cerebral circulation is regulated by various metabolic and
reflex homeostatic mechanisms to maintain adequate perfusion in the
face of systemic hypotension. Pre-existing vascular disease (athero-
sclerotic, hypertensive, etc.) or profound hypotension, however, may
overcome these mechanisms. Neurologic change depends on extent and
duration of hypoxia and length of survival after the insult. The
first changes are seen 6-12 hours after the insult in the hippo-
campal neurons and cerebellar purkinje cells which develop acute
ischemic necrosis. Cortical neurons may be haphazardly affected, but
with extensive anoxia, laminar necrosis of the cortex may result.
B. INFARCTION - results from loss of blood supply with ensuing necrosis.
Symptoms relating to cerebral infarcts depend on the location and
size of the infarct.
1. ETIOLOGY

a. Vascular thrombosis - thrombus formation within a vessel


is usually associated with underlying, pre-existing
damage to the vessel wall (atherosclerosis, vasculitis,
etc) and tends to occur within the larger vessels
(internal carotid, vertebral, basilar) of older

180
individuals. Infarcts due to thrombosis are often
heralded by transient ischelllic attacks (TIA) and tend to
produce ischemic infarcts.
b. Emboli - most arise from thrombi within the left heart
or carotid arteries, from atheromatous material at the
carotid bifurcation, or from vegetations attached to the
mitral or aortic valves. Emboli tend to be multiple and
involve smaller vessels (middle cerebral, etc). Infarcts
due to emboli are usually characterized by the sudden
onset of neurologic deficit and tend to produce hemor-
rhagic infarcts due to dissolution of embolus and
reperfusion of the infarcted necrotic tissue.
2. LOCATION

a. KiddIe cerebral artery distribution - is the most common


and usually due to carotid thrombosis or thrombosis/-
embolus of middle cerebral artery.
b. Lacunar infarcts - are small infarcts in deep white
matter due to occlusion of deep penetrating arteries and
often seen in association with hypertension.
c. Border zone infarcts - are hypotensive infarcts occur-
ring in regions between major vessel distribution
patterns.
3. GROSS APPEARANCE - slight discoloration of the parenchyma appears
within 6-12 hours followed by disintegration of tissue and
surrounding edema occurring between 2-3 days. Later, there is
liquefaction,. cyst formation, and glial reaction around the
periphery.
4. MICROSCOPIC APPEARANCE - ischemic neuronal necrosis with myelin
disintegration and loss of astrocytes and oligodendroglia
develops within 6-12 hours. After about two days, there is a
mild-moderate acute inflammatory infiltrate which is soon
replaced by proliferating macrophages which reach a peak at
about two weeks. Glial fibrosis then ensues.
C. HEIIORRHASE (non-traumatic) - hemorrhages greater than 3.0 cm. diameter
within the cerebral hemispheres or greater than 1.5 cm. diameter
within the brain stem and cerebellum are considered massive hemor-
rhages. Although less common than infarcts, hemorrhages account for
more deaths. With cerebral hemorrhage, opening pressure of the CSF
is usually increased. Bloody fluid is present in 75% but it may be
xanthochromic (10%) or clear (15%). Protein usually is increased and
glucose is normal. In subarachnoid hemorrhage, opening pressure is
elevated. Fluid is bloody in first 24 hours and then turns
xanthochromic. Protein is increased but less than with cerebral
hemorrhage. Glucose is normal.
1. HYPERTENSIVEHEMORRHAGE - is the most common cause of non-
traumatic hemorrhage (25%) and occurs most frequently in the
basal ganglia. The exact mechanism is not known, but theories
include primary rupture of vessels whose walls have been
previously damaged by chronic hypertension, hemorrhage into
lacunar infarcts, and rupture of hypertensive microaneurysms
(Charcot-Bouchard aneurysllls). Unlike infarcts, hemorrhages are
not confined to an arterial distribution, and they tend to
compress and disrupt the parenchyma rather than destroy it.
The surrounding tissue and ventricles are distorted and the
hemorrhage may rupture into the ventricular system or, less
often, the subarachnoid space. Herniation may occur secondary
to the expanding mass and the associated cerebral edema. If
the patient survives, resolution may occur with surprisingly
little parenchymal destruction.

181
2. ANEURYSItS - account for about 15% of non-traumatic hemorrhages.
a. Saccular ("berry") aneurysms - are the most common and
are present in 5-6% of the general population. Although
some feel these arise from a congenital defect in the
internal elastic membrane, others feel they are
developmental in origin. Most occur in the middle
cerebral artery and its branches although those of the
anterior circulation (anterior cerebral and anterior
communicating) are more prone to rupture and cause
clinical symptoms. They arise at the bifurcation of
vessels and are multiple in 25% of cases. Complications
(in order of clinical significance) include:
(1) Rupture subarachnoid, intraparenchymal, and
intraventricular hemorrhage is the most lethal
complication and occurs in about half of ruptures
while subarachnoid hemorrhage alone occurs in
25%.
(2) Infarction - vascular spasms may occur resulting
in parenchymal necrosis. Most often seen in
association with anterior cerebral aneurysms.
(3) Mass effects - large aneurysm (which are less
prone to rupture due to laminated mural thrombus
within the aneurysm) may uncommonly cause space-
occupying problems (compression of third
ventricle and hydrocephalus).
b. Inflammatory (mycotic) aneurysms - usually seen in
small, peripheral vessels due to septic emboli where
organisms invade vessel wall with subsequent dilatation
and bleeding.
c. Atherosclerotic aneurysms - produce a fusiform dilata-
tion usually involving vertebral or basilar artery but
occasionally involving internal carotid. Usually do not
rupture but may thrombose or, due to size, cause cranial
nerve dysfunction.
D. are relatively common (5% of general population)
VASCULAR IlALFORI'IATIQH -
and due to incomplete and/or abnormal resolution of embryonic
vasculature.
1. ARTERIOVENOUS HALFORI'IATIONS (12%) - consist of tangled admixture of
veins, arteries, and arterialized veins which are separated by
gliotic parench~a. They are usually located in cerebral hemi-
spheres and may "grow" by incorporating adjacent vessels. They
are the type of malformation most likely to cause seizures,
"steal" (diverting blood from arterial to venous side
resulting in poor perfusion of the intervening and surrounding
parenchyma), and intracerebral hemorrhage from rupture.
2. VENOUS HALFORI'IATIONS (67%) - are the most common malformation but
are usually asymptomatic. They consist of loose aggregates of
venous channels.
3. CAVERNOUS ANGIOMA (7%) - compact aggregate of sinusoidal channels
without intervening parenchyma. Often located at the surface
of brain and may act as epileptogenic foci.
4. TELANGIEcrASIS (11%) - small aggregate of capillaries usually
found at base of pons and is usually asymptomatic.
5. VARICES (3%) - single abnormally dilated vein within the neural
parenchyma that is generally asymptomatic.

182
V.~Cll0US DISEASE - is becoming increasingly frequent particularly in
patients with AIDS (primarily viral and fungal infections).
A. BACTERIAL

1. LEPTOIENINGITIS - is most commonly caused by E. coli (neonates),


H. influenza (infants and young children), N. meningitidis
(older children, adolescents, young adults), and Strep.
pneumoniae (adults, young children). Grossly, meninges become
congested and clouded. Purulent exudate may be observed and
microscopic exam shows acute inflammatory infiltrate. CSF
shows increased pressure, increased neutrophils, increased
protein, and decreased glucose. Common symptoms include
agitation, irritability, headache, photophobia, stiff neck,
and positive Kernig's sign. Complications consist of adhesions
which can constrict cranial nerves, obliterate subarachnoid
space around brainstem, or obstruct foramina of Luschka and
Magendie resulting in hydrocephalus; arteritis or phlebitis
which may cause obstructive thrombus formation lending to
small cerebral infarcts; intraparenchymal abscesses; and
death.
2. INTRAPARENCHYHAL ABSCESS most abscesses have no definable
source. When an organism can be cultured, anaerobic strep is
often fOlmd. Others will be related to sinus infection, ear
infection, mastoiditis, direct implantation, or hematogenous
spread. Septic emboli often cause multiple abscesses. Infil-
tration of polys with central liquefactive necrosis is
associated with extensive cerebral edema. Fibroblasts and
astrocytes attempt to wall off the necrotic area and, as dead
tissue is removed, an abscess cavity evolves. Symptoms are
related to the edema, increased intracranial pressure, and
area of involvement. Without surgical drainage, abscesses tend
to expand in size and act as mass lesions. Rupture into
ventricular cavities is usually fatal.
B. VIRAL -viral meningitis and encephalitis are uncommon but important
complications of systemic viral disease and AIDS. Portals of entry
are primarily from hematogenous spread (blood-brain barrier does not
inhibit passage of viruses) and spread along peripheral nerves. CNS
damage may occur by direct destruction of tissue without inflam-
matory or immunologic response (slow viruses); cell lysis resulting
from immunologic reaction against virus infected cells (SSPE);
immune complexes deposited in walls of vessels; and autoimmune
reactions to myelin. Histologic changes characteristic of viral
disease include perivascular mononuclear infiltrates (lymphocyte,
plasma cell, macrophage); intranuclear or intracytoplasmic inclu-
sions - Cowdry A bodies (eosinophilic intranuclear inclusions with
a hyaline or granular appearance often surrounded by a halo) or
Cowdry B bodies (small round indistinct inclusions which are less
suggestive of viral disease); glial nodules; and neuronophagia. The
CSF shows increased mononuclear cells, normal glucose, and normal or
slightly increased protein. Many viruses are site-specific and
clinical symptoms depend on virulence and site affected. Diagnosis
usually rests on serologic proof.
1. - as a group, these are the most common cause of
ENTEROVIRUS
viral meningitis. They are spread from person to person and
cases usually seen in late summer or fall.
a. Poliomyelitis is heralded by nonspecific upper
respiratory symptoms, GI symptoms, and fever which may
progress to headache and stiff neck. Most cases resolve
without sequelae but may progress to involve large motor
neurons in spinal cord and brainstem with resultant
lower motor neuron paralysis.
b. Coxsackie B may cause meningoencephalitis and
myocarditis especially in newborn period. Character-
istically involves inferior olivary nucleus.
183
2. ARBOVI~S - are transmitted by mosquito and tick vectors. The
causative agents of Eastern equine encephalitis and Western
equine encephalitis, the majority of cases are subclinical.
3. HERPES VIRUS
a. Heryes Simplex Type I - the initial infection is usually
chi dhood gingivostomatitis. The virus is harbored in
the trigeminal ganglia and may cause recurrent "cold
sores". Encephalitis is uncommon but when it occurs, it
is heralded by a "flu-like" syndrome which progresses to
olfactory and gustatory hallucination, focal neurolog-
ical signs, bizarre behavior, seizures, obtundation and
coma. Characteristically involves inferior surface of
frontal lobes and medial surfaces of temporal lobes.
Areas of hemorrhagic necrosis, edema, perivascular in-
filtrates, and Cowdry type A inclusions are seen micro-
scopically. Virus cannot be isolated from blood or CSF.
b. Herpes Simplex Type II - venereally transmitted, the
virus is harbored in sacral ganglia, and uncommonly
produces encephalitis in adults. Newborns, however, can
be infected during passage through the birth canal or,
less often, in utero to produce a systemic infection.
Characteristic cutaneous vesicles may be present and
about half will develop CNS involvement. Cowdry Type A
inclusions are present. Virus can be recovered from
blood and CSF.
c. Herpes Zoster - virus is harbored in dorsal root gang-
lia, produces "shingles", and rarely results in
disseminated disease.
d. Cytomegalovirus - transplacental infection of fetus
causes a systemic disease resulting in microcephaly,
periventricular mineralization, hepatosplenomegaly,
purpura, pneumonitis, chorioretinitis, and G. I. lesions.
Microscopically characterized by large cells with
prominent nuclear inclusions and granular cytoplasm.
4. HutIAN DlHUNOOEFICIEHCY VIRUS (HIV-l) - most patients will develop at
some point a peripheral neuropathy related to focal demyelin-
ation of peripheral nerves. The virus may also, independent of
its immunosuppressive effects, produce an aseptic meningitis
similar to other forms of viral meningitis and/or a dementive
encephalitis characterized by the presence of virally infected
multinucleated giant cells in the white matter.
S. MUMPS VIRUS - is the virus most likely to affect CNS but is
usually only mild and transient with headache and stiff neck.
6. RABIES - the incubation period is 1-3 months and onset is
characterized by fever, malaise, and headache which progresses
to profound CNS sensitivity (pain, convulsions, photophobia),
coma, and death. Brain shows edema and congestion with
widespread neuronal degeneration particularly within basal
ganglia, midbrain, and floor of 4th ventricle. When seen (70%
of cases), mUltiple eosinophilic intracytoplasmic inclusions
(Negri bodies) are diagnostic. Immunofluorescent techniques
are now used for diagnosis.
7. RUBELLA - transplacental infection during first trimester
causes reduction in cell growth and division. Many fetuses are
stillborn or aborted, but those born alive may show cardio-
vascular and pulmonary defects, blindness, deafness, mental
retardation, and low birth weight.
8. RUBEOLA - although rare, measles infection at an early age, may
later lead to a progressive, fatal, immune-mediated
encephaHtis called subacute sclerosing panencephalitis
(SSPE). It is more common in young males and heralded by
intellectual deterioration, personality change, and gait
disturbances and progresses to death within months to years.
It probably represents antibody or cell mediated reaction
against virus infected cells. There is increased IgG in CSF.

184
9. ~urr-~MXB D~ is a slow-virus infection with
incubation period of many years. Onset in middle-aged to older
adults and manifested by personality changes and disturbances
in coordination followed by progressive severe dementia. It is
characterized by spongiform encephalopathy (vacuolization of
neurons and glial processes) of cortex and, to a lesser
extent, the basal ganglia.
10. PROGRESSIVE ItULTIFOCAL LEUKOEHCEPHALOPA11IY (PItL) - is a slow virus
infection of oligodendroglia that causes death of the cells
and foci of related demyelination. Intranuclear inclusions can
be identified. This occurs in immunosuppressed or
immunocompromised patients and the clinical manifestations
vary greatly but are progressive.
C. lWEBCULOUS IlENINGIIIS - at one time was most common cause of meningitis
in children, but now seen primarily in adults and secondary to
hematogenous spread from pulmonary focus. Tends to localize at base
of brain where abundant shaggy, necrotic yellowish exudate
accumulates and may cause compression of cranial nerves or brain
stem, hydrocephalus from obliteration of subarachnoid space, or
ischemic necrosis secondary to obliterative vasculitis. Always
results in death of untreated; 30% mortality if treated.
D. SYPHILIS - neurosyphilis occurs in about 30% of untreated cases. The
spirochete enters the CNS hematogenously during the secondary stage
and becomes quiescent.
1. meninges are infiltrated by plasma cells and
IlENINGOVASCULAR -
lymphocytes perivascularly. Scarring may cause nerve compres-
sion or hydrocephalus. Vascular involvement (obliterative
vasculitis} may result in parenchymal infarction and present
as "stroke". Congenital syphilis produces meningovascular
symptoms at, or soon after, birth.
2• PARENCHYIIAL

a. General paresis of the inSane - results from cerebral


involvement and produces psychotic dementias with onset
about 15 years after initial infection. Spirochetes are
abundant in the gray matter (particularly prefrontal and
temporal lobes). There is a diffuse co.rtical atrophy
with neuronal loss, hyperplastic microglia (rod cells)
and perivascular infiltrates.
b. Tabes dorsalis - results from involvement of spinal cord
with onset 10-25 years after primary infection. Spiro-
chetes are absent but there is degeneration of the large
diameter fibers of the posterior columns of spinal cord
(pain, vibratory, position) resulting in ataxia, urinary
retention, and pupils reactive to accommodation but not
to light (Argyll-Robertson pupil).
E. ~

1. CRYPrOCOCCUS - is the most common fungal infection of CNS and


has an insidious onset with headache or behavioral changes.
The organisms give a "soap-bubble" appearance to the brain and
do not elicit much inflammatory reaction.
2. HUCORIIYCOSIS - is a commensal organism in nasal turbinates which
when pathogenic, enters cavernous sinus via ophthalmic veins,
enters internal carotid artery, and is disseminated to brain
where it obstructs vessels and produces parenchymal infarc-
tion. Seen predominantly in diabetics with ketoacidosis or
immunocompromised patients.
3. CANDIDA, COCCIDlmrtCOSIS, BLASTOtIYCOSIS, HISfOPLASItOSIS, ASPERGILLOSIS,
etc. - may involve CNS in disseminated disease.
F. - is a parasitic protozoan that can cross the placental
TOxopLASMA
barrier between mother and fetus. Maternal infection can cause
extensive areas of necrosis of the developing fetal brain. These
lesions also appear in patients with AIDS.

185
VI. DEMENTIAS AND DEGENERATIVE DISEASES - dementia refers to impairment
of orientation, memory, intellect, and judgement with associated
alterations in mood and behavior.
A. ALZIIEDtER' S DISEASE -most of the cases occur sporadically but a few
familial cases (autosomal dominant pattern) have been identified.
There is evidence that the pathogenesis involves abnormalities of
Chromosome 21. Clinically, there is an insidious onset of behavioral
changes (anxiety, depression, insomnia, visual hallucinations,
paranoia), recent memory loss, and progressive intellectual
impairment leading to eventual inability to comprehend, communicate,
or care for oneself. Death is often secondary to respiratory
infections. Grossly, the meninges are thickened, gyri atrophic with
corresponding widening of sulci, and dilatation of ventricles due to
the cortical atrophy (hydrocephalus ex vacuo). Characteristic
histologic changes include increased numbers of senile plaques
(consisting of granular or filamentous argyrophilic material
arranged around a central amyloid core), neurofibrillary tangles
(neuronal intracytoplasmic masses of fibrillary argyrophilic
structures), granulovacuolar degeneration (intracytoplasmic vacuoles
of hippocampal pyramidal cells which contain an argyrophilic
granule), and amyloid angiopathy (abnormal amyloid deposition in
subarachnoid and penetrating cortical vessels).
B. PICK'S DISEASE - clinically indistinguishable from Alzheimer's disease
but occurs more frequently in females. The cortical atrophy occurs
primarily in frontal and temporal lobes except for sparing of the
posterior 2/3 of the first temporal gyrus. There is gliosis of
subcortical white matter (lobar sclerosis) and occasional
involvement of the anterior portions of the caudate and putamen.
Characteristic intracytoplasmic spherical inclusions (Pick bodies)
may be seen in cortical and basal ganglia neurons.
C. mmrr~'s amm£A - autosomal dominant inherited disease with onset
in 4th decade heralded by choreiform movements and facial grimaces.
This is followed by personality changes, emotional disturbances, and
dementia. Gross ly, there is marked atrophy of the head of the
caudate and putamen with corresponding ventricular enlargement.
Microscopically, there is loss of small neurons of the caudate and
putamen with marked astrocytosis.
D. PARKINSON'S DISEASE (paralysis agitans) - is a relatively common disease
with onset in 5th and 6th decades heralded by resting "pill-rolling"
tremors and bradykinesia which progressed to rigidity and postural
changes, poor balance, shuffling gate, festination, and a mask-like
facies. Less than half will actually develop dementia. Histologic-
ally, there is degeneration and loss of the neuromelanin containing
cells especially within the substantia nigra but also in the loci
cerulei and dorsal motor nucleus of the vagus, accompanied by
astrocytosis. Characteristic, but not pathognomonic, single or
multiple round, eosinophilic laminated intracytoplasmic inclusions
(Lewy bodies) may be seen in remaining pigmented neurons.
E. AMYOTROPHIC LATERAL SCLEROSIS (ALS) is a relatively uncommon
degenerative disease which appears in the 6th decade and is
characterized by gradual progressive motor weakness leading to
respiratory failure. Sensory function and mental function remain
intact. Basically, there is degeneration of upper motor neurons in
the spinal cord and brain stem. Damage to the corticospinal tracts
and degeneration of anterior horn cells result in denervation
atrophy of muscle groups. Other "motor-neuron diseases" such as
Werdnig-Hoffman ("floppy infant") may be related.
F. SPINOCEREBELLAR ATAXIAS - refer to a group of inherited degenerative
diseases characterized clinically by a progressive disturbance of
equilibrium and movement resulting from primary neuronal atrophy.
These include Friedreich's ataxia and Olivopontocerebellar ataxia.

186
VII. DEMYELINATING DISEASE - is typified by multiple sclerosis, the most
common of the demyelinating diseases. The etiology is unknown but may be
autoimmune reaction against oligodendroglia (GSF IgG is increased in
approximately 2/3 of patients). Onset appears in young adults with highly
variable clinical symptoms. Patients may show episodic exacerbations and
remissions and may progress to blindness, incontinence, ataxia, and
paraplegia. Intelligence is usually not impaired. In the brain, there are
multiple irregular gray sclerotic plaques involving both the white and
gray matter. In the cerebral hemispheres, these are most prominent in the
periventricular white matter, but are also almost always present in brain
stem, spinal cord, and optic nerves. Early, active lesions show perivenous
demyelination and perivascular mononuclear infiltrates. Oligodendroglia
are markedly reduced in number or absent. These areas may coalesce to form
larger areas of demyelination with inflammatory reaction along their
borders. Older plaques show less mononuclear infiltrate but there is
astrocytosis with proliferation of glial processes imparting a firmer
consistency to the plaque.
VI I I. NUTRITIONAL AND TOXIC DISEASE

A. morphologic changes are often the result of


ALCOHOLIC ENCEPHALOPATHY -
trauma or complicating hypoxia, liver disease, or nutritional
deficiencies.
1. WERNICKE'S ENCEPHALOPATIlY - is the result of thiamine deficiency
and manifested as ataxia, nystagmus, and extra-ocular palsies.
Morphologic changes (proliferation and swelling of small
vessel endothelial cells and reactive gliosis) are found in
mammillary bodies, wall of third ventric.1e, floor of fourth
ventricle, and periaqueductal gray matter.
2. KORSAKOFF SYNDROME - often seen in association with Wernicke's
encephalopathy and characterized by profound memory
impairment.
3. CEREBELLAR DEGENERATION degeneration of purkinje cells
predominately in anterior superior portion of vermis produces
ataxic symptoms.
4. CENTRAL PONTINE MYELINOLYSIS - demyelinating lesion in the center
of pons. Usually, no associated clinical symptoms.
5. ALCOHOLIC NEUROPATHY segmental demyelination and axonal
degeneration of peripheral nerves (especially lower
extremities) lead to weakness, pain, and paresthesias.
B. D12 DEFICIENCY - demyelination and gliosis of posterior and lateral
columns of spinal cord leads to weakness and paresthesias which, if
untreated, may progress to ataxic paraplegia.
G. HEIHANOL POISONING - methanol is metabolized to formaldehyde and formic
acid with profound metabolic acidosis. Low doses cause blindness
secondary to retinal degeneration. Higher doses result, if patient
survives, in bilateral often symmetric necrosis of putamen.
D. CARBON MONoxmE POISONING - cherry-red discoloration of brain. If patient
survives, pallidal necrosis ensues.
IX. METABOUC DISEASES

A. STORAGEDISEASES - genetic (usually autosomal recessive) diseases


reSUlting from various enzyme deficiencies which interrupt normal
metabolic pathways and cause accumulation of precursor substances in
GNS and systemic organs.
1. SPHINGOLIPIDOSES

a. Tay'-Sachs - hexosaminidase A deficiency with accumu-


lation of GM2 - ganglioside. Infants are normal at birth
but within first year develop progressive motor and
mental retardation leading to blindness and vegetative
state. Macular degeneration produces a characteristic
"cherry-red spot" in fundi.

187
b. Gaucher's disease - beta glucosidase deficiency with
accumulation of glucocerebroside. Mental retardation,
hepatosplenomegaly, and yellow-brown cutaneous
pigmentation. Gaucher cells are present in HE system.
Infantile form involves CNS and appears earlier than
adult form which has little or no CNS involvement.
c. Niemann-Pick disease - sphingomyelinase deficiency with
accumulation of sphingomyelin. Mental retardation,
hepatosplenomegaly, olive-brown cutaneous pigmentation.
2. HUCOPOLYSACCHARIDOSES - approximately eight closely related
variants including Hurler's syndrome.
3. GLYCOGEN STORAGE DISEASES - approximately six variants including
Pompe's disease (Type II glycogenosis).
B. LEUKOOYSTROPHIES -inherited (usually autosomal recessive) biochemical
abnormalities in the development and maintenance of myelin include
metachromatic leUkodystrophy (cerebroside sulfatase deficiency) and
Krabbe's disease (galactoside beta galactosidase deficiency).
C. HIISJN'S DISEASE (hepatolenticular degeneration) - autosomal recessive
disorder of copper metabolism. There is increased copper absorption
from GI tract and decreased copper excretion in bile. The defect in
production of ceruloplasmin results in deposition of copper in
tissue. About 1/3 show onset in childhood with liver failure.
Remainder usually have onset in early adult life. Characteristic,
virtually pathognomonic, brownish-green discoloration occurs around
limbus of cornea (Kayser-Fleischer ring). Laboratory data shows
decreased serum ceruloplasmin (a-2 globulin), decreased serum
copper, increased hepatic copper, increased urinary copper
excretion, and abnormal liver function tests.
X. NEOPLASTIC DISEASE - there are no specific signs or symptoms of
neoplastic disease. Clinical manifestations depend on local effects
(irritative, compressive, or destructive effects on the particular sites
involved) and generalized effects (increased pressure, edema, hemorrhage,
etc.). All CNS neoplasms are potentially biologically malignant but
extraneural metastases are rare. Although the overall incidence of CNS
neoplasms tend to increase with age, they represent the second most common
group of tumors in childhood. About 70% of primary childhood intracranial
tumors develop in the posterior fossa while about 70% of primary adult
intracranial tumors are supratentorial.
A. MENINGEAL ORIGIN

1. MENINGIOMA(15%) - these are generally s low growing benign


neoplasms which occur most frequently in middle-aged adults
and mayor may not be symptomatic. They arise from
meningothelial cells which are most numerous in the arachnoid
villi. Although they may occur anywhere along the meninges,
favored sites include the parasagittal regions and lateral
cerebral convexities. Symptoms may include headache, focal
seizures and visual disturbances. Grossly, they grow as firm,
lobulated, white-grey to red-tan masses which compress and
indent, but are usually well circumscribed from, the brain
parenchyma. Occasionally, they may grow in a diffuse manner
(en plaque) along the meninges - particularly those that occur
along the sphenoid wings. They are usually attached to the
dura, may invade and obstruct the dural venous sinuses, are
often associated with hyperostos is. Microscopically, these
tumors have a heterogeneous appearance which allows
subclassification into meningotheliomatous (syncytial),
fibroblastic, transitional, and angioblastic forms. Psammoma
bodies may be present. Subclassification has disputed
prognostic significance but those tumors with cortical
invasion, increased cellularity, and increased mitoses indicate
a more aggressive behavior. The prognosis depends primarily on
accessibility to surgical removal.

188
2. these are actually of vascular origin but have
IlEKAHGIlBl.ASTCIIA -
histologic characteristics of the "angioblastic" meningioma.
They most often arise in the cerebellum but may occur in
medulla or spinal cord. They may occur as isolated neoplasms
or in association with the familial Von Hipple-Lindau syndrome
(retinal hemangioblastoma, visceral cysts, renal cell carcin-
oma, pheochromocytoma). Symptoms result from obstruction of
CSF leading to headache, vomiting, papilledema. Tonsillar
herniation may be fatal consequence. Grossly appear as
discrete masses or small red to yellow nodule on the wall of
a cyst which contains yellow to brown fluid. Varied micro-
scopic appearance but basically composed of vascular and
stromal cells. Prognosis is generally good.
B• NEUROGLIAL ORIGIN

1. AST~ - age incidence, appearance, and prognosis may vary


depending on location.
a. Well differentiated astrocytoma (Grade I) - these tumors
arise in the white matter with a peak incidence in the
3rd and 4th decades. Symptomatology may include
seizures, increased pressure, motor deficits, and mental
changes. Grossly, these are solid (there may be foci of
cystic degeneration), grey masses with ill-defined
boundaries which expands and distorts the brain
parenchyma. Microscopically there is an unordered
increased cellularity with an uneven distribution and
infiltrative margins. When infiltrating the grey matter,
there tends to be clustering around neurons
(satellitosis). The cells have a generous eosinophilic
cytoplasm which gives off multiple processes to form a
fibrillary background. An occasional variant, the
gemistocytic astrocytoma, is composed largely of neo-
plastic astrocytes with abundant eosinophilic glassy
cytoplasm that resembles reactive gemistocytes.
Differential diagnosis lies primarily with reactive
gliOSis and other forms of gliomas. The well
differentiated lesions have a slow growth rate but may
undergo more malignant degeneration (particularly the
gemistocytic variant). Average post-operative survival
is 3-10 years.
b. Anaplastic astrocytoma (Grade II, III) - these have the
same distribution pattern as the well differentiated
tumors but have a peak incidence in the 5th decade. Many
may be simply dedifferentiation of a preexisting well
differentiated tumor. Gross ly, they may be somewhat more
firm than the well differentiated tumors. Microscop-
ically, there is increased cellularity with greater
pleomorphism, increased mitoses, and increased N/C
ratio. Average post-operative survival is 2 years.
c. Glioblastoma multiforme (Grade IV) - unfortunately, this
highly malignant neoplasm is the most common glioma
(50-60% of gliomas). The peak incidence is in the 5th to
6th decade. They usually arise in the frontal or
temporal lobes but may be deeper seated within the basal
ganglia or thalamus. Symptomatology is the same as other
astrocytomas but with more rapid progression and
deterioration. Grossly, they may appear deceptively
discrete but there is extensive infiltration of
surrounding white and grey matter. Cystic degeneration,
necrosis, and hemorrhage impart a variegated coloration.
Microscopically, diagnostic features include necrosis
(often marginated by pseudopalisading of neoplastic
cells) and vascular and endothelial proliferation.
Untreated, the average survival . is 14 weeks. With
radiation, survival may be extended up to 10-11 months.

189
d. Cerebellar astrocytoma (spongioblastoma) - these tumors
predominate in childhood (1st or 2nd decade) and arise
in the cerebellar hemispheres or vermis. Symptoms
include cerebellar dysfunction and the effects of
increased pressure. Grossly appears as discrete lesions
often with cystic degeneration. Malignant degeneration
is rare. Prognosis is good with long survivals (and some
cures) even following partial resection.
e. Brain stem glioma - the peak incidence occurs in child-
hood. Most originate in the pons and cause diffuse
enlargement often at the expense of the 4th ventricle.
The tumors are unresectable and length of survival
depends on degree of differentiation.
2. (5% of gliomas) - these are primarily tumors of
OLlGOOENDROGLICl1A
middle-aged adults and may first be manifested by seizures.
Grossly, the lesions arise in the white matter and are
relatively discrete, soft, grey-pink masses that are often
focally calcified and occasionally cystic. Microscopically
consist of patternless sheets of monotonous small round cells
with round to oval nuclei, delicate chromatin, and perinuclear
halos. Vascular patterns may be prominent. Calcification is
common (40%). Most are well differentiated but there is a
variable spectrum of anaplasia characterized by increased
cellularity, increased N/C ratio, increased pleomorphism,
increased mitoses, etc. These are usually unresectable and
prognosis depends on the degree of differentiation and
aggressiveness of the tumor.
3. E~~ (6% of intracranial gliomas but 65% of intraspinal
gliomas) - may occur anywhere in the ventricular system but
tend to occur most frequently in 4th ventricle with a peak
incidence in the 1st and 2nd decade (5-10% of childhood
intracranial tumors) . Symptoms due to obstructive effects
reflected by hydrocephalus or increased pressure. Fourth
ventricle lesions tend to grow into the ventricle while
supratentorial lesions have greater tendency to expand into
the parenchyma but, compared to other gliomas, usually have a
sharper demarcation from the surrounding brain. Ependymal
cells are derived from neuroglia but also have epithelial
properties and ependymomas may reflect both glial and
epithelial traits. Glial (most common form) shows unpatterned
proliferation of· cells which tend to form vascular
pseudorosettes (clear zones around vessels composed of
radiating cytoplasmic processes). Epithelial traits are
reflected by ependymal rosettes (columnar cells radially
oriented around clear central lumina) . Presence of
blepharoplasts are pathognomonic but usually found only in
better differentiated tumors which contain ependymal rosettes.
Prognosis depends on location, surgical accessibility, and
degree of differentiation. They tend to recur locally and may
seed the subarachnoid space. Survival averages 5-6 years.
C. MESENCHYMAL ORIGIN - although primary CNS lymphoma was classically felt
to be derived from microglia, this has been called into question due
to apparent B-cell origin. Occur most commonly in the cerebrum of
adul ts. Symptoms due to increased pressure and local effects.
Grossly appear as grey, granular, discrete or confluent masses which
tend to arise deep in basal ganglia or thalamus.
D. NEURONAL ORIGIN - these tumors may affect CNS but more commonly involve
peripheral nerve ganglia and adrenal medulla.
E. utmIFFERENTIATED CELL ORIGIN - medulloblastoma arises in cerebellum
(usually midline vermis) with a peak incidence in 1st and 2nd decade
(second in frequency after astrocytoma). Symptoms due to cerebellar
dysfunction and increased pressure. Grossly, consist of fairly
discrete, soft, grey-pink masses. Microscopically has variable
morphology. Cell of origin disputed but may arise from cells of

190
external granular layer of cerebellum and may show differentiation
along neural or glial lines. The tumor frequently seeds the
subarachnoid space. Survival averages 3-4 years.
F. I'JETASTATIC ORIGIN - incidence varies with age but metastatic tumors
comprise between 15-30% of intracranial tumors. Symptoms (headache,
seizures, motor deficits, mental changes, etc.) are the result of
local expansion and surrounding edema. In order of frequency,
primary sites include lung > breast > melano¥Da ~ kidney > GI.
Melanomas and choriocarcinomas have a higher incidence of CNS
metastases but are less common tumors. Most metastatic lesions
appear in the cortex at the grey-white junction and may be single or
multiple. They tend to occur most frequently in distribution of
middle cerebral artery. Usually well demarcated spherical nodules
surrounded by edema. Microscopic appearance depends on primary site.

PERIPHERAL NERVOUS SYSTEM


I. RESPONSE TO IN.JURY

A. KALLERIAN DEGENERATIOH - is a term originally applied to degenerative


changes in distal segment of a mechanically transected nerve. The
term has now been extended to include other types of transection
such as vascular, radiation, compression, etc. The initial axonal
changes are the accumulation of organelles in the proximal and
distal axonal stumps with subsequent fragmentation of the distal
axoplasm. Schwann cell changes consist of initial retraction of
paranodal myelin with subsequent fragmentation of myelin sheaths.
Schwann cells proliferate and regeneration may occur by axoplasmic
sprouting from the proximal axonal stump. If regeneration fails,
Schwann cells atrophy and are replaced by fibrous tissue.
Regenerated axons may be remyelinated.
B. AXONAL DY;rrROPHY (axonal degeneration, "dying back" neuropathy) - does
not involve transection. These terms are applied to incompletely
characterized changes that are encountered in a wide variety of
toxic and metabolic neuropathies. There may be selective involvement
of certain fiber populations and certain organelles. The distal or
even terminal parts of axon are affected more severely, and it may
be accompanied by secondary segmental demyelination.
C. AXONAL ATROPHY - is a term used to describe changes encountered in
chronic, progressive neuropathies, many of which are hereditary. It
may simply represent a more chronic form of axonal dystrophy.
D. SEGHENTAL DEHYELINATIOH - is a term employed to describe changes that
result from disorders primarily damaging Schwann cells and/or myelin
sheaths. It results in demyelinated internodes with intact axons.
Remyelinated internodes are abnormally short (intercalated segments)
and have abnormally thin myelin.
E. SECONDARY SEGtlEHI'AL DEI1YELINATIOH - disorders formerly considered as
"classic" examples of segmental demyelination have now been found to
be accompanied by significant axonal alterations. It may be preceded
or accompanied by axonal dystrophy or atrophy.
F. "ONIOH BULBS" - are concentric lamellae of Schwann cell processes
around a nerve fiber. It is a non-specific response to repeated
episodes of segmental demyelination and remyelination.
G. BNm§ OF ~R - represent stacks of Schwann cell processes within
a common basement membrane and reflect complete degeneration of
axons.
II . PERIPHERAI~ NEUROPATHIES

A. DIABETIC NEUROPATHY the most common form is a symmetrical


polyneuropathy that can involve sensory and autonomic nerves. There
tends to be extensive segmental demyelination probably secondary to
ax.onal degeneration. Mononeuropathies (possibly due to diabetic
vascular disease) also occur and may involve cranial nerves.

191
B. ALCOHOLIC NEUROPA11IY is also common and involves segmental
demyelination secondary to axonal degeneration, possibly due to
thiamine deficiency.
C. GUILLAIH-BARRE DISEASE - is an autoimmune disorder causing progressive,
but usually self-limiting, ascending motor paralysis which may
affect both spinal and cranial nerves. It is often preceded by a
respiratory or gastrointestinal viral illness, and is characterized
histologically by segmental demyelination and perivascular
mononuclear inflammatory cell infiltrates. Serum antibodies to
peripheral nerve myelin are present in the acute stages.
D. CHAmxn-KARIE-~ D~ - is an autosomal dominant disorder leading
to weakness and atrophy of the foot and leg muscles. There are two
forms: the hypertrophic form which has an onset in 2nd to 4th decade
with palpably enlarged nerves in 25% of patients, loss of larger
myelinated fibers, axonal atrophy with secondary segmental
demyelination, and prominent "onion bulbs"; and the neuronal form
which has a late onset, few enlarged nerves, and few or no "onion
bulbs" although some demyelination and axonal atrophy does occur.
E. DE.JERIHE-sarrAS' DISEASE - is an autosomal recessive disorder with very
early onset. The peripheral nerves are strikingly enlarged,
gelatinous, and contain numerous, very prominent "onion bulbs".
F. REFsutI'S DISEASE - is an autosomal recessive disorder involving a
defect in the metabolism of phytanic acid. Prominent "onion bulbs"
are present on histologic examination. There is also persistent
elevation of CSF protein.
II I. TUMORS OF CRANIAL AND PERIPHERAL NERVES

A. cranial nerve involvement usually occurs in


NEURILBItIQtIA (SCHWAHHOO) -
the acoustic nerve. Peripheral involvement more often affects sen-
sory branches. Grossly appears as single or multiple whitish-gray
firm encapsulated masses. Histologic patterns include the Antoni A
(interlacing bundles of spindle cells with tendency toward nuclear
palisading) and Antoni B (loose arrangement of stellate cells within
a myxoid background). Rarely, they may undergo malignant transforma-
tion.
B. NEUROFIBROIA - shows a fusiform swelling of cranial or peripheral
nerves and nerve roots. Comprised of spindle cells arranged in
loose, disorganized pattern with wavy nuclei. In contrast to
neurilemmoma, nerve fibers ma~ be present. Multiple tumors are also
seen with Von Recklinghausen s disease and may undergo malignant
transformation.

SKELETAL MUSCLE
I. MUSCULAR DYSTROPHIES - are genetically determined myopathies in which
progressive atrophy and/or degeneration of muscle fibers is the hallmark.
A. DUCHEHHE I'IUSCULAR DYSTROPHY - is the most common form} is X -linked
recessive (males are affected, females are carriers), and
characterized by the absence of dystrophin, a normal muscle protein
that appears to stabilize the muscle fiber membrane. Although there
are markedly elevated "muscle enzymes" (CK and aldolase) and
abnormal biopsies from birth, clinical weakness is usually not
apparent until age 3 or 4. The muscular weakness is progressive and
most patients are can no longer walk by age 10. Death usually occurs
by age 20 from pneumonia or cardiac involvement. In the early
stages, although there is patchy necrosis and regeneration of
muscle, the involved muscles appear hypertrophied due to replacement
by fibrofatty tissue, so-called "pseudohypertrophy". In the late
stages, little identifiable muscle tissue is observable.
B• BECKER I1USCULAR DYSTROPHY - is also X-linked recess i ve and may be a
milder form of Duchenne' s. It has a later onset and may not be
noticed until after the age of 25. It tends to have a chronic
course, and many patients live a normal lifespan.

192
C. FACIOSCAPULOJIUItERAL DYSTROPHY - is autosomal dominant and usually becomes
manifest during adolescence as progressive weakness of face,
shoulder girdle and upper arms.
D. LlIB-GIIIDLE QXSTROnY - is autosomal recessive and causes slowly
progressive weakness of the proximal muscle groups of the shoulder
and pelvic girdles.
E. IIXQ'lPIIC DXSl'BOPJIY - is autosomal dominant and has two forms: a
congenital form in which there is extreme weakness from birth with
death occurring in the neonatal period, and an adult form in which
the presenting complaint is usually distal muscular weakness and
atrophy plus myotonia (tonic muscular spasms persisting after
cessation of stimulation). Patients may have subnormal or borderline
intelligence, and males may display frontal balding. Weakness and
atrophy are slowly progressive and may result in severe
incapacitation in late life.
II. MYASTIIENIA GRAVIS - is a relapsing, remitting autoimmune disease in
which weakness and extreme muscular fatigue are characteristic.
Autoantibodies against acetylcholine receptors on the post-synaptic
membranes of the neuromuscular junction are present in the serum of the
majority of patients. Mores commonly seen in females, it may appear at any
age but the peak age of onset is about 20. Almost all of the patients have
abnormal thymus glands, either a thymoma or thymic hyperplasia. The
disorder is progressive but the prognosis is highly variable. Thymectomy
and possibly plasmapheresis may be of benefit.

193
SELF ASSESSMENT EXAM A

DIRECTIONS: SEl,ECT THE ONE BEST ANSWER OR COMPLETION

1. Osteomalacia is associated with each of the following EXCEPT:


A. Chronic renal insufficiency.
B. Immobilization.
C. Malabsorption states.
D. Rickets.
E. Vitamin D deficiency.
2. Which of the following alterations that can be seen in fibrocystic
change of the breast is most associated with an increased risk of the
development of carcinoma:
A. Apocrine metaplasia.
B. Cystic ductal dilation.
C. Epithelial hyperplasia.
D. Stromal fibrosis.
3. Of the following, the cell that is most likely to be damaged by
ionizing radiation is:
A. Fibroblast.
B. Hepatocyte.
C. Intestinal mucosal cell.
D. Neuron.
E. Pancreatic islet cE~I1.
4. Which of the following may show clinical symptoms related to low cardiac
output:
A. Dilated cardiomyopathy.
B. Hypertrophic cardiomyopathy.
C. Both.
D. Neither.
5. The most accurate predictor of melanoma behavior is:
A. Anatom:ic location.
B. Degree of inflammatory reaction.
C. Lesion thickness.
D. Number of mitoses per square centimeter.
E. Presence or absence of ulceration.
6. The MOST common type of thyroid carcinoma is:
A. Anaplastic.
B. Fo11icular.
C. Hurthle ce11.
D. Medullary.
E. Papi11ary.
7. The MOST COMMON cause of hematosalpinx is:
A. Ectopic tubal pregnancy.
B. Endometriosis.
C. Gonorrheal salpingitis.
D. Ruptured luteal cyst.

194
8. Which of the following statements concerning Crohn's disease is TRUE:
A. Always begins in the rectal area and extends proximally.
B. Invariably associated with non-caseating granulomatous inflammation of
the submucosal tissue.
C. May produce symptoms of partial bowel obstruction due to stenosis and/or
strictures of the bowel lumen.
D. Patients are at high risk of developing colonic carcinoma.
9. Early diagnosis of pancreatic cancer is most likely if the tumor:
A. Has metastasized to the brain.
B. Has metastasized to the liver.
C. Is located in the head of the pancreas.
D. Is located in the tail of the pancreas.
10. The development of edema would be expected when there is:
A. Decreased intravascular hydrostatic pressure.
B. Increased oncotic pressure of blood.
C. Both.
D. Neither.
11. Reticulocytosis would be a feature of:
A. Aplastic anemia.
B. Autoimmune hemolytic anemia.
C. Megaloblastic anemia.
D. Sideroblastic anemia.
12. The primary vascular mechanism for the edema in acute inflammation:
A. venous congestion.
B. lymphatic obstruction.
C. increased arterial flow.
D. increased arterial pressure.
E. increased vascular permeability.
13. The MOST malignant of testicular tumors is the:
A. Choriocarcinoma.
B. Embryonal carcinoma.
C. Gonadal stromal tUDlor.
D. Seminoma.
14. Which of the following features is most characteristic of a neoplasm:
A. Autonomous growth.
B. Increased vascularity.
C. Large size.
D. Necrosis.
E. Rapid growth.
15. Traumatic skull fracture with tearing of the middle meningeal artery
results in:
A. Epidural hemorrhage.
B. Hemorrhage in the base of the pons.
C. Hemorrhage in the external capsule.
D. Subarachnoid hemorrhage.
E. Subdural hemorrhage.

195
16. The most distinctive structural defect in pulmonary emphysema is:
A. Cellular infiltration of the walls of small airways.
B. Dilatation and thickening of bronchial walls.
C. Enlargement of air spaces and loss of alveolar septa.
D. Increase in total mass of lung tissue.
E. Loss of alveolar epithelium.
17. Necrosis of the renal papillae is MOST commonly seen in patients with
which of the following diseases:
A. Chronic glomerulonephritis.
B. Chronic pyelonephritis.
C. Diabetes mellitus.
D. Lupus erythematosus.
18. The buildup of excessive amounts of collagen during wound healing is called:
A. Contraction.
B. Exuberant granulation.
C. Keloid.
D. Organization.
E. Proud flesh.
19. A 40 year old woman who complains of a low-grade fever, malaise, and
stiffness in her joints each morning most likely has which of the
following diseases:
A. Gout.
B. Metastatic carcinoma.
C. Osteoarthritis.
D. Rheumatoid arthritis.
E. Vi110nodular synovitis.
20. Which of the following clinical features is LEAST likely to indicate
a malignancy of the breast:
A. Alterations of breast contour.
B. Edema of the skin.
C. Nipple inversion.
D. Skin dimpling.
E. Tenderness to palpation.
21. Radiation is most likely to induce cell injury because of its effects
on the:
A. Cell membrane.
B. DNA.
C. Endoplasmic reticulum.
D. Lysosomes.
E. Mitochondria.
22. Manifestations of right-sided heart failure might include all of the
following EXCEPT:
A. Ascites.
B. Hepatic congestion.
C. Pulmonary edema.
D. Splenomegaly.
23. Mycosis fungo ides is a lymphoma of which cell type:
A. B-cel1.
B. Langerhans cell.
C. M~rckle cell.
D. Null cell.
E. T-cell.

196
24. Following the injection of TRF, thyrotropin (TSH) would NOT be expected
to rise in a patient:
A. Who is hyperthyroid.
B. Who is hypothyroid.
C. With a defect in thyroid hormone synthesis.
D. With hypothalamic disease.
25. Which of the following is a masculinizing tumor of the ovary:
A. Dysgerminoma.
B. Granulosa cell tumor.
C. Krukenberg tumor.
D. Mucinous cystadenoma.
E. Sertoli-Leydig cell tumor.
26. Which of the following would favor a diagnosis of ulcerative gastric
carcinoma over a diagnosis of gastric peptic ulcer:
A. Clean ulcer base.
B. Indurated ulcer margins.
C. Mucosal folds radiating from ulcer.
D. All of the above.
27. Common complications of alcoholic cirrhosis of the liver include all
of the following EXCEPT:
A. Ascites.
B. Bleeding abnormalities.
C. Cholelithiasis.
D. Enlarged spleen.
E. Jaundice.
28. Pooling of blood in capillary beds and venules due to impaired blood
flow is known as:
A. Congestion.
B. Hyperemia.
C. Hypovolemia.
D. Shock.
E. Vasoconstriction.
29. Which of the following typically causes a macrocytic anemia:
A. Acute blood loss.
B. Folate deficiency.
C. Both.
D. Neither.
30. Fluid exudation and neutrophil emigration in acute inflammation occur
predominantly from:
A. arteries.
B. arterioles.
C. capillaries.
D. venules.
E. veins.
31. Distant metastasis in prostatic carcinoma MOST COMMONLY involves:
A. Adrenal.
B. Bone.
C. Brain.
D. Liver.
E. Lung .•

197
32. In most instances, metaplasia is the result of:
A. Chronic irritation.
B. Developmental defect.
C. Immunologic reaction.
D. Somatic mutation.
E. Viral infection.
33. Infarcts of the brain which occur in the boundary zone between major
arterial supplies are usually the result of:
A. Atherosclerosis.
B. Emboli.
C. Hypotension.
D. Thrombosis.
34. The lung cancer which most commonly produces and secretes hormone-like
substances is:
A. Adenocarcinoma.
B. Larfe cell undifferentiated carcinoma.
C. Sma I cell undifferentiated carcinoma.
D. Squamous cell carcinoma.
35. The most common renal malignancy in childhood would be:
A. Neuroblastoma.
B. Renal cell carcinoma.
C. Transitional cell carcinoma.
D. Wilms' tumor.
36. The phenomenon of wound contraction may be:
A. Advantageous in that it reduces the amount of scar tissue needed.
B. Deleterious in that slows down the healing process.
C. Both.
D. Neither.
37. Of the following, the most common tumor that involves bone is:
A. Chondrosarcoma.
B. Giant-cell tumor.
C. Metastatic tumor from an extraosseous site.
D. Multiple myeloma.
E. Osteogenic sarcoma.
38. Factors predisposing to the development of breast. cancer in women
include each of the following EXCEPT:
A. Fibrocystic change of the breast.
B. Increasing age.
C. Maternal history of breast cancer.
D. Multiparity.
E. Previous mastectomy for cancer.
39. Which of the following features characterizes irreversible cell injury:
A. Cellular swelling.
B. Cytoplasmic franulation.
C. Mitochondria swelling.
D. Nuclear pyknosis.
E. Polysome disaggregation.

198
40. A 62 year old insulin dependant diabetic with a past history of myocardial
infarction suddenly develops acute abdominal pain and bloody diarrhea
followed by ileus and abdominal rigidity. Exploratory laparotomy would most
likely reveal which of the following:
A. Bleeding duodenal peptic ulcer.
B. Encroacnment of mesenteric fat over the serosal surfaces of the bowel.
C. Thrombosis of the portal vein.
D. Thrombotic occlusion of proximal superior mesenteric artery.
41. The characteristic cutaneous immune deposits in systemic lupus
erythematosus are seen in the:
A. Dermal blood vessels.
B. Dermal papillae.
C. Epidermal basement membrane zone.
D. Epidermal intercellular spaces.
42. A localized, destructive lesion of the posterior pituitary would most
likely affect which of the following serum values:
A. Calcium.
B. Creatinine.
C. Glucose.
D. Potassium.
E. Sodium.
43. Metastatic, mucin-producing, signet-ring cancer cells in the ovary most
frequently come from:
A. Astrocytoma.
B. Endometrial carcinoma.
C. Gastrointestinal carcinoma.
D. Histiocytic lymphoma.
E. Malignant melanoma.
44. Adenocarcinoma of the esophagus would most likely arise in the:
A. Distal esophagus.
B. Mid esophagus.
C. Proximal esophagus.
D. Equal distribution between all three of the above.
45. Of the following, hepatocellular carcinoma is most often associated with:
A. Biliary atresia.
B. Chronic pancreatitis.
C. Gallstones.
D. Hepatic cirrhosis.
46. Hypovolemic shock would be most likely to develop in patients with:
A. Bee stings.
B. Cardiac failure.
C. Extensive burns.
D. Head trauma.
E. Septicemia.
47. Renal failure is characteristically associated with:
A. Acute myelogenous leukemia.
B. Chronic lymphatic leukemia
C. Hodgkin's disease.
D. Multiple myeloma.
E. NodulaF lymphoma.

199
48. All of the following cell types are found in acute and/or chronic
inflammatory reactions EXCEPT:
A. Lymphocytes.
B. Macrophages.
C. Megakaryocytes.
D. Plasma cells.
E. Polymorphonuclear leukocytes.
49. Metastatic spread of a prostatic adenocarcinoma would be most likely
to occur through the:
A. Arterial system.
B. Lymphatic system.
C. Peritoneal cavity.
D. Venous system.
50. Which of the following is a benign neoplasm of epithelial origin:
A. Astrocytoma.
B. Chondroma.
C. Lipoma.
D. Melanoma.
E. Papilloma.
51. Which of the following intracranial neoplasms has the highest incidence
in adults:
A. Ependymoma.
B. Ganglioglioma.
C. Glioblastoma multiforme.
D. Neuroblastoma.
E. Oligodendroglioma.
52. A decrease in surfactant activity is the underlying problem in patients
with:
A. Alpha-1-antitrypsin deficiency.
B. Asbestosis.
C. Asthma.
D. Bronchiectasis.
E. Respiratory distress of the newborn.
53. The disease most likely to cause nephrotic syndrome in a child is:
A. Focal sclerosis.
B. Lipoid nephrosis.
C. Membranous glomerulonephritis.
D. Membranoproliferative glomerulonephritis.
54. Each of the following cell types are capable of regeneration EXCEPT:
A. Astrocyte.
B. Cardiac muscle.
C. Fibroblast.
D. Hepatocyte.
E. Keratinocyte.
55. Generation of free radicals is the major mechanism of cell injury in:
A. Ischemia.
B. Oxygen toxicity.
C. Both.
D. Neither.

200
56. Elevated serum levels of which of the following is NOT associated with
the development of clinically significant atherosclerosis:
A. Alpha lipoproteins.
B. Beta lipoproteins.
C. Low density lipoproteins.
D. Pre-beta lipoproteins.
57. Most of the human papilloma viruses (HPV) associated with human
malignant tumors are strains related to:
A. Epidermodysplasia.
B. Flat warts.
C. Immunosuppression.
D. Sexual transmission.
58. Secondary hyperparathyroidism will characteristically show:
A. Elevated serum calcium.
B. Elevated serum PTH.
C. Both.
D. Neither.
59. Each of the following is characteristic of carcinoma of the vulva EXCEPT:
A. Affects post-menopausal age group.
B. May be preceded by leukoplakia.
C. Majority are derived from subcutaneous sweat glands.
D. Regional node metastases often present at time of diagnosis.
60. The MOST common type of polyp of the colon and rectum is:
A. Adenomatous polyp.
B. Juvenile adenoma.
C. Pseudopolyp.
D. Villous adenoma.
61. Total functional regeneration of acutely injured liver is dependent
upon the degree of injury to:
A. Bile ducts.
B. Central veins.
C. Hepatocytes.
D. Portal veins.
E. Reticular framework.
62. Which of the following has the LEAST influence on slowing hemorrhage
from venules:
A. Blood coagulation.
B. Direct pressure on bleeding site.
C. Formation of platelet plugs.
D. Vasoconstriction.
63. Massive splenomegaly would be most likely to accompany which of the
following:
A. Acute lymphoblastic leukemia.
B. Chronic myelogenous leukemia.
C. Multiple myeloma.
D. Hodgkin's disease.
E. Sickle cell anemia.

201
64. The most characteristic cell of the acute inflammatory response is the:
A. Giant cell.
B. Lymphocyte.
C. Monocyte.
D. Plasma cell.
E. Polymorphonuclear leukocyte.
65. Which of the following disorders would be LEAST likely to present as
gross hematuria in a 46 year old male:
A. Adult polycystic disease.
B. Carcinoma of the urinary bladder.
C. Renal calculi.
D. Renal cell carcinoma.
E. Wilms' tumor.
66. A malignant neoplasm of connective tissue origin is a/an:
A. Adenoma.
B. Carcinoma.
C. Hamartoma.
D. Sarcoma.
E. Teratoma.
67. Which of the following microscopic findings is LEAST suggestive of
Alzheimer's disease:
A. Granulovacuolar degeneration.
B. Increased neuronal lipofuscin pigmentation.
C. Neurofibrillary tangles.
D. Senile plaques.
68. The MOST frequent cause of symptomatic pulmonary edema is:
A. Anaphylaxis.
B. Infection.
C. Left heart failure.
D. Shock.
69. Intermittent severe flank pain which radiates to the groin would be the
dominant symptom of:
A. Acute pyelonephritis.
B. Papillary necrosis.
C. Renal infarct.
D. Ureter obstructed by calculus.
70. Which one-of the following is LEAST likely to regenerate:
A. hepatocytes.
B. axons.
C. bone.
D. cardiac muscle.
E. renal tubular epithelium.

202
SELF ASSESSMENT EXAM A

ANSWER KEY

1. B 11. B 21. B 31. B 41. C 51. C 61. E


2. C 12. E 22. C 32. A 42. E 52. E 62. D
3. C 13. A 23. E 33. C 43. C 53. B 63. B
4. C 14. A 24. A 34. C 44. A 54. B 64. E
5. C 15. A 25. E 35. D 45. D 55. B 65. E
6. E 16. C 26. B 36. A 46. C 56. A 66. D
7. A 17. C 27. C 37. C 47. D 57. D 67. B
8. C 18. C 28. A 38. D 48. C 58. B 68. C
9. C 19. D 29. B 39. D 49. B 59. C 69. D
10. D 20. E 30. D 40. D 50. E 60. A 70. D

203
SELF ASSESSMENT EXAM B

DIRECTIONS: SELECT mE ONE BEST ANSWER OR COMPLETION

1. Inadequate mineralization of bone matrix is characteristic of:


A. Achondroplasia.
B. Hypoparathyroidism.
C. Osteogenesis imperfecta.
D. Osteoporosis.
E. Rickets.
2. A 22-year-old woman presents with a discrete upper/outer quadrant
breast mass. The most likely diagnosis is:
A. Fat necrosis.
B. Fibroadenoma.
C. Fibrocystic change.
D. Infiltrating ductal carcinoma.
E. Intraductal papilloma.
3. Quantitatively, which of the following serum proteins is present in the
greatest concentration in a healthy individual:
A. Albumin.
B. Beta lipoproteins.
C. Gammaglobulins.
D. Haptoglobin.
4. Complications of cystitis include each of the following EXCEPT:
A. Acute pyelonephritis.
B. Cystitis cystica.
C. Formation of bladder stones.
D. Transitional cell carcinoma.
5. Pleural mesotheliomas are most closely associated with:
A. Anthracosis.
B. Asbestosis.
C. Berylliosis.
D. Silicosis.
E. Smoking.
6. Of the following, the cerebrospinal fluid finding most consistent with
acute bacterial meningitis is:
A. Increased glucose.
B. Increased lymphocyte count.
C. Increased protein.
D. Xanthochromia
7. An elderly male presents with a 3-month history of severe throbbing
pain and tenderness over the temple. The most likely diagnosis is:
A. Giant cell arteritis.
B. Raynaud IS di.sease.
C. Rheumatoid arteritis.
D. Syphilitic arteritis.
E. Thromboangiitis obliterans.

204
8. After activation, each of the following chemical mediators of
inflammation often proceeds in a cascade EXCEPT:
A. Arachidonic acid.
B. Complement.
C. Fibrinopeptides.
D. Kinin.
E. Neutral proteases.
9. Factor VIII deficiency is associated with prolonged:
A. Bleeding time.
B. Clotting time.
C. Both.
D. Neither.
10. Massive pulmonary thromboemboli cause sudden death because of:
A. Acute cor pulmonale with arrhythmias.
B. Acute pulmonary infarction.
C. Asphyxia.
D. Cerebral anoxia.
E. Massive hemoptysis.
11. Primary biliary cirrhosis is typically associated with each of the
following EXCEPT:
A. Alcohol abuse.
B. Hyperlipidemia.
C. Jaundice.
D. Middle-aged women.
E. Pruritus.
12. Hirschsprung's disease is characterized by absence of ganglion cells in the:
A. Adrenal medulla.
B. Body of the stomach.
C. Distal esophagus.
D. Rectum.
E. Ureters.
13. Each of the following statements concerning squamous cell carcinoma of
the uterine cervix is true EXCEPT:
A. It begins at the cervical/endocervical squamo-columnar junction.
B. It is associated with human papilloma virus infection.
C. It is usually preceded by dysplastic epithelial changes.
D. It shows early, widespread metastases.
14. Which of the following tumors would be most likely associated with
abdominal stria, easy bruising, and osteoporosis are associated with:
A. Adrenal adenoma.
B. Craniopharyngioma.
C. Parathyroid adenoma.
D. Pheochromocytoma.
E. Thyroid adenoma.
15. Which of the following is most characteristic of polyarteritis nodosa:
A. Cutaneous rash.
B. Hypothyroidism.
C. Jaundice.
D. Necrotizing arteritis.
E. Urtic8,J"ia.

205
16. A myocardial infarct which is grossly detectable, yellow to gray in color,
and microscopically contains necrotic debris and macrophages but little
evidence of granulation tissue is probably:
A. 1 hour old.
B. 12 hours old.
C. 1 day old.
D. 1 week old.
E. 1 month old.
17. Light microscopic examination of a pulmonary infarct would reveal:
A. Caseous necrosis.
B. Coagulation necrosis.
C. Enzymatic fat necrosis.
D. Gangrenous necrosis.
E. Liquefaction necrosis.
18. Liver cells are an example of:
A. Alternate cells.
B. Labile cells.
C. Permanent cells.
D. Stable cells.
19. A 24 year old female presents to the emergency room with a three day
history of fever, malaise, and dysuria. Physical examination reveals
costo-vertebral angle tenderness. BUN and creatinine levels are
normal.UA shows a few red blood cells, white blood cell casts, and
bacteria. The most likely diagnosis would be:
A. Acute pyelonephritis.
B. Chronic pyelonephritis.
C. Goodpasture's syndrome.
D. Necrotizing papillitis.
20. Which of the following is associated with an increased incidence of cancer:
A. Anthracosis.
B. Asbestosis.
C. Berylliosis.
D. Siderosis.
E. Silicosis.
21. In an axonal reaction, degeneration and disintegration of the myelin
sheath and axon cylinder is called:
A. Axonal dystrophy.
B. Central chromatolysis.
C. Clasmatodendrosis.
D. Gliosis.
E. Wallerian degeneration.
22. Which of the following terms would best describe the presence of squamous
epithelium lining the renal pelvis:
A. Desmoplasia.
B. Dysplasia.
C. Ectopia.
D. Hyperplasia.
E. Metaplasia.

206
23. The organism most frequently responsible for acute bacterial
endocarditis is:
A. Beta hemolytic streptococcus.
B. Diplococci pneumoniae.
C. Hemophilus influenzae.
D. Staphylococcus aureus.
E. Streptococcus viridans.
24. The benefit of the acute inflammatory response ion include:
A. Localization or walling off of site of damage.
B. Removal of dead cells and debris.
C. Preparation of area for repair ..
D. Laying foundation for hypersensitivity.
25. Which of the following non-Hodgkin's lymphomas has the best prognosis:
A. Follicular small cleaved cell (nodular poorly differentiated).
B. Large cell immunoblastic (diffuse histiocytic).
C. Lymphoblastic.
D. Small non-cleaved cell (diffuse undifferentiated).
26. Infarcts tend to be hemorrhagic when they occur in the:
A. Heart.
B. Intestine.
C. Kidney.
D. Pancreas.
E. Spleen.
27. The pathogenic factor which appears to be responsible for the maj~rity
of instances of cholelithiasis is:
A. Acute cholecystitis.
B. Autoantibodies to bile.
C. Hemolysis.
D. High fat diet.
E. Supersaturation of bile with cholesterol
28. Diverticulosis of the colon:
A.. Increases in incidence with age.
B. Is a necessary precursor to the development of hemorrhoids.
C. Is complicated by diverticulitis in over half of the cases.
D. Is inherited as an autosomal dominant trait.
E. Predisposes to carcinoma of the colon.
29. Causes of infertility include each of the following EXCEPT:
A. Cervical dysplasia.
B. Chronic salpingitis.
C. Endometriosis.
D. Leiomyomas.
E. Polycystic ovary disease.
30. A patient with Addison's disease is likely to exhibit all of the
following EXCEPT:
A. Adrenal atrophy.
B. Central obesity.
C. Hyperpigmentation of skin.
D. Hypotension.

207
31. After activation, each of the following chemical mediators of
inflammation often proceeds in a cascade EXCEPT:
A. Arachidonic acid.
B. Complement.
C. Fibrinopeptides.
D. Kinin.
E. Neutral proteases.
32. Each of the following is a feature of Tetralogy of Fallot EXCEPT:
A. Atrial septal defect.
B. Pulmonary stenosis.
C. Right ventricular hypertrophy.
D. Ventricular septal defect.
33. Coagulation necrosis:
A. Grossly appears soft with ill-defined borders.
B. Is characteristic of granulomatous inflammation.
C. Is often associated with ischemia.
D. Results from lysosomal digestion of tissue.
34. The greatest enlargement of the spleen is usually found in which of the
following diseases:
A. Acute granulocytic leukemia.
B. Acute lymphocytic leukemia.
C. Chronic granulocytic leukemia.
D. Hodgkin's disease.
E. Multiple myeloma.
35. The tensile strength of a healing wound appears to be primarily a
function of:
A. Collagen deposition.
B. Dystrophic calcification.
C. Exuberant granulation tissue.
D. Inflammatory infiltrate.
E. Vascular proliferation.
36. On radiologic exam, small scarred kidneys which show blunting of the
pyramids would be most consistent with:
A. Chronic pyelonephritis.
B. Membranoproliferative glomerulonephritis.
C. Renal cell carcinoma.
D. Renal infarction.
37. Each of the following could be considered an obstructive disease EXCEPT:
A. Bronchial asthma.
B. Chronic bronchitis.
C. Mucoviscidosis.
D. Pulmonary emphysema.
E. Pulmonary fibrosis.
38. In the brain, which of the following cells are most sensitive to anoxic
damage:
A. Astrocytes.
B. Ependyma.
C. Oligodendroglia.
D. Microglia.
E. Neurons.

208
39. A teratoma is most likely to arise in whi.ch of the following locations:
A. Breast.
B. Kidney.
C. Lung.
D. Ovary.
E. Prostate.
40. During a pre-employment physical exam, a 35 year old female was found
to have a peripherally located "coin lesion" on chest X-ray. This would
most likely represent a/an:
A. Abscess.
B. Carcinoma.
C. Granuloma.
D. Hamartoma.
E. Infarct.
41. A granulomatous inflammatory response is diagnostic of:
A. Alcoholism.
B. Bacterial dise.ase.
C. Fungal disease.
D. Tuberculosis.
E. None of the above.
42. Reed-Sternberg cells are characteristic of which of the following diseases:
A. Cat scratch disease.
B. Chronic lymphocytic leukemia.
C. Histiocrtosis X.
D. Hodgkin s disease.
E. Thymoma
43. The most frequent site of venous thrombosis is the:
A. brain.
B. kidney.
C. leg.
D. liver.
E. lung.
44. Morphologic features of viral hepatitis include each of the
following EXCEPT:
A. Ballooning degeneration of hepatocytes.
B. Focal necrosis of hepatocytes.
C. Hypertrophy and hyperplasia of Kupffer cells.
D. Microabscesses.
E. Periportal inflammation.
45. Which of the following disorders would be most likely to present with
hematemesis:
A. Achalasia.
B. Hiatal hernia.
C. Mallory~Weiss syndrome.
D. Plummer-Vinson syndrome.
E. Zenker's diverticulum.
46. Characteristic features of polycystic ovary disease include each of the
following EXCEPT:
A. Amenorrhea.
B. Bilateral ovarian enlargement.
C. Hirsutism.
D. Obesity.
E. Ovarian "chocolate" cysts.

209
47. Which of the following tumors is most likely to be associated with
essential hypertension:
A. Adrenal adenoma.
B. Parathyroid adenoma.
c. Pheochromocytoma.
D. Pituitary adenoma.
E. Thyroid adenoma.
48. Tobacco smoking is closely associated with the development of:
A. Buerger's disease.
B. Giant cell arteritis.
C. Polyarteritis nodosa.
D. Raynaud's disease.
E. Wegener's granulomatosis.
49. In which of the following clinical settings might you expect acute cor
pulmonale to most likely arise:
A. A patient with a two year history of stable angina.
B. A patient with an 85 pack-year smoking history and chronic bronchitis.
C. A patient with kyphoscoliosis due to childhood polio.
D. An overweight patient in bed recovering from surgery for cervical cancer.
50. Of the following, dystrophic calcification would be most closely
associated with:
A. Hyperphosphatemia.
B. Necrotic tissue.
C. Osteoporosis.
D. Parathyroid hyperplasia.
E. Renal calcium excretion.
51. Huntington's chorea is characterized by:
A. Atrophy of the caudate nucleus.
B. Degeneration of upper motor neurons.
C. Demyelination of posterior spinocerebellar tract.
D. Depigmentation of substantia nigra.
E. Necrosis of Purkinje cells.
52. Osteogenic sarcoma is known to be a complication of:
A. Bone trauma.
B. Osteitis deformans (Paget's disease).
C. Osteoarthritis.
D. Osteogenesis imperfecta.
E. Multiple myeloma.
53. As a scar matures, it becomes:
A. Less vascular.
B. More cellular.
C. Both.
D. Neither.
54. Which of the following is LEAST likely to be seen in the renal biopsy of an
individual who has had non-insulin dependent diabetes mellitus for 15 years:

A. Diffuse glomerular sclerosis.


B. Hyaline arteriolosclerosis.
C. Nodular mesangial sclerosis.
D. TPickening of the glomerular capillary basement membranes.

210
55. In which of the following would an interstitial inflammatory infiltrate
be most striking:
A. Bronchopneumonia.
B. Fungal pneumonia.
C. Lobar pneumonia.
D. Viral pneumonia.
56. In the central nervous system, myelin formation and maintenance is a
function of:
A. Astrocytes.
B. hons.
C. Microglia.
D. Neuron.
E. Oligodendroglia.
57. A fibroadenoma of the breast is an example of:
A. Anaplasia.
B. Dysplasia.
C. Hyperplasia.
D. Metaplasia.
E. Neoplasia.
58. Of the following, the most life-threatening complication of ulcerative
colitis would be:
A. Enterocutaneous fistulas.
B. Hemorrhage.
C. Peritoneal adhesions.
D. Stenosis of ileo-cecal valve.
E. Toxic megacolon.
59. The first blood cells to aggregate at the site of injury usually are:
A. Lymphocytes.
B. Macrophages.
C. Plasma cells.
D. Polymorphonuclear leukocytes.
60. Which of the following is characteristic of acute lymphocytic leukemia:
A. Cytoplasmic auer rods.
B. Hepatosplenomegaly.
C. High incidence in children.
D. Lymphadenopathy.
E. Philadelphia chromosome.
61. Predisposing factors for thrombosis include all of the following EXCEPT:
A. Anemia.
B. Atherosclerosis.
C. High serum protein levels.
D. Thrombocytosis.
E. Venous obstruction.
62. The typical outcome of hepatitis A is:
A. Chronic active hepatitis.
B. Chronic persistent hepatitis.
C. Hepatic cirrhosis.
D. Massive hepatic necrosis.
E. Resolution.

211
63. In contrast to carcinoma of the right colon, carcinoma of the left
colon tends to:
A. Be annular and obstruct the colon earlier.
B. Be clinically silent or asymptomatic.
C. Cause anemia and anorexia.
D. Produce steatorrhea.
64. Which of the following is embryologically related to the paramesonephric
duct:
A. Gartner's duct.
B. Mesonephric duct.
C. Mullerian duct.
D. Vitelline duct.
E. Wolffian duct.
65. Each of the following neoplasms are associated with the multiple
endocrine neoplasia (~mN) syndromes EXCEPT:
A. Islet cell adenoma.
B. Medullary thyroid carcinoma.
C. Neuroblastoma.
D. Pheochromocytoma.
E. Pituitary adenoma.
66. The most frequently identified neoplasm in the lung is:
A. Adenocarcinoma.
B. Bronchioloalveolar carcinoma.
C. Squamous carcinoma.
D. Undifferentiated carcinoma.
E. Metastatic carcinoma.
67. Which heart valve is most commonly affected in rheumatic heart disease:
A. Aortic valve.
B. Mitral valve.
C. Pulmonary valve.
D. Tricuspid valve.
68. In hypoxic cell injury, cellular swelling occurs because of:
A. Active reabsorption of interstitial water.
B. Osmotic influx of water due to increased cellular glycogen.
C. Both.
D. Neither.
69. The MOST common cause of sudden death from myocardial infarction is:
A. Angina pectoris.
B. Arrhythmia.
C. Cardiac tamponade.
D. Cardiogenic shock.
E. Ventricular aneurysm.
70. The primary cartilage change in osteoarthritis is characterized by:
A. Degeneration.
B. Dysplasia.
C. Hyperplasia.
D. Inflammation.
E. Regeneration.

212
SELF ASSESSMENT EXAM B

ANSWER KEY

1. D 11. A 21. E 31. E 41. E 51. A 61. A


2. B 12. D 22. E 32. A 42. D 52. B 62. E
3. A 13. D 23. D 33. A 43. C 53. A 63. A
4. D 14. A 24. A 34. C 44. D 54. C 64. C
5. B 15. D 25. A 35. A 45. C 55. D 65. C
6. C 16. D 26. B 36. A 46. E 56. E 66. E
7. A 17. B 27. E 37. E 47. C 57. E 67. B
8. E 18. D 28. A 38. E 48. A 58. E 68. D
9. B 19. A 29. A 39. D 49. D 59. D 69. B
10. A 20. B 30. B 40. C 50. B 60. C 70. A

213

You might also like